You are on page 1of 158

CHAPTER – 1

SIMPLE EQUATIONS
There will be linear equations of one or two unknowns Step II: Solve for the value of one variable from the
invariably in every problem. A linear equation is one equation (in one unknown) obtained from Step 
where each variable occurs only in its first power and not above. Therefore, y = 2.
in any higher powers. Some times we get three
equations in three unknowns. In general, we need as Step III: Substitute this value of the variable in one of
many equations as the variables we will have to solve the two equations to get the value of the
for. So, for solving for the values of two unknowns, we second variable.
need two equations (or two conditions given in the
problem) and for solving for the values of three Substituting the value of y in equation (1) or
unknowns, we need three equations (and hence the equation (2), we get x = 1. Therefore the values
problem should give three conditions from which we can of x and y that satisfy the given set of
frame three equations). Solving the equations by itself is equations are x = 1 and y = 2.
not a difficult task. The most important part of the
problem is framing the equation/equations. Once the THREE EQUATIONS IN THREE UNKNOWNS
equations are framed, solving them is very easy. In this
chapter, we will deal with problems involving as many A set of equations like
equations (of first degree) as the number of unknowns. x + 2y + 3z = 14 .................. (1)
Later on, we will look at equations of second degree 2x + y + 2z = 10 .................. (2)
(Quadratic Equations) and linear equations where the 3x + 3y + 4z = 21 .................. (3)
number of equations will be less than that of the number is a system of three equations in three unknowns.
of variables (under the chapter Special Equations).
Here we have three unknowns x, y and z which we have
ONE EQUATION IN ONE UNKNOWN to solve for from the three given equations. The
procedure for the same is as follows:
An equation like 2x + 4 = 26 is an equation in one
unknown. We have only one variable x whose value we Step I: Take two out of the three equations [say, eqn.
have to find out. The steps in solving this are: (1) and (2)] and eliminate one variable (say x)
so that we get an equation in two unknowns
Step I: Take all quantities added to (or subtracted (y and z in this case).
from) the x term (term with the unknown) to the
right side with a change of sign. For this purpose, take equations (1) and (2).
i.e., 2x = 26 – 4 = 22. Multiply equation (1) by 2 and subtract
equation (2) from it.
Step II: Take the co-efficient of x from left hand side
and divide right hand side with this term to get Equation (1)  2  2x + 4y + 6z = 28
the value of x. 2x + y + 2z = 10
i.e., x = 22/2 = 11. Therefore, x = 11. ----------------------------
3y + 4z = 18..... (4)
TWO EQUATIONS IN TWO UNKNOWNS
Step II: Repeat Step  for two other equations [say
A set of equations like equations (2) and (3)] and eliminate the same
2x + 3y = 8 ......... (1) variable (x in this case) so that we get one
5x + 4y = 13 ........ (2) more equation in two unknowns (y and z).
is called a system of simultaneous equations in two
unknowns. Here, we have two variables (or unknowns) For this purpose, take equations (2) and (3).
x and y whose values we have to find out. This can be Multiply equation (2) by 3 and from that
done using the two given equations. The steps for this subtract equation (3) multiplied by 2.
are as follows:
Equation (2) x 3  6x + 3y + 6z = 30
Step I: Using both the equations we first eliminate Equation (3) x 2  6x + 6y + 8z = 42
one variable (so that we can then have --------------------------
one equation in one unknown). –3y – 2z = –12 .... (5)
For this purpose, we multiply equation (1) with
5 (the co-efficient of x in the second equation) Step III: Now the equations in two unknowns that have
and multiply equation (2) with 2 (the co-efficient been obtained from the above two steps have
of x in the first equation) to eliminate x. Thus to be solved as discussed previously
we have (in TWO EQUATIONS IN TWO UNKNOWNS)
(1)  5  10x + 15y = 40 ...… (3) to get the values of two of the three variables
(2)  2  10x + 8y = 26 ...... (4) (y and z in this case).
Now, subtracting equation (4) from equation (3)
we have 7y = 14 ...... (5) In this case, solving equations (4) and (5), we
This is one equation in one unknown. get y = 2 and z = 3.
Triumphant Institute of Management Education Pvt. Ltd. (T.I.M.E.) HO: 95B, 2nd Floor, Siddamsetty Complex, Secunderabad – 500 003.
Tel : 040–27898195 Fax : 040–27847334 email : info@time4education.com website : www.time4education.com SM1001906/1
Step IV: Substitute these values of the two variables in x  y = 2 or y  x = 2 --- (1)
one of the three equations to get the value of 10 (10x + y)  5 (10y + x + x + y) = 90
the third variable. 90x  45y = 90
Substitute the value of y and z in equation (1) 2x  y = 2
to get the value of x = 1. 2x  (x  2) = 2
Thus the values of the three variables x, y and x = 4 or 0
z that satisfy the three given equations are As x cannot be 0, x = 4
x = 1; y = 2 and z = 3 y=6
 the number is 46
Examples
1.04. The age of Fahim 4 years ago was 4 times of
1.01. The cost of 3 tables and 4 chairs is `2500. The Sachin’s age. Fahim’s age three years ago was
cost of 4 tables and 3 chairs is `2400. Find the thrice Sachin’s age. How many times Sachin’s
costs of each table and each chair. age was Fahim’s age 5 years ago?

Sol: Let the present ages of Fahim and Sachin be


Sol: Let the cost of each table be `x.
f years and s years respectively.
Let the cost of each chair be `y.
f  4 = 4 (s  4)  f = 4s  12
3x + 4y = 2500 --- (1)
f  3 = 3 (s  3)  f = 3s  6
4x + 3y = 2400 --- (2)
f = 4s  12 = 3s  6
Method 1: s = 6, f = 12
Multiplying (1) by 3 and subtracting it from f  5 = 7 (s  5)
(2) multiplied by 4, we get  5 years ago Fahim’s age was 7 times
7x = 2100 Sachin’s age.
x = 300
Substituting x = 300 in (1), 1.05. The age of a man 15 years ago was 5 times his
y = 400 son’s age. His age 10 years ago was thrice his
Method 2: son’s age. After how many years from now will
Adding both the equations (1) and (3), we get their combined age become 80 years?
7 (x + y) = 4900 Sol: Let the present age of the man and his son be
x + y = 700 --- (3) f years and s years respectively.
subtracting (2) from (1), f  15 = 5 (s  15)  f = 5s  60
 x + y = 100 --- (4) f  10 = 3 (s  10)  f = 3s  20
Adding (3) and (4), 2y = 800
f = 5s  60 = 3s  20
y = 400
s = 20, f = 40
Substituting y = 400 in either (3) or (4), x = 300
Their combined present age is 60 years. For the
combined present age to become 80 years, the
1.02. Raju bought 6 pens, 5 erasers and
age of each of them must increase by 10 years.
4 sharpeners for `32. Had he bought 4 pens,
 Their combined age will become 80 years
3 erasers and 5 sharpeners, his total
after 10 years.
expenditure would have been `23. Had he
bought 7 pens, 2 erasers and 6 sharpeners, his 1.06. If the numerator and the denominator of a
total expenditure would have been `31. Find the fraction are both increased by 1, the fraction
cost of 1 pen, 1 eraser and 2 sharpeners. 3
becomes . If both are decreased by 1, it
Sol: Let the prices of each pen, each eraser and 5
each sharpener be `p, `e and `s respectively. 5
becomes . Find the fraction.
6p + 5e + 4s = 32 --- (1) 9
4p + 3e + 5s = 23 --- (2)
7p + 2e + 6s = 31 --- (3) x
Sol: Let the fraction be .
Multiplying (1) by 2 and subtracting from (2) y
multiplied by 3, –e + 7s = 5 --- (4)
x 1 3
Multiplying (3) by 4 and subtracting it from (2) 
multiplied by 7, 13e + 11s = 37 --- (5) y 1 5
Multiplying (4) by 13 and adding it to (5), 102s = 102  5x + 5 = 3y + 3
s=1 5x + 2 = 3y --- (1)
Substituting s = 1 in (4), x 1 5
e=2 
Substituting values of e and s in (1), p = 3 y 1 9
 9x  9 = 5y  5
1.03. In a two digit number, the digits differ by 2. 9x  4 = 5y --- (2)
10 times the number exceeds 5 times the sum Multiplying (1) by 5 and subtract it from (2) after
of the number formed by reversing its digits and multiplying by 3, 5 (5x + 2) = 3 (9x  4)
the sum of its digits by 90. Find the number.  x = 11
substituting x = 11 in (1), y = 19
Sol: Let the number be xy. Hence the value of the 11
number is 10x + y. The fraction =
19
Triumphant Institute of Management Education Pvt. Ltd. (T.I.M.E.) HO: 95B, 2nd Floor, Siddamsetty Complex, Secunderabad – 500 003.
Tel : 040–27898195 Fax : 040–27847334 email : info@time4education.com website : www.time4education.com SM1001906/2
1.07. Find the values of x and y satisfying the Sol: Let the prices of each shirt, each trouser and
equations below. each pair of shoes be `x, `y and `z
35 18 respectively.
  11 2x + 4y + 5z = 3600 --- (1)
xy xy
6x + 5y + 15z = 8700 --- (2)
28 33 Multiplying (1) by 3 and subtracting (2) from it,
  15
xy xy 7y = 2100
y = 300
1
Sol: Let  p and (3) Even in case of indeterminate equations, when
xy
some additional conditions are either implicitly built
1 into the problem or explicitly imposed by specifying
q
xy some constraints on the values of the variables, we
may some times be able to determine the values of
35p + 18q = 11 --- (1)
28p + 33q = 15 --- (2) the variables uniquely or find out a finite set of
values that the variables may take. Such problems
Multiplying (1) by 4 and subtracting it from (2)
are separately considered under the chapter
multiplied by 5, 93q = 31
"SPECIAL EQUATIONS."
1
q=
3 (4) Sometimes, even if we have equations less in
1 number than the number of variables (i.e.,
Substituting q  in (1), indeterminate equations), while we cannot find out
3
the values of ALL the variables uniquely, it may be
1
p possible to find out the value of some specific
7 combination of the variables.
1 1 1 1
  and  1.09. The cost of 3 dosas, 5 idlis and 7 vadas is `154.
xy 3 xy 7
The cost of 5 dosas, 8 idlis and 11 vadas is
 x  y = 3 and x + y = 7. `246. Find the total cost of one idli, one dosa
Solving these equations, x = 5 and y = 2. and one vada.

ADDITIONAL CASES IN LINEAR EQUATIONS Sol: Let the cost of each dosa, each idli and each
vada be `d, `i and `v respectively.
(1) If the number of equations is less than the number 3d + 5i + 7v = 154 --- (1)
of unknowns, then we say the variables are 5d + 8i + 11v = 246 --- (2)
'indeterminate' or we have an "indeterminate" Multiplying (1) by 3 and subtracting it, from
system of equations. Here, we cannot uniquely twice (2), d + i + v = 30
determine the values of all the variables. There will
be infinite sets of solutions that satisfy the (5) Sometimes, even if we have three equations in three
equations. unknown, we may not be able to uniquely determine
the values of the variables if the equations are not
For example, if we take the following two equations "INDEPENDENT," i.e., one of the given equations
in three unknowns, can be written as a "linear combination" of the other
x + y + 2z = 8 two equations.
2x – y + 3z = 13
this system of equations have infinite number of For example, let us take the following system of
solutions and no unique solution is possible. For any three equations in three unknowns.
value we take for x, we can find a corresponding set 3x + 5y + 7z = 12 ----- (1)
of values for y and z. x – 3y + 9z = 16 ----- (2)
9x + 8y + 31z = 54 ----- (3)
(2) However, even in case of indeterminate equations,
say, of three variables, it is possible that the value of If we try to solve these equations, we will find that
one of the variables may be uniquely determined, we cannot get a unique solution. That is because
i.e., if we have two equations and three unknowns, these equations are not independent. In this case,
we may be still able to determine the value of one equation (3) can be obtained by multiplying equation
variable uniquely but the other two variables will (1) by 2.5 and equation (2) by 1.5 and adding them.
have infinite number of values. This will happen if
the ratio of the coefficients of two variables in one If there are three equations l1, l2and l3 in three
equation is the same as the ratio of the coefficients unknowns, we say that they are linearly dependent if
of the same two variables in the second equation. one of the three equations can be written as a linear
combination of the other two, i.e., l3 = l1 + kl2 where
This depends on the equations given. Example 1.08
k is any constant.
will clarify this aspect.
In such a case, the system of equations will have
1.08. Tarun bought 2 shirts, 4 trousers and 5 pairs of infinite number of solutions.
shoes for `3600. Had he bought 6 shirts,
5 trousers and 15 pairs of shoes, his total If it is not possible to write the three equations in the
expenditure would have been `8700. Find the form above, then they are linearly independent and
price of each trouser. the system of equations will have a unique solution.

Triumphant Institute of Management Education Pvt. Ltd. (T.I.M.E.) HO: 95B, 2nd Floor, Siddamsetty Complex, Secunderabad – 500 003.
Tel : 040–27898195 Fax : 040–27847334 email : info@time4education.com website : www.time4education.com SM1001906/3
(6) Sometimes, we can have "inconsistent" equations. 1.10. Find the value of k for which the following
For example, if we know that x + 2y = 4, then the system of equations will be consistent.
value of 2x + 4y has to be 8. The expression 2x  5y = 10 and 6x  15y = k
(2x + 4y) cannot take any other value. If it is given
any other value, there will be inconsistency in the Sol: In the given system of equations, the ratio of the
data because then we will effectively be saying that coefficients of x equals the ratio of the
x + 2y = 4 and at the same time x + 2y  4. coefficients of y.
So, if we have the system of equations  They would be consistent only if this ratio
equals the ratio of the constant terms.
x + 2y = 4 and 2x + 4y = k, this system of equations 10 2 5
will be consistent ONLY If the value of k = 8. For any  If   i.e.,
other value of k, the system of equations will be k 6  15
INCONSISTENT. if k = 30, the given system of equations would
In the above system of equations, when k = 8, there be consistent.
will be infinite number of solutions (and not a unique
solution).

Triumphant Institute of Management Education Pvt. Ltd. (T.I.M.E.) HO: 95B, 2nd Floor, Siddamsetty Complex, Secunderabad – 500 003.
Tel : 040–27898195 Fax : 040–27847334 email : info@time4education.com website : www.time4education.com SM1001906/4
Concept Review Questions
Directions for questions 1 to 30: For the Multiple Choice Questions, select the correct alternative from the given
choices. For the Non-Multiple Choice Questions, write your answer in the box provided.

1. Thrice a number is 24 more than one-third of it. Find it. 14. A two digit number has a tens digit of T and a units
digit of U. The digit 4 is placed immediately to the
right of U forming a three digit number. The new
number is equal to _____.
(A) 10T + U + 4 (B) 100T + 10U + 4
2. Four added to half of one-third of one-sixth of a
(C) 1000T + 10U + 4 (D) T + U + 4
number is equal to one-twelfth of the number. What
is the number?
15. A two-digit number is such that twice the tens digit
(A) 48 (B) 36 (C) 72 (D) 24
added to eleven times the units digit is equal to the
number itself. Find the number.
3. The sum of two numbers is 18. The difference of the
numbers is 4. Find the numbers.
(A) 11, 7 (B) 12, 8 (C) 16, 2 (D) 8, 10

4. How many pairs (x, y) satisfy the equations 16. The digits of a two digit number differ by 3. Find the
3x + 6y = 12 and 4x + 8y = 16? difference of the number and the number formed by
(A) 0 (B) 1 (C) 2 (D)  reversing its digits.
(A) 18 (B) 27 (C) 36 (D) 45
5. How many pairs (x, y) satisfy the equations
3x + 9y = 21 and 6x + 18y = 45? 17. The difference between a three-digit number and the
(A) 0 (B) 1 (C) 2 (D)  number formed by reversing its digits is 198. Find
the difference of its first and last digits.
6. How many pairs (x, y) satisfy the equations
3x + 5y = 21 and 4x + 7y = 29?
(A) 0 (B) 1 (C) 2 (D) 
18. The greatest number that always divides the
7. If 8x + y = 10 and 4x + 2y = 13, then the respective difference between a three digit number and the
values of x and y are number formed by reversing its digits is
(A) 3, 4/3 (B) 2/3, 4/3
(C) 3/4, 2 (D) 7/12, 16/3

8. Find the value of x, if 3x + 9y + 12z = 18 and 19. A fraction is such that the numerator is five less than
2x + 3y + 4z = 8. the denominator. Also four times the numerator is
(A) 2 (B) 3 (C) 1 (D) 4 one more than the denominator. Find the fraction.
(A) 4/9 (B) 3/8 (C) 2/7 (D) 7/12
9. Five puffs and two samosas cost `48. The cost of a
puff is twice the cost of a samosa. Find the cost of a 20. The denominator of a fraction is two more than its
puff (in `). numerator. If one is added to the denominator, the
fraction becomes 3/4. Find the fraction.
(A) 7/9 (B) 5/7
(C) 11/13 (D) 9/11
10. Four pens and five erasers cost `32. Five pens and
four erasers cost `31. Find the cost of each pen (in `). 21. Three consecutive even integers are such that
(A) 2 (B) 3 (C) 4 (D) 5 one-third of the second number is equal to
one-fourth of the third number. Find the
11. Four pens and three erasers cost `17. Four erasers and three numbers.
3 pens cost `18. Find the total cost of a pen and a (A) 4, 6, 8 (B) 8, 10, 12
eraser. (C) 12, 14, 16 (D) 2, 4, 6

22. Nalini has an amount of `20 in coins of


denominations of 50 paise and `1. If she has a total
12. The cost of ten mangoes and nine apples is `104. of 30 coins with her, how many `1 coins does she
The cost of twenty-seven mangoes and twenty-five have?
apples is `285. What is the cost of three mangoes
and two apples?
(A) `24 (B) `30 (C) `27 (D) `32
23. Shankar has an amount of `7,000 in the
13. Two chocolates, three milk shakes and four cakes denominations `500 and `100. How many
cost `190. Four chocolates and eight cakes cost `500 notes does he have, if he has a total of
`320. Find the cost of a milkshake. (in `). 22 notes with him?
(A) 10 (B) 20
(C) 30 (D) Cannot be determined
Triumphant Institute of Management Education Pvt. Ltd. (T.I.M.E.) HO: 95B, 2nd Floor, Siddamsetty Complex, Secunderabad – 500 003.
Tel : 040–27898195 Fax : 040–27847334 email : info@time4education.com website : www.time4education.com SM1001906/5
24. Amar, Bhavan, Chetan and Dinesh have a total of 28. Four years ago, Ashok’s age, was four times that of
`150 with them. Amar has one-fourth of the total Bala. Three years hence, Ashok’s age will be thrice
amount with the others. Find the amount with Amar the age of Bala. Find Ashok’s present age. (in years).
(in `). (A) 56 (B) 63 (C) 60 (D) 66
(A) 20 (B) 25 (C) 30 (D) 37.5
29. Mrudhula gave birth to twins, when she was twenty
25. P, Q and R had a total of `1250 among them. P had years old. After x years, the sum of the ages of
three-seventh of the total share with Q and R. Find Mrudhula and her twin children is 50 years. What is
the amount that P had (in `). the value of x?
(A) 300 (B) 450 (C) 525 (D) 375

26. Twenty years from now, Sreedhar will be six times


as old as he was twenty years ago. What is the
30. Twenty seven years ago, a man's age was half his
present age of Sreedhar?
wife’s present age. Sum of their present ages is
(A) 20 years (B) 24 years
90 years. What are the respective present ages
(C) 28 years (D) 32 years
(in years) of the man and his wife?
(A) 48, 42 (B) 52, 38
27. Ramesh is thrice as old as Suresh. Two years
(C) 54, 36 (D) 46, 44
hence, Ramesh will be twice as old as Suresh. Find
Ramesh’s present age (in years).
(A) 2 (B) 3 (C) 4 (D) 6

Triumphant Institute of Management Education Pvt. Ltd. (T.I.M.E.) HO: 95B, 2nd Floor, Siddamsetty Complex, Secunderabad – 500 003.
Tel : 040–27898195 Fax : 040–27847334 email : info@time4education.com website : www.time4education.com SM1001906/6
Exercise – 1(a)
Directions for questions 1 to 30: For the Multiple Choice Questions, select the correct alternative from the given
choices. For the Non-Multiple Choice Questions, write your answer in the box provided.

1. Solve 10. Nitya and Satya have some marbles with them.
7(x + 2) + 3(y  2) = 8 Nitya says to Satya, "If you give one marble to me,
3(x  2) + 7(y + 2) = 8 we will have equal number of marbles. Satya says to
Nitya, “If you give me one marble, I will have twice
(A) x = 2, y = 0 (B) x = 0, y = 2 the number of marbles you have”. How many
(C) x = 0, y = 0 (D) x = 2, y = 2 marbles do Nitya and Satya have respectively?
(A) 4, 6 (B) 5, 7 (C) 6, 4 (D) 7, 5
2. 2x + 3y  6z = 18
4 11. Gopi, Murthy and Hari had some amount of money.
2y  x  4z  12 Gopi gives half his amount to Murthy, who then
3
gives half of what he now has to Hari. Hari gives half
Solve for y.
of what he now has to Gopi, who, now has exactly
what he started with. If the sum of Murthy’s initial
amount and twice Hari’s initial amount is `45, what
was the amount (in rupees) Gopi started with?
3. If x + 2y + 3z = 14,
2x + 3y + z = 11 and
3x + y + 2z = 11,
find (x + y + z). 12. A shopkeeper has a few 100 gm weights and a few
(A) 6 (B) 5 (C) 4 (D) 0 500 gm weights. He can weigh a maximum of 8 kg
in one weighing. If he has 20 pieces of weights,
4. If 3x + y – 3z = 11 and 2x – 2z + 5y = 29 what is the what is the maximum weight that he can weigh with
value of x + y – z? only 100 gm weights?
(A) 7 (B) 14 (C) 9 (D) 8 (A) 800 gm (B) 600 gm
(C) 450 gm (D) 500 gm
5. What is the value of p, if the equations are
consistent and x  0  y? 13. A child went to a shop to buy a pen, a pencil and a
5x + 4y = 32 ruler where costs are integral values (in `) and are in
2py + 15x = 96 decreasing order. Each item costs at least `4. The
(A) 5 (B) 6 (C) 4 (D) 3 total cost is `15 and the cost of a pencil is `5. How
many pencils can he purchase with the amount
6. For which of the following values of k will required to purchase ten rulers?
3x + (k + 3)y = 1 and kx + 6y = 4 have a unique solution?
(A) 3
(B) 6
(C) 6 14. A housewife, with a given amount, can buy either
(D) Any value except 3 and 6 10 apples or 15 oranges or 2 watermelons. Find the
maximum number of oranges which she can buy
7. Six times the denominator of a fraction is two less with six times the initial amount such that she gets
than eight times the numerator and ten times the each of the three varieties of fruits.
numerator equals 20 less than ten times the (A) 75 (B) 81 (C) 60 (D) 72
denominator. The reciprocal of the fraction is
(A) 6/7 (B) 3/8 (C) 7/9 (D) 9/7 15. Ten years ago, the age of a man was 35 years more
than twice his son’s age. After how many years from
8. John covers 10 km per hour more than Peter while now will the man be
driving. On doubling his speed, Peter covers (i) twice his son’s age?
15 km per hour more than John who is driving at his (A) 15 (B) 20
normal speed. What is John’s speed? (in km/hr) (C) 25 (D) Cannot be determined

(ii) thrice his son’s age?


(A) 10 (B) 15
(C) 20 (D) Cannot be determined
9. Classes A and B have 35 students each. If
seven girls shift from class A to class B, then the 16. Runs scored by Bhangar in a match are 10 more
number of girls in the classes would interchange. If than the balls faced by Karthik. The number of balls
four girls shift from class B to class A, then the faced by Bhangar is 5 more than the number of runs
number of girls in class A would be twice the original scored by Karthik. Together they have scored
number of girls in class B. What is the number of 50 runs and Bhangar has faced 15 balls less than
boys in Class A and in Class B? Karthik. What is the number of runs scored by
(A) 18 and 11 Bhangar?
(B) 24 and 17
(C) 18 and 27
(D) 17 and 24
Triumphant Institute of Management Education Pvt. Ltd. (T.I.M.E.) HO: 95B, 2nd Floor, Siddamsetty Complex, Secunderabad – 500 003.
Tel : 040–27898195 Fax : 040–27847334 email : info@time4education.com website : www.time4education.com SM1001906/7
17. With the data from the above question what is the younger to him. This type of distribution continues
number of runs scored by Karthik per ball? for the next two transactions. In the end the total
(A) 2.67 (B) 2.33 (C) 0.5 (D) 0.33 sum with the children who have given money to
other children is half the sum of money with the
18. An amount of `9000 is divided among four people children who did not give any money. What was the
A, B, C and D. The sum of the shares of A, C and D original amount with each child?
is four times the share of B. The sum of the shares (A) `36 (B) `48 (C) `50 (D) `32
of B and D is equal to four-fifths the sum of shares
of A and C. Find the share of D. 25. A three-digit number is eleven times the two-digit
(A) `1,800 (B) `2,400 number formed by using the hundreds and the units
(C) `2,200 (D) Cannot be determined digit of the three-digit number respectively, in the
tens and units place of the two-digit number. If the
19. P, Q and R completed a certain project working difference between the digit in tens place and the
together. The number of hours for which P worked digit in hundreds place is 1, then what is the digit in
2 rd the units place?
on the project was of that for which Q worked
3
2 th
on the project, which in turn was of that for
5
26. In addition to the data in the above question, if the
which R worked on the project. The number of hours difference between the digit in tens place and that in
for which R worked is what fraction of the total the units place of the three-digit number is also 1,
number of hours for which all three worked? then what is the number?
2 3 7 1
(A) (B) (C) (D)
5 5 10 2

20. The cost of four apples, six bananas and eight


oranges is p. The cost of five apples, eight bananas 27. A grocer uses a weighing balance in which one pan
and eleven oranges is q. The cost of eight apples, weighs 0.5 kg and the other 0.75 kg. He puts a
fourteen bananas and twenty oranges is r. Which of certain quantity of food grains in 0.5 kg pan and
the following is always true? finds the weight (in kg) as a two-digit number.
However, as the customer insists, he puts it in
3r r
(A) 3p + 4q = (B) 3p + 4q = 0.75 kg pan. Now the indicated weight is 9.5 kg
4 2 more than the weight which is obtained by reversing
r the digits of previous weight. Which of the following
(C) 4q + 3p = (D) 3p + 4q = r
2 cannot be the actual weight (in kg) of the
food grains?
21. While adding ten two-digit numbers the digits of one (A) 43.25
of the numbers were interchanged. As a result the (B) 36.25
sum of all the ten numbers increased by a value (C) 41.5
which was four less than that number. Three times (D) More than one of the above
the sum of the digits of that original number is ten
less than the number. What is the product of the 28. Using the data of the above question, which of the
digits of that number? following can be the weight as indicated in the
(A) 56 (B) 18 (C) 36 (D) 42 first case?
(A) 65 kg (B) 36 kg (C) 64 kg (D) 85 kg
Directions for questions 22 and 23: Answer these
questions based on the information below. 29. In t minutes from now, the time will be noon.
44 minutes ago, the time was 7t minutes past 9 a.m.
A shopkeeper sold a certain number of toys. The number Find the present time. :
of toys as well as the price of each toy (in `) was a : a.m.
two digit number. By mistake, he reversed the digits of
both the number of toys he sold and the price of each
toy. As a result, he found that his stock account at the
end of the day showed 81 items more than it actually was. 30. In an exam, there are 35 questions. Each correct
answer, each wrong answer and each unanswered
22. Find the actual number of toys sold. question carries 5 points, 0 points and 1 point
(A) 92 (B) 81 (C) 90 (D) 29 respectively. Had each correct answer, each wrong
answer and each unanswered question carried
23. If the faulty calculations show a total sale of `882, 4, – 1 and 0 points respectively, the number of
find the actual selling price of each toy (in `). points a student would score in the exam is 25. Find
(A) 89 (B) 98 (C) 97 (D) 79 the actual number of points that he would score in
the exam.
24. There are ten children (aged 1 to 10 years) who
have equal amounts of money. In the first
transaction the eldest child gives one rupee to every
child younger to him. In the second transaction, the
second eldest child gives one rupee to every child

Triumphant Institute of Management Education Pvt. Ltd. (T.I.M.E.) HO: 95B, 2nd Floor, Siddamsetty Complex, Secunderabad – 500 003.
Tel : 040–27898195 Fax : 040–27847334 email : info@time4education.com website : www.time4education.com SM1001906/8
Exercise – 1(b)
Directions for questions 1 to 35: For the Multiple Choice Questions, select the correct alternative from the given
choices. For the Non-Multiple Choice Questions, write your answer in the box provided.

1. (a) Solve 8. Ashok has a total of 30 notes in denominations of


x x x `20 and `5. The total value of the notes with him is
x   5 `300. Find the number of `20 notes with him.
2 3 4
(A) 5 (B) 10 (C) 8 (D) 6
(A) 60/19 (B) 4 (C) 12/5 (D) 19/60
9. The cost of printing the first fifteen hundred copies of
(b) Solve a book, is `1500. It costs y rupees to print each
99x + 101y = 400 subsequent copy. The cost of printing the first
99y + 101x = 400 7500 copies of the book is `7080. Find y.

(A) x = 2, y = 1 (B) x = 1, y = 2
(C) x = 1, y = 1 (D) x = 2, y = 2
10. The cost of four pens, five erasers and
6 7 six sharpeners is `47. The cost of six pens,
(c)  7
( 2 x  y ) (3 y  x ) seven erasers and nine sharpeners is `69. Find the
cost of an eraser (in `).
4 14 (A) 3 (B) 6 (C) 5 (D) 7
 6
( 2 x  y ) (3 y  x )
11. The cost of five pens, eight erasers and
(A) x = 2, y = 3 (B) x = 3, y = 2 eleven sharpeners is `54. The cost of three pens,
(C) x = 4, y = 7 (D) x = 7, y = 13 five erasers and seven sharpeners is `34. Find the
cost of one pen, one eraser and one sharpener (in `).
2. Divide 1 kg weight into two parts such that the sum
of the parts is 5/4th the difference.
(A) 550 gm, 450 gm
(B) 200 gm, 800 gm 12. Cost of two pens, five pencils and seven erasers is
(C) 900 gm, 100 gm `37. Cost of seven pens, one eraser and two pencils
(D) 400 gm, 600 gm is `49. What is the cost of nine pencils and
forty seven pens?
3. P, Q and R are successive even natural numbers in (A) `184 (B) `276
ascending order. Five times R is eight more than (C) `284 (D) None of these
seven times P. Find Q.
13. The cost of two pens, one eraser and
three sharpeners is `23. The cost of six pens,
three erasers and one sharpener is `45. The cost of
4. A is greater than B by 1/3rd the sum of A and B. If
fourteen pens, seven erasers and twenty one
B is increased by 40, it becomes greater than twice
sharpeners is `161. Find the cost of each pen (in `).
A by 10. Find A, B.
(A) 3 (B) 4
(A) 30, 20 (B) 60, 30
(C) 5 (D) Cannot be determined
(C) 20, 10 (D) 20, 40
14. Anand bought a total of 30 white and brown pens for
5. In three hours Meena makes one basket more than
a total of `32. The cost of each white pen is 70 paise
what Reena makes in two hours. In five hours,
more than the cost of each brown pen. Which of the
Meena makes one basket less than what Reena
following represents a possible value of the cost of
makes in four hours. How many baskets can Meena
each brown pen (in paise)?
make in an hour?
(A) 40 (B) 35 (C) 45 (D) 60
(A) 4 (B) 2 (C) 3 (D) 6
Directions for questions 15 to 17: Answer these
6. Ajay was asked to find (2/9)th of a number. He
questions based on the information below.
instead multiplied the number by (9/2) and obtained
an answer which was 4235 more than the correct Rakesh went to a stationery shop to purchase a total of
answer. Find the number. 38 pens, erasers and sharpeners. He purchased at least
11 items of each. He purchased more sharpeners than
erasers and more erasers than pens.

15. How many pens did he purchase?


7. Ashok and Balu have some coins. If Ashok gives
(A) 11 (B) 12 (C) 13 (D) 14
20 coins to Balu, he would have thrice as many
coins as Balu. If Ashok gives 30 coins to Balu, he
would have twice as many coins as Balu. Find the 16. If each pen cost `2, each eraser cost `3 and each
number of coins with Balu. sharpener cost `4, find the minimum expenditure be
(A) 8 (B) 10 (C) 12 (D) 14 could have incurred on the items (in `).
(A) 116 (B) 118 (C) 117 (D) 119
Triumphant Institute of Management Education Pvt. Ltd. (T.I.M.E.) HO: 95B, 2nd Floor, Siddamsetty Complex, Secunderabad – 500 003.
Tel : 040–27898195 Fax : 040–27847334 email : info@time4education.com website : www.time4education.com SM1001906/9
17. If in the previous question the condition that he 27. A test had 200 questions. Each correct answer
purchased more sharpeners than erasers and more carried 2 marks. Each wrong answer carried
erasers than pens is removed, find the minimum  1/2 mark and unanswered questions carried no
expenditure he could have incurred on the items (in `). mark. Ajay attempted all the questions in the test
(A) 108 (B) 109 (C) 110 (D) 111
and scored 360 marks. What would his marks be, if
for each correct answer he got only 1/2 mark and for
18. An amount of `5,600 is divided among A, B and C.
The sum of the shares of B and C is equal to thrice the each wrong answer he lost 2 marks?
share of A. The sum of the shares of A and C is equal (A) 80 (B) 60 (C) 40 (D) 100
to nine-fifths the share of B. What is the share of C?
(A) `1,400 (B) `2,400 (C) `2,200 (D) `2,000 28. Ten children are standing in a line. Each child has
some chocolates with him. If the first child attempted
19. Aswin went on a tour to three cities. In each city, he to double the number of chocolates with each of the
spent `2 more than half of the amount he had with others he would fall short by two chocolates. If the
him when he arrived in the city. At the end of the second child took two chocolates from each of the
tour he had `150. Find the amount that he started remaining, he would have three chocolates less than
with (in `). what the first child initially had. Find the total number
of chocolates with the third to the tenth child.

20. If both the numerator and denominator of a fraction


are increased by one, the fraction formed is 2/3. If
29. Ashok went to a casino to play a card game. In each
both the numerator and denominator are decreased
by two, the fraction formed is 3/5. What is the round he happened to double the amount with
fraction formed, if both numerator and denominator himself and in each round he gave `x to his friend.
are increased by four? At the end of three rounds he was left with no
(A) 7/10 (B) 9/11 (C) 5/7 (D) 3/4 money. If the amount he gave to his friend in each
round was `20 more than the amount he started
21. Find the value of k if the equations 3x  (k/3+ 2)y = 1 with, find the amount (in `) that he started with.
and kx + 2ky = 4 have infinite solutions. (A) 110 (B) 120 (C) 130 (D) 140

30. Six years ago, Ram’s age was four times Shyam’s
age. Six years hence, Ram’s age will be thrice
22. Four times the sum of the digits of a two-digit
Shyam’s age. After how many years from now will
number is 18 less than the number and is also
9 less than the number formed by reversing its their combined age be 150 years?
digits. Find the product of its digits. (A) 21 (B) 9 (C) 36 (D) 18
(A) 12 (B) 20 (C) 30 (D) 42
31. Ashok told his son, “My age x years ago was twice
23. The sum of a two-digit number and its reverse is 99. your age. My age 5x years ago was thrice your age.”
How many two-digit numbers satisfy this condition? If the difference of their present ages is 24 years,
find the sum of their present ages (in years).
(A) 91 (B) 65 (C) 78 (D) 52
24. The sum of the digits of a three-digit number is eight.
The middle digit is thrice the sum of the other 32. The sum of two numbers is 4 and the reciprocal of
two digits. The difference between the number and one exceeds the reciprocal of the other by twice the
the number obtained by reversing the order of the product of their reciprocals. What is the product of
digits is zero. What is the number? the reciprocals of the two numbers?
(A) 161 (B) 242 (C) 192 (D) 125 (A) 3 (B) 2 (C) 1/3 (D) 1/2
25. Five three-digit numbers including N, were to be
33. The sum of two numbers is 250. The difference of
added. While adding, the reverse of N was added by
mistake instead of N. Hence, the sum increased by their squares is 12500. Find the larger number.
11 times the sum of the digits of N. Eight times the (A) 130 (B) 140
difference of N’s units and hundreds digits is 6 more (C) 150 (D) 160
than twice its hundreds digit. Find its tens digit.
(A) 4 (B) 6 (C) 8 (D) 2 34. Dinesh, Eswar, Ganesh and Harish had a total of
120 marbles. The number of marbles with Dinesh
26. In a four-digit number, the sum of the middle was half the total number of marbles with the others.
two digits is twice the units digit. The sum of the
The number of marbles with Eswar was one-third
hundreds digit and six times the thousands digit is
twice the sum of the other two digits. The sum of the the total number of marbles with the others. The
units digit and five times the thousands digit is twice number of marbles with Ganesh one fourth the total
the hundreds digit. How many values can the number of marbles with the others. Find the number
four-digit number assume? of marbles with Harish.

Triumphant Institute of Management Education Pvt. Ltd. (T.I.M.E.) HO: 95B, 2nd Floor, Siddamsetty Complex, Secunderabad – 500 003.
Tel : 040–27898195 Fax : 040–27847334 email : info@time4education.com website : www.time4education.com SM1001906/10
35. Amar, Bhavan, Chetan and Dinesh each had some 37. What is the difference between the costs of 12 shirts
money. Dinesh doubled the amounts with the and 36 pairs of pants?
others. Chetan then doubled the amounts with the . Total cost of 12 shirts and 36 pairs of pants is
others. Bhavan then doubled the amounts with the `1200.
others. Amar then doubled the amounts with the . If the cost of each shirt is doubled, 12 shirts will
others. At this stage, each of them has `80. Find the cost the same as that of 72 pairs of pants.
initial amount with Chetan (in `).
(A) 45 (B) 65 (C) 95 (D) 85 38. How many questions did Aarti attempt in the Data
Sufficiency test?
Directions for questions 36 to 40: Each question is . There were 25 questions in the test.
followed by two statements  and . Indicate your . Aarti got 21 marks in the test, in which every
responses based on the following directives: correct answer fetched her 1 mark and for every
incorrect answer 1/3 mark was deducted from
Mark (A) if the question can be answered using one the total.
of the statements alone, but cannot be
answered using the other statement alone. 39. Is the age of a person equal to the sum of the ages
Mark (B) if the question can be answered using of his son and wife?
either statement alone. . The person is two years older than his wife.
Mark (C) if the question can be answered using . The age of his son is twelve years, who was
 and  together but not using  or  alone born when his wife was 30 years old.
Mark (D) if the question cannot be answered even
using  and  together. 40. Which car gives more mileage, Uno or Zen?
. Uno gives a mileage of 45 miles per 10 litres of
36. Find the value of 2x + 3y. petrol, which is three-fourths of the mileage of
x 3y Alto.
.  = 1.
2 4 . The mileage of Palio is four-fifths of the mileage
. 9y + 6x = 12. of Alto, which is two miles less than that of Zen.

Key
Concept Review Questions

1. 9 7. D 13. A 19. C 25. D


2. C 8. A 14. B 20. D 26. C
3. A 9. 8 15. 54 21. A 27. D
4. D 10. B 16. B 22. 10 28. C
5. A 11. 5 17. 2 23. 12 29. 10
6. B 12. C 18. 99 24. C 30. A

Exercise – 1(a)

1. C 9. D 16. 40 24. A
2. 6 10. B 17. D 25. 1
3. A 11. 30 18. C 26. 121
4. A 12. D 19. B 27. D
5. B 13. 8 20. D 28. A
6. D 14. B 21. C 29. 11, 43
7. D 15. (i) C 22. C 30. 60
8. 35 (ii) D 23. A

Exercise – 1(b)

1. (a) A 6. 990 13. D 20. C 27. B 34. 26


(b) D 7. B 14. D 21. 12 28. 23 35. D
(c) A 8. B 15. A 22. B 29. D 36. B
2. C 9. 0.93 16. C 23. 9 30. B 37. A
3. 8 10. A 17. B 24. A 31. C 38. D
4. C 11. 6 18. C 25. B 32. C 39. C
5. C 12. D 19. 1228 26. 2 33. C 40. C

Triumphant Institute of Management Education Pvt. Ltd. (T.I.M.E.) HO: 95B, 2nd Floor, Siddamsetty Complex, Secunderabad – 500 003.
Tel : 040–27898195 Fax : 040–27847334 email : info@time4education.com website : www.time4education.com SM1001906/11
CHAPTER – 2
RATIO – PROPORTION – VARIATION
RATIO Examples
If the values of two quantities A and B are 4 and 6 2.01. The scores of Mohan and Sohan in a test are in
respectively, then we say that they are in the ratio the ratio 5 : 4. If their total score is 135, find
4 : 6 (read as "four is to six”). Ratio is the relation which Mohan’s score.
one quantity bears to another of the same kind, the
comparison being made by considering what multiple, Sol: As their scores are in the ratio of 5 + 4, let their
part or parts, one quantity is of the other. The ratio of two scores be 5k and 4k.
quantities "a" and "b" is represented as a : b and read as Given that the sum of their scores = 5k = 4k
"a is to b". Here, "a" is called antecedent, "b" is the = 135
consequent. Since the ratio expresses the number of ⇒ 9k = 135
times one quantity contains the other, it's an abstract ⇒ k = 15
quantity. ∴ Mohan's score = 5k = 5 × 15 = 75
Ratio of any number of quantities is expressed after Alternative method:
removing any common factors that ALL the terms of the
ratio have. For example, if there are two quantities Mohan’s score =
5
(135 ) = 75
5+4
having values of 4 and 6, their ratio is 4 : 6, i.e., 2 : 3
after taking the common factor 2 between them out. 2.02. If a : b = 3 : 4, find 3a + 4b : 4a + 5b.
Similarly, if there are three quantities 6, 8 and 18, there
is a common factor among all three of them. So, dividing Sol: 3a + 4b : 4a + 5b
each of the three terms by 2, we get the ratio as 3 : 4 : 9. 3a + 4b
3a + 4b b
If two quantities whose values are A and B respectively = =
are in the ratio a : b, since we know that some common 4a + 5b 4a + 5b
factor k(>0) would have been removed from A and B to b
get the ratio a : b, we can write the original values of the a 3
3  + 4 3  + 4
two quantities (i.e., A and B) as ak and bk respectively.  b 4 25
For example, if the salaries of two persons are in the = = =
a 3 32
ratio 7 : 5, we can write their individual salaries as 7k 4  + 5 4  + 5
and 5k respectively. b 4

A ratio a : b can also be expressed as a/b. So if 2.03. The ratio of the number of marbles with Ram
two items are in the ratio 2 : 3, we can say that their ratio and Shyam is 19 : 13. If Ram gives Shyam
is 2/3. If two terms are in the ratio 2, it means that they 30 marbles, both will have equal number of
are in the ratio of 2/1, i.e., 2 : 1. marbles. Find the number of marbles with Ram.

"A ratio is said to be a ratio of greater or less inequality Sol: Let the number of marbles with Ram and
or of equality according as antecedent is greater than, Shyam be 19x and 13x respectively. Total
less than or equal to consequent". In other words, number of marbles with them = 32x
If Ram gives Shyam 30 marbles each will have
- the ratio a : b where a > b is called a ratio of greater 32 x
inequality (example 3 : 2) = 16 x marbles.
- the ratio a : b where a < b is called a ratio of less 2
inequality (example 3 : 5) ∴ 19x − 16x = 30
- the ratio a : b where a = b is called a ratio of equality x = 10
(example 1 : 1) 19x = 190

From this we can find that a ratio of greater inequality is 2.04. Two numbers are in the ratio 3 : 4. What part of
diminished and a ratio of less inequality is increased by the larger number must be added to each
adding same quantity to both terms, i.e., in the ratio a : b, number so that their ratio becomes 5 : 6?
when we add the same quantity x (positive) to both the
terms of the ratio, we have the following results Sol: Let the two numbers be 3x and 4x.
3x + k 5
if a < b then (a + x) : (b + x) > a : b =
if a > b then (a + x) : (b + x) < a : b 4x + k 6
if a = b then (a + x) : (b + x) = a : b 18x + 6k = 20x + 5k
k = 2x
This idea can also be helpful in questions on Data ∴Half of the larger number must be added to
Interpretation when we need to compare fractions to find each number.
the larger of two given fractions.
If two quantities are in the ratio a : b, then the 2.05. 1400 is divided into 4 parts such that twice the
first quantity will be a/(a + b) times the total of the first part, thrice the second part, 4 times the
two quantities and the second quantity will be equal to third part and 12 times the last part are all
b/(a + b) times the total of the two quantities. equal. Find the 4 parts.
Triumphant Institute of Management Education Pvt. Ltd. (T.I.M.E.) HO: 95B, 2nd Floor, Siddamsetty Complex, Secunderabad – 500 003.
Tel : 040–27898195 Fax : 040–27847334 email : info@time4education.com website : www.time4education.com SM1001906/12
Sol: Let the first part, second part, third part and last Sol: Let the present age of the husband be x years.
part be a, b, c and d respectively. ⇒ Present age of the wife = (x − 6) years.
2a = 3b = 4c = 12d 10 years ago, the ages of the husband and the
a = 6d, b = 4d, c = 3d wife will be (x − 10) years and (x − 16) years
Given, a + b + c + d = 1400 respectively.
∴ 6d + 4d + 3d + d = 1400
Given x − 10 = (x − 16 )
5
⇒ d = 100
4
∴a = 600, b = 400, c = 300
∴ x = 40
2.06. 1400 is divided into 4 parts such that half of the Alternative method:
first part, one third of the second part, Let the age of the husband 10 years ago be
1 5x years. Age of his wife at that time = 4x years.
one fourth of the third part and th of the last The husband would then also be 6 years older
12
than his wife.
part are all equal. Find the 4 parts.
∴ 5x = 4x + 6 ⇒ x = 6
Sol: Let the first part, second part, third part and Hence the present age of the husband
fourth part be a, b, c and d respectively. = 5x + 10 i.e. 40 years
1 1 1 1
a= b= c= d PROPORTION
2 3 4 12
3 When two ratios are equal, then the four quantities
d = 6a, b = a , c = 2a
2 involved in the two ratios are said to be proportional i.e.,
Given, a + b + c + d = 1400 if a/b = c/d, then a, b, c and d are proportional.
3
⇒ a + a + 2a + 6a = 1400 This is represented as a : b : : c : d and is read as "a is to
2
b (is) as c is to d".
400
⇒a =
3 When a, b, c and d are in proportion, then a and d are
800 called the EXTREMES and b and c are called the
∴ b = 200, c = , d = 800 MEANS. We also have the relationship:
3
Product of the MEANS = Product of the EXTREMES i.e.,
2.07. If a : b = b : c = 2 : 3, find a : b : c b c = adv
If a:b = c : d then
a b 2 2 2 b:a = d:c ..... (A)
Sol: = = ⇒ a = b and b = c a:c = b:d ..... (B)
b c 3 3 3
(a + b) : b = (c + d) : d ..... (C)
22  4
∴ a =  c  = c
3 3  9 (obtained by adding 1 to both sides of the given
relationship)
4 2
∴a : b : c = c: c:c = 4 : 6 : 9 (a − b) : b = (c − d) : d ..... (D)
9 3
(obtained by subtracting 1 from both sides of the given
Alternative method: relationship)
As b is common to both ratios and since it is (a + b) : (a – b) = (c + d) : (c – d) ...… (E)
divisible by 3 (from the first ratio) and it is
divisible by 2 (from the second ratio), it is {obtained by dividing relationship (C) above by (D)}
divisible by L.C.M (3, 2) i.e., 6. Hence if b = 6, a
= 4 and c = 9 Relationship (A) above is called INVERTENDO
∴a:b:c=4:6:9 Relationship (B) is called ALTERNENDO;
Relationship (C) is called COMPONENDO;
2.08. There are 2 classes A and B. If 10 students Relationship (D) is called DIVIDENDO;
leave class A and join class B, then the ratio Relationship (E) is called COMPONENDO –
of the number of students in class A and DIVIDENDO.
class B would reverse. Find the difference in
The last relationship, i.e., COMPONENDO-DIVIDENDO
the numbers of students in class A and class B.
is very helpful in simplifying problems. By this rule,
whenever we know a/b = c/d, then we can write
Sol: Let the numbers of students in class A and class
(a + b) / (a − b) = (c + d) / (c − d).
B be ax and bx respectively.
ax − 10 b
Given, = The converse of this is also true - whenever we know
bx + 10 a that (a + b) / (a − b) = (c + d)/(c − d), then we can
a2x − 10a = b2x + 10b ⇒ a2x – b2x – 10a – 10b = 0 conclude that a/b = c/d.
⇒ (ax – bx – 10) (a + b) = 0
∴ ax – bx = 10 a c e
If = = …..., then each of these ratios is equal to
b d f
2.09. A husband’s age exceeds that of his wife by
a + c + e + .....
6 years. 10 years ago, the ratio of their ages .
was 5 : 4. Find the present age of the husband. b + d + f + .....

Triumphant Institute of Management Education Pvt. Ltd. (T.I.M.E.) HO: 95B, 2nd Floor, Siddamsetty Complex, Secunderabad – 500 003.
Tel : 040–27898195 Fax : 040–27847334 email : info@time4education.com website : www.time4education.com SM1001906/13
If three quantities a, b and c are such that a : b : : b : c, If X varies inversely with Y and we have two sets of
then we say that they are in CONTINUED values of X and Y – X1 corresponding to Y1 and X2
PROPROTION. We also get b² = ac. In such a case, c is corresponding to Y2, then since X and Y are inversely
said to be the third proportional of a and b. Also, b is said related to each other, we can write down
to be the mean proportional of a and c. X Y
X1Y1 = X2Y2 or 1 = 2
X2 Y1
VARIATION
Two quantities A and B may be such that as one JOINT VARIATION
quantities changes in value, the other quantity also
changes in value bearing certain relationship to the If there are three quantities A, B and C such that
change in the value of the first quantity. A varies with B when C is constant and varies with C
when B is constant, then A is said to vary jointly with
B and C when both B and C are varying. i.e., A α B when
DIRECT VARIATION C is constant and Aα C when B is a constant; ⇒ A α BC
One quantity A is said to vary directly as another quantity
A α BC ⇒ A = kBC where k is the constant of
B if the two quantities depend upon each other in such a
proportionality.
manner that if B is increased in a certain ratio, A also
increases in the same ratio and if B is decreased in a
2x + 5 x + 2
certain ratio, A also decreases in the same ratio. 2.10. Find the value(s) of x if = .
This is denoted as A α B (A varies directly as B). x +1 x −1

If A α B then A = kB, where k is a constant. It is called Sol: (2x + 5) (x − 1) = (x + 2) (x + 1)


the constant of proportionality. 2x2 + 5x − 2x − 5 = x2 + 2x + x + 2
⇒ x2 = 7
For example, when the quantity of sugar purchased by a
∴x=± 7
housewife doubles from the normal quantity, the total
amount she spends on sugar also doubles, i.e., the
quantity and the total amount increase (or decrease) in 2.11. X varies directly with Y2 + 18. When Y = 18,
the same ratio. X = 18. Find Y when X = 1.

2
From the above definition of direct variation, we can see X1 Y + 18
that when two quantities A and B vary directly with each Sol: = 1
X 2 Y2 2 + 18
other, then A/B = k or the ratio of the two quantities is
a constant. Conversely, when the ratio of two 18 18 2 + 18
quantities is a constant, we can conclude that they = 2
1 Y2 + 18
vary directly with each other.
Y2 + 18 = 19
2

If X varies directly with Y and we have two sets of values


Y2 = ± 1
of the variables X and Y – X1 corresponding to Y1 and X2
corresponding to Y2, then, since XαY, we can write down
In these types of problems on variation, there are
X1 X X Y typically three parts:
= 2 or 1 = 1
Y1 Y2 X2 Y2
- the relationship between different variables is
defined to frame an equation involving the
INVERSE VARIATION variables and the constant of proportionality
- one set of values of all the values of all the
A quantity A is said to vary inversely as another quantity variables is given to enable us find the value of the
B if the two quantities depend upon each other in such a constant of proportionality
manner that if B is increased in a certain ratio, A gets - the values of all but one variable of a second set
decreased in the same ratio and if B is decreased in a are given and we are asked to find the value of the
certain ratio, then A gets increased in the same ratio. one variable whose value is not given.
It is the same as saying that A varies directly with 1/B.
It is denoted as A α 1/B i.e., A = k/B where k is k the 2.12. The volume of a cone varies jointly as the area
constant of proportionality. of its base and its height. The volume of a cone
whose base area is A sq. cm and height is
For example, as the number of men doing a certain work 10 cm is 270 cubic cm. Find the volume of a cone
increases, the time taken to do the work decreases and whose base area is 2A sq. cm and height is 30 cm.
conversely, as the number of men decreases, the time
taken to do the work increases. Sol: VαAh
V1 A h
From the definition of inverse variation, we can see that = 1 1
V2 A 2 h2
when two quantities A and B vary inversely with each
other, then AB = a constant, i.e., the product of the 270  A  10
two quantities is a constant. Conversely, if the = 
product of two quantities is a constant, we can
V2  2A  30
conclude that they vary inversely with each other. V2 = 1620 cubic cm
nd
Triumphant Institute of Management Education Pvt. Ltd. (T.I.M.E.) HO: 95B, 2 Floor, Siddamsetty Complex, Secunderabad – 500 003.
Tel : 040–27898195 Fax : 040–27847334 email : info@time4education.com website : www.time4education.com SM1001906/14
Note that the there should be consistency of the units 2.15. The monthly expenses of Raja on his car are
used for the variables, i.e., whatever be the units used to partly constant and partly vary with the number
express the variables when the constant of of kilometres he travels in a month. If he travels
proportionality is being calculated, the same units should 100 km in a month his total car expenses will be
be used for different variables later on also when finding `3500. If he travels 200 km in a month, his total car
the value of the variable which we are asked to find out. expenses will be `4000. If he travels 250 km in a
month, what will be his total car expenses.
2.13. The ratio of the monthly incomes of A and B is
4 : 3. The ratio of their monthly expenditures is Sol: Let his total car expenses be `T. Let the fixed
5 : 4. If A saves one fourth of his monthly expense be `F. Let the variable expense be `V.
income, find the ratio of their monthly savings.
T=F+V
V
Sol: Let the monthly incomes of A and B be `4x and If he travels D km in a month, = K, where K
`3x respectively. D
Let the monthly expenditures of A and B be `5y is a constant.
and `4y respectively. Monthly savings of ∴T=F+KD
A = `(4x − 5y). Monthly savings of B = `(3x − 4y). Given that
Given that 4x − 5y =
1
(4x ) 3500 = F + 100K --- (A)
4 4000 = F + 200K --- (B)
3x = 5y. Solving (A) and (B),
∴ Monthly saving of B = 3x – 4y = 5y – 4y i.e. `y. F = 3000 and K = 5.
Required ratio =
1
(4x ) : y = x : y = 5 : 3 Total car expenses if he travels 250 km
4 = F + 250K = `4250.

2.14. If x varies directly with y, check whether The problems involving ratio and proportion are just
x3 + y3 varies directly with x3 − y3. different forms of the models of the basic problems we
saw above. For example, the problem we just solved
x above might be reframed bringing in Mangoes, Bananas,
Sol: Let = K , where K is a constant Baskets, etc. Here, practice and perseverance pay you a
y
lot. In entrance exams, there will be either direct problems
x=Ky on ratio, proportion and variation or indirect problems of
x3 + y3 = y3 (K3 + 1) application of these concepts just discussed to areas like
x3 − y3 = y3 (K3 − 1) Time and Work or Time and Distance.
x3 + y3 K 3 +1
= a constant
x −y
3 3
K 3 −1
∴ (x3 + y3)varies directly with (x3 − y3)

Triumphant Institute of Management Education Pvt. Ltd. (T.I.M.E.) HO: 95B, 2nd Floor, Siddamsetty Complex, Secunderabad – 500 003.
Tel : 040–27898195 Fax : 040–27847334 email : info@time4education.com website : www.time4education.com SM1001906/15
Concept Review Questions
Directions for questions 1 to 30: For the Multiple Choice Questions, select the correct alternative from the given
choices. For the Non-Multiple Choice Questions, write your answer in the box provided.

9 3b 13. The strength of a class is 70. Which of the following


1. If a = b, then = cannot be the ratio of the number of boys and girls
8 4a
in the class?
(A) 2/3 (B) 1/3 (C) 4/3 (D) 2 (A) 2 : 5 (B) 3 : 2
(C) 11 : 3 (D) 9 : 2
q+p
2. If 2.4p = 0.08 q, then = 14. The ratio of the number of ladies and gentlemen in a
q−p
party is 3 : 4. If eight ladies join the party, the ratio of
(A) 31/28 (B) 31/27 ladies and gentlemen would become 5 : 4. Find the
(C) 31/29 (D) 39/35 number of gentlemen in the party.

3a + 4b
3. If a : b = 2 : 3, find .
4a + 5b
(A) 6 : 7 (B) 18 : 23 (C) 19 : 23 (D) 20 : 23 15. The ratio of the present ages of a mother and a son
is 4 : 1. Twelve years from now, the ratio of their
4. If a : b = b : c = 2 : 3, find a : b : c. ages will be 2 : 1. Find the present ages of mother
(A) 4 : 6 : 9 (B) 2 : 3 : 3 and son respectively. (in years)
(C) 3 : 3 : 4 (D) 9 : 6 : 4 (A) 36, 9 (B) 28, 6
(C) 24, 6 (D) 32, 8
5. If p : q = 5 : 6 and q : r = 4 : 7, then find p : q : r
(A) 10 : 12 : 21 (B) 20 : 24 : 35 16. The ratio of the number of chocolates with A and B is
(C) 15 : 18 : 28 (D) 25 : 30 : 35 3 : 4. If A gives four chocolates to B, the ratio of the
number of chocolates with them becomes 5 : 9. How
6. Find a : e, if a : b = 2 : 3, b : c = 6 : 7, c : d = 14 : 25 and many chocolates did A have initially?
d : e = 1 : 2.
(A) 14 : 25 (B) 6 : 25
(C) 12 : 25 (D) 4 : 25
17. The ratio of the number of students in three classes
7. If p : q = 5 : 4 and p = a + b and q = a − b, find a : b. A, B and C is 3 : 7 : 8. If ten students, are
(A) 1 : 9 (B) 9 : 1 (C) 5 : 4 (D) 4 : 5 transferred from C to B, B will have 80 students.
Find the total number of students in the
8. Thirty five sweets are distributed among Sita and three classes.
Gita in the ratio 4 : 3. Find the number of sweets
received by Sita.

18. The ratio of the incomes of A and B is 4 : 3. The


ratio of the savings of A and B is 5 : 6. Find the ratio
9. Two numbers are in the ratio of 2 : 5. If 4 is added to of the expenditures of A and B, if A spends
each, they would be in the ratio 4 : 9. Find the three-fourth of his income.
numbers. (A) 3 : 2 (B) 5 : 3
(A) 16 and 40 (B) 18 and 45 (C) 3 : 5 (D) 2 : 3
(C) 22 and 55 (D) 20 and 50
19. The ratio of the monthly incomes of A and B is 3 : 4.
10. The ratio of the ages of A, B and C is 7 : 5 : 4. If The ratio of the monthly expenditures of A and B is
C’s age is 32 years, find the sum of the ages of 4 : 5. Which of the following represents a possible
A, B and C (in years). value of the ratio of their savings?
(A) 9 : 10 (B) 3 : 4
(C) 13 : 20 (D) 4 : 5

11. The ratio of the ages of four members of a family is 20. The ratio of the present ages of a husband and a
9 : 8 : 3 : 2. The average age of the family is wife is 5 : 4.
22 years. What is the age of the eldest person in the
(i) Which of the following can be a possible ratio of
family?
their ages 20 years ago?
(A) 36 years (B) 32 years
(A) 5 : 4 (B) 6 : 5
(C) 12 years (D) 6 years
(C) 23 : 20 (D) 13 : 10
12. The number of marbles with A and B are in the ratio
(ii) Which of the following can be a possible ratio of
of 10 : 11. Which of the following cannot be a
their ages 20 years hence?
possible number of marbles with A and B together?
(A) 5 : 4 (B) 6 : 5
(A) 189 (B) 210
(C) 7 : 5 (D) 13 : 10
(C) 231 (D) 153
Triumphant Institute of Management Education Pvt. Ltd. (T.I.M.E.) HO: 95B, 2nd Floor, Siddamsetty Complex, Secunderabad – 500 003.
Tel : 040–27898195 Fax : 040–27847334 email : info@time4education.com website : www.time4education.com SM1001906/16
21. Four positive numbers are in the ratio of 1 : 2 : 3 : 4. 26. For a given distance, speed is inversely proportional
If the sum of their squares is equal to 480, find the to the time taken. Raju would take six hours to cover
numbers. it if his speed was 50 km/hr. Find the time taken
(A) 2, 4, 8 10 (B) 8, 12, 16, 20 (in hours) by him to cover it if his speed was 100 km/hr.
(C) 4, 8, 12, 16 (D) 8, 10, 12, 16 (A) 2 (B) 3 (C) 4 (D) 6

22. Three positive numbers p, q and r satisfy 27. P, Q and R are three quantities, P varies directly
q+r p+r p+q with the sum of Q and R. If both Q and R decrease
= = = K. K = _____. by 1, find the change in P.
p q r
(A) No change
(A) 3/2 (B) 5/2 (C) 3 (D) 2 (B) A decrease of 2
(C) An increase of 2
23. x varies directly as y² and when y = 12, x = 4. Find x (D) Cannot be determined
when y = 18.
(A) 91/16 (B) 9 (C) 1/9 (D) 16/9 28. A, B and C are three quantities. A varies directly
with B when C is constant. A varies directly with C
24. The area of a square varies with the square of its when B is constant. A = 6000 if B = 20 and C = 30.
diagonal. The area of a square whose diagonal Find A if B = 40 and C = 60.
measures eight units is 32 sq.units. Find the
diagonal of a square of area 72 sq.units (in units).

29. A varies directly with B when C is constant and


inversely with C when B is constant. When A = 26,
B = 65 and C = 30. Find A when B = 84 and C = 42.
25. The monthly salary of an employee in an office (A) 24 (B) 36 (C) 14 (D) 32
varies directly with the number of days he works in
that month. An employee gets `10400 as salary in a 30. X, Y and Z are three quantities. X varies inversely
month in which he works for 26 days. Find his salary with Y when Z is constant. Y varies inversely with Z
(in `) in a month in which he works for 24 days. when X is constant. When Y = 8, and Z = 7, X = 30.
Find X if Y = 16 and Z = 21.

Triumphant Institute of Management Education Pvt. Ltd. (T.I.M.E.) HO: 95B, 2nd Floor, Siddamsetty Complex, Secunderabad – 500 003.
Tel : 040–27898195 Fax : 040–27847334 email : info@time4education.com website : www.time4education.com SM1001906/17
Exercise – 2(a)
Directions for questions 1 to 30: For the Multiple Choice Questions, select the correct alternative from the given
choices. For the Non-Multiple Choice Questions, write your answer in the box provided.

4a + 5b 10. Three different types of balls priced at `5, `8 and


1. If a : b = 3 : 7, what is the value of ? `13 per piece are displayed in three different boxes
2a + 2b
by a trader. Mr. Paul bought from this shop all three
(A) 47/20 (B) 3/2 (C) 7/4 (D) 5/2 types of balls spending a total sum of `768. The
numbers of the balls he bought, taken in the order in
2. If a : b = 2 : 3 b : c = 4 : 3 and c : d = 2 : 3, then find which the prices are mentioned above, are in the
a : b : c : d. ratio 5 : 4 : 3. How many balls of the costliest variety
(A) 8 : 12 : 9 : 27 did he buy?
(B) 16 : 24 : 18 : 27 (A) 104 (B) 64 (C) 48 (D) 24
(C) 18 : 27 : 36 : 8
(D) 12 : 18 : 15 : 20 11. A certain amount of money is divided among
nine brothers. The second brother gets `2 more
3. The weights of Bimal and Basu are in the ratio 2 : 3 than twice the amount given to the first brother. The
and the weights of Basu and Bali are in the ratio third brother gets `3 more than thrice the amount
4 : 3. What is Basu’s weight (in kg) if the sum of the given to the first brother, and so on till the
weights of Bimal, Basu and Bali is 203 kg? ninth brother. If the ratio of the amount with the
ninth brother to the amount with the first brother is
10 : 1, how much did the fifth brother get (in `)?
(A) 100 (B) 80 (C) 90 (D) 50
4. The ratio of the number of boys to the number of
girls in a school is 7:3. If an additional 15 girls were 12. Some apples are divided among four people Karan,
to join the class, the ratio of the number of boys to Kiran, Kumar and Khanna. The ratio of the number
the number of girls would become 2:3. What is the of apples given to Kiran to the total number of
initial number of girls in the class? apples given to Karan and Khanna is 1 : 2. The ratio
(A) 4 (B) 10 (C) 12 (D) 6 of the number of apples given to Kumar to that of the
remaining apples is 2 : 5. Khanna gets 2 apples more
5. If 3x − 4y + 2z = 0 and 4x − 2y − z = 0, find x : z : y. than Kiran. Karan gets half the number of apples
(A) 8 : 10 : 11 (B) 8 : 11 : 40 that Kumar gets. What is the total number of apples
(C) 11 : 40 : 8 (D) 8 : 40 : 11 distributed?
(A) 21 (B) 18
6. What must be subtracted from p and added to q so (C) 19 (D) 24
that the ratio of the resultants becomes 1 : 3?
p+q 3p − q 13. Alok distributed a certain number of toffees among
(A) (B)
3 4 his brothers Bala, Chetan and David in the ratio
p−q 4 : 4 : 9. David distributed the toffees that he
q − 3p
(C) (D) received among his sisters Amitha, Bama and
p+q 4 Chandra in the ratio 1 : 7 : 8. If Amitha received
18 toffees, the number of toffees distributed by Alok is
7. A certain number is added to each of a pair of
numbers which are in the ratio 4 : 7. The sum of the
resulting numbers is 75 and their ratio (taken in the
same order as mentioned above) is 8 : 17. What is
the number added? Directions for questions 14 and 15: These questions
are based on the data given below.

The amount used to purchase one litre of petrol can be


8. Rohan and Sohan had certain amounts of money used to purchase three litres of diesel or five litres of
with them. The ratio of the amounts with them is kerosene. Out of a certain amount, `510 is spent on
8 : 5. Each day Rohan spends a certain amount and diesel.
Sohan earns one – sixth of the amount that Rohan
spends. After 9 days, the ratio of the amounts with 14. How much is spent on kerosene if equal volumes of
them is 10 : 11. After how many more days, will the the three liquids are purchased with the total amount?
ratio of the amounts with them be 18 : 35? (A) `300 (B) `306
(C) `382 (D) `354

15. What will be the amount spent on petrol if the total


9. Vipin’s present age is twice the age of Kishore one year
amount referred in the above question is instead
ago. What is the sum of their present ages (in years), if
spent to purchase equal volumes of petrol and
the ratio of the sum of their present ages to the
kerosene only?
difference of their present ages is 19 : 5?
(A) `1250 (B) `1275
(C) `1955 (D) `1360

Triumphant Institute of Management Education Pvt. Ltd. (T.I.M.E.) HO: 95B, 2nd Floor, Siddamsetty Complex, Secunderabad – 500 003.
Tel : 040–27898195 Fax : 040–27847334 email : info@time4education.com website : www.time4education.com SM1001906/18
a+c c + e a+e 24. A writer gets a fixed amount for his book apart from
16. If k = = = , when all quantities opthe royalty he gets per book sold. He gets
b+ d d+ f b+ f
`46000 and `66000 for 2000 and 3000 books sold
are positive, then which of the following must be respectively. Find his income per book when
true? 6000 books are sold (in `).
e a
(A) k = (B) k =
f b
c 25. ‘A’ varies directly as the sum of two quantities
(C) k = (D) All of the above ‘B’ and ‘C’. ‘B’ in turn varies directly as ‘x’ and ‘C’
d
varies inversely as ‘x’. When x = 1 or 2, A = 3. Find
17. A number of coins can be divided in the ratio 13 : 14 : 15. the value of A when x = 4.
They can also be divided in the ratio 9 : 10 : 11 : 12.
If the difference between the greatest parts in the
two cases is 6, the number of coins is .
26. A garrison of 900 soldiers had food-stock sufficient
for 30 days when the rate of consumption is
2.5 kg/day/soldier. After some days of consumption
at that rate, 300 soldiers were transferred to another
18. The mean proportional between two numbers is 9 garrison and the balance food lasted for 25 days for
and the third proportional of the two numbers is 243. the remaining soldiers. If the rate of consumption of
Find the larger of the two numbers. the remaining soldiers was 3.0 kg/day/ soldier, after
(A) 27 (B) 81 (C) 9 (D) 54 how many days from the start, were the soldiers
transferred?
19. P and Q are distinct two-digit numbers. Ps and Qs
denote the sums of the digits in P and Q
P Q
respectively. If = , then find the minimum
Ps Q s 27. The speed of a locomotive without any wagons
possible value of Ps+ Qs. attached to it is 40 kmph. It diminishes by a quantity
(A) 8 (B) 9 (C) 6 (D) 3 which is proportional to the cube root of the number
of wagons attached. If the speed of the locomotive is
20. For a body starting from rest, the distance travelled 34 kmph when 27 wagons are attached, what is the
(d) is directly proportional to the square of the time maximum number of wagons that can be attached if
elapsed from the start (t). When t = 4 sec, d = 48 m. the condition is that the speed should not fall below
What is the value of d (in metres) at t = 7sec? 30 kmph?
(A) 128 (B) 150 (C) 115 (D) 147 (A) 64 (B) 125 (C) 216 (D) 343

21. A certain sum of money is sufficient to pay the 28. A precious stone worth `10872 fell and broke into
wages of worker A for 55 days. It is also sufficient to three pieces, the weights of which are proportional
pay the wages of worker B for 66 days. The number to 1 : 2 : 3. The value of each stone is directly
of days that the sum of money is sufficient to pay the proportional to the square of its weight. Find the loss
wages of both workers is in the value caused by the breakage.
(A) `3624 (B) `4228 (C) `6644 (D) `7510

29. The distance (in meters) to which a boy can throw a


stone is inversely proportional to its weight (in kg).
22. The volume of a cone varies as the square of the He breaks the stone into 3 pieces whose weights
radius of its base when its height is fixed and also (in kg) are in the ratio 1 : 3 : 2. He then throws the
varies as its height when radius of its base is fixed. stones one by one. The sum of the distances they
If a radius of 3 cm and height of 7 cm give it a cover is 22 meters. To what distance can he throw
volume of 66 cubic centimetre, then what will be the the unbroken stone? (in m)
radius, (in centimetres), if the cone has a height of
6 cm and volume of 308 cubic centimetres?
(A) 9 (B) 15 (C) 7 (D) 11
30. The amount collected per month from a consumer of
23. The kinetic energy of a body is directly proportional electrical power consists of two parts – a fixed
to the square of its speed when the mass is kept charge for providing the service and a variable
constant and is directly proportional to mass when charge which is directly proportional to the number
its speed is kept constant. A body with a mass of of units of power consumed. An amount of `700 is
2 kg and a speed of 10 m/s has a kinetic energy of collected from the consumer when he consumed
100 joules. What is the kinetic energy of a body 50 units in a month. It is also noticed that when the
whose mass is 20 kg and speed is 1 m/s? consumption increases from 100 units per month to
(A) 100 joules 200 units per month, the bill amount increases to
(B) 1000 joules 5/4 times that of the former. How much is the fixed
(C) 10 joules charge per month?
(D) 20 joules (A) `600 (B) `350 (C) `500 (D) `400

Triumphant Institute of Management Education Pvt. Ltd. (T.I.M.E.) HO: 95B, 2nd Floor, Siddamsetty Complex, Secunderabad – 500 003.
Tel : 040–27898195 Fax : 040–27847334 email : info@time4education.com website : www.time4education.com SM1001906/19
Exercise – 2(b)
Directions for questions 1 to 40: For the Multiple Choice Questions, select the correct alternative from the given
choices. For the Non-Multiple Choice Questions, write your answer in the box provided.

1. If a + b − c : b + c − a : a + c − b = 5 : 6 : 7, then find 9. Divide 66 into three parts such that the sum of the
a : b : c. first two parts equals the third part and the
(A) 12 : 13 : 11 second part is three less than twice the first part.
(B) 12 : 11 : 13 What is the ratio of the parts as arranged in the
(C) 13 : 12 : 11 ascending order?
(D) 13 : 11 : 12 (A) 14 : 17 : 19 (B) 5 : 9 : 11
(C) 11 : 7 : 4 (D) 4 : 7 : 11
2. Which of the following represents a possible value of
10. A person has with him a certain number of weighing
20p 2 − 40pq
p : q satisfying = 20? stones of 100 gm, 500 gm and 1 kg in the ratio of
pq + 4q 2
3 : 5 : 1. If a maximum of 5 kg can be measured
using weighing stones of 500 gm alone, then what is
(A) 3 : 1 (B) 1 : 4 (C) 4 : 1 (D) 5 : 1 the number of 100 gm stones he has?
(A) 6 (B) 3 (C) 9 (D) 5
3. If a : b = 3 : 4 and c : d = 2 : 3, then find
a 3 c 2 + b 3 d2 11. In a class, the ratio of the number of boys and girls
. is 1 : 2. Among the boys, one-third are day scholars
ab 2 d2 + a 2bcd
and one-fourth of these day scholars travel to school
19 19 by bus. Among the girls, two-thirds are day scholars
(A) (B) and half of these day scholars travel to school by
3 4
bus. What part of the students in the class are day
19 19 scholars travelling to school by bus?
(C) (D)
9 18 1 1 1 2
(A) (B) (C) (D)
3 5 4 5
4. Ajay and Vijay wrote a test. The sum of Ajay’s score
and twice Vijay’s score is 310. The sum of Vijay’s 12. In a bag, there are balls of three colours - white,
score and twice Ajay’s score is 290. Find the ratio of black and green. The ratio of the number of
the scores of Ajay and Vijay. white and black balls is 3 : 4. The ratio of the
(A) 9 : 11 (B) 13 : 17 number of black and green balls is 3 : 5. Which of
(C) 11 : 19 (D) 7 : 13 the following can be a possible value of the total
number of white and green balls in the bag?
5. Ninety three is divided into two parts such that thrice (A) 42 (B) 32 (C) 58 (D) 41
the first part and twice the second part are in the
ratio 25 : 4. Find the first part. 13. Amar, Bhavan and Chetan have some coins with
them. The total number of coins with Bhavan and
Chetan and those with Amar and Chetan are in the
ratio 4 : 5. The total number of coins with Bhavan
6. If three is subtracted from the numerator and five is and Chetan and those with Amar and Bhavan are in
added to the denominator of a fraction, the the ratio 4 : 3. Which of the following is the ratio of
new fraction formed is 1/2. If two is added to the the number of coins with Bhavan and Chetan?
numerator of the initial fraction, the ratio of the (A) 7 : 4 (B) 3 : 2
new numerator to the denominator becomes 1 : 1. (C) 7 : 5 (D) 1 : 3
Find the original fraction.
(A) 11/13 (B) 18/23 14. The ratio of the prices of tea, last year and this year
(C) 13/15 (D) 13/11 is 5 : 6. The ratio of the prices of coffee last year and
this year is 7 : 8. The sum of prices of a kg of tea
7. The ratio of the number of students in classes and a kg of coffee this year is `48. Find the price of
A, B and C is 3 : 7 : 8. If 10 students leave C and 20
join B, the ratio of the number of students in B and C tea (in `) last year if it was of the price of coffee
21
would be reversed. Find the total number of
last year.
students in the classes A, B and C.

8. Ajay distributed a total of 60 sweets among his sons 15. `117 was supposed to be divided among Rohan,
Ram, Shyam and Tarun. For every five sweets Sohan and Mohan in the ratio 2 : 3 : 4. By mistake, it
received by Ram, Shyam received four sweets. For 1 1 1
every two sweets received by Shyam, Tarun was divided in the ratio : : . Find the loss / gain
received three sweets. Find the number of sweets 2 3 4
received by Tarun. of Rohan due to his mistake (in `)
(A) gain, 26 (B) loss, 26
(C) gain, 28 (D) loss, 28

Triumphant Institute of Management Education Pvt. Ltd. (T.I.M.E.) HO: 95B, 2nd Floor, Siddamsetty Complex, Secunderabad – 500 003.
Tel : 040–27898195 Fax : 040–27847334 email : info@time4education.com website : www.time4education.com SM1001906/20
16. Five vessels have equal capacities. Each vessel a2 + c 2 b 2 + c 2
contains some milk. The ratio of the quantities of 24. If = = k and a ≠ b, which of the
milk in these vessels is 3 : 4 : 5 : 6 : 7. The total a+c b+c
quantity of milk is three-fifth of the combined following is equal to k?
capacity of the vessels. How many vessels contain (A) a + c (B) b + c
at least half milk?
(C) a + b (D) a − c

p+ q q+r p+r
25. If = = = k, find the sum of all the
r p q
17. The monthly incomes of Ashok and Bala be 3 : 4.
The ratio of the expenditures of Ashok and Bala is possible values of k.
4 : 5. Which of the followinjg cannot be the ratio of
the savings of Ashok and Bala? (A) 1 (B) 2 (C) 0 (D) 3
(A) 2 : 3 (B) 4 : 5 (C) 8 : 11 (D) 5 : 7
2 x 2 − 4 x + 3 2x ² − 3 x + 5
18. Manoj and Shiva, who are colleagues in an office, 26. If = , find the value(s) of x.
4x − 3 3x − 5
have their monthly savings in the ratio of 2 : 3.
Manoj spends two-thirds of his income every month. (A) 0 (B) 2
If the ratio of their monthly incomes is 3 : 4, what is (C) − 2 (D) Both (A) and (C)
the ratio of their expenditures?
(A) 3 : 2 (B) 4 : 5 (C) 4 : 3 (D) 5 : 3 27. There are 2 two-digit numbers. Their product equals
the product of the numbers formed by reversing their
19. In a three digit number, the units digit is the sum of digits. Which of the following holds true if the
the other two digits and it exceeds the tens digit by numbers are denoted by ab and cd?
as much as the latter exceeds the hundreds digit. (A) a, b, d and c are in proportion.
How many such three digit numbers exist? (B) a, b, c and d are in proportion.
(A) 1 (B) 2 (C) 3 (D) 4 (C) a, d, c and b are in proportion.
(D) a, c, b, d are in proportion.
20. The sum of the present ages of a mother and her
daughter is 60 years. When the mother attains her 28. A string is cut into two parts. The ratio of the lengths
husband’s present age, the ratio of her husband’s of the string and the smaller part is six times the
age and her daughter’s age would be 2 : 1. Find the ratio of the lengths of the smaller and the larger
present age of the daughter. (in years) parts. Find the ratio of the lengths of the larger and
smaller parts of the string.
(A) 2 : 1 (B) 3 : 1 (C) 4 : 1 (D) 5 : 1

21. A person had two plots of land. In each plot, he 29. The mean proportional between two numbers is 9.
cultivated maize and barley. The ratio of the area The third proportional of the same numbers is 6561.
under maize cultivation and barley cultivation in the Find the greater of the 2 numbers.
larger plot is 8 : 9. The ratio of the total areas under
maize cultivation and barley cultivation in the
two plots together is 29 : 33. The ratio of the areas
under maize cultivation and barley cultivation in the 30. The ratio of Ahmed’s age to Mohammed’s age is the
smaller plot is 13 : 15. The ratio of the area same as the ratio of the ages of their respective
under maize cultivation in the larger plot and that elder brothers. The ratio of the difference of the
under barley cultivation in the smaller plot is _____. ages of Ahmed and Mohammed to that of the
(A) 8 : 15 (B) 16 : 15 difference of the ages of their respective brothers is
(C) 15 : 16 (D) 15 : 8 1 : 2. What is the ratio of the sum of the ages of their
respective brothers to the sum of the ages of
Directions for questions 22 and 23: These questions Ahmed and Mohammed?
are based on the data given below. (A) 2 : 1 (B) 4 : 1 (C) 3 : 1 (D) 2 :1
A test of 60 minutes contains questions on Mathematics 31. x is directly proportional to 4 more than the square
and English only. The time taken to solve a Mathematics of y . x is 39 when y is 3. What is the positive value
question is twice the time taken to answer an English of y when x is 60?
question and the ratio of time taken to solve all
Mathematics questions to time taken to answer all
English questions is 8/7.

22. What is the ratio of the number of English questions 32. In a three digit number the tens digit is the average
to that of Mathematics? of the other two digits. The ratio of the number
(A) 11/7 (B) 7/4 (C) 9/4 (D) 7/5 formed by its first two digits and their sum equals the
ratio of the number formed by its last two digits and
23. If the total number of questions is 22, how many their sum. How many three digit numbers satisfy
English questions can be answered in 18 minutes? these conditions?
(A) 7 (B) 8 (C) 9 (D) 6
(A) 8 (B) 10 (C) 11 (D) 9

Triumphant Institute of Management Education Pvt. Ltd. (T.I.M.E.) HO: 95B, 2nd Floor, Siddamsetty Complex, Secunderabad – 500 003.
Tel : 040–27898195 Fax : 040–27847334 email : info@time4education.com website : www.time4education.com SM1001906/21
33. The distance that a body falls through when dropped 40. The telephone bill consists of a fixed monthly rent
from a certain height varies directly with the square and a variable part that varies with the number of
of the time of fall. A body falls through a total of units of calls made per month. The monthly rents
500 m in 10 seconds. Find the distance it falls charged for a telephone connection in Bangalore
through, in the 9th second (in m). and Patna are in the ratio 5 : 3. The bill amounts
corresponding to 100 calls made from a phone in
Patna and 300 calls made from a phone in
Bangalore are respectively `500 and `1100. What is
34. A quantity Q is obtained by adding three quantities. the difference in the fixed monthly rent of a
The first is a constant, the second varies directly telephone connection in Bangalore and one in
with the square root of y and the third varies directly Patna, if call rate per unit is same in both cities?
with the cube root of y. When y = 1, Q = 60. When (A) `200 (B) `100
y = 64, Q = 230 and when y = 729, Q = 660 (C) `500 (D) `300
Find the constant.
Directions for questions 41 to 45: Each question is
followed by two statements Ι and ΙΙ. Indicate your
responses based on the following directives:
35. The cost of supply of a commodity equals the sum
Mark (A) if the question can be answered using one
of three quantities. The first one is a constant, the
of the statements alone, but cannot be
second varies directly as the price per unit of the
answered using the other statement alone.
commodity and the third varies directly as the
Mark (B) if the question can be answered using
square of the price per unit. The costs of supply are
either statement alone.
`9, `24 and `47 when the prices per unit are `1, `2
Mark (C) if the question can be answered using
and `3 respectively. Find the cost (in `) of supply
Ι and ΙΙ together but not using Ι or ΙΙ alone
when price per unit of commodity is `4.
Mark (D) if the question cannot be answered even
(A) 80 (B) 72 (C) 76 (D) 78
using Ι and ΙΙ together.
36. The pressure of a gas varies directly with the
temperature when the volume is constant and varies 41. A bowl contains `1, 50 paise and 25 paise coins.
inversely with the volume when temperature is What is the total amount in the bowl?
constant. If the present temperature is 100 K what Ι. The total number of coins in the bowl is 20.
will be the increase in temperature if the pressure ΙΙ. The total value of `1 coins is `6 and the number
triples and the volume doubles? (in K) of 50 paise coins and 25 paise coins are in the
ratio 6 : 1.

42. The ratio of technical staff and non-technical staff in


37. Two diaphragms partition a cylinder into a company is 3 : 10. What percentage of the staff
three chambers, whose volumes are in the ratio of are graduates?
1 : 2 : 3. Pressure of a gas, which is in joint variation Ι. 80% of the technical staff and 40% of the
with its mass and volume, is directly proportional to non-technical staff are graduates.
its mass and inversely proportional to its volume. If ΙΙ. The ratio of the number of graduates to the total
the masses of the gas in the three chambers are staff in the company is 32 : 65.
same, find the ratio of the pressures of the gases in
the three chambers taking them in the same order 43. How many girls are there in the class?
as the volumes have been taken. (Assume no factors Ι. The ratio of the number of boys to girls is 5 : 4.
other than those mentioned play a role). ΙΙ. If four boys leave the class, the ratio of number
(A) 3 : 2 : 1 (B) 1 : 2 : 3 of boys to girls in the class will be 1 : 1.
(C) 6 : 3 : 2 (D) 1 : 1 : 1
44. What is the percentage of students who are not
38. The heat radiated by a certain body per unit time intelligent?
varies directly with the square root of the excess of Ι. The ratio of number of boys and girls is 3 : 2.
the temperature of the body over the ambient ΙΙ. 20% of the boys and 20% of the girls are
temperature which is 20oC. The heat radiated by the intelligent.
body in one second is 16 joules when the
temperature of the body is 36ºC. Find the 45. If a cake is distributed among two boys A and B and
temperature of the body when the heat radiated in three girls X, Y and Z, who received the largest part?
one second is 20 joules. Ι. X received one-eighth part of the cake, which is
(A) 56ºC (B) 45ºC (C) 50ºC (D) 60ºC two-thirds of B’s share.
ΙΙ. A received twice B’s share, which is equal to
39. In a colony, the expenses for yoga classes are partly thrice the share of Y.
constant and partly vary directly with the number of
members. If there are 50 members, each member
has to bear `110 per month. If there are 30 more
members, each member has to bear `80 per month.
How many members are there if the share of
each member is `130 per month?

Triumphant Institute of Management Education Pvt. Ltd. (T.I.M.E.) HO: 95B, 2nd Floor, Siddamsetty Complex, Secunderabad – 500 003.
Tel : 040–27898195 Fax : 040–27847334 email : info@time4education.com website : www.time4education.com SM1001906/22
Key
Concept Review Questions

1. A 9. D 17. 180 24. 12


2. C 10. 128 18. B 25. 9600
3. B 11. A 19. C 26. B
4. A 12. D 20. (i) D 27. D
5. A 13. D (ii) B 28. 24000
6. D 14. 16 21. C 29. A
7. B 15. C 22. D 30. 5
8. 20 16. 24 23. B

Exercise – 2(a)

1. A 6. B 11. D 16. D 21. 30 26. 10


2. B 7. –12 12. A 17. 84 22. C 27. B
3. 84 8. 6 13. 544 18. A 23. C 28. C
4. D 9. 19 14. B 19. D 24. 21 29. 2
5. A 10. D 15. C 20. D 25. 4.5 30. A

Exercise – 2(b)

1. B 6. C 11. C 16. 3 21. B 26. D 31. 4 36. 500 41. C


2. C 7. 180 12. C 17. B 22. B 27. A 32. C 37. C 42. B
3. D 8. 24 13. D 18. B 23. D 28. A 33. 85 38. B 43. C
4. A 9. D 14. B 19. C 24. C 29. 81 34. 30 39. 40 44. A
5. 75 10. A 15. C 20. 20 25. A 30. A 35. D 40. A 45. C

Triumphant Institute of Management Education Pvt. Ltd. (T.I.M.E.) HO: 95B, 2nd Floor, Siddamsetty Complex, Secunderabad – 500 003.
Tel : 040–27898195 Fax : 040–27847334 email : info@time4education.com website : www.time4education.com SM1001906/23
CHAPTER – 3
PERCENTAGES – PROFIT & LOSS – PARTNERSHIPS
PERCENTAGE If the production in 1994 is given as 400 MT and the
increase from 1993 to 1994 is given to be 25%, then the
"Percent" implies "for every hundred". This concept is production in 1993 will be equal to 400/1.25 = 320 MT
developed to make the comparison of fractions easier by (where 1.25 = 1 + 0.25, 0.25 being the decimal equivalent
equalising the denominators of all fractions to hundred. of 25%).

Similarly, if there is a decrease of 12% on a quantity of


For example, 7/11 as percentage is represented as
225, then the new quantity will be equal to 225 x 0.88
7 7 × 100 (7 × 100 ) / 11 63.63
= = = = 63.63% (where 0.88 = 1 – 0.12, 0.12 being the decimal
11 11× 100 100 100 equivalent of 12%).

Percentages can also be represented as decimal If the production in 1994 is given as 400 MT and it is a
fractions. In such a case it is effectively equivalent to the decrease of 13% from 1993, then the production in 1993
proportion of the original quantity. will be equal to 400/0.87 (where 0.87 = 1 – 0.13, 0.13
being the decimal equivalent of 13%).
20
For example, 20% is the same as , i.e, 0.2. On the basis of percentage increase, we can write down
100 how many times the old value gives the new value.
Any percentage can be expressed as a decimal fraction For example, if the percentage increase is 100%, then
by dividing the percentage figure by 100 and conversely, we can conclude that the new value is 2 times the
any decimal fraction can be converted to percentage by old value. If the percentage increase is 300%, the new
multiplying it by 100. value is 4 times the old value. If the percentage increase
is 450%, then the new value is 5.5 times the old value. In
PERCENTAGE INCREASE or DECREASE in a quantity general, if the percentage increase is p%, then the
is the ratio, expressed in percentage, of the actual  p 
new value is  + 1 times the old value.
INCREASE or DECREASE in the quantity to the original  100 
amount of the quantity, i.e.,
Conversely, if we know how many times the old value
Actual increase
PERCENTAGE INCREASE = x 100 gives the new value, we can find out the percentage
Original quantity increase in the old value to get the new value.
For example, if the new value is 3 times the old value,
Actual decrease the percentage increase in the old value to get the
PERCENTAGE DECREASE = x 100
Original quantity new value is 200%. If the new value is 4.25 times the
old value, then the percentage increase is 325%. In
For example, if the production of rice went up from general, if the new value is k times the old value, then
225 MT in 1993 to 242 MT in 1994, then the percentage the percentage increase is (k – 1) x 100.
increase in rice production from 1993 to 1994 is
calculated as follows: Examples
Actual increase = 242 – 225 = 17 MT
3.01. Rice production in a country increased by 25%
Percentage increase
from 2000 to 2004. It increased by 20% from
Quality increase from 1993 to 1994 2004 to 2008. Find the percentage increase in
= x 100
Actual production of rice in 1993 the rice production from 2000 to 2008.
17
= x 100 = 75/9%
225 Sol: Let the rice production in 2000 be 100 tonnes.
Rice production in 2004
Ratio of any two quantities also can be expressed as
percentage. = 100 +
25
(100 ) = 125 tonnes
100
Rice production in 2008
For example, if the ratio of A and B is 3 : 2, we can say
the ratio of A : B is 60% : 40%. = 125 +
20
(125 ) = 150 tonnes
100
Whenever there is any percentage increase or decrease ∴ Percentage increase in the rice production
in a quantity, we can directly calculate the new value of 150 − 100
the quantity instead of calculating the actual from 2000 to 2008 is ×100 i.e. 50%.
100
increase/decrease and then adding to/subtracting from
the original quantity. 3.02. The population of a country increased by 10%
from 2001 to 2002. It increased by 20% from
For example, if the increase on a value of 350 is 15%, the 2002 to 2003. It increased by 30% from 2003 to
new quantity is 1.15 x 350 = 402.5 (where 1.15 = 1 + 0.15, 2004. Find the simple average yearly percentage
0.15 being the decimal equivalent of 15%). increase in the population from 2001 to 2004.
Triumphant Institute of Management Education Pvt. Ltd. (T.I.M.E.) HO: 95B, 2nd Floor, Siddamsetty Complex, Secunderabad – 500 003.
Tel : 040–27898195 Fax : 040–27847334 email : info@time4education.com website : www.time4education.com SM1001906/24
Sol: Let the population of the country in 2001 be 1
100 million. Population in 2002 = 110 million. 3.06. The ratio of the salaries of A and B is 2 : 2.
7
Population in 2003 = 110 × 1.2 = 132 million. By what percentage is B’s salary greater than
Population in 2004 = 132 × 1.3 A’s salary?
= 171⋅6 million
1
Simple average percentage increase Sol: Let the salary of A be 2x ⇒ Salary of B = ` 2
x
171 .6 − 100 7
=
100
(100 ) = 23 13 % ∴ The salary of B is more than the salary of A
15
1
3 x
by 7 (100 ) = 7 1 %
2x 7
3.03. The price of a Swiss watch was `10000 in 2001.
Due to devaluation of the rupee it becomes 3.07. The height of a triangle as well as its base are
`12000 in 2002. Find the percentage increase in increased by 30%. Find the percentage
its price from 2001 to 2002. increase in its area.

Sol: Percentage increase Sol: Let the original height as well as the base be
Final price − Initial price (100 ) (100 ) = 100 2
= (100 ) 100 cm. Original area =
2 2
Initial price
New Height = New Base = 130 cm
12000 − 10000
= (100 ) = 20% (130 )(130 )
10000 New area =
2
3.04. In 2002, Rakesh’s salary was `24000. In 2001, =
1
( ) (
(1 ⋅ 3)(100 )2 = 1 ⋅ 69 100 )
2

his salary was equal to that of Ramesh. 2 2


Rakesh's salary in 2001 was 20% less than his = 1⋅69 (original area)
salary in 2002. Ramesh’s salary in 2002 was ∴ the area increased by 69%.
20% more than his salary in 2001. By what
percentage is the sum of the salaries of 3.08. The price of an article is increased by 25%. By
what percentage must this price be reduced to
Ramesh in both the years more / less than that
bring it down to the original price?
of the Rakesh in both the years?
Sol: Let the original price be `100
Sol: Salary of Ramesh in 2001 = Salary of Rakesh in New price = 100 × 1.25 = `125
2001 To bring back the price to `100 it has to be
 20  reduced by `25
= 24000 1 −  = 19200
 100  Percentage reduction =
25
(100 ) = 20%
Salary of Ramesh in 2002 125
 20 
= 19200 1 +  = `23040. 3.09. If Ram’s salary is 20% less than Shyam’s
 100  salary, by what percentage is Shyam’s salary
Total salary of Rakesh in both the years more than Ram’s salary?
= 19,200 + 24,000 = `43200. Total salary of
Ramesh in both the years = 19200 + 23040 Sol: Let Shyam’s salary be `100
= `42240, which is `960 less than that of Ram’s salary = `80
Shyam's salary is more than Ram's salary by
Rakesh.
∴ Required percentage 20
(100 ) = 25%
80
=
960
(100 ) = 2 2 %
43200 9
1
3.10. If the price of tea goes up by 33 % , what
3
3.05. 64% of a number is 416. Find 85% of that
should be the percentage by which its
number.
consumption must be reduced so that the
expenditure on it remains unchanged?
Sol: Let the number be x
64 Sol: Let the original price be `300 per kg and the
x = 416
100 original consumption be 100 kg. Original
x = 650 expenditure = New expenditure = `30000.
85  1
x = 552 ⋅ 5 New price = 300 1 +  = `400
100  3 
30000
Alternative method: New consumption = i.e. 75 kg.
400
64% = 416 Percentage reduction in consumption
85% =
85
( 416 ) = 552 ⋅ 5 =
25
(100 ) = 25%
64 100

Triumphant Institute of Management Education Pvt. Ltd. (T.I.M.E.) HO: 95B, 2nd Floor, Siddamsetty Complex, Secunderabad – 500 003.
Tel : 040–27898195 Fax : 040–27847334 email : info@time4education.com website : www.time4education.com SM1001906/25
In the above three examples, if the percentage given The SELLING PRICE (S.P) and the COST PRICE (C.P)
100 x of an article determine the profit or loss made on the
initially is x, what is asked to be found is .
(100 + x ) particular transaction.
We can generalize each of the three cases as below: The computation is done as follows:
If the value of an item goes up/down by x%,
Profit = Sale Price – Cost Price = S.P. – C.P.
the percentage reduction/increment to be now
made to bring it back to the original level is S.P. − C.P. Pr ofit
Percentage Profit = x 100 = x 100
100 x C.P. C.P.
%
(100 ± x ) Loss = C.P. - S.P.
Loss
Percentage Loss = x 100
C.P.
If A is x% more/less than B, then B is It is customary to express Profit/Loss as percentage of
100 x Cost Price. However, in some problems it may
% less/more than A.
(100 ± x ) specifically be given that profit/loss percentage has been
calculated on the selling price or the student may be
asked to calculate the profit/loss percentage on the
selling price. Unless such specific directions are given,
If the price of an item goes up/down by x%,
the profit/loss percentage is always to be calculated on
then the quantity consumed should be
the cost price.
100 x
reduced/increased by % so that the
(100 ± x ) Given Profit/Loss percentage along with S.P., C.P. can
total expenditure remains the same. be found out and similarly, given Profit/Loss percentage
along with C.P., S.P. can be found out by using the
concepts discussed at the beginning of this chapter
(where, if percentage increase or decrease is given, we
PERCENTAGE POINTS can find out the new value from the old value or the
old value from the new value).
The concept of "percentage points" is important in the
usage of percentages. Percentage points is the The following simple rules can be remembered for this
difference of two percentage figures. purpose.
Let us understand this with an example.
Suppose that rice forms 20% of total food grain
production in Year I and 30% of total food grain Given the cost price (C.P.) and profit
production in Year II. percentage p%, the selling price will be given
If we are asked to find out the percentage increase in the by S.P. = C.P. x
(100 + p)
production of rice, calculating percentage increase from 100
30 − 20
20 to 30 as x 100 and saying it is 50% increase
20 Given the cost price (C.P.) and loss
is NOT correct. With the available data, we cannot find percentage p%, the selling price will be
out the percentage increase in the production of rice
from Year I to Year II. We can only say that the given by S.P. = C.P. x
(100 − p)
production of rice as a percentage of total food grain 100
production went up by 10 PERCENTAGE POINTS (the
10 being the increase from 20 to 30 – both percentage
figures) Given the selling price (S.P.) and profit
percentage p%, the cost price will be given
We can see by taking the following figures that the 100
by C.P. = S. P. x
percentage increase in rice production need not be 50%.
(100 + p)
Year I Year II
Rice 1000 960
Total foodgrains 5000 3200 Given the selling price (S.P.) and loss
Rice as percent of 20% 30% percentage p%, the cost price will be given by
total foodgrains 100
C.P. = S.P. x
Here, while rice is 20% of total food grains in Year I and
30% of total food grains in Year II, we find that the actual
(100 − p)
production of rice has not even increased – it decreased
from 1000 in Year I to 960 in Year II.
When two articles are SOLD at the same price (i.e., their
S.P. is the same) such that there is a PROFIT of p% on
PROFIT AND LOSS one article and a LOSS of p% on the other (i.e., common
profit or loss percentage), then, irrespective of what the
In any business/commercial environment the most
S.P. actually is, the net result of the transaction is LOSS.
important concern is about the profit/loss of the
This percentage loss is given by
transaction conducted.
Triumphant Institute of Management Education Pvt. Ltd. (T.I.M.E.) HO: 95B, 2nd Floor, Siddamsetty Complex, Secunderabad – 500 003.
Tel : 040–27898195 Fax : 040–27847334 email : info@time4education.com website : www.time4education.com SM1001906/26
Sol: Let his selling price be `100
(i) Profit = `25 ⇒ CP = SP – Profit = `75
Loss percentage
Actual profit percentage
(Common profit or loss)2 p2
=
100
=
100
=
25
(100 ) = 33 1 %
75 3
(ii) Loss = `25 ⇒ CP = SP + Loss = `125
Actual loss percentage
MARKED PRICE or LIST PRICE is the price that is
indicated or marked on the product or it is the price =
25
(100 ) = 20%
125
which is given in the price list. This is the price at which
the product is intended to be sold. However, there can 3.14. The cost of 4 apples equals the selling price of
be some DISCOUNT given on this price and 3 apples. Find the profit/loss percentage.
consequently, the actual SELLING PRICE of the product
may be less than the MARKED PRICE. Sol: Let the cost of each apple be `x
Cost of 3 apples = `3x
SELLING PRICE = MARKED PRICE – DISCOUNT Selling price of 3 apples = Cost price of
4 apples = `4x
The amount of discount given can also be expressed as ∴ Profit on selling 3 apples = `x
a percentage. DISCOUNT is always expressed as a
percentage of the MARKED PRICE or the LIST PRICE. Profit % =
x
(100 ) = 33 1 %
3x 3
DISCOUNT percent 3.15. The profit made by selling 5 m of a cloth equals
Marked Pr ice − Selling Pr ice the selling price of 2 m of that cloth. Find the
= x 100
Marked Pr ice profit percentage made.
Discount Sol: SP (5m) = CP (5m) + Profit (5m)
= × 100
Marked price As Profit (5m) = SP (2m),
SP (5m) = CP (5m) + SP (2m)
Certain discount is given on an article whose selling ⇒ SP (3m) = CP (5m)
price is S.P. If further discounts are given on this SP 5
⇒ =
discounted price, such discounts are referred to as CP 3
successive discounts. If the successive discounts are
5 −3 2
p%, q% and r%, on a product whose selling price is S.P., ∴ Profit % = ×100 = 66 %
then the effective price after all the discounts is given by 3 3

3.16. A trader promised his customers to sell at cost


(100 - p)(100 - q)(100 - r) price. But he cheats his customers by giving
Discounted price = S.P. x 100 gms less for every kg that he sells. Find his
100 x 100 x 100
profit percentage.

Sol: Let the cost of each gm to the trader be `1. Cost


price of 1000 gms = Selling price of 900 gms.
3.11. Alok bought a watch for `250 and sold it for ost of 900 gms = `900
`300. Find his profit percentage. elling price of 900 gms = `1000
1000 − 900
Sol: Given that rofit percentage = (100 ) = 11 1 % .
S.P = `300 C.P = `250 900 9
⇒ Profit = S.P – C.P = `300 – `250 = `50
3.17. A sold a chair to B at 10% profit. B sold it to C at
Profit % =
50
(100 ) = 20% 20% profit. If C bought it for `660, find the price
250 at which A bought the chair.

3.12. Anand gained 20% by selling a book at `30. Sol: et the cost price of A be `x
Find his gain percentage if he sells it for `36. Selling price of A = Cost price of B = `1⋅1x
Selling price of B = Cost price of C = `1⋅32x
Sol: Let his cost price be ` x Given that 1⋅32x = 660 ⇒ x = 500
 20  ∴ A bought it at `500
Given, 1 +  x = 30 ⇒ x = 25
 100 
3.18. Kiran sold a table at 20% loss. If he sold it at
Gain percent when sold at `36
10% profit, he would have earned `300 more.
36 − 25
= (100 ) = 44% Find the cost price of Kiran.
25
Sol: Let the cost price of Kiran be `x
3.13. Ajay calculated his profit / loss percentage on Selling price of Kiran = `0⋅8x
his selling prices. Find his actual profit/loss If he sold it at 10% profit, his selling price = `1⋅1x
percentage if he calculated Given that
(i) his profit percentage to be 25% 1⋅1x = 0⋅8x + 300
(ii) his loss percentage to be 25% ∴x = 1000
Triumphant Institute of Management Education Pvt. Ltd. (T.I.M.E.) HO: 95B, 2nd Floor, Siddamsetty Complex, Secunderabad – 500 003.
Tel : 040–27898195 Fax : 040–27847334 email : info@time4education.com website : www.time4education.com SM1001906/27
3.19. Ashok bought 15 kg of cashew nuts for `540. (ii) Selling price of the radio sold at profit
He was forced to sell them at a loss equal to the = 1.2 × 1000 = `1200
amount he would have realized by selling 3 kg. Selling price of the radio sold at loss
Find his selling price per kg. = 0.9 × 1000 = `900
Total selling price = `2100
Sol: CP = SP + Loss Overall profit = `100
⇒ CP (15kg) = SP (15kg) + SP (3kg)
⇒ CP (15kg) = SP (18 kg) = `540 (given) Overall profit % =
100
(100 ) = 5%
2000
`540
∴ selling price per kg = = `30
18 (iii) Similar to a method as in (ii), it can be
shown that overall loss = 5%
3.20. Shyam purchased some goods for `1800. He
Note: Overall profit/loss percentage is always
sold one-third of the goods purchased at 20%
half of the difference of x and y.
loss. Find the profit percentage at which the rest
of the goods must be sold to realize an overall
profit of 20%. 3.23. Balu sold two TV sets, one at 10% profit and the
other at 10% loss. Find his overall profit/loss
Sol: Suppose that Shyam purchased 3 kg for `1800 percentage if he sold both the sets at the same
Cost price of 1 kg = `600 price.
He sold one kg at a loss of 20%. Hence his
selling price of this kg = `480 Sol: When two items are sold at the same selling
His total selling price must be (1.2) × 1800 price with one being sold at x% profit and the
= `2160 other being sold at x% loss, on the overall a
Hence selling price of the remaining 2 kg must loss is always made and the overall loss
be `1680. Cost price of 2kg = 2 × 600 x2
percentage is given by % . As Balu sold
= `1200 100
1680 − 1200
∴Profit percentage = (100 ) = 40% both TV sets at the same price and x = 10, he
1200 10 2
must have made an overall loss of i.e. 1%
3.21. Usha bought a certain number of chocolates at 100
the rate of 16 chocolates for `12 and the same
number of chocolates at the rate of 3.24. If Ram sold an article at four-fifth of its actual
24 chocolates for `20. She sold all of them at selling price, he would have incurred a loss of
the rate of 30 chocolates for `30. Find her gain / 40%. Find his actual profit / loss percentage.
loss percentage.
Sol: Let the cost price of Ram be `100
Sol: Let 2x be the total number of chocolates that Given that four-fifth of its actual selling price
she bought. 0.6 × 100 = `60. Hence its actual selling price
otal cost price of Usha 5
is × 60 = `75
12 20 19 4
( x) + ( x ) =` x
16 24 12 ∴ Ram made a loss of 25%.
30
Total selling price of Usha = ( 2 x ) = ` 2x 3.25. A trader marked his goods 40% above his
30
Hence the gain% of Usha cost price. He then gave a discount of 20%.
Find his profit percentage.
19
2x − x
= 12 (100 ) = 500 % = 26 6 % Sol: Let the cost price be `100.
19 19 19 Marked price = 1.4 × 100 = `140.
x
12 Selling price = 0.8 × 140 = `112.
112 − 100
3.22. Anwar bought two radios at the same price. He ∴ Profit % = × 100 = 12%.
100
sold one at x% profit and the other at y% loss.
Find his overall profit/loss percentage, if 3.26. Sachin gave a discount of 20% on the marked
(i) x = y = 10 price of his watch and then sold it. He made a
(ii) x = 20, y = 10 profit of 25%. By what percentage did he mark
(iii) x = 10, y = 20 the watch above its cost price?
Sol: Let the cost price of each radio be `1000. Sol: Let the cost price of the watch be `100
Total cost price = `2000 Let the marked price be `x.
(i) Selling price of the radio sold at profit
Selling price = `0⋅8x
= 1.1 × 1000 = `1100
Also given that the selling price = `125
Selling price of the radio sold at loss
⇒ 0⋅8x = 125
= 0.9 × 1000 = `900
625
Total selling price = Total cost price = `2000 x= = 156 ⋅ 25
∴Neither profit nor loss is made in the 4
entire transaction. ∴Percentage of mark up = 56⋅25

Triumphant Institute of Management Education Pvt. Ltd. (T.I.M.E.) HO: 95B, 2nd Floor, Siddamsetty Complex, Secunderabad – 500 003.
Tel : 040–27898195 Fax : 040–27847334 email : info@time4education.com website : www.time4education.com SM1001906/28
3.27. Alex sold his goods after announcing 3.28. Ram and Shyam invested `18000 and `21600
two successive discounts of 30% each. Find his respectively in a business and at the end of the
effective discount percentage. year shared a profit of `22000. Find the profit
share of Ram.
Sol: Let the marked price be `100. Price after the
first discount = `70. Price after the second Sol: Ratio of profits when ivested for the same
discount periods = ratio of investments = 18000 : 21600
0.7 × 70 = `49 =5:6
∴ Effective discount percentage is 100 – 49
= 51%. Profit share of Ram =
5
(22000 ) = `10000
11

PARTNERSHIPS 3.29. David started a business with `40000.


Three months later, Edward joined him with
Two or more people can get together to do business by `50000. Find the ratio in which they must share
pooling their resources. The money put in by each of the the annual profit.
partners is called his "INVESTMENT" or "CAPITAL."
Sol: Ratio of the profits of David and Edward = The
All the people who have invested money in the ratio of the product of their investments and the
partnership are called PARTNERS. period of investment.
= (40000) (12) : (50000) (9) = 16 : 15
While two or more partners would have invested money,
it is not necessary that all of them should be involved in 3.30. Sachin started a business with `20000 and after
the day-to-day running of the business. The partners 4 months Sunil joined him with `40000. Sachin
involved in the day-to day activities of the business are received `39000 as his annual profit share
called "working partners" and the others are called which included a salary of 16% of the annual
"sleeping partners" or "dormant partners." profit. Find the annual profit share of Sunil.

The profits left after paying the working partners' Sol: Let the total annual profit be `x
remuneration/commission are shared amongst all the Ratio of the part of the total annual profit which
partners. is shared in the ratio of the (investments × time)
= (20000) (12) : (40000) (8) = 3 : 4.
Sometimes, the partners also take interest on their Hence the share of Sachin
investments and only the remaining profits are shared by 3  84  16 x
the partners. =  x + = 39000
7  100  100
x = 75000
Sharing of profits among the partners also depends on
∴ Annual profit share of Sunil
the understanding between the partners. However, if no
= `75000 – `39000 = `36000
special scheme of sharing the profits is specified (in a
problem), then the profits are shared based on the
investments of the partners. There are three different 3.31. A started a business with `20000. After
possibilities that exist here. 3 months, B joined him with `40000. After some
more months, C joined them with `100000.
B received `18000 out of the total annual profit
- If the partners invest DIFFERENT amounts each for of `55000. How many months after A started
the SAME period of time, then the profits at the end the business did C join?
of the year are shared in the ratio of their
investments. Sol: Let us say C joined after x months.
Profit is shared in the ratio
- If the partners invest the SAME amounts for
DIFFERENT periods of time, then the profits at the (20000) (12) : (40000) (9) : 100000 (12 − x)
end of the year are shared in the ratio of the time = 24 : 36 : 10 (12 − x)
periods for which their respective investments have 36 18 36
Given = =
been in business. 180 − 10 x 55 110
180 − 10x = 110
- If the partners invest DIFFERENT amounts and the
∴x=7
time periods for which their investments are in the
business are also DIFFERENT, then the profits at
3.32. Mohan started a business with `20000. After
the end of the year are shared in the ratio of the
4 months, Sachin joined him with `30000. At the
products of (investment x time period) calculated for
beginning of the fifth month, Mohan added
each partner.
`10000. Find the ratio in which the annual profit
will be shared.
There CAN be problems that are modelled along the
sharing of profits in partnerships. An example of this type
Sol: Ratio of the profit shares
is where a particular facility (like renting a tractor for
= [(20000) (12) + (10000) (8)] : [30000 (8)] = 4 : 3
ploughing their fields by three different people) is used
by more than one party and the rent has to be shared by
3.33. The salary of a working partner equals 20% of
all the concerned parties – similar to sharing of profits in a
the annual profit remaining after his salary is paid.
partnership.
If his salary is `10000, find the annual profit.
Triumphant Institute of Management Education Pvt. Ltd. (T.I.M.E.) HO: 95B, 2nd Floor, Siddamsetty Complex, Secunderabad – 500 003.
Tel : 040–27898195 Fax : 040–27847334 email : info@time4education.com website : www.time4education.com SM1001906/29
Sol: Let the annual profit be `x If somebody is holding `1000 "worth of stock", it means
that the face value of stock he is holding is `1000. If the
Given, 10000 =
20
(x − 10000 ) face value of the stock is `100, that person will be
100
holding 10 units of such stock.
⇒ x = 60000
Typical problems in Shares and Stocks may include
STOCKS AND SHARES finding as to which out of given investment is a better
one or finding the annual income or change in income
A limited company raises capital by floating shares. It is from a certain investment or change in portfolio, etc.
also referred to as stock. The capital required is divided
into small units called shares. In India, the generally These problems are very similar to problems in Profit
accepted value for such a unit is `10 or `100. This is and Loss Percentages except for involving the
called the Face Value or Par Value. terminology as given above.
The shares of a public limited company are traded in the For all the examples we are going to look at, the face
market place and depending on the demand for the value of the stock is to be taken as `100 unless
share, the price fluctuates. The rate at which a share is otherwise specified.
bought or sold in the market is the Market Value of the
share. This fluctuates. If the market value is more than 3.34. What is the annual income from `32400
the face value of the share, then we say that such a invested in 12% stock at 8% premium?
share is quoting at a "premium." If the market value is
less than the face value of the share, then we say that Sol: Market value = `108
such a share is quoting at a "discount." 32400
Number of units purchased = = 300
The people who are holding the shares are called 108
shareholders. The company distributes a part of its Each unit purchased gives `12 as income.
profits from its operations as dividend to the ∴ Annual income = (12) (300) = `3600
shareholders. The dividend is expressed as a
percentage of the Par Value. Whenever any company 3.35. Which of the following is a better investment
quotes a dividend percentage figure, it goes without − 6% stock at 84 or 8% stock at 96?
saying that it is a percentage of the face value.
Sol: In the first investment, `84 must be invested to
Dividend Amount
% of dividend = x 100 obtain `6 as income.
Par Value ∴ `14 must be invested to obtain `1 as income.
Dividend is always calculated only on the 'FACE VALUE' In the second investment, `96 must be invested
or the 'PAR VALUE' irrespective of the price at which to obtain `8 as income.
the share was purchased.
∴ `12 must be invested to obtain `1 as income.
The government also deals with stock where it issues ∴ The second investment is a better
bonds or other form of stock with a certain face value investment.
and a certain assured rate of interest. This stock is then
3.36. Sastry invests half his salary in 4% stock at 95
traded in the market as per the regulations of the
and the other half in 8% stock at 105. Sharma
government. Since the government stock comes with
invests the same amounts in 4% stock at
fixed rate of return, the stock is normally referred to by
90 and 8% stock at 110. Who receives a better
the percentage of the return. For example, if 5% is the
percentage yield?
rate of return (of stock whose face value is `100), then
such stock is referred to as 5% stock. The face value of
the government bond is normally `100. Supposing this Sol: Let the total Amount be (2 × 95 × 105 × 90 × 110);
stock yielding 5% return (on face value) is purchased by (Common multiple of prices)
somebody at `95, then we say that person has Sastry makes (4 × 105 × 90 × 110) + (8 × 95 × 90
purchased "5% stock at 95". Instead, if he purchases it at × 110) = 90 × 110 × 4 (105 + 190) ……. (1)
`108, then we say that he has purchased "5% stock at
295 × 90 × 110 × 4
108". Rate of return = × 100
2 × (95 × 105 × 90 × 110 )
In the case where he purchased 5% stock at 95, to buy = 5⋅09%
one unit of that stock, he pays `95. But since the face Sharma makes
value is `100, the return or income he gets at the end of
the year will be 5% of 100, i.e., `5. (4 × 105 × 95 × 110) + (8 × 105 × 95 × 90)
= (110 + 180) × 4 × 105 × 95 …….. (2)
In this case, since he receives an income of `5 per year 290 × 4 × 105 × 95
Rate of return = × 100
5 2 × (110 × 90 × 95 × 105
by investing `95, his rate of return is x 100 which is
95 = 5⋅08%.
55/19%
Note: Even without calculating the rates of
To compare two investments (i.e., investments in return, the numbers under (1) and (2) can be
two different stocks), we compare the rate of return for compared and the result obtained.
both investments and whichever gives a higher rate of
return is a better investment.

Triumphant Institute of Management Education Pvt. Ltd. (T.I.M.E.) HO: 95B, 2nd Floor, Siddamsetty Complex, Secunderabad – 500 003.
Tel : 040–27898195 Fax : 040–27847334 email : info@time4education.com website : www.time4education.com SM1001906/30
Concept Review Questions
Directions for questions 1 to 50: For the Multiple Choice Questions, select the correct alternative from the given
choices. For the Non-Multiple Choice Questions, write your answer in the box provided.

1. If 40% of 50 = x% of 80, what is the value of x? 14. Ashok secured 70% of the votes polled in an
(A) 20 (B) 25 (C) 30 (D) 40 election and was elected by a majority of 168 votes.
All the votes polled were valid. Find the number of
2. What percentage of 80 is 200? votes polled if there were only two contestants.
(A) 490 (B) 420 (C) 350 (D) 560

15. The price of an article becomes `260 after it


3. 32% of what number is 256? increases by 30%. Find the original price (in `).
(A) 1024 (B) 800 (C) 640 (D) 400

4. If 60% of x is 60 more than 60% of 60, then 60% of


x = _______. 16. The cost price of an article and its selling price are
(A) 90 (B) 94 (C) 96 (D) 92 `600 and `900 respectively. Find the profit percentage.
1 2
5. Which of the following fractions equals 1013/5%? (A) 50% (B) 33 % (C) 100% (D) 16 %
3 3
(A) 508/5 (B) 254/5
(C) 51/25 (D) 127/125
17. P and Q started a business in which P invested
`10000 and Q invested `20000. They received a
6. If 60% of 70% of a number is 1680, find the number.
profit of `9600 at the end of a year. Find Q’s share
in the profit (in `).

7. If 40% of a number y is 75 more than 20% of 1500,


y = _______. 18. The price of an article becomes `63 after
(A) 962.5 (B) 952.5 a decrease of 30%. Find the original price (in `).
(C) 937.5 (D) 912.5 (A) 100 (B) 96 (C) 90 (D) 80

A A A 19. In a test, Mohan’s mark was 25% more than


8. If A% of + % of A = C% of , C = A.
B B B Sohan’s mark. Mohan got the minimum mark
required to pass the test. The pass mark was 35.
Find Sohan’s mark.

9. If a% of b + b% of a is equal to 331/3% of (a + b),


 1 1 20. Ravi’s salary before he got an increment was
 +  = ________.
a b 20% of the total income of his family. His increment
(A) 6% (B) 9% (C) 12% (D) 18% was one-fourth of his salary after the increment.
What percentage of the total income of his family is
10. Ganesh owns 831/3% of a property. Three-fourths of his new salary?
his share is worth `5 lakhs. Find the value of the 2
(A) 16 % (B) 20%
property (in ` lakhs). 3
1
(C) 33 % (D) 25%
3

11. A student secured 80% of the total marks and got 21. The salaries of two persons are equal. If the salary
720 marks. How many marks did a student who of one of them is increased by 20% and the salary of
scored 90% get? the other is decreased by 20%, find the percentage
(A) 750 (B) 1000 change in the total salary of the two persons.
(C) 810 (D) 900 (A) 4% increase (B) 4% decrease
(C) 0% (D) None of these
12. In a test, the pass percentage was 35%. A student
who wrote it got 230 marks and failed by 15 marks. 22. The population of a country quadrupled from 2001 to
Find the maximum mark in it. 2002. Find the percentage increase in the population.
(A) 700 (B) 630 (C) 770 (D) 840 (A) 400% (B) 500% (C) 300% (D) 4%

13. The income tax rate is reduced from 25% to 171/2%. 23. In an exam, Arun got 20% more marks than Bala.
The tax to be paid by Bala whose taxable income is By what percentage are the marks of Bala less than
`9200 would reduce by (in `) . that of Arun?
2 1
(A) 20% (B) 16 % (C) 25% (D) 33 %
3 3
Triumphant Institute of Management Education Pvt. Ltd. (T.I.M.E.) HO: 95B, 2nd Floor, Siddamsetty Complex, Secunderabad – 500 003.
Tel : 040–27898195 Fax : 040–27847334 email : info@time4education.com website : www.time4education.com SM1001906/31
24. The price of a TV is decreased by 20%. By what 34. An amount of `36 is gained when an article was sold
percent must it be increased to bring it back to the at a profit of 6%. What is the cost price of the
original price? article? (In `)
2
(A) 25% (B) 20% (C) 16 % (D) 15%
3

25. The price of a TV is increased by 20%. By what 35. Two articles are bought at the same price. One is
percent must it be decreased to bring it back to the sold at 20% profit and the other is sold at 10% loss.
original price? Find the overall profit/loss percentage.
2 (A) 10% profit (B) 10% loss
(A) 25% (B) 20% (C) 16 % (D) 15%
3 (C) 5% loss (D) 5% profit

26. The ratio of two numbers is 5/6 : 2/3. By what 36. The cost price of a camera is 90% of its selling
percentage is the second number more/less than price. Find the profit percentage.
the first number? 1 1
(A) 20% less (B) 25% more (A) 9 % (B) 10% (C) 11 % (D) 12%
11 9
(C) 25% less (D) 20% more
37. The cost price of a product is `50. It is increased by
27. The price of fan A is twice that of another fan B. The
price of A is increased by 10% and that of B is 1
20% and then by 33 % . Find its latest price (in `)
decreased by 20%. Find the percentage decrease in 3
the sum of the prices of the fans.

38. A shopkeeper sells an item for `60 at a profit of 20%.


At what price (in `) should he sell it to gain 30%?
28. There are 30 employees in a company. The salaries (A) 63 (B) 65 (C) 68 (D) 70
of 20 of them were each increased by 10%. The
salaries of the rest were each increased by 20%. 39. The profit made on selling 5 m of a cloth equals the
Find the percentage increase in the total salary of cost price of 2 m of that cloth. Find the profit
the employees. percentage in selling each metre of the cloth.
1 2 4
(A) 13 % (B) 15% (A) 66 % (B) 50% (C) 40% (D) 28 %
3 3 7
2
(C) 16 % (D) Cannot be determined
3 40. A company manufactures a product for `50. It sold it
to a dealer for `60. The dealer sold it to a shopkeeper
29. The price of an article is decreased by 20% and for `75. The shopkeeper sold it to a customer for
then increased by 20%. Find the net percentage `100. Find the profit percentage of the company.
change in the price.
2
(A) 0% (B) 4% increase (A) 16 % (B) 25%
(C) 4% decrease (D) None of these 3
1
30. The price of on article is increased by 10% and then (C) 20% (D) 33 %
3
decreased by 10%. Find the net percentage change
in the price. 41. In question above who got the highest profit on
(A) 0% (B) 1% increase selling the product?
(C) 1% decrease (D) None of these (A) Company
(B) Dealer
31. A positive number was decreased by 20% and then (C) Shopkeeper
increased by 20%. Find the percentage change in it. (D) All of them got the same profit
(A) 2% decrease (B) 0%
(C) 4% decrease (D) 4% increase
42. The cost price of 80 articles is `12⋅50 per article.
32. There are three natural numbers. The first and Twenty of them were sold for `18 each. At what price
second are less than the third by 40% and 50% should each of the remaining articles be sold so as to
respectively. What percentage of the second get an overall profit of `4.50 per article?
number is the first number? 2 1
(A) `15 (B) `16 (C) ` 17 (D) `18
(A) 125% (B) 1331/3% 3 3
(C) 1162/3% (D) 120%
43. Two successive discounts of 30% and 10% are
33. Raja receives an increment in his salary. His salary equivalent to a single discount of
presently forms 30% of the total income of his
family. Before the increment his salary used to form %.
20% of the total income of his family. By how many
percentage points did the salary of Raja as a
percentage of his family’s total income increase? 44. Govind marked an article 25% above its cost price and
2 2 allowed a discount of 30%. Find his loss percentage.
(A) 14 (B) 16 (C) 10 (D) 8 (A) 10% (B) 15% (C) 7.5% (D) 12.5%
7 3
Triumphant Institute of Management Education Pvt. Ltd. (T.I.M.E.) HO: 95B, 2nd Floor, Siddamsetty Complex, Secunderabad – 500 003.
Tel : 040–27898195 Fax : 040–27847334 email : info@time4education.com website : www.time4education.com SM1001906/32
45. Rohit marked his goods 40% above his cost price. 48. Ramesh and Suresh started a business. Ramesh
He sold it after a discount at 12% profit. Find his invested `9000 for ten months and Suresh invested
discount percentage. `6000 for a year. If the profit at the end of a year
was `4500, find Suresh’s share.
% (A) `3600 (B) `2700
(C) `2500 (D) `2000
46. A shopkeeper bought an article for `360. The profit
made by the shopkeeper after selling it after a 49. Ashok invested `12000 in a 6% stock at par. Find
1 his annual income (in `).
11 % discount is `40. Find the marked price (in `)
9
of the article.

50. Ajay invested `12600 in a 5% stock at 5% premium.


Find his annual income (in `).
47. Ajay and Vijay invested `8000 each to start a (A) 600 (B) 630 (C) 660 (D) 720
business. Ajay invested his money for nine months
and Vijay invested his money for a year. At the end
of a year, if the profit was `3500, find Ajay’s share.
(A) `1500 (B) `2000
(C) `2100 (D) `2800

Triumphant Institute of Management Education Pvt. Ltd. (T.I.M.E.) HO: 95B, 2nd Floor, Siddamsetty Complex, Secunderabad – 500 003.
Tel : 040–27898195 Fax : 040–27847334 email : info@time4education.com website : www.time4education.com SM1001906/33
Exercise – 3(a)
Directions for questions 1 to 35: For the Multiple Choice Questions, select the correct alternative from the given
choices. For the Non-Multiple Choice Questions, write your answer in the box provided.

1. In 2004, the price of a shampoo bottle increases by 7. The production of rice in the year 2001 was
10% with respect to that in 2003. By what 1000 tonnes which was 25% of the total food grain
percentage is its price in 2003 less than that in production in that year. In the next year if the
2004? production of rice decreased by 4% and production of
1 rice as a percentage of total food grain production
(A) 10% (B) 9 % increased by 5 percentage points, what is the total
11
food grain production in 2002? (in tonnes)
1
(C) 11% (D) 10 %
11

2. A student secures 38% of the total marks in an 8. In a public sector unit (PSU), there are
exam and gets 18 marks more than the pass mark. 45600 employees. When the PSU offered a
A second student secures 27% of the total marks in voluntary retirement scheme (VRS), 40% of the
the same exam and fails by 37 marks. What is the employees applied for the VRS. After scrutinizing,
pass mark as a percentage of the total marks? the PSU has rejected 15% of the applications. But
only 9120 employees took the retirement through the
scheme. What percentage of the total number of
employees did not take retirement even though their
3. Due to inflation the total cost of monthly household applications are not rejected?
items has gone up by 20%, but the salary of the (A) 25% (B) 14% (C) 24% (D) 12.75%
family increased by only 10%. Initially, the family
used to spend 20% of the salary on household 9. A school has 2000 students, out of which 40% of the
items. What percentage of the present salary should students are in higher secondary. Out of the
the family spend to buy the same quantities of students in the higher secondary, 28% are girls and
household items? 50% of them passed. The overall pass percentage in
the higher secondary is 40%. How many boys passed?
2 (A) 208 (B) 320
(A) 10% (B) 20 %
11 (C) 200 (D) 96
2 9 10. In an election among three contestants P, Q and R,
(C) 22 % (D) 21 %
11 11 P gets 120% more votes than Q. P beats R by
3,50,000 votes. Q beats R by 5% of the total votes.
4. In 2001, the transportation cost of an item was 30% Find the total number of votes polled (in lakhs).
of its manufacturing cost. From 2001 to 2002, its (A) 12 (B) 10 (C) 9 (D) 11
manufacturing cost went up by 20%. Its
1 11. In a certain month, the total income of A, B, C is
transportation cost in 2002 was 33 % of its
3 `644000. In that month, A spends 75% of his
manufacturing cost in that year. By what percent income, B spends 80% of his income, C spends
should the total cost in 2002 be reduced to bring it 62.5% of his income. The ratio of the amounts spent
back to that in 2001? by A, B, C in that month is 42 : 64 : 55. Find the
income of A in the month. (in `)
%

(Total cost = Manufacturing cost + Transportation cost).


12. The success rate at one stage, of the Indian cricket
5. In a town, the population of males decreased by team in Australia was 25% from 60 matches. If India
25% from 2001 to 2002. The population of females lost the next 12 matches, what was the minimum
increased by 20% in this period. If females formed number of total matches that were played if the
4 overall success rate of India was 50%?
44 % of the population in 2002, what percentage (A) 42 (B) 104
9
of the population in 2001 were males? (C) 118 (D) 114
2
(A) 50% (B) 66 % 13. In 2003, Brijesh paid a tax of 20% of his salary. In
3 2004, his salary increased by 933/4% and the tax
(C) 80% (D) 75% scheme changed. Under the new tax scheme he
had to pay a fixed sum of `1000 and an additional
6. The length of a rectangle is increased by 20% and 20% on the amount above `10000. His salary in
its breadth is increased by 10%. Which of the 2004 was more than `10,000 and he paid a tax of
following is a possible value of the percentage `500 more than what he paid in 2003. What is his
increase in its perimeter? salary in 2004?
(A) 13% (B) 14% (A) `15500 (B) `17000
(C) 12% (D) 16% (C) `18200 (D) `20000
Triumphant Institute of Management Education Pvt. Ltd. (T.I.M.E.) HO: 95B, 2nd Floor, Siddamsetty Complex, Secunderabad – 500 003.
Tel : 040–27898195 Fax : 040–27847334 email : info@time4education.com website : www.time4education.com SM1001906/34
14. At a certain stationery shop, the cost of each ruler is 21. Initially a television was being offered at a discount
2.5 times the cost of each sharpener. Anil bought of 40%. The dealer reduced this discounted price
25 rulers and a certain number of sharpeners. Had further by 20% because the customer bargained.
he bought as many rulers as the number of If the selling price of the television is `9600, then
sharpeners that he bought and vice versa, his total what is its marked price?
expenditure on the two items would have been (A) `19200 (B) `16000
1 (C) `20000 (D) `25000
9 % less. How many sharpeners did he buy?
11
2 th
22. A shopkeeper sold of his stock at 18% profit,
9
three – fifth of his stock at 27.5% profit and the rest
15. In 2000, the market shares of the toilet soaps Margo, at 30% profit. The overall profit percentage made by
Palmolive and Dove were 40%, 30% and 30% the shopkeeper is
respectively. Starting from the next year, a new soap 1 5
enters the market each year and gets 10% of total (A) 21 % (B) 25 %
6 6
market share in that year. The existing soaps in that
year share the remaining market in the same ratio 7 1
(C) 19 % (D) 28 %
as they did in the previous year. What percent of the 2 3
total market share will Margo have in 2002?
(A) 32% (B) 32.4% 23. A man bought 100 mangoes at a certain price, with
(C) 28.8% (D) 34% the intention of selling each at a profit of 25%. But
20 mangoes got spoilt. If he sold the rest at the
16. At the beginning of a year, the owner of a jewel shop intended price, what was his profit or loss
raised the prices of all the jewels in his shop by x% percentage?
and the lowered then by x%. The price of one jewel (A) 0% (B) 6.66% profit
after this up and down cycle reduced by `100. The (C) 6.25% loss (D) 12.5% profit
owner carried out the same procedure after a
month. After this second up-down cycle, the price of 24. A trader sells an article at a profit of `25. If the cost
that jewel was `2304. Find the original price of that price is reduced by `25 and consequently the selling
jewel (in `). price is reduced by 25% he would make a profit of
(A) 2600 (B) 2550 (C) 2650 (D) 2500 25%. What is his initial cost price? (in `)

17. At the beginning of 2002, Sunil had four dozen goats


with him. He increased this stock by x%. At the end
of 2002, he sold off y% of his stock. At the beginning 25. A shopkeeper always weighs 20% less than the
of 2003, he again increased his stock by x% and at correct weight. One day, he weighed 20% more than
the end of 2003, he again sold off y% of his stock. the quantity usually weighed by him. If the profit on
There were no other changes in the number of the correct weight is 20%, what is the effective
goats. At the end of 2003, he had five dozen goats percentage of profit in this transaction?
with him after his sales. Which of the following (A) 10% (B) 25% (C) 20% (D) 15%
always holds true?
100 y 100 y 26. A dealer bought 50 television sets at `10000 each.
(A) x > (B) >x>y
100 − y 100 − y For every set purchased from him, he gave one set
free. The loss made by him is equal to the selling
100 y price of 15 sets. What is the selling price of each
(C) x < y (D) < x<y
100 − y set, that is bought?
(A) `10000 (B) `15000 (C) `12500 (D) `20000
18. A bicycle was sold at a loss of 18%. Had it been
sold for `990 more, there would have been a profit 27. Anwar bought an article listed at `18000 at
of 15%. At what price (in `) should the bicycle be 1
sold to make a profit of 10%? 30% discount. He sold it to Balu at 33 % profit.
3
Balu marked up the price of the article by 25% and
2
then gave 16 % discount. Find the profit
19. A person sold two shirts each for `880. On one he 3
gained 10% and on the other he lost 20%. What is
the overall profit or loss percentage? percentage made by Balu. %
6
(A) 55% profit (B) 5.5% loss
(C) 7.36% profit (D) 7.36% loss
28. A printer undertook the task of printing 2000 pages
for a client. The client provided the paper.
20. The cost of production of a motorbike which is sold
The dimension of each page was 24 cm x 14 cm.
at 20% profit went up by 40%. What should be the
percentage increase in the selling price to maintain A 2 cm gap was left on each page on all the 4 sides
the profit percentage the same even at the new cost of the margin. The cost of printing was `8 per sq. cm.
of production? If the printer wants to make 8% profit, how much
should he charge per page?
% (A) `1728 (B) `2281 (C) `1628 (D) `1901

Triumphant Institute of Management Education Pvt. Ltd. (T.I.M.E.) HO: 95B, 2nd Floor, Siddamsetty Complex, Secunderabad – 500 003.
Tel : 040–27898195 Fax : 040–27847334 email : info@time4education.com website : www.time4education.com SM1001906/35
29. Serena and Venus start a business with `50000 32. Ganga and Gayathri start an enterprise by investing
each. At the end of 3 months Venus withdraws `24000 and `36000 respectively. Their agreement is
`10000 from his investment. What percent of the to share half of the total profit equally and then share
total profit should Venus receive at the end of the the remaining half in the ratio of their capitals. If they
year? share the entire profit in the ratio of their capitals,
5 Gayathri would have got `2500 more than what she
(A) 33.3% (B) 45 % would have got otherwise. What is the total profit?
6 (A) `75000 (B) `72000
35 (C) `108000 (D) `50000
(C) 50% (D) 45 %
37
33. A man invests `1395 in a 3% stock at `93. After holding
the stock for a month, he sells `1000 worth of stock at
30. Mehta, Mehra and Mihir start a business with
`95 and the remaining at `80. What is the overall profit
investments in the ratios of 1 : 2 : 3. They make
or loss in the above purchase and sale
a profit of `600000. If Mihir and Mehra leave after
transaction?(in `)
4 and 8 months respectively, then what is Mehta’s
(A) 75 loss (B) 15 loss (C) 22 profit (D) 45 loss
share of the total profit? (in `)
34. Ismail invests in a 6% stock at 5% premium. He gets
a dividend of `3600 at the end of the year. What is
his investment? (in `)
31. Goyal and Gokul invest `20000 and `30000
respectively and start a business. Since Goyal
manages the business, he gets a salary which is equal
to 20% of the balance profit after deducting this salary. 35. A man invests half of the amount he has in 4% stock
If Goyal’s total earnings are `4800 at the end of one at 90 and the other half in 8% stock at 110. If he had
year, what is the total profit made by the business? invested the total amount in the 8% stock at 110 he
(A) `4000 would have made `3500 more. How much did he
(B) `5200 invest?
(C) `9600 (A) `250000 (B) `247500
(D) `6400 (C) `300000 (D) `225000

Exercise – 3(b)
Directions for questions 1 to 45: For the Multiple Choice Questions, select the correct alternative from the given
choices. For the Non-Multiple Choice Questions, write your answer in the box provided.

1. A’s salary is 20% less than B’s salary. If C’s salary is 5. In a college of total strength 1000, 30% of the
`10000 and it is 25% more than B’s salary, then students are girls. There are 600 PGs and 120 more
what is A’s salary? male UGs than female UGs. What percent of the
(A) `6000 (B) `9600 males are the female UGs?
(C) `8000 (D) `6400 (A) 20% (B) 15% (C) 25% (D) 10%

6. A machine costs `4,00,000. It depreciates by 18% in


2. School A has 30% more students than school B.
value in the first year, 16.5% in the second year,
If 120 more students join school B, the two schools 15% in the third year and so on. Find the amount by
will have the same number of students. What is the which it depreciates in the seventh year (in `)
sum of the number of students in school A and (Assume all percentages apply to the original cost of
school B initially? the equipment).

3. The price of petrol increased by 2% in a certain 7. One month Mrs. and Mr. Rai take home `20000
week and increased by 4% in the next week. Find each. These amount respectively represent an
the net percentage increase in the price of petrol increase of 25% over Mrs. Rai’s take home in the
over these two weeks. previous month and a decrease of 33.33% over
(A) 6.12% (B) 6.08% Mr. Rai’s take home in the previous month. What
(C) 6.16% (D) 6.20% was their total income in the previous month?
(A) `40000 (B) `50000
4. Rahul got 150 marks in a test. He scored 25% more (C) `46000 (D) `41666
marks than the pass mark in it. Rajesh got
165 marks in it. By what percent did his mark 8. The monthly income of Ram increased by 26%. His
expenditure which is 70% of his monthly income
exceed the pass mark?
increased by 20%. His savings must have increased by
% (A) 40% (B) 30%
(C) 50% (D) 25%

Triumphant Institute of Management Education Pvt. Ltd. (T.I.M.E.) HO: 95B, 2nd Floor, Siddamsetty Complex, Secunderabad – 500 003.
Tel : 040–27898195 Fax : 040–27847334 email : info@time4education.com website : www.time4education.com SM1001906/36
9. Anand cut a rectangular piece of paper twice. With Directions for questions 16 and 17: These questions
the first cut, he decreased its area by 40%. With the are based on the data given below.
second cut, he decreased its area by 50%. Its final
area was 30 sq cm less than its area after the first A family has 4 earning members A, B, C, D. In 2001 their
cut. Find its original area (in sq cm). respective shares in the total family income are 25%,
35%, 10% and 30%.
They spend 30% of the total income and save the
remaining. In 2002, A’s salary went up by 20%, B’s salary
10. The breadth of a rectangle decreases by 20% and increased by 20%, C’s salary decreased by 10% and D’s
then increases by 30%. By what percent should the salary decreased by 20%.
length decrease, so that the area remains unchanged?
9 8 16. In 2002, what percent of the total income should
(A) 3 % (B) 3 % they spend to save the same amount as in 2001?
13 13 (A) 35% (B) 30%
10 11 (C) 33.33% (D) 22.22%
(C) 3 % (D) 3 %
13 13
17. The total income increases further by 20% in 2003
11. A laptop was sold at a profit of 15%. If it was sold at solely due to an increase in A’s salary. What should
a price that was 10% lower, the profit would have be the percentage increase in A’s salary from
been `1050. What is the cost price of the laptop? 2002 to 2003?
(A) `21000 (B) `35000 (A) 20% (B) 30% (C) 70% (D) 60%
(C) `30000 (D) `42000
Directions for questions 18 and 19: Answer the
12. A new coach was appointed in the middle of a season questions based on the following data:
for a football team. After he took over, the team won
80% of the 60 matches it played. But the overall In an exam, every question correctly answered fetches
success rate of the team was only 60%. Find the 2 marks. Every question wrongly answered loses
minimum number of matches the team must have 1 mark. Unanswered questions have no marks
played that season before the new coach took over. associated with them. Ram and Shyam wrote this exam.
Ram attempted a certain number of questions and 30%
of them went wrong. Shyam attempted a certain number
of questions and 40% of them went wrong. Ram got
40 marks more than the pass mark. Shyam got 25 marks
13. Mr. Singh is paid as per the number of hours he puts more than the pass mark. The two of them together
in per month. The rate of pay was increased by 20% attempted a total of 100 questions.
per hour, but the number of hours put in by him went
down by 20%. What is the percentage 18. Find the pass mark in the exam.
increase/decrease in his income? (A) 15 (B) 20 (C) 25 (D) 30
(A) 2% decrease (B) 2% increase
(C) 4% decrease (D) 4% increase 19. If there are 80 questions in the exam, find the
percentage of marks secured by Ram.
14. In December 2014, Bala spent 75% of his income on (A) 40.625% (B) 45.75%
food, shopping and utility bills. The amounts that he (C) 50.75% (D) 34.375%
spent on food, shopping and utility bills is 7 : 6 : 2. In
February 2015, Bala spent 72% of his income on 20. The price of a PC has been decreasing every year
food, shopping and utility bills. The amount that he by a constant percentage over the last 4 years. If
spent on food, shopping and utility bills is 22 : 19 : 7. cost of a PC was `50000 4 years ago and it costs
The amount he spent on utility bills in February 2015 `32805 now, find the yearly rate of decrease.
2 (A) 8% (B) 5% (C) 10% (D) 15%
was 16 % more than that in December 2014. The
3
percentage by which his income in December 2014 21. In a certain year, the wholesale price index
was less than that in February 2015 is fluctuated as given below:
%. Percentage of increase
Period or decrease over
the preceding period.
15. A motorist used 10% of his fuel to cover 20% of his
total journey. He covered another 40% of his total (a) 1st April to 30th April Increased by x%
journey under similar conditions. For the rest of st st
journey the conditions were different. Find the (b) 1 May to 31 May Decreased by x%
maximum percentage by which his fuel efficiency st
(c) 1 June to 30 Juneth
Increased by x%
(distance covered per unit quantity of fuel) can drop,
so that he can still cover the remaining journey (d) 1st July to 31st July Decreased by x%
without a refill.
If the decrease for the period 1st April to 31st May
3 6 was 160 points and that for the period from 1st June
(A) 71 % (B) 72 %
7 7 to 31st July was 134.4 points, what was the price
1 2 index on 1st April? (in points)
(C) 27 % (D) 74 % (A) 800 (B) 900 (C) 1000 (D) 1200
7 7
Triumphant Institute of Management Education Pvt. Ltd. (T.I.M.E.) HO: 95B, 2nd Floor, Siddamsetty Complex, Secunderabad – 500 003.
Tel : 040–27898195 Fax : 040–27847334 email : info@time4education.com website : www.time4education.com SM1001906/37
22. To manufacture a product X, a company needs raw 31. If the discount and profit percentage are both 20%
materials P, Q and R and others. The cost price of by what percent is the marked price above the
P is 10% of the total cost price of X. The cost price cost price?
of each of Q and R is 20% of the total cost of X. The %
cost of P is increased by 20%. The costs of each of
Q and R is increased by 10%. The total cost is
increased by 15%. Find the percentage increase in 32. A car dealer sold a car at a discount of `100000.
the cost of the others. Even after the discount, he made a profit of 15%.
%. What is the marked price of the car, if the marked
price is 25% more than the cost price?
(A) `1000000 (B) `1115000
23. The length of a rectangle is increased by 10%. The (C) `2500000 (D) `1250000
breadth of the rectangle is increased by 20%. The
perimeter of the rectangle increased by x%. The 33. Simon gets a discount of 25% on purchasing
range of x is_____. 100 VCD’s from Samuel. He sells them and makes
(A) [15, 20) (B) (10, 15] a profit equal to the undiscounted price of 25 VCD’s.
(C) (10, 20) (D) None of these What is the gain percentage?
(A) 25% (B) 30%
24. P sold an article to Q at 20% profit. Q sold it to (C) 66.66% (D) 33.33%
R at 25% profit. If the difference in the cost prices of
Q and R is `90, find the cost price of the article for P. 34. Mr. Londa imported 10000 hard disks from Korea at
(A) `270 (B) `360 (C) `300 (D) `250 a discount of 10% on the marked price. Out of
these, 20% of the hard disks were damaged in
25. A man gets back the amount he had invested in transit. If the selling price of an undamaged disk is
buying 100 radios by selling 65 of them. What is his 20% more than its cost price, what is the profit/loss
profit percentage? percentage on total sales?
(A) 6.66% profit (B) 10% profit
11 9
(A) 53 % (B) 49 % (C) 6.66% loss (D) 4% loss
13 13
(C) 35% (D) 65% 35. Ashwin bought an article at `200 and marked it at
`300. He offered a discount and then sold it his
26. Ravi manufactures watches. Each day he manufactures profit/loss percentage and discount percentage are
as many watches as the cost price per watch (in `). in the ratio 3 : 2. Find his profit/loss percentage.
Each day he sells all his watches at a profit of (A) 29% profit (B) 25% profit
`60 per watch and at the end of the day his profit (C) 20% loss (D) 25% loss
percentage is 10%. Find his daily profit (in `).
36. Feroze marks up an article by 30% and sells it at a
discount of 20% to Sohail. Sohail marks up the price
of the article to a certain amount which happens to
27. Rakesh bought some chocolates at 10 for `1.50 and be 20% more than Feroze’s cost price. What is the
an equal number at 20 paise each. He sold them at maximum discount Sohail can offer without going
a rate of 20 for `4 and made a profit of `10. How into loss?
many chocolates did he purchase? (A) 30% (B) 20%
(A) 100 (B) 200 (C) 300 (D) 400 2 1
(C) 16 % (D) 13 %
3 3
28. Kanchan has bought 50 articles. He sells 20% of the
articles and makes a profit of `1200, which is also 37. Girish marked an article at 60% above his cost
equal to the cost of 5 articles. If the selling price for all price. What is the maximum number of successive
50 articles is the same, what is the value of the discounts, each being 10%, he can give such that
remaining articles at the selling price? he does not make a loss?

29. Ajay sold an article at 20% profit to Balu. Balu sold it 38. Govind bought an article at `300 and marked it at
at 30% profit to Chetan. Dinesh sold a similar article `400. He offered a certain discount but sold it at a
at 20% loss to David. David sold it at 30% loss to profit. Which of the following holds true?
Edward. The sum of the price that Chetan and (A) If his discount percentage is at least equal to his
Edward paid for their respective articles is `28000 profit percentage, his profit will be at least equal
more than what Ajay paid. If Ajay and Dinesh bought to is discount.
the article for the same price, Find the sum of the (B) If his discount percentage is at most equal to his
prices paid by Ajay and Dinesh (in rupees) for their profit percentage, his profit will be at most equal
respective articles. to his discount.
(A) 50000 (B) 40000 (C) 45000 (D) 52000 (C) If the profit percentage is at least equal to the
discount percentage, the discount is at most
30. An article is marked up by 35%. What should be the equal to the profit.
discount percentage so that the profit is 20%? (D) If the profit percentage is at most equal to the
1 discount percentage, the discount is at least
(A) 11 % (B) 10% (C) 15% (D) 20%
9 equal to the profit.
Triumphant Institute of Management Education Pvt. Ltd. (T.I.M.E.) HO: 95B, 2nd Floor, Siddamsetty Complex, Secunderabad – 500 003.
Tel : 040–27898195 Fax : 040–27847334 email : info@time4education.com website : www.time4education.com SM1001906/38
39. In a provision store, a packet of dal contains 20% Mark (B) if the question can be answered using
less than the labelled weight. But it is sold to a either statement alone.
customer at a discount of 10%. How much more, as Mark (C) if the question can be answered using Ι and
a percentage by weight, should the customer get, so ΙΙ together but not using Ι or ΙΙ alone
that he gets the dal at the rate he would have got, if Mark (D) if the question cannot be answered even
the packet contained the correct quantity and there using Ι and ΙΙ together.
was no discount?
46. Was the discount percentage offered on item x less
than 16%?
Ι. Its marked price was at most `30 and the profit
40. Gopal, Hari and Karthik started a business with made was 25%.
investments of `8000, `12000 and `16000 ΙΙ. Its cost price was `20.
respectively. Hari and Karthik left the business after
x months. Out of the annual profit share, Gopal got 47. What is the cost price of the cycle?
more than Hari but less than Karthik. If x is an integer, Ι. The cost price is ten times the profit, and the
find the ratio of Gopal’s, Hari’s and Karthik’s shares. selling price is `165
(A) 27 : 21 : 28 (B) 24 : 21 : 28 ΙΙ. The profit is 10% of `160, the selling price.
(C) 30 : 27 : 36 (D) 32 : 30 : 40
48. Did the transaction result in loss or gain?
41. P and Q started a business. They made an annual Ι. Selling price of the article is `320.
profit of `50000. Q being a working partner received ΙΙ. Selling price of 18 articles is equal to the cost
20% of the annual profit as his salary. If the entire price of 21 articles.
profits were divided in the ratio of their investments,
P would have received `8000 more as his profit 49. What percentage of income does the man save?
share than what he actually got. Find P’s actual Ι. He spends 35% of income on house rent.
profit share (in `). ΙΙ. He spends 75% of his income remaining after
paying house rent on other items.

50. A and B are in a business, in which B joined A after


42. A, B, C and D started a business with investments in x months. What is the value of x?
the ratio 3 : 4 : 5 : 6. As B and C were working Ι. They shared the profits equally at the end of the
partners they were paid equal salaries. The ratio of year.
B’s and C’s total annual income is 9 : 10. If the total ΙΙ. The capitals of A and B are in the ratio 7 : 12.
annual profit is `84000, find B’s salary (in `).
51. In a partnership business, did A and B invest equal
capitals, given that they are in the business for the
same period where A did not receive any
43. In an election between two candidates P and Q, only remuneration?
90% of the registered voters cast their vote. Only Ι. B looked after the business and got a
90% of the polled votes were valid. P got 65% of the remuneration of 25% of the profit for it.
valid votes and won by a margin of 9720 votes. How ΙΙ. A and B got equal amounts at the end of the
many registered voters were there? year.

52. What is the percentage of literacy in females?


Ι. Out of the total population, 60% are males. 25%
44. Satish, Sanjay and Sunil started a business with an
of the males are literates and 25% of the total
investment of `20000 each. At the start of each
population are literates.
month starting from the second, Satish adds `1000.
ΙΙ. For every four females one is literate.
At the start of each even month starting from the
second, Sanjay adds `2000. At the start of the 53. What were the sales of a salesman in a month?
3rd, 6th, 9th and 12th months, Sunil adds `3000. Who Ι. The salesman earns `1500 in that month.
gets the greatest share out of the annual profit? ΙΙ. Every month the salesman gets a flat
(A) Satish commission of 7.5% on all the sales and a bonus
(B) Sanjay of 2.5% on all the sales exceeding `9000.
(C) Both Satish and Sunil
(D) Sunil 54. In a certain store, item X sells for 10% less than item
Y. What is the ratio of the store's revenue from the
45. A man invests `19950 in a 4% stock at 5% premium
sales of item X to that from the sales of item Y?
and the same amount in a 6% stock at 5% discount.
Ι. The store sells 20% more units of item Y than
What is the approximate effective yield percentage
item X.
on his total investment?
(A) 6% (B) 7% (C) 5% (D) 4% ΙΙ. The store's revenue from the sales of item X is
`6000 and from the sales of item Y is `8000.
Directions for questions 46 to 55: Each question is
followed by two statements Ι and ΙΙ. Indicate your 55. If the original price of an article was `120, and the
responses based on the following directives: original price was reduced by `d, then d is what
percent of the reduced price?
Mark (A) if the question can be answered using one
Ι. d is 25% of the original price.
of the statements alone, but cannot be
answered using the other statement alone. ΙΙ. d is 1/3rd of the reduced price.
Triumphant Institute of Management Education Pvt. Ltd. (T.I.M.E.) HO: 95B, 2nd Floor, Siddamsetty Complex, Secunderabad – 500 003.
Tel : 040–27898195 Fax : 040–27847334 email : info@time4education.com website : www.time4education.com SM1001906/39
Key
Concept Review Questions

1. B 6. 4000 11. C 16. A 21. C 26. A 31. C 36. C 41. C 46. 450
2. 250 7. C 12. A 17. 6400 22. C 27. 0 32. D 37. 80 42. B 47. A
3. B 8. 2 13. 690 18. C 23. B 28. D 33. C 38. B 43. 37 48. D
4. C 9. A 14. B 19. 28 24. A 29. C 34. 600 39. C 44. D 49. 720
5. D 10. 8 15. 200 20. D 25. C 30. C 35. D 40. C 45. 20 50. A

Exercise – 3(a)
1. B 6. D 11. 16100 16. D 21. C 26. C 31. C
2. 34.4 7. 3200 12. D 17. A 22. B 27. 25 32. D
3. D 8. B 13. A 18. 3300 23. A 28. A 33. D
4. 18.75 9. A 14. 20 19. D 24. 100 29. D 34. 63000
5. B 10. B 15. B 20. 40 25. B 30. 180000 35. B

Exercise – 3(b)
1. D 8. A 15. A 22. 18 29. A 36. D 43. 40000 50. C
2. 920 9. 100 16. C 23. B 30. A 37. 4 44. B 51. C
3. B 10. D 17. C 24. C 31. 50 38. D 45. C 52. B
4. 37.5 11. C 18. A 25. A 32. D 39. 12.5 46. D 53. C
5. A 12. 20 19. D 26. 36000 33. D 40. B 47. B 54. B
6. 36000 13. C 20. C 27. D 34. D 41. 32000 48. A 55. B
7. C 14. 10 21. C 28. 14400 35. B 42. 15000 49. C

Triumphant Institute of Management Education Pvt. Ltd. (T.I.M.E.) HO: 95B, 2nd Floor, Siddamsetty Complex, Secunderabad – 500 003.
Tel : 040–27898195 Fax : 040–27847334 email : info@time4education.com website : www.time4education.com SM1001906/40
CHAPTER – 4
SIMPLE INTEREST AND COMPOUND INTEREST
INTEREST COMPOUND INTEREST
Interest is money paid to the lender by the borrower for Under Compound Interest, the interest is added to the
using his money for a specified period of time. Various principal at the end of each period to arrive at the new
terms and their general representation are as follows: principal for the next period.

(a) INTEREST In other words, the amount at the end of first year
Money paid by borrower for using the lender’s (or period) will become the principal for the second year
money. Denoted by I. (or period); the amount at the end of second year
(b) PRINCIPAL (or period) becomes the principal for the third year
The original sum borrowed. Denoted by P. (or period) and so on.

(c) TIME If P denotes the principal at the beginning of Period 1,


Time for which money is borrowed. Denoted by n. then, principal at the beginning of Period 2
(n is expressed in number of periods, which is  r 
normally one year) = P1 + 
 100 
(d) RATE OF INTEREST = PR = Amount at the end of Period 1, where
Rate at which interest is calculated on the original
sum. Denoted by r and is expressed as a   r 
R = 1 +  
percentage or decimal fraction.   100 
(e) AMOUNT P at the beginning of Period 3
Sum of Principal and Interest. Denoted by A. 2
 r 
= P1 + 
SIMPLE INTEREST  100 
= PR2 = Amount at the end of Period 2
When interest is calculated every year (or every time P at the beginning of Period (n + 1)
n
period) on the original principal, i.e., the sum at the  r 
beginning of first year, such interest is called Simple = P1 +  = PRn
Interest.  100 
= Amount at the end of Period n
Here, year after year, even though the interest gets
accumulated and is due to the lender, this accumulated Hence the amount after n years (periods)
interest is not taken into account for the purpose of = PRn = A
calculating interest for latter years. Interest = I = A – P = P [Rn – 1]

Pnr The following table gives an example of how simple


Simple Interest =
100 interest and compound interest operate, i.e., how the
where P, n, r are as explained above. Principal is for various years under simple interest and
Pnr compound interest. A principal at the beginning of
Total Amount A = P + 1st year, of `100 and a rate of 10% p.a. are considered.
100 The details are worked out for three years and shown
 nr  below.
= P1 + 
 100 

(All figures pertaining to Principal, interest and amount are in Rupees)

Under Simple Interest Under Compound Interest


Principal at Interest Interest till Amount at Principal at Interest Interest till Amount at
Year the beginn. for the the end of the end of the beginn. for the the end of the end of
of the year year the year the year of the year year the year the year
1 100 10 10 110 100 10 10 110
2 100 10 20 120 110 11 21 121
3 100 10 30 130 121 12.1 33.1 133.1

As can be seen from the table,

In case of Simple Interest, In case of Compound Interest,


- The amount at the end of an year is the Principal for
- The principal remains the same every year the next year
- The interest for any year is the same as that for any - The interest for different years is not the same
other year.

Triumphant Institute of Management Education Pvt. Ltd. (T.I.M.E.) HO: 95B, 2nd Floor, Siddamsetty Complex, Secunderabad – 500 003.
Tel : 040–27898195 Fax : 040–27847334 email : info@time4education.com website : www.time4education.com SM1001906/41
The compound interest for the first year (where
compounding is done every year) is the same as the If the number of times compounding is done
simple interest for one year. in a year is increased to infinity, we say that
the compounding is done EVERY MOMENT
and then the amount is given by P.enr/100,
COMPOUNDING MORE THAN ONCE where r is the rate% p.a. and n is the number
A YEAR of years
We just looked at calculating the amount and interest
when the compounding is done once a year. But,
The following points should also be noted which are
compounding can also be done more frequently than once
helpful in solving problems.
a year. For example, the interest can be added to the
principal every six months or every four months and so on.
The difference between the Compound
If the interest is added to the principal every six months, Interest and Simple Interest on a certain sum
we say that compounding is done twice a year. If the for two years is equal to the interest
interest is added to the principal every four months, we calculated for one year on one year's Simple
say that compounding is done thrice a year. If the Interest.
interest is added to the principal every three months, we
say that compounding is done four times a year.
In mathematical terms, the difference between
The formula that we discussed above for calculating the Compound Interest and Simple Interest for two years will
amount will essentially be the same, be equal to P(r/100)2, which can be written as P (r/100)
 r 
n (r/100). In this Pr/100 is the simple interest for one year
i.e., Amount = P1 +  and when this is multiplied by r/100 again, it gives
 100 
interest for one year on Pr/100 i.e., interest for one year
where r = rate % per annum and n = number of years, on one year's simple interest.
but the rate will not be for ONE YEAR but for the time
period over which compounding is done and the power
to which the term inside the bracket is raised (n in the The difference between the Compound Interest
above case) will not be the number of years but the for k years and the Compound Interest for
number of years multiplied by the number of times (k + 1) years is the interest for one year on the
compounding is done per year (this product is referred to amount at the end of kth year.
as the total number of time periods). This can also be expressed in terms of the
amount as follows:
For example, if a sum of `10000 is lent at the rate of The difference between the amount for k years
and the amount for (k + 1) years under
10% per annum and the compounding is done for every
four months (thrice a year), then the amount will be compound interest is the interest for one year on
equal to the amount at the end of the kth year.
2 x3
 10 1  
10000 1 + x 
 3 100 
 The difference between the Compound Interest
for the kth year and the Compound Interest for
the (k + 1)th year is equal to the interest for one
Here, the dividing factor of 3 in the rate and the
year on the compound interest for the kth year.
multiplying factor of 3 in the power (multiplying the
number of years) - both shown by arrow marks - are
nothing but the NUMBER OF TIMES compounding is
done in a year. PRESENT VALUE
Consider a given sum P and a rate of interest r.
If compounding is done k times a year (i.e., once
every 12/k months), at the rate of r% p.a. then in We have seen that interest is cost of using the money
n years, the principal of P will amount to over a period of time. That means a sum at the
kn beginning of a period is always higher than the same
 r 
= P1 + 
k ⋅ 100 
amount after a period greater than or equal to 1.

Let the sum P that is being considered at a rate of
interest r% p.a., becomes Y at the end of Year 1 and Z at
When compounding is done more than once a year, the the end of Year 2 (i.e., Y and Z are the amounts at the end of
rate of interest given in the problem is called NOMINAL first and second years respectively on a principal of P).
RATE OF INTEREST.
Then we can say that what is P today is equal to Y at the
We can also calculate a rate of interest which will yield end of one year and equal to Z at the end of the second
simple interest in one year equal to the interest obtained year. In other words, if an amount of Y were to come at
under the compound interest at the given nominal rate of the end of one year from now, its value today is equal to P.
interest. The rate of interest so calculated is called Similarly, if an amount of Z were to come at the end of
EFFECTIVE RATE OF INTEREST. two years from now, its value today is equal to P.

Triumphant Institute of Management Education Pvt. Ltd. (T.I.M.E.) HO: 95B, 2nd Floor, Siddamsetty Complex, Secunderabad – 500 003.
Tel : 040–27898195 Fax : 040–27847334 email : info@time4education.com website : www.time4education.com SM1001906/42
So, P is the PRESENT VALUE of Y coming at the end of REPAYMENT IN EQUAL INSTALMENTS –
one year and P is the PRESENT VALUE of Z coming at
COMPOUND INTEREST
the end of two years.
If a sum P borrowed, is repaid in n equal instalments
Similarly, if we consider n years (or n periods in general),
compound interest being calculated at r% per period of
and X is the amount that P will become in n periods, then
instalment, we can find out the value of each instalment.
we say that P is the PRESENT VALUE of X coming at
Let us consider the case of n equal ANNUAL instalments
the end of n periods.
(Even if the instalments are not annual, but monthly, the
approach will remain the same except that the rate of
If we consider a series of payments Y1 at the end of
interest taken should then be the rate per month and not
first year, Y2 at the end of second year and so on, the
rate per annum).
present value of the series of payments will then be
equal to the sum of the present values of each of the
Let each instalment (i.e., the amount paid at the end of
payments calculated separately. If Z1 is the present
each year) be X.
value of Y1, Z2 is the present value of Y2 and so on, then
the present value of the series of payments Y1, Y2, ... is
Instalment X paid after year 1 gives a present value of
equal to Z1 + Z2 + ....
X
Present Value can be looked at both under Simple (
1 + r / 100
.
)
Interest and Compound Interest.
Instalment X, paid at the end of year 2 gives a present
If an amount of Y whose present value is P1 comes at X
( )
the end of Year 1, and an amount of Z whose present value of 2
value is P2 comes at the end of Year 2, then the present 1 + / 100
r

value of both the amounts together will be equal to (P1 + P2), Similarly, instalment X paid for nth period (at the end of
i.e., the present value of the stream of payments that
X
come at different points of time is equal to the sum of the
( )
year n) gives a present value of n
.
present values of the individual amounts coming in at 1 + / 100
r

various points of time.


The sum of all these present values would be equal to
Present Value Under Simple Interest the loan amount P (because only if the amount borrowed
is equal to the amount repaid can we say that the loan is
The principal P is amounting to X in n periods. From this repaid).
we know that
X X X

X = P 1 +

nr 
100  ⇒ P = 

X
nr 
(
1 + / 100
r
+
) (
1 + / 100
r 2
+ …… +
1 + / 100
r
)
n
=P
( )
1 + 
 100  1 100
Hence, in general, the present value P of an amount X
Call
(1 + r
/ 100 ) =k ⇒k=
100 + r
coming (or due) after n periods is given by
The above equation can then be rewritten as
X X {k + k2 + ......... + kn} = P
P=
 nr 
1 +  The terms within the brackets form a G.P with first term
 100  k and common ratio k.
k (k n - 1)
The sum of this G.P. = ;
where r is the rate percent per time period. (k - 1)
X • k (k n - 1) P (k − 1)
Present Value Under Compound Interest Thus
(k - 1)
= P⇒X=
k kn − 1 ( )
The principal P is amounting to X in n periods. From this
we know that
=
[P{ 100
} ]
/ (100 + r ) − 1

[{ }] { } − 1
n
 r  X
/ (100 + r )  100 / (100 + r )
n
X = P 1 +  ⇒ P = n
100
 100   r  
1 + 
 100  P•r
=
{ }
Hence, in general, the present value P of an amount
100 1 − / (100 + r ) 
100 n
X coming (or due) after n periods is given by
 
X
P= n
 r  P•r
1 +  Each Instalment =
 100  100 1 −

{
100
/ (100+r ) 
n


}
where r is the rate percent per time period.

Triumphant Institute of Management Education Pvt. Ltd. (T.I.M.E.) HO: 95B, 2nd Floor, Siddamsetty Complex, Secunderabad – 500 003.
Tel : 040–27898195 Fax : 040–27847334 email : info@time4education.com website : www.time4education.com SM1001906/43
Examples  R 
4
⇒ 1+  = 3 –––––– (1)
4.01. Find the simple interest on a sum of `1000 at  100 
10% p.a. for 4 years. Let it take K years to become 9 times.
K K
 R   R 
PNR P 1+  = 9P ⇒ 1+  =9
Sol: Simple interest =  100   100 
100
(1000 )(4 )(10 )
K

Interest = = ` 400  R  4
4
100 ⇒ 1+   = 32
 100  
4.02. A sum of `4000 becomes `4500 in 2 years from (1),
under simple interest. In how many years will K
`5000 become `5625 under simple interest at K
3 4 = 32 ⇒ =2
the same rate of interest? 4
∴K=8
Sol: Let the rate of interest be R% p.a.
Interest on `4000 = `500 4.07. If `4000 is lent at 10%p.a, interest being
 R  compounded annually, find the interest for the
500 = (4000 )   (2) fourth year.
 100 
R = 6⋅25 Sol: Interest for the fourth year = Amount at the end
Interest on `5000 = `625 of the first 4 years − Amount at the end of the
Let the required time be T years. first 3 years
4 3
 6 ⋅ 25   10   10 
625 = (5000 )   T ⇒ T = 2 = 4000 1 +  − 40001 + 
 100   100   100 
= 4000 (1.4641 – 1.3310)
4.03. Find the value that `1000 would amount to = 4000 (0.1331) i.e. ` 532.40
under compound interest at 20% p.a., interest
being compounded annually in 3 years. 4.08. Find the value that `8000 will amount to in
2 years at 20% p.a., interest being compounded
 R 
N half yearly.
Sol: Amount = P 1+ 
 100  Sol: Rate of interest = 10% per half year
3 Number of time periods (i.e. half years) = 4
 20 
= 1000 1 +  = `1728  10 
4
 100  Amount = 8000 1 +  = 8000 (1.4641) i.e.
 100 
4.04. Find the sum that would amount to `6600 under `11712.80
simple interest in 4 years at 8% p.a.
4.09. The interest on a sum is compounded every
Sol: Let the sum be `P. 3 months. If the rate of interest is 40% p.a., find
the effective rate of interest per annum.
  8 
Given that P1 + 4   = 6600
  100   Sol: Let the sum be `100
P = 5000 Amount at the end of a year
4
 40 
= 100 1 +  = `146⋅41
4 (100 ) 
4.05. If a sum triples in 4 years under simple interest,
find the time that it would take to become 
5 times itself at the same rate of interest. ∴ effective rate of interest = 46⋅41%

Sol: If the sum triples, the interest obtained will be 4.10. A sum amounts to `28800 in two years and to
twice the sum. This takes 4 years. If the sum `34560 in three years under compound interest,
becomes 5 times, the interest must be interest being compounded annually. Find the
four times the sum. sum and the rate of interest.
∴ This takes a total of 8 years. Sol: Let the sum be `P and the rate of interest be
R% p.a.
4.06. A sum triples in 4 years under compound 2
interest at a certain rate of interest, interest  R 
P1 +  = 28800 ––––––––– (1)
being compounded annually. Find the time it  100 
would take to become 9 times itself. 3
 R 
Sol: The sum triples in 4 years. If it becomes P1 +  = 34560 ––––––––– (2)
 100 
9 times itself, it has tripled twice.
Dividing (2) by (1),
∴ This takes 8 years.
Let the sum of `P, triple in 4 years at R% p.a. R
1+ = 1 ⋅ 2 ⇒ R = 20
4 100
 R 
⇒ P 1+  = 3P Substituting R = 20 in (1) or (2),
 100  P = 20000
Triumphant Institute of Management Education Pvt. Ltd. (T.I.M.E.) HO: 95B, 2nd Floor, Siddamsetty Complex, Secunderabad – 500 003.
Tel : 040–27898195 Fax : 040–27847334 email : info@time4education.com website : www.time4education.com SM1001906/44
4.11. The compound interest and the simple interest 4.13. Sanjay borrowed `15000 at 20% p.a. under
on a sum at certain rate of interest for 2 years compound interest, interest being compound
are `2760 and `2400 respectively. Find the sum annually. He repaid `12000 at the end of the
and the rate of interest. first year. Find the amount he must repay at the
end of the second year to clear the loan.
Sol: Let the sum be `P and let the rate of interest be
R% p.a. Sol: Value of `15000 at the end of the first year
Difference between the compound interest and = `18000
the simple interest = `360 As Sanjay repaid `12000, he has to repay the
 R 
2 value of the balance of `6000 at the end of the
∴ P  = 360 ––––––––– (1) second year. He has to repay 6000 × 1.2
 100  i.e., `7200.
 R  PR
P ( 2)   = 2400 ⇒ = 1200 ––––– (2)
 100  100 4.14. Praveen borrowed `26400 at 20% p.a. under
compound interest, interest being compounded
PR  R   R  annually. If he has to repay this in two equal
∴   = 1200   = 360
100  100   100  annual instalments, find the value of each
R = 30 instalment.
Substituting R in (1) or (2),
P = 4000 Sol: Let each instalment be `x
Value of `26400 at the end of the first year
4.12. The compound interest on a certain sum for the = 26400 × 1.2 i.e. `31680
2nd year and the 3rd year are `3300 and `3630 As Praveen repaid `x at the end of the first
respectively. Find the sum and the rate of year, he has to repay the value of the balance
interest. of `(31680 − x) at the end of the second year.
⇒ 1⋅2 (31680 − x) = x
Sol: Let the sum be `P ∴ x = 17280
Let the rate of interest be R% p.a.
Interest for the 2nd year 4.15. Find the sum of the present values of the
 R 
2
 R  payments received at 10% p.a. under
= P1 +  − P1 +  = 3300 compound interest, interest being compounded
 100   100  annually, if `7700 and `7260 are received at the
 R  R  end of the first year and second year
P  1 +  = 3300 ––––––––– (1) respectively.
 100   100 
Interest for the 3rd year Sol: Sum of the present values of the payments
3 2
 R   R  received
= P1 +  − P1 +  = 3630
 100   100  7700 7260
= + 2
= `13000
10  10 
 R 
2
R  1+
P  1 + 100  = 3630 ––––––––– (2) 100 1 + 100 
 
 100  
Dividing (2) by (1),
R
1+ = 1 ⋅ 1 ⇒ R = 10
100
Substituting R in (1) or (2),
P = 30000.

Triumphant Institute of Management Education Pvt. Ltd. (T.I.M.E.) HO: 95B, 2nd Floor, Siddamsetty Complex, Secunderabad – 500 003.
Tel : 040–27898195 Fax : 040–27847334 email : info@time4education.com website : www.time4education.com SM1001906/45
Concept Review Questions
Directions for questions 1 to 25: For the Multiple Choice Questions, select the correct alternative from the given
choices. For the Non-Multiple Choice Questions, write your answer in the box provided.

1. `5000 is invested for two years under simple interest 12. Bhavan invested two equal amounts at simple
at 10% p.a. Find the interest earned (in `) interest. He invested one amount at 15% p.a. and
(A) 1000 (B) 2000 the other at 20% p.a. The total interest earned at the
(C) 500 (D) 1050 end of a year was `1400. Find the total amount
invested (in `).
2. The simple interest for the second year on a certain
sum at a certain rate of interest is `1000. Find the
sum of the interests accrued on it for the 6th, 7th and
8th years. (in `) 13. An amount of `2400 is due after six years under
simple interest at 10% p.a. Find its present value (in `)
(A) 2000 (B) 1600
(C) 1800 (D) 1500
3. What sum will yield an interest of `306 in six years
at 5% p.a. simple interest? 14. If `3000 amounts to `3630 in two years under
(A) `840 (B) `765 compound interest, interest being compounded
(C) `1,210 (D) `1,020 annually, what is the annual rate of interest?
(A) 10% (B) 21%
4. What sum will fetch a simple interest of `25,410 in (C) 11% (D) 10.5%
five and a half years at 6% p.a. rate of interest?
(A) `55,000 15. `5000 is invested for two years under compound
(B) `88,000 interest at 10% p.a, interest being compounded
(C) `77,000 annually. Find the interest earned (in `).
(D) `70,000

5. In how many months will a sum of `15,000 at 5% p.a.


rate of simple interest yield an interest of `4,000? 16. A sum under compound interest, interest being
compounded annually amounts to `6000 in two years
and `7200 in three years. Find the rate of interest.
(A) 10% p.a. (B) 20% p.a.
6. A sum yields `54 more interest when lent at 37% (C) 18% p.a (D) 15% p.a
p.a. simple interest than when lent at 33% p.a. simple
interest in six years. Find the sum. 17. The compound interest on a sum for the third year is
(A) `225 (B) `250 `2420, interest being compounded annually.
(C) `450 (D) `255 The interest on it for the fourth year is `2662.
Find the rate of interest.
7. If `3000 amounts to `3600 in two years under % p.a
simple interest. What is the rate of interest?
(A) 20% p.a (B) 10% p.a
(C) 5% p.a (D) None of these
18. A sum of money becomes four times itself in eight
years at compound interest. In how many years will
8. Find the amount obtained by investing `24,000 at
the same sum become sixteen times itself?
18% p.a. simple interest for five years (in `)
(A) 64 (B) 32
(C) 44 (D) 16

19. A sum becomes 2.197 times of itself in three years at


9. In how many years will a sum of money become compound interest. Find the annual rate of interest.
sixteen times itself at 30% p.a. simple interest?
%.
(A) 25 (B) 40 (C) 30 (D) 50

10. A sum becomes 84% more in seven years when


20. A sum becomes 33.1% more in three years when
invested at simple interest. Find the annual rate of
invested under compound interest, interest being
interest.
compounded annually. Find the annual rate of interest.
(A) 28% (B) 24%
(A) 10.5% (B) 11.03%
(C) 12% (D) 18%
(C) 10% (D) 16.55%
11. A sum of money becomes ten times itself at simple
21. Find the interest (in `) earned in the first year on
interest. If the time period (in years) is numerically equal
`200 at 20% p.a. compound interest, interest
to the rate of interest, find the annual rate of interest.
(A) 25% (B) 20% compounded every six months.
(C) 30% (D) 90%

Triumphant Institute of Management Education Pvt. Ltd. (T.I.M.E.) HO: 95B, 2nd Floor, Siddamsetty Complex, Secunderabad – 500 003.
Tel : 040–27898195 Fax : 040–27847334 email : info@time4education.com website : www.time4education.com SM1001906/46
22. Which of the following schemes of computing 24. Find the effective rate of interest if the rate of interest is
interest yields the maximum interest for a year? 40% p.a., and the interest is compounded quarterly.
(A) Interest compounded monthly at 2% per month. (A) 42% p.a (B) 40% p.a.
(B) Interest compounded per quarter 6% per quarter. (C) 44% p.a (D) 46.41% p.a.
(C) Interest compounded every six months at 12%
for every 6 months. 25. The interests earned on a sum under a certain rate
(D) Interest compounded every year at 24% p.a. of compound interest, interest being compounded
annually, for the xth year and the (x + 1)th year differ
23. If the interest on a sum is compounded quarterly, by `Ι1. The difference in the interests earned on it for
which of the following is necessarily true regarding the yth year and the (y + 1)th year differ by `Ι2.
the effective rate of interest per annum? If Ι1 > Ι2, which of the following can be concluded?
(A) It is the same for each year. (A) x > y
(B) It quadruples every year. (B) x < y
(C) Neither (A) nor (B) (C) Neither (A) nor (B)
(D) Both (A) and (B)

Triumphant Institute of Management Education Pvt. Ltd. (T.I.M.E.) HO: 95B, 2nd Floor, Siddamsetty Complex, Secunderabad – 500 003.
Tel : 040–27898195 Fax : 040–27847334 email : info@time4education.com website : www.time4education.com SM1001906/47
Exercise – 4(a)
Directions for questions 1 to 25: For the Multiple Choice Questions, select the correct alternative from the given
choices. For the Non-Multiple Choice Questions, write your answer in the box provided.

1. What is the principal, if after five years at 11% p.a. 9. Prasad invests `34000, part of it in scheme A at
simple interest, it amounts to `18600? (in `) 20% p.a. and the rest of it in scheme B at 60% p.a.,
both at compound interest. If, after two years, the
amount in scheme A is nine times that in scheme B,
how much was invested in scheme B? (in `)
2. Given that a principal amounts to `10080 at 10% p.a
simple interest after two years what is the compound
interest for two years on this principal at the same
rate? 10. A sum was lent for a year, another sum was lent for
(A) `10164 (B) `1764 2 years and another sum was lent for 3 years. Each
(C) `1640 (D) `3764 sum was lent at 5% p.a compound interest. If each
sum amounted to the same value, the ratio of the
3. Prashanth borrows a sum at compound interest and first, second and third sums is _____.
it amounts to `1,38,240 at the end of three years (A) 400 : 420 : 441 (B) 20 : 21 : 22
and to `1,99,065.60 at the end of five years. What is (C) 22 : 21 : 20 (D) 441 : 420 : 400
the sum borrowed?
(A) `76,000 (B) `80,000 11. Suhaas borrowed a certain amount at 28% p.a.
(C) `90,000 (D) `96,000 compound interest and repays it in one year. Bhanu
borrows a certain amount at a certain interest rate
4. `10000 was lent at compound interest, interest under simple interest and returns it after four years.
being compounded annually for 3 years. The annual If the amounts repaid by Suhaas and Bhanu are the
rates of interest for the first, second and third years same and that is equal to `38400 and the sum of
were 10%, 20% and 30% respectively. If it was their principals borrowed is `54000, at what rate
instead, lent at 20% p.a. simple interest for the same Bhanu paid the interest?
time, how much more / less interest would be (A) 10 % p.a (B) 12% p.a
realized? (C) 15% p.a (D) 18% p.a
(A) ` 530 more (B) `530 less
(C) `1160 more (D) `1160 less 12. Puneet borrows `32000 from Jalpesh at 10% simple
interest. He lends it to Govinda at 20% compound
5. The compound interest on a certain sum for the third interest. How much more would Puneet have
and the fourth years is `1815 and `1996.50 gained, if Govinda had returned the amount in three
respectively. What is the annual rate of interest? years instead of two years?
(A) 5% (B) 8% (C) 10% (D) 12% (A) `5016 (B) `7016 (C) `6016 (D) `4016

6. Prakash invested a certain amount in a six-year 13. The difference between the amounts to be repaid by
fixed deposit scheme, interest being compounded a man at the end of two years, at 20% per annum
annually. The interests accrued on this deposit for compounded annually and half yearly, is `1084.50.
the fourth and fifth years, respectively, are `1331 What is the principal? (in `)
are `1464.10. If George deposited `12000 in the
same scheme, how much interest would be accrued
on this deposit for the first two years?
(A) `1320 (B) `1452 (C) `2520 (D) `1440 14. Venkat lends a sum P at r% compound interest,
compounded every moment for ten years. It
7. Mr. Patel borrows `10000 at 20% p.a. for five years becomes “a” times P after ten years. What is the
at simple interest. But, from the fourth year onwards, annual interest rate? (Assume that a = e2)
on the entire amount due at the end of three years, (A) 100% (B) 20% (C) 40% (D) 200%
the lender begins to charge 20% p.a interest
compounded annually. What is the amount repaid 15. `2.4 crores invested at a certain rate of compound
by Mr. Patel after five years from the beginning? interest becomes `19.2 crores in 18 years. The
(A) `15342 (B) `18432 value that `4 crores invested at the same rate of
(C) `17324 (D) `23040 compound interest would amount to in 24 years is
(in ` crores) .
8. Krishna takes a loan of `8000 at simple interest.
After four years he takes an additional loan of
`14440. From that point, compound interest at
10% per annum is calculated on the total amount 16. The compound interest and the simple interest on a
repayable on the first loan as well as the second certain sum for two years are `1230 and `1200
loan. He repays a total of `30250 after two more respectively. The rate of interest is same for both
years to clear the entire loan amount. What is the compound interest and simple interest. What is the
annual rate of simple interest? principal?
% (A) `10000 (B) `12000
(C) `12500 (D) `16000
Triumphant Institute of Management Education Pvt. Ltd. (T.I.M.E.) HO: 95B, 2nd Floor, Siddamsetty Complex, Secunderabad – 500 003.
Tel : 040–27898195 Fax : 040–27847334 email : info@time4education.com website : www.time4education.com SM1001906/48
17. The difference between the compound and simple 22. A man took a loan of `100000 at 8% per annum
interests on a sum for three years at 20% p.a. is compound interest. He repays `10000 per annum.
`3200. Find the sum. What is the amount due from him at the beginning of
(A) `5000 (B) `10000 (C) `15000 (D) `25000 the third year?
(A) `91917 (B) `81917
18. Sourabh borrows `2500000 at 12% compound (C) `93970 (D) `95840
interest from a bank and invests in shares.
The investment gives him a return of 20% per annum 23. The difference between the compound interests on
and he repays `500000 at the end of first year. How a certain sum for the second and the third years at
much does he make for himself after paying all the 5% p.a. is `42. Find the sum (in `).
outstanding amount at the end of the second year?
(A) `424000 (B) `356241.50
(C) `525000 (D) `484241.80
24. Srikanth buys a car worth `525000. He pays
19. Karthik borrows `24000 from a bank at 12% p.a
`125000 as down payment and agrees to pay the
simple interest. He repays 20% of the principal at the
remaining amount in instalments. What is the
end of every year. What is the simple interest that has
approximate yearly instalment amount to be paid by
accrued for the first four years? (in `)
him, if at 12% p.a. compounded annually, he repays
the remaining amount in three more years?
Given that (1/1.12)3 = 0.71
(A) `160000 (B) `165000
20. What annual instalment will discharge a debt of (C) `180000 (D) `183000
`1450 due after five years at 8% p.a simple interest?
(A) `320 (B) `450 25. A sum was borrowed at 20% p.a. compound
(C) `250 (D) `400 interest. It was repaid in three annual instalments
with each instalment being paid at the end of an
21. What is the present worth of `20000 due after year. The first, second and third instalments were
three years at 10% p.a. compound interest? (Given `1200, `1152 and `2592 respectively. Find the sum
(1/1.1) = 0.90) borrowed. (in `)
(A) `13970 (B) `14120
(C) `14580 (D) `15730

Exercise – 4(b)
Directions for questions 1 to 40: For the Multiple Choice Questions, select the correct alternative from the given
choices. For the Non-Multiple Choice Questions, write your answer in the box provided.

1. A sum of money invested at simple interest amounts to 5. A sum was invested at simple interest. At the end of
`2832 at the end of three years and `3120 at the end of four years, the total interest was equal to the sum.
five years. Find the principal. At the end of five years the total interest was
`12500. Find the interest on the sum at the end of
(A) `2400 (B) `2000
three years (in `).
(C) `3500 (D) `2500
(A) 6000 (B) 7500 (C) 9000 (D) 4500

2. If the annual rate of simple interest at which a sum is 6. Rohan lent a total of `1,00,000 to two people.
lent for two years increases by 10 percentage He lent a certain amount to P at 10% p.a. simple
points, the interest realized would be `4000 more. interest and lent the remaining amount to Q at
Find the sum (in `). 20% p.a. simple interest. He lent each amount for
two years and received `32000 as the total interest.
Find the sum he lent to P (in `).

3. If a sum was `10000 more it would fetch `4000


extra as simple interest, if it was lent at a certain rate 7. In question 6 find the effective annual rate of simple
of interest for two years. Find the annual rate of interest at which he lent the total sum.
interest. (A) 12% (B) 14% (C) 16% (D) 18%
(A) 5% (B) 10%
(C) 20% (D) 25% 8. Rajesh, a money lender, lends in the following
manner. For the first year, he charges 2% p.a., for
4. If a sum invested at simple interest doubles itself in the second year, he charges 4% p.a for the
third year, he charges 6% p.a. and so on. If he lends
ten years, how long will it take to become five times
a sum in this way at simple interest, find the least
itself? integral number of years in which it will fetch an
(A) 20 (B) 40 interest at least equal to itself.
(C) 25 (D) 30 (A) 9 (B) 10 (C) 11 (D) 12

Triumphant Institute of Management Education Pvt. Ltd. (T.I.M.E.) HO: 95B, 2nd Floor, Siddamsetty Complex, Secunderabad – 500 003.
Tel : 040–27898195 Fax : 040–27847334 email : info@time4education.com website : www.time4education.com SM1001906/49
9. A sum was split into four parts (P1, P2, P3, P4) where 19. A sum when lent at a certain rate compounded
P1 : P2 : P3 : P4 = 1 : 4 : 5 : 2. Each part was lent at annually amounts to `1200 in two years and when
simple interest. P1 was lent at 10% p.a. for a year. lent at twice the previous rate amounts to `1323 in
P2 was lent at 20% p.a. for 5 years. P3 was lent at the same time. What is the rate at which it was
2% p.a. for 4 years. P4 was lent at 6% p.a. for initially lent?
10 years. The greatest of the interests on the parts 3
exceeds the least of the interests on the parts by (A) 4% p.a (B) 4 % p.a
7
`7800. Find the total interest fetched by the parts
(in `). 5 1
(C) 5 % p.a (D) 6 p.a
19 4%

10. Ajay borrowed `20,000 from Balu at 10% p.a. simple 20. Akshay deposits a sum P in a bank at r% compound
interest and twice that amount from Chetan at interest. The amount becomes 27P after three years
20% p.a. simple interest. He then added an amount by compounding annually. Instead, if the bank had
of his own and lent the entire sum at 18% p.a. compounded half yearly, what is the additional
simple interest. All the deals were for a year. At the amount Akshay would have received in terms of P?
end of the year, he earned `4400. Find the amount (A) 54P (B) 37P
that he added. (C) 18P (D) 5P
(A) 15000 (B) 26000 (C) 24000 (D) 20000
21. The excess interest (in `) realised when `10000 is
11. Find the present value of `17280 due 3 years from lent at compound interest at 20% p.a for 3 years
now at 20% p.a. simple interest (in `). than when it is lent at 10% p.a, 20%p.a, 30%p.a for
the first, second and third years respectively, is

12. A sum amounts to `266,200 in three years and


22. If a sum is lent at 20% p.a. compound interest,
`292,820 in four years under compound interest,
interest being compounded annually, for a year it
interest being compounded annually. Find the
would fetch an interest which is `400 less than it
annual rate of interest.
would fetch if it is lent at the same rate of interest,
(A) 8% (B) 10% (C) 12% (D) 15%
interest being compounded half yearly. Find the sum
13. A sum was invested under compound interest, (in `).
interest being compounded annually. It fetches (A) 48000 (B) 40000
`14400 as interest in the second year and `17280 (C) 36000 (D) 50000
as interest in the third year. Find the annual rate of
interest. 23. A man invested `40,000 in a bond which gives
(A) 10% (B) 15% (C) 20% (D) 25% 10% p.a. interest, compounded half yearly. If the
annual rate of interest is increased by 20 percentage
14. The interest on a sum under compound interest points at the end of every half year, what will be the
when interest is compounded annually for the sixth interest for the first one and half years?
year and the seventh year are `1,256 and `1,413 (A) `15875 (B) `16750
respectively. Find the rate of interest. (C) `20375 (D) `19875
(A) 10% p.a (B) 15% p.a
(C) 71/2% p.a (D) 121/2% p.a 24. A sum becomes `60,000 in five years at 20% p.a.
compound interest, interest being compounded
15. A loan has to be repaid after three years under continuously. Find the sum to the nearest thousand
compound interest, interest being compounded (e = 2.718). (in `)
annually at 30% p.a. Find the amount borrowed if
the amount to be repaid is `87880 (in `).

25. Sunil takes a loan from a financier at 100% p.a.


interest. When he was repaying it after three years,
16. In how many years will a sum of `1875 amount to he had to pay `952000 more because the loan was
`2187 at 8% p.a. compound interest? compounded every moment, instead of annually.
(A) 1 (B) 2 (C) 3 (D) 4 What was the loan taken? [Take e = 2.71 and
(2.71)3 = 19.9]
17. If a sum of `12000 is lent at 5% p.a compound (A) `70000 (B) `75000
interest, what is the interest for the third year? (C) `80000 (D) `85000
(A) `441 (B) `661.50
(C) `531.20 (D) `742.75 26. Srikar saves `20000 at the beginning of each year in
a savings bank account that pays 5% p.a, interest
18. A sum was lent at 20% p.a. compound interest, being compounded annually. If, at the beginning of
interest being compounded annually. The interest the third year, instead of depositing `20000, he
for the first year was `4000. Find the total interest withdraws `10000, how much would be the total
for the second and third years (in `). savings of the man at the end of three years?
(A) `28124.24 (B) `29324.20
(C) `31349.75 (D) `34702.50

Triumphant Institute of Management Education Pvt. Ltd. (T.I.M.E.) HO: 95B, 2nd Floor, Siddamsetty Complex, Secunderabad – 500 003.
Tel : 040–27898195 Fax : 040–27847334 email : info@time4education.com website : www.time4education.com SM1001906/50
27. Anwar borrowed `72000 at 20% p.a. compound 35. A certain sum becomes twice itself in exactly
interest, interest being compounded annually. five years at r% p.a. simple interest. In which year
He repaid `x at the end of the first year. He repaid does the sum amount to twice itself, under
`57600 at the end of the second year and thereby r% p.a compound interest?
cleared the loan. Find x.

36. A sum of money when kept at simple interest


28. Aswin borrowed `3,20,320 at 20% p.a. compound doubled in eight years four months. If the rate of
interest, interest being compounded annually. interest is doubled, in which year the same sum
He planned to repay it in two equal annual becomes twice of itself under compound interest?
instalments. But he actually repaid it in three equal (A) 4th year (B) 5th year
annual instalments. Find the difference between the (C) 3rd year (D) 6th year
value of each instalment he had planned to pay and
each instalment that he actually paid (in `). 37. A sum was divided into two equal parts. One part
(A) 48000 (B) 54000 was lent at 20% p.a. simple interest. The other part
(C) 29952 (D) 57600 was lent at 20% p.a. compound interest, interest
being compounded annually. The difference in the
29. Ashok borrowed a total of `84000 from two banks at interests fetched by the parts in the second year is
compound interest, interest being compounded `400. Find the difference in the interests fetched by
annually. One of the banks charged interest at the parts in the fourth year (in `).
10% p.a. while the other charged interest at 20% p.a. (A) 1414 (B) 1442
If Ashok paid `13200 as the total interest after a (C) 1456 (D) 1484
year, find the difference of the sums he borrowed
(in `). 38. A sum of `4000 was split into two parts in the ratio
1 : 3. The smaller part was lent at 10% p.a. simple
interest for two years. The greater part was lent at
30% p.a. simple interest for two years. If the total
30. A sum takes T1 years to double at R1% p.a. simple sum of `4000 was instead lent at 20% p.a. simple
interest. If it is lent at R2% p.a. compound interest, interest for two years, how much more / less interest
interest being compounded annually, it would take would be realized? (in `)?
the same time to double. Which of the following is (A) 1200 less (B) 1200 more
always true if T1 > 1? (C) 400 less (D) 400 more
(A) R1 > R2 (B) 0.5R2 < R1 < R2
(C) R1 = R2 (D) R 2 3 < R1 < R2 39. In the above question, if all the interests considered
had been compound interest, interest being
compounded annually, how much more/less interest
31. Anil gave a certain sum to Bala. Bala lent 75% of the would be realized? (in `)
sum he received at 10% p.a. compound interest for (A) 1632 more (B) 1632 less
3 years. He received `3993 at the end of 3 years. (C) 2014.40 more (D) None of these
Find the sum that Anil gave Bala (in `).
40. The value of a machine depreciates at the rate of
10% p.a. every year. What was the value of the
machine two years ago, if its present value is
32. The difference between the interest under compound `24,300? (in `)
interest, interest being compounded annually and
simple interest, for two years, for the same sum and
at the same rate of interest is `112.5. Find the sum if
the rate of interest is 15%p.a.
(A) `3500 (B) `5000 Directions for questions 41 to 45: Each question is
(C) `7500 (D) `10000 followed by two statements Ι and ΙΙ. Indicate your
responses based on the following directives:
33. The difference between the simple interest and the
compound interest, interest being compounded Mark (A) if the question can be answered using one
annually, on a sum of `8000 for two years is `320. of the statements alone, but cannot be
Find the annual rate of interest, if it is the same for answered using the other statement alone.
both types of interest. Mark (B) if the question can be answered using
either statement alone.
%
Mark (C) if the question can be answered using
Ι and ΙΙ together but not using Ι or ΙΙ alone
34. A sum takes two years to become 40% more under Mark (D) if the question cannot be answered even
simple interest at a certain rate of interest. If it was using Ι and ΙΙ together.
lent at the same interest rate for the same time
under compound interest, interest being 41. What is the rate of interest per annum?
compounded annually, it would amount to x% more Ι. The interest is `500 for two years.
than itself. Find x. ΙΙ. At twice the rate of interest, the interest for
(A) 36 (B) 48 (C) 40 (D) 44 four years is `2000.

Triumphant Institute of Management Education Pvt. Ltd. (T.I.M.E.) HO: 95B, 2nd Floor, Siddamsetty Complex, Secunderabad – 500 003.
Tel : 040–27898195 Fax : 040–27847334 email : info@time4education.com website : www.time4education.com SM1001906/51
42. What is the rate of simple interest on the deposit? 44. What is the rate of compound interest per annum?
Ι. The interest earned on the deposit for 8 years is Ι. `100 becomes `125 in three years compounded
`3600. annually.
ΙΙ. The amount at the end of 8 years is double the ΙΙ. `x becomes `1.5x in 3 years compounded
principal. annually.

43. A man invests a total of `x on the names of his 45. What is the total compound interest accrued on a
three sons A, B and C in such a way that they get sum of money after five years?
the same amount after 2, 3 and 4 years respectively. Ι. Rate of interest is 6% per annum.
What is the ratio of amounts invested on A, B and C? ΙΙ. The total simple interest on the same amount
Ι. x = 15,860. after 5 years at the same rate will be `600.
ΙΙ. The man invested at the rate of 5%.p.a. simple
interest.

Key
Concept Review Questions

1. A 6. A 11. C 16. B 21. 42


2. 3000 7. A 12. 8000 17. 10 22. A
3. D 8. 45600 13. D 18. D 23. A
4. C 9. D 14. A 19. 30 24. D
5. 64 10. C 15. 1050 20. C 25. A

Exercise – 4(a)

1. 12000 6. C 11. C 16. B 21. C


2. B 7. D 12. C 17. D 22. D
3. B 8. 8 13. 45000 18. A 23. 16000
4. D 9. 2000 14. B 19. 8064 24. B
5. C 10. D 15. 64 20. C 25. 3300

Exercise – 4(b)

1. A 10. D 19. C 28. D 37. C


2. 20000 11. 10800 20. B 29. 12000 38. C
3. C 12. B 21. 120 30. A 39. D
4. B 13. C 22. B 31. 4000 40. 30000
5. B 14. D 23. C 32. B 41. D
6. 40000 15. 40000 24. 22000 33. 20 42. A
7. C 16. B 25. C 34. D 43. A
8. B 17. B 26. D 35. 4 44. B
9. 11400 18. 10560 27. 38400 36. A 45. C

Triumphant Institute of Management Education Pvt. Ltd. (T.I.M.E.) HO: 95B, 2nd Floor, Siddamsetty Complex, Secunderabad – 500 003.
Tel : 040–27898195 Fax : 040–27847334 email : info@time4education.com website : www.time4education.com SM1001906/52
CHAPTER – 5
TIME AND DISTANCE
In this chapter, we will look at problems in the following If a body travels from point A to point B with a speed of p
different areas: and back to point A (from point B) with a speed of q, then
the average speed of the body can be calculated as
– General problems on Time, Speed and Distance 2pq/(p + q). Please note that this does not depend on
– Relative Speed the distance between A and B.
– Boats and Streams
– Races and Circular Tracks If a body covers part of the journey at speed p and the
remaining part of the journey at speed q and the
Before we look at problems in various areas, let us first distances of the two parts of the journey are in the ratio
look at some basic concepts pertaining to speed, time m : n, then the average speed for the entire journey is
and distance. (m + n)pq/(mq + np).

SPEED Examples
Distance covered per unit time is called speed. 5.01. Express 72km/hr in m/sec.
i.e., Speed = Distance/time
 5 
The above relationship between the three variables Sol: 72 km/hr = (72 )   i.e. 20 m/sec
distance, speed and time can also be expressed as  18 
follows:
5.02. A car can cover 90 km in 3 hours. If its speed is
Distance = Speed x Time or increased by 15 kmph, find the time taken by it
Time = Distance/Speed to cover this distance.

– If two bodies travel with the same speed, Distance Sol: Speed of the car = 30 kmph
covered α Time (Direct Variation). If its speed is increased by 15 kmph,
– If two bodies travel for the same period of time, 90
time taken = = 2 hours.
Distance covered α Speed (Direct Variation). 30 + 15
– If two bodies travel the same distance, 5.03. Ashok covered a certain distance at a certain
1 speed. If his speed was 20% more, he would
Time α (Inverse Variation) take 10 minutes less to cover the same
Speed
distance. Find the time he takes to cover the
Distance is normally measured in kilometres, metres or distance.
miles; time in hours or seconds and speed in km/hr (also
denoted by kmph), miles/hr (also denoted by mph) or Sol: If his speed was 20% more, it would be 120%
metres/second (denoted by m/s). 6
i.e., times his actual speed.
5
To convert speed in kmph to m/sec, multiply it with 5/18.
5
∴ Time taken would be times his actual time.
To convert speed in m/sec to kmph, multiply it with 18/5. 6
1
In the case of moving trains, three different situations Reduction in time = (actual time)
6
need to be considered. = 10 minutes
∴Actual time = 60 minutes.
When a train passes a stationary point, the distance
covered (in the passing) is the length of the train. If the 5.04. A car covered a certain distance at 90 kmph
train is crossing a platform (or a bridge), the distance and returned back at 60 kmph. Find its average
covered by the train (in the crossing) is equal to the length speed for the entire journey.
of the train plus the length of the platform (or bridge). If
two trains pass each other (travelling in the same direction Sol: Let x km be the distance to be covered, each
or in opposite directions), the total distance covered (in the way.
crossing or the overtaking, as the case may be) is equal to Total time of travel (in hours)
the sum of the lengths of the two trains. x x 5x x
= + = =
90 60 180 36
AVERAGE SPEED Average speed (in km / hr)
Average speed of a body travelling at different speeds is Total distance travelled 2x
= = = 72
defined as follows: Total time taken x
36
Total distance travelled
Average Speed =
Total time taken
5.05. Find the time taken by a 200 m long train
Please note that the Average speed of a moving body is running at 36 kmph to cross a boy standing on a
Not equal to the Average of the speeds. platform.

Triumphant Institute of Management Education Pvt. Ltd. (T.I.M.E.) HO: 95B, 2nd Floor, Siddamsetty Complex, Secunderabad – 500 003.
Tel : 040–27898195 Fax : 040–27847334 email : info@time4education.com website : www.time4education.com SM1001906/53
 5  5.09. Find the time taken by a train 175 m long
Sol: Speed of the train = (36 )   = 10 m / sec running at a speed of 54 kmph to overtake another
 18 
train 75 m long running at a speed of 36 kmph.
Distance to be travelled by the train to cross the
stationary boy = length of the train Sol: Relative speed = 18 kmph = 5 m/sec
Time taken by the train to cross the boy Time taken for the faster train to overtake the
200 slower train =
= = 20 sec
10 (Length of the faster train) + (Length of the slower train)
5.06. Find the time taken by a train, 100 m long, Their relative speed
travelling at a speed of 63 kmph to cross a 175 + 75
platform of length 250 m. = = 50 seconds
5
 5  35
Sol: Speed of the train = (63 )   = m / sec 5.10. A train overtakes two persons, cyling at 9 kmph
 18  2 and 18 kmph in 40 seconds and 48 seconds
Distance to be travelled by the train to cross the respectively. Find its length and speed.
platform = length of the train + length of the platform.
100 + 250 Sol: Let the length and the speed of the train be ℓ m
Time taken to cross the platform = and s kmph respectively.
35
ℓ = 40 (s − 9 ) = 48 (s − 18 )
2 5 5
= 20 seconds 18 18
s− 9 48
5.07. Find the length of the platform which a train 400 m ⇒ = ⇒ s = 63
long, travelling at 45 kmph can cross in 40 seconds. s − 18 40
5
 5  ∴ ℓ = 40 (63 – 9) × = 600m
Sol: Speed of the train = (45 )  = 12 ⋅ 5 m / sec 18
 18 
Length of the platform = distance travelled by 5.11. Two trains running at 36 kmph and 45 kmph
the train – length of the train cross each other in 20 seconds when they run
= (12⋅5) (40) − 400 = 100 m in the opposite directions. When the trains run
in the same direction, a person in the faster
5.08. Ashok would reach his office 15 minutes early if train observed that he overtook the slower train
he walked at 4 kmph from his house. He would in 48 seconds. Find the lengths of the trains.
reach it 45 minutes late if he walked at 3 kmph
from his house. Find the distance between his Sol: Let the lengths of the faster and the slower
house and office. trains be f and s respectively.
Sol: Let the distance be x km. Time taken by Ashok f +s
Given that, = 20
if he walked at 4 kmph =
x
hours. (36 + 45 ) 5
4 18
Time taken by Ashok if he walked at 3 kmph f + s = 450 ––––––––– (1)
x When the trains run in the same direction, the
= hours. In this case he would take time taken by the person in the faster train to
3
s
one hour more to reach his office compared to overtake the slower train = = 48
the time taken if he had walked at 4 kmph. (45 − 36 ) 5
x x 18
∴ − = 1 ⇒ x = 12.
3 4 s = 120
From (1), f = 330
In general, if a person travelling between two
points reaches p hours late travelling at a 5.12. Two trains have lengths of 300 m and 200 m.
speed of u kmph and reaches q hours early When they run in the same direction, the
travelling at v kmph, the distance between faster train will take 100 seconds to cross the
vu slower train. When they run in the opposite
the two points is given by (p + q) .
v −u directions, they will take 20 seconds to cross
each other. Find the speeds of the trains.

Relative speed Sol: Let the speeds of the trains be x m/sec and
y m/sec where x > y.
The speed of one (moving) body in relation to another
300 + 200
moving body is called the relative speed of these = 100
two bodies, i.e., it is the speed of one moving body as x−y
observed, from the second moving body. 5 = x − y ––––––––– (1)
300 + 200
If two bodies are moving in the same direction, the = 20
relative speed is equal to the difference of the speeds of x+y
the two bodies. 25 = x + y ––––––––– (2)
Solving (1) and (2)
If two bodies are moving in opposite directions, the relative
x = 15 and y = 10
speed is equal to the sum of the speeds of the two bodies.
Triumphant Institute of Management Education Pvt. Ltd. (T.I.M.E.) HO: 95B, 2nd Floor, Siddamsetty Complex, Secunderabad – 500 003.
Tel : 040–27898195 Fax : 040–27847334 email : info@time4education.com website : www.time4education.com SM1001906/54
BOATS AND STREAMS 5.14. Anand can row 20 km in 2 hours in still water. If
the speed of the stream is 6 kmph, he would
Problems related to boats and streams are different in take 3⋅75 hours to cover a round trip journey.
the computation of relative speed from those of Find the distance that he would then cover each
trains/cars. way.

When a boat is moving in the same direction as the 20


Sol: Speed of the boat in still water = = 10 kmph
stream or water current, the boat is said to be moving 2
WITH THE STREAM OR CURRENT. Let the total distance covered be 2x km.
x x
When a boat is moving in a direction opposite to that of Given that, + = 3 ⋅ 75
10 + 6 10 − 6
the stream or water current, it is said to be moving
x = 12
AGAINST THE STREAM OR CURRENT.

If the boat is moving with a certain speed in water that is 5.15. Arun rowed a distance of 36 km in still water in
not moving, the speed of the boat is then called the 3 hours. He rowed 30 km in 2 hours with the
SPEED OF THE BOAT IN STILL WATER. current. Find the time he would take to row
27 km against the current.
When the boat is moving upstream, the speed of the
Sol: Let the speed of his boat in still water and the
water opposes (and hence reduces) the speed of the boat.
speed of the current be x kmph and y kmph
respectively
When the boat is moving downstream, the speed of the
water aids (and thus adds to) the speed of the boat. 36
x= = 12
Thus, we have 3
30
Downstream speed = x + y = = 15
Speed of the boat against stream 2
= Speed of the boat in still water – Speed of ⇒ y = 3 (As x = 12)
the stream 27 27
Speed of the boat with the stream Required time = = = 3 hours
x−y 12 − 3
= Speed of the boat in still water + Speed of
the stream 5.16. In a certain time, a boat can cover either a
5
These two speeds, the speed of the boat against the certain distance upstream or times that
2
stream and the speed of the boat with the stream, are distance downstream. If the speed of the
RELATIVE SPEEDS. current is 6 kmph, find the speed of the boat in
still water.
If u is the speed of the boat down the stream and v is the
speed of the boat up the stream, then we have the Sol: Let the speed of the boat in still water be
following two relationships. x kmph.
x+6 5
Speed of the boat in still water = (u + v)/2 = ⇒ 2x + 12 = 5x – 30
x−6 2
Speed of the water current = (u – v)/2
∴ x = 14
In problems, instead of a boat, it may be a swimmer but 5.17. Balu can row 2 km upstream in 20 minutes and
the approach is exactly the same. Instead of can return in another 15 minutes. Find his
boats/swimmers in water, it could also be a cyclist average speed for the entire journey.
cycling against or along the wind. In some problems it
can be person(s) going up/down an ascending or Sol: Average speed of Balu
descending escalator. The approach to solving the 2+2 48
= = kmph
problems still remains the same. 20 15 7
+
60 60
5.13. A boat travels 30 km upstream in 5 hours and
100 km downstream in 10 hours. Find the 5.18. An escalator is moving downwards. Anil takes
speed of the boat in still water and the speed of 60 steps to reach the bottom of the escalator
the stream. starting from the top. Bala takes 140 steps to
30 reach the top of the escalator starting from the
Sol: Upstream speed = = 6 kmph bottom. The speeds of Anil and Bala are equal.
5 Find the number of steps visible when the
100 escalator is stationary.
Downstream speed = = 10 kmph
10 Sol: Let the number of steps that the escalator
moves for ten steps of Anil or Bala be e.
6 + 10 Let the number of steps visible when the
Speed in still water = = 8 kmph
2 escalator is stationary be N.
N = 60 + 6e = 140 – 14e
10 − 6 20e = 80
Speed of the stream = = 2 kmph
2 e=4
N = 60 + 6e = 84
Triumphant Institute of Management Education Pvt. Ltd. (T.I.M.E.) HO: 95B, 2nd Floor, Siddamsetty Complex, Secunderabad – 500 003.
Tel : 040–27898195 Fax : 040–27847334 email : info@time4education.com website : www.time4education.com SM1001906/55
5.19. A takes 120 steps to reach the bottom of an When TWO people are running around a circular track
escalator which is moving upwards. B takes
Let the two people A and B with respective speeds of a
60 steps to reach the top of the escalator. If
and b (a > b) be running around a circular track
A takes two steps for every step of B, find the
(of length L) starting at the same point and at the same
number of steps visible when the escalator is
time. Then,
stationary.
When the two
Sol: Let us say the number of steps that the When the two
persons are
escalator takes when A takes 120 steps is e. In persons are
running in
the time that A takes 120 steps, B takes running in the
OPPOSITE
60 steps. SAME direction
directions
Let the number of steps on the escalator when it
is stationary be N. Time taken
to meet for L L
N = 120 – e = 60 + e
e = 30 the FIRST (a − b ) (a + b )
N = 90 TIME EVER

RACES AND CIRCULAR TRACKS Time taken


to meet for L L  L L 
When two persons P and Q are running a race, they can the first time LCM of  ,  LCM of  , 
start the race at the same time or one of them may start at the STARTING a b  a b 
a little later than the other. In the second case, suppose POINT
P starts the race and after 5 seconds, Q starts. Then we
say P has a "start" of 5 seconds. Alternatively, in a race Please note that when we have to find out the time taken
between P and Q, P starts first and then when P has by the two persons to meet for the first time at the
covered a distance of 10 metres, Q starts. Then we say starting point, what we have to do is to find out the time
that P has a "start" of 10 metres. taken by each of them to complete one full round and
In a race between P and Q where Q is the winner, by the then take the LCM of these two timings (L/a and L/b are
time Q reaches the winning post, if P still has another the timings taken by the two of them respectively to
15 metres to reach the winning post, then we say that Q complete on full round).
has won the race by 15 metres. Similarly, if P reaches When THREE people are running around a circular track
the winning post 10 seconds after Q reaches it, then we
say that Q has won the race by 10 seconds. Let the three people A, B and C with respective speeds
of a, b and c (a > b > c)) be running around a circular
In problems on RACES, we normally consider a track (of length L) starting at the same point at the same
100 m race or a 1 km race. The length of the track time. In this case we consider the three persons running
NEED NOT necessarily be one of the two figures in the same direction as the general case.
mentioned above but can be as given in the problem.
Time taken to meet  L L 
When two or more persons running around a circular for the FIRST LCM of  , 
track (starting at the same point and at the same time), TIME EVER  ( a − b ) (b − c) 
then we will be interested in two main issues:
Time taken to
- When they will meet for the first time and meet for the first L L L 
- When they will meet for the first time at the starting LCM of  , , 
time at the a b c 
point STARTING POINT
To solve the problems on circular tracks, you should The logic in obtaining the above is as follows:
keep the following points in mind.
A and B will be together with a time gap of L/(a − b);
When two persons are running around a circular track in B and C will be together with a time gap of L/(b − c); for
OPPOSITE directions A, B and C to be together, A and B should be together
- the relative speed is equal to the sum of the speeds as well as B and C should be together. Hence the LCM of
of the two individuals and the two timings L/(a − b) and L/(b − c) will give the time
- from one meeting point to the next meeting point, when A, B and C will all be together.
the two of them TOGETHER cover a distance equal
to the length of the track. When we have to find out the time taken by the
three persons to meet for the first time at the starting
When two persons are running around a circular track in point, what we have to do is to find out the time taken by
the SAME direction each of them to complete one full round and then take
the LCM of these three timings (L/a, L/b and L/c are the
- the relative speed is equal to the difference of the timings taken by the three of them respectively to
speeds of the two individuals and complete one full round).
- from one meeting point to the next meeting point,
the faster person covers one COMPLETE ROUND Even if we are given a case where three persons are
more than the slower person. running around a circular track with two persons running
in the same direction and the third in the opposite
We can now tabulate the time taken by the persons to direction, we can work out the time taken by them to
meet for the first time ever or for the first time at the meet for the first time ever and for the first time at the
starting point in various cases. starting point by extending the above logic.
Triumphant Institute of Management Education Pvt. Ltd. (T.I.M.E.) HO: 95B, 2nd Floor, Siddamsetty Complex, Secunderabad – 500 003.
Tel : 040–27898195 Fax : 040–27847334 email : info@time4education.com website : www.time4education.com SM1001906/56
5.20. In a 200 m race, A beats B by 10 m or 2 seconds. (ii) Time taken to meet for the first time
Find B’s speed and A’s speed. 1800
= 80 seconds
Sol: A beats B by 10 m or 2 seconds. (45 + 36 ) 5
⇒ When A reached the finishing line B was 18
10 m behind the finishing line and took 2 seconds
5.25. In the previous example, find the time after
to cover it.
which they will meet at the starting point for the
10
∴ B’s speed = = 5 m / sec first time.
2
200 m Sol: Time taken to meet at the starting point for the
Time taken by B to complete the race = first time
5m / s
 
= 40 seconds  
∴ Time taken by A to complete the race  1800 1800 
= L.C.M  , 
 (36 ) 5  (45 ) 5  
= 38 seconds
200 100  
A’s speed = = m / sec   18   18  
38 19 = LCM [180, 144] = 720 seconds
5.21. Ramu is 50% faster than Somu. In a race,
Ramu gave Somu a head start of 200 m. Both 5.26. On a circular track of length 1200 m, P, Q and R
finished the race simultaneously. Find the start from the same point simultaneously with
length of the race. speeds of 18 kmph, 27 kmph and 36 kmph
respectively. Find the minimum time after which
Sol: Let the length of the race be x m. they will meet if they are running in the same
x 150 direction.
=
x − 200 100 Sol: If three runners with speeds of a m/sec, b m/sec
⇒ x = 600. and c m/sec respectively start from the same
point in the same direction on a circular track
5.22. In a 1200 m race, Ram beats Shyam by 300 m.
and a > b > c, they will meet for the first time
In the same race, Shyam beats Tarun by 400 m.
after
Find the distance by which Ram beats Tarun.
 L L L 
LCM any two of , ,  seconds
Sol: Let the speeds of Ram, Shyam and Tarun be  a−b b−c a−c
r m/sec, s m/sec, and t m/sec respectively
In the problem given, the required time
r 1200 4
= =  
s 1200 − 300 3 
1200 1200

s 1200 3 = LCM  , 
= =  5 
t 1200 − 400 2  (36 − 27 ) 5
(27 − 18 ) 
 18 18 
r  r  s 
=    = 2 = LCM [480, 480] = 480 seconds
t  s  t 
⇒ By the time Ram covers 1200 m, Tarun CLOCKS
covers 600 m.
The hours and minutes hands of a clock move in relation
∴Ram beats Tarun by (1200 – 600) i.e. by 600 m. to each other continuously and at any given point of time,
5.23. In a 600 m race, P gives Q a start of 200 m. they make an angle between 0° and 180°with each oth er.
Ratio of the speeds of P and Q is 5 : 4. Who
If the time shown by the clock is known, the angle
wins the race? By what distance does the
between the hands can be calculated. Similarly, if the
winner beat the loser?
angle between two hands is known, the time shown by
Sol: Q has to run 400 m to finish the race. In the the clock can be found out.
time Q runs 400 m, P can run 500 m. When we say angle between the hands, we normally
∴ When Q finished the race, P would have refer to the acute/obtuse angles (upto 180°) betwee n the
another 100 m to run. two hands and not the reflex angle (> 180).
∴ Q wins the race and he beats P by 100 m. For solving the problems on clocks, the following points
5.24. On a circular track of length 1800 m, X and Y will be helpful.
start from the same point simultaneously with
speeds of 36 kmph and 45 kmph respectively. Minutes hand covers 360° in 1 hour, i.e., in
Find the minimum time after which they will 60 mins. Hence MINUTES HAND COVERS
meet if they are running in 6° PER MINUTE.
(i) the same direction. Hours hand covers 360° in 12 hours. Hence
(ii) opposite direction to each other. HOURS HAND COVERS 30° PER HOUR or
HOURS HAND COVERS 1/2° PER MINUTE
Sol: (i) Time taken to meet for the first time
1800 All angles are measured in the clockwise direction
= = 720 sec onds starting from the vertical line at 12 o' clock.
(45 − 36 ) 5
18 Note: We can also solve the problems on clocks by
the route of "Relative Speed"
Triumphant Institute of Management Education Pvt. Ltd. (T.I.M.E.) HO: 95B, 2nd Floor, Siddamsetty Complex, Secunderabad – 500 003.
Tel : 040–27898195 Fax : 040–27847334 email : info@time4education.com website : www.time4education.com SM1001906/57
In 1 minute, Minutes hand covers 6° and Hours hand Sol: Let the time be 8:x p.m.
covers 1/2°. Angle between the hands = 180°
Angle effectively covered by the hour hand by
Therefore, Relative Speed = 6 – 1/2
 x
= 5½° per minute. 8:x p.m. =  240 + 
 2 
Alternately, in 1 hour, minutes hand covers 60 minute Angle effectively covered by the minute hand by
divisions whereas hours hand covers 5 minute divisions. 8:x p.m. = 6x°
∴Relative Speed = 60 – 5 = 55 minute divisions per hour. °
 x
However, taking the route of actual angles covered is by  240 +  − 6 x° = 180° (or)
 2 
far the simplest and easy to understand as well as
°
helpful in solving ALL the different models of problems  x
on CLOCKS. Hence, we will look at mainly that method 6 x° −  240 +  = 180°
 2
only and not the relative speed method. This will not
create any confusion. 11
x = 60° or 420
2
The following additional points should also be
120 840
remembered. x= or
11 11
In general, every angle is made TWICE in
840
ONE HOUR by the two hands of the clock. If x = , x > 60 which is not possible.
11
In a period of 12 hours, the hands make an angle of
120 10
- 0° with each other (i.e., they coincide with each ∴x = = 10
other) 11 times and hence the time gap between 11 11
two successive coincidences is 12/11 hours, i.e., 10
∴ The required time is 8 : 10 p.m
11/11 hours, i.e., 655/11 minutes. 11
- 180° with each other (i.e., they lie on the
same straight line) 11 times. 5.30. In 24 hours, how many times do the hands of a
- 90° or any other angle with each other 22 times. clock coincide?
Sol: In 12 hours, the hands of a clock coincide 11 times.
5.27. Find the angle between the hands of a clock In 24 hours, they coincide (11) (2) = 22 times.
when the time is 3:40 p.m.
5.31. If the hands of a clock coincide every 66 minutes,
Sol: From 12:00 p.m. to 3:00 p.m. the minute hand how much time does the clock gain/lose per day?
would effectively cover 3 (0°) = 0°. In the next
40 minutes, it would cover 40 (6°) = 240° Sol: In a clock showing correct time the hands
∴ The minute hand would have covered a total 5
of 240° effectively by 3:40 p.m. Similarly the coincide after every 65 min utes. If the hands
11
hour hand would cover a total of 110° effectively
coincide every 66 minutes, it take 66 minutes to
by 3:40 p.m. from the 12:00 noon.
5
∴The angle between the hands is 240° – 110° i.e., cover 65 minutes of correct time.
130° at 3:40 p.m. 11
6
5.28. At what time between 5 O’ clock and 6 O’ clock ∴It is losing time. It loses minutes every
11
in the evening will the hands of a clock be at an 66 minutes
angle of 90° with each other?
6
Sol: Let the time be 5:x p.m.
Angle effectively covered by the hour hand by
∴In a day it loses 11 (24 ) (60 ) = 11 109 minutes
66 121
°
 x
5:x p.m. = 150 +  5.32. At a certain time between 9 a.m. and 10 a.m.
 2  the angle between the hands of a clock is 6°.
Angle effectively covered by the minute hand by This time can be ______.
5:x p.m. = 6 x°
° Sol: Let all possible times be denoted by 9 : x a.m.
 x
150 +  − 6 x° = 90° (or) From 12 midnight to 9 : x a.m, the hour hand
 2 °
 x
° would have covered  270 +  , from 9:00 a.m. to
 x  2
6x° − 150 +  = 90°
 2 9:x a.m. the minute hand would have covered (6x)°.

x=
120 480
or (6x ) −  270 + x  = 6 ⇒ 11 x = 276
11 11  2 2
∴ The required time is ⇒ x = 50
2
OR
10 7 11
5 : 10 p.m. or 5 : 43 p.m.
11 11 x 11x
270 + − 6x = 6 ⇒ = 264 ⇒ x = 48
2 2
5.29. At what time between 8 O’ clock and 9 O’ clock
in the evening will the hands of a clock be in a 2
∴The time can be 9 : 50 a.m (or 9 : 48 a.m.)
straight line and in opposite directions? 11
Triumphant Institute of Management Education Pvt. Ltd. (T.I.M.E.) HO: 95B, 2nd Floor, Siddamsetty Complex, Secunderabad – 500 003.
Tel : 040–27898195 Fax : 040–27847334 email : info@time4education.com website : www.time4education.com SM1001906/58
Concept Review Questions
Directions for questions 1 to 50: For the Multiple Choice Questions, select the correct alternative from the given
choices. For the Non-Multiple Choice Questions, write your answer in the box provided.

1. Convert the following speeds into meters per second 11. Ashok left X and reached Y in 4 hours. His average
(a) 36 km/hr speed for the journey was 90 kmph. Find the
(A) 10 (B) 12 (C) 15 (D) 20 distance between X and Y (in km).

(b) 12.6 km/hr


(A) 3.5 (B) 4 (C) 0.35 (D) 6
12. Rakesh travelled for 2.5 hours at 40 kmph and for
(c) 252/35 km/hr another 2.5 hours at 60 kmph. Find his average
(A) 2.2 (B) 2.4 (C) 2 (D) 2.6 speed for the journey (in kmph).
(A) 48 (B) 50
2. If a man rides a cycle at 2.5 m/s, what distance will (C) 45 (D) 52
he cover in 4 hours? (in km)
13. Bala travelled for 3 hours at 40 kmph and then for
5 hours at 60 kmph. Find his average speed
(in kmph) for the journey.
3. If a man runs at 6 metres per second, what distance (A) 52.5 (B) 50
(in km) will he cover in 3 hours and 45 minutes? (C) 42 (D) 48
(A) 81 (B) 96 (C) 91 (D) 27
14. Alok travelled from Hyderabad to Tirupati at
4. The ratio of the speeds of X, Y and Z is 3 : 4 : 6. 60 kmph and returned at 90 kmph. Find his average
Find the ratio of the time that they take to travel a speed for the journey (in kmph)
certain distance.
(A) 4 : 3 : 2 (B) 3 : 4 : 6 (C) 6 : 4 : 3 (D) 2 : 3 : 4

5. A train travels for a total of 16 hours. The first half of the


distance at 35 km/hr and the second half at 45 km/hr. 15. When Ajay was about to leave for home from his
The total distance travelled is office he noticed that his car had a problem. Hence
km. 5
he travelled at a reduced speed and took times
4
the usual time. His average speed must have been
6. Travelling at 5/6th of his usual speed a man is 4
10 minutes late. What is the usual time he takes to (A) th of the usual average speed.
5
cover the same distance?
(A) 50 minutes (B) 70 minutes 5
(B) th of the usual average speed.
(C) 1 hour (D) 75 minutes 4
3
(C) th of the usual average speed.
7. A person saves 6 minutes by increasing his speed 4
by 25%. What is the time taken to cover the distance 4
at his usual speed? (in minutes) (D) rd of the usual average speed.
3

16. Alok maintained his average speed for a journey at


25% more than his usual average speed. The time
8. X and Y are 270 km apart. At 9:00 a.m, buses A and he must have taken to cover it must be
B left X and Y for Y and X respectively. If the speeds 5
of A and B are 50 kmph and 40 kmph respectively, (A) th of the usual time.
find their meeting time. 4
(A) 11:00 a.m (B) 12:00 p.m 4
(B) th of the usual time.
(C) 1: 00 p.m (D) 2:00 p.m 5

9. Car A left X for Y at 9:00 a.m. Car B left Y for X at 4


(C) rd of the usual time
10:00 a.m. XY = 180 km. Speeds of A and B are 3
30 kmph and 20 kmph respectively. Find their 3
meeting time. (D) th of the usual time
4
(A) 12:36 p.m. (B) 1:36 p.m.
(C) 1:00 p.m (D) 2:00 p.m
17. A car travels x km at 60 km/hr and another 3x km at
10. Car P started driving north from X at 7:00 a.m. Car 90 km/hr. Find its average speed over the entire
Q started driving north from X at 10:00 a.m. The distance.
ratio of the speeds of P and Q is 3:4. Find their (A) 84 km/hr
meeting time. (B) 75 km/hr
(A) 7:00 p.m. (B) 12:15 p.m. (C) 72 km/hr
(C) 6:00 p.m (D) 10:00 p.m (D) 80 km/hr

Triumphant Institute of Management Education Pvt. Ltd. (T.I.M.E.) HO: 95B, 2nd Floor, Siddamsetty Complex, Secunderabad – 500 003.
Tel : 040–27898195 Fax : 040–27847334 email : info@time4education.com website : www.time4education.com SM1001906/59
18. A car left X for Y and travelled at an average speed 29. Trains P and Q have lengths of 400 m, and
of 60 kmph. It reached Y, 2 hours after the 600 m respectively. They are running on parallel
scheduled time. If it had left X for Y and travelled at tracks towards each other. Find the total distance
an average speed of 80 kmph it would have reached covered by the two trains from the time they start to
Y 2 hours before the scheduled time. At what cross each other to the time they completely cross
average speed must it travel in order to reach Y at each other.
the scheduled time? (in kmph) (A) 200 m
440 (B) 1000 m
(A) (B) 70 (C) 1200 m
7
(D) Cannot be determined
500 480
(C) (D)
7 7 30. In a kilometer race, A beats B by 250 m. Find the
ratio of the speeds of A and B.
19. The ratio of the speeds of Ram and Shyam is 5 : 4. (A) 3 : 4 (B) 4 : 3 (C) 5 : 4 (D) 4 : 5
Shyam is ahead of Ram by 12 km. If they are
travelling in the same direction, what is the distance 31. In a 200 m race, Eswar gives Girish a start of 10 m
(in km.) that Ram has to cover to catch up with Shyam? and beats him by 10 m. Find the ratio of their
speeds
(A) 1 : 1 (B) 9 : 10 (C) 10 : 9 (D) 19 : 20

20. A and B start simultaneously from P and Q towards 32. X beats Y by 200 m in a kilometer race. Find Y’s
Q and P respectively. A takes 20 minutes to cover speed (in m/sec) if X’s speed is 10 m/sec (in m/sec).
PQ. B takes 30 minutes to cover QP. Find the time (A) 10 (B) 12.5 (C) 12 (D) 8
they would take to meet each other (in minutes).
33. In a 100 m race, Ganesh beats Harish by 10 m or
2 seconds. Find Harish’s speed (in m/sec).
5 1
(A) 5 (B) 5 (C) 4 (D) 6
21. What is the time taken by a train 800 m long to cross
9 2
an electric pole, if the speed of the train is 46 km/hr?
(A) 60 sec (B) 80 sec (C) 75 sec (D) 72 sec 34. In the previous question, find Ganesh’s speed.
(in m/sec).
22. A train of length 500 m takes 25 seconds to 4
(A) 5 (B) 5
completely cross a pole. Find its speed (in kmph). 9
(A) 54 (B) 72 (C) 60 (D) 90 5
(C) 5 (D) 6
9
23. What is the time taken by a train 650 m long
travelling at 72 km/hr to cross a 750 m long
35. In a 100 m race, Akbar gives Birbal a start of
platform?
2 seconds. Birlbal covers 10 m by the time Akbar
(A) 60 sec (B) 65 sec (C) 70 sec (D) 75 sec
starts. If both of them finish together, find Akbar’s
speed. (in m/sec)
24. A train 400 m long travels at a speed of 36 kmph.
Find the time it would take to cross a bridge 300 m 5
(A) 5 (B) 5
long. (in sec) 9
1
(C) 4 (D) 4
2
25. A train, 300 m long, crosses a bridge 1200 m long in 36. In a race, P beats Q by 20 seconds. Q beats R by
3 minutes. Find the speed of the train. (in kmph) 30 seconds. By how many seconds did P beat R?

26. A train took 12 seconds to cross a person standing 37. In a 100 m race, A beats B by 10 m and B beats C by
on a platform. If the speed of the train is 72 kmph 20 m. Find the distance by which A beats C. (in m)
then, find the length of the train. (A) 30 (B) 28 (C) 32 (D) 36
(A) 180 m (B) 240 m (C) 300 m (D) 120 m
38. Anand can row a boat in still water at a speed of
27. How long will a train 250 m long travelling at 5 kmph. The speed of the stream is 3 kmph. Find
90 kmph take to cross a train 200 m long travelling the time taken by him to row 40 km downstream
at 36 kmph in the same direction? (in hours).
(A) 20 seconds (B) 22 seconds
(C) 30 seconds (D) 18 seconds

28. What is the time taken by a 750 m long train 39. A and B are 2 points on a river. Akbar took 6 hours
travelling at 99 km/hr to cross a boy running at to row from A to B and 8 hours to row from B to A.
9 km/hr towards the train? (in sec) Find the ratio of the speeds of his boat in still water
and the river.
(A) 7 : 1 (B) 4 : 3 (C) 5 : 3 (D) 3 : 1
Triumphant Institute of Management Education Pvt. Ltd. (T.I.M.E.) HO: 95B, 2nd Floor, Siddamsetty Complex, Secunderabad – 500 003.
Tel : 040–27898195 Fax : 040–27847334 email : info@time4education.com website : www.time4education.com SM1001906/60
40. Ram, Shyam and Tarun started cycling from a point 45. Find the angle (in degrees) between the hands of a
on a circular track 600 m long with speeds of clock at 2:30 p.m. in degrees.
10 m/sec, 15 m/sec and 20 m/sec respectively. Find
the time taken by them to meet at the starting point
for the first time (in seconds).
(A) 120 (B) 60 (C) 240 (D) 600
46. Find the number of times in a day that the hands of
41. Amar and Bhavan started simultaneously from a a clock will be at an angle of 63°.
point on a circular track of length 600 m with speeds
of 15m/sec and 5 m/sec respectively and cycled
around it. Find the time taken by them to meet for
the first time (in seconds) if 47. Find the number of times in a day that the hands of
(i) they cycled in the same direction a clock coincide.
(A) 60 (B) 30 (C) 20 (D) 40 (A) 11 (B) 44 (C) 22 (D) 12

(ii) they cycled in opposite directions 48. Find the time interval (in minutes) between
(A) 60 (B) 30 (C) 20 (D) 40 two successive instants when the hands of a clock
concide.
42. In the previous question, find the time taken 6 5
(A) 55 (B) 55
(in seconds) by them to meet at the starting point for 11 11
the first time if Bhavan’s speed doubled. 5
(C) 65 (D) None of these
11

49. What is the time when the hands of a clock coincide


43. Ram and Shyam are running along a circular track between 12:00 p.m and 1:00 p.m?
of length 60 m. If their speeds are 2 m/sec and 5
4 m/sec respectively, how many more rounds than (A) 1 : 05 p.m (B) 1:05 p.m
11
Ram would Shyam have completed in an hour?
(A) 240 (B) 60 (C) 120 (D) 30 5 5
(C) 1 : 06 p.m (D) 1 : 07 p.m
11 11
44. Ashwin and Bhaskar started running simultaneously
from a point on a 300 m long circular track. They ran 50. The minute and hour hands of a clock coincide after
in opposite directions with speeds of 6 m/sec and every 67 minutes. Is the clock losing time?
4 m/sec respectively. After meeting for the first time, (A) Yes (B) No (C) Cannot say
they exchange their speeds. Who will reach the
starting point first?
(A) Ashwin (B) Bhaskar
(C) Both reach simultaneously

Triumphant Institute of Management Education Pvt. Ltd. (T.I.M.E.) HO: 95B, 2nd Floor, Siddamsetty Complex, Secunderabad – 500 003.
Tel : 040–27898195 Fax : 040–27847334 email : info@time4education.com website : www.time4education.com SM1001906/61
Exercise – 5(a)
Directions for questions 1 to 35: For the Multiple Choice Questions, select the correct alternative from the given
choices. For the Non-Multiple Choice Questions, write your answer in the box provided.

1. Walking from home at 3/4ths of his usual speed, a 9. How far is the point where the rear ends of the
man reaches his office 20 minutes late. Had the two trains cross each other, from the point of entry
person walked at 4/3rds of his usual speed, find the of the slower train?
time taken by the man to reach his office. (in minutes) (A) 40.5 m (B) 81 m
(C) 84.5 m (D) 68.75 m

10. P, Q and R start from the same place X at a kmph,


2. A boy walks from his home to school at 5 kmph, (a + b) kmph and (a + 2b) kmph respectively. If
reaches the school 4 minutes early. If he walks from Q starts p hours after P, how many hours after
his home to school at 4 kmph, he reaches the Q should R start, so that both Q and R overtake P at
school 2 minutes late. Find the time taken by him to the same time? (a > 0, b > 0)
go to school if he walks at 6 kmph.
(A) 15 minutes (B) 20 minutes pa a
(A) (B)
(C) 25 minutes (D) 30 minutes a+b p(a + b )

3. M and N start at the same time from two ends p(a + b ) pa


(C) (D)
S and T, at speeds of 3 m/sec and 4 m/sec towards a + 2b a + 2b
each other, ST being 7 m. After reaching the
opposite ends, they retrace their path and start 11. Two men, M and N started walking towards each
moving towards each other. Find the total distance other simultaneously from two places F and G
travelled by M when he meets N for the second time. respectively, which are 50 km apart. They meet after
(A) 20 m (B) 15 m (C) 9 m (D) 26 m 5 hours. After their meeting, M reduced his speed by
1 kmph and N increased his speed by 1 kmph. They
4. A person travels a distance AB of 15.625 m in arrived at G and F respectively at the same time.
5 seconds. In the last four seconds, every second Find the initial speed of M. (in kmph)
he covers a distance equal to one-fourth of the total
distance covered upto the end of the previous
second. Find the distance covered in the 1st second.
(in m)
12. A thief was running along to a median on a road at
6 kmph. He crossed a police jeep moving at
9 kmph in the opposite direction. The jeep had to
5. A stone, allowed to fall under gravity falls 5t2 metres continue for 10 more minutes before it could find a
in t seconds. Find the distance travelled by the stone gap in the median and start chasing the thief. Find
in the last two seconds before it reaches the ground, the total time taken by the jeep to catch the thief
if the stone is dropped from a height of 180 m. from the time it crossed him (in minutes).
(A) 30 m (B) 60 m (C) 100 m (D) 90 m (A) 45 (B) 50 (C) 55 (D) 60

6. A train crosses two bridges 430 m and 550 m long in 13. Anand and Bala started from towns P and Q
30 seconds and 36 seconds respectively. Find the simultaneously towards Q and P respectively. After
speed of the train. 6 hours, they crossed each other at town R. After
(A) 36 kmph (B) 72 kmph that, Anand took 5 hours more to reach Q than Bala
(C) 27 kmph (D) 45 kmph took to reach P. Find the sum of the times taken by
them to reach their destinations from R (in hours).
7. A train travelling at 60 kmph, crosses a cyclist who (A) 15 (B) 11 (C) 13 (D) 17
is cycling at 12 kmph in the opposite direction in
20 sec. Find the time taken by the train to cross 14. A man left X for Y. Another man left Y for X at the
another cyclist travelling at 20 kmph in the same same time. After reaching their destinations, they
direction as the train does. (in sec) turned back without stopping. They crossed each
other at a point 30 km from X on their onward trip
and 20 km from Y on their return trip. Find XY.
(in km)
Directions for questions 8 and 9: These questions are
based on the following data.
Train T of length 100 metres moving at 54 kmph and
train U of length 150 metres moving at 90 kmph running 15. P started from town X at 10 a.m. and went towards
on parallel tracks enter a 200-metre long tunnel at the town Y. Q started from Y at 12 p.m. and went
same instant from opposite directions. towards X. They met at 2 p.m. P reached his
destination 20 minutes after Q did. At what time did
8. Which train exits the tunnel first and at the moment it Q reach his destination?
exits, what length of the other train is still in the tunnel? (A) 4:20 p.m. (B) 4:40 p.m.
(A) T, 50 m (B) U, 10 m (C) 5 p.m. (D) 5:20 p.m.
(C) U, 90 m (D) T, 75 m
Triumphant Institute of Management Education Pvt. Ltd. (T.I.M.E.) HO: 95B, 2nd Floor, Siddamsetty Complex, Secunderabad – 500 003.
Tel : 040–27898195 Fax : 040–27847334 email : info@time4education.com website : www.time4education.com SM1001906/62
Directions for question 16: This question is based on 22. Find the angle between the hands of the clock at
the following data. 4:30 p.m.
o
Three friends - Anil, Bala and Chetan, from X wanted to 1
(A) 30° (B) 52
travel from town X to town Y which was 40 km. Anil who 2
had a bike started along with Bala while Chetan started (C) 60° (D) 45°
simultaneously on foot. After some time, Anil dropped Bala
on the way and went back to pick up Chetan while Bala 23. There are two clocks which have been correctly set
proceeded to Y on foot. Anil picked up Chetan and at 8:00 a.m. on Sunday, 1st January. The first clock
reached Y at the same time as Bala. Anil traveled at gains 4 minutes every hour uniformly, while the
50 km/hr. The speed at which Bala and Chetan walked second clock loses 6 minutes every hour uniformly.
was 10 km /hr. When is the next time that the two clocks show
8:00 a.m. simultaneously?
16. (i) Find the time after which Anil turned back. (A) 9th January
(A) 24 minutes (B) 48 minutes (B) 15th January
(C) 36 minutes (D) 30 minutes (C) 13th January
(ii) Find Bala’s average speed for the entire trip. (D) 31st January
(A) 12 km/hr (B) 24 km/hr
(C) 20 km/hr (D) 25 km/hr 24. If the hands of a clock coincide every 80 minutes.
and the hands of another clock coincide every
17. Peter and Paul are running in opposite directions 65 minutes, what is approximate time difference
along a circular track of 600 m with initial speeds of between the two clocks in exactly 24 hours time
3 m/s and 6 m/s. They start at the same point and at interval as shown by a correct clock?
the same time. Whenever they meet, they exchange (A) 272 min (B) 145 min
their speeds and carry on in their respective (C) 220 min (D) 200 min
directions. Find the distance between Peter and
Paul (when measured along the track) when Peter 25. Flight A started from a city P at 4 a.m. local time and
completes 41/4 rounds. (in m) reached city Q at 8 a.m. local time. It started back
from Q at 8 p.m. local time and reached P at 10 p.m.
local time. If the speed of A for either direction was
800 km/hr, the distance between P and Q is (in km)
18. Two runners run in the same direction along a .
circular track 3 km in length. The faster runner
overtakes the slower every 1 hour. Find the speed of 26. Anwar leaves home everyday at 4 p.m. to pick his
the slower runner, if the faster one completes one wife from office and returns home at 6 p.m. One
length of the track 2 minutes sooner than the other. day, the office was over at 4 p.m. and his wife
(A) 20 kmph (B) 18 kmph started walking home from office. Anwar, unaware of
(C) 15 kmph (D) 12 kmph this, starts from home as usual and meets his wife
on the way and returns home with wife 15 minutes
19. Prakash and Pramod are running along a circular early. If the speed of Anwar’s car is 30 kmph, find
track having started at the same time from the same the walking speed of his wife.
point, in the same direction. How much more (A) 30/7 kmph (B) 3.6 kmph
distance would Prakash have travelled compared to (C) 34/5 kmph (D) 5.4 kmph
Pramod by the time they meet for the 11th time given
that the radius of the track is 7 m, Prakash and 27. In a race of 500 m, L beats M by 40 seconds and
Pramod run at 22 m/s and 11 m/s respectively? beats N by 125 m. If M and N run a 500 m race,
(in m) M beats N by 40 seconds. Find the time taken
(in seconds) by M to run the race.
(A) 160 (B) 240 (C) 280 (D) 320
20. P and Q started moving simultaneously from points 28. Rahim and Saleem are competing in a 1000 m race.
X and Y respectively on a circular track in the same Rahim gives Saleem a head start of 200 m, but his
direction. XY = 2 km and the length of the track is speed is twice that of Saleem. When Rahim reaches
6 km. The initial position of P was behind that of Q. the 600 m mark, Rahim reduces his speed by half
P caught up with Q for the second time in but still beats Saleem by 20 s. What is Saleem’s
40 minutes. Had P started in the opposite direction, speed?
they would have met for the first time in 12 minutes. (A) 14 m/s (B) 5 m/s
Find the speed of P (in km/hr). (C) 3.5 m/s (D) 7 m/s
(A) 16 (B) 14 (C) 6 (D) 4
29. Akbar takes a certain time to go downstream on a
21. Anant started answering a mathematics test river between two villages. He takes 4 times as
sometime between 2:00 p.m. and 3:00 p.m. and much time to cover the same distance upstream.
ended at sometime between 5:00 p.m. and 6:00 p.m. If What is the speed of the stream, if the distance
it is known that the position of the minute hand and between the villages is 10 km, which can be covered
the hour hand at the start interchanged with the position in 2 hours if the boat travels in still water? (in kmph)
at the end, for how long did Anant take the test?
(A) 25/11 hours (B) 210/13 hours
(C) 25/7 hours (D) 213/14 hours
Triumphant Institute of Management Education Pvt. Ltd. (T.I.M.E.) HO: 95B, 2nd Floor, Siddamsetty Complex, Secunderabad – 500 003.
Tel : 040–27898195 Fax : 040–27847334 email : info@time4education.com website : www.time4education.com SM1001906/63
30. A boat travels from point P to point Q upstream and 34. A car travels a total distance of 300 km. After
returns from point Q to point P downstream PQ = 96 km. travelling a part of the distance without any trouble,
If the round trip takes 9 hours and the speed of the the car develops an engine problem and proceeds
boat in still water is 8 kmph more than the speed of at 3/4th of its former speed and arrives at the
the stream, find the time taken for the downstream destination 80 minutes late. Had the problem
journey. developed 50 km further on, the car would have
(A) 3 hours (B) 4 hours arrived 20 minutes sooner. Find the original distance
(C) 8 hours (D) 6 hours it travelled without any problem, and the speed over
that part of the journey.
31. There are two identical escalators, one is ascending (A) 100 km, 50 kmph
and the other is descending. A person took (B) 50 km, 75 kmph
90 seconds to go up using the descending escalator (C) 150 km, 50 kmph
and 18 seconds to go up using the ascending (D) 50 km, 100 kmph
escalator. Both times walking at the same speed.
If the speed of the two escalators is the same, then 35. One day, Hari walked along a circular track for
the time taken to go up/down using either of the 3 hours. He then cycled along the track for the time
escalators when they are switched off is he would take to walk one round around the track and
sec. thereby covered a total of 9 rounds. Another day, he
walked along the track for 4 hours and then drove his
motorcycle along the track for the time he would take
32. A ship 154 km from the shore springs a leak which to walk one round around the track and thereby
admits 4½ tonnes of water in 11 minutes. When covered a total of 14 rounds. If the speeds of his
184 tonnes of water enter the ship, the ship would cycling and motorcycle were 5 km/hr and 10 km/hr
start sinking. But the pumps can throw out 1 tonne respectively, then find his walking speed.
every five minutes. Find the average speed so that
she may just reach the shore as she begins to sink. 5 4
(A) km/hr (B) km/hr
(A) 10.5 kmph (B) 12 kmph 3 3
(C) 10 kmph (D) 13 kmph 7
(C) 2 km/hr (D) km/hr
3
33. An artillery gun is fired twice at an interval of
24 seconds. A motorist moving towards the gun
heard the sounds of the gunfire at an interval of
22 seconds. If the speed of sound is 330 m/s, what
is the speed of the car?
m/s

Exercise – 5(b)
Directions for questions 1 to 50: For the Multiple Choice Questions, select the correct alternative from the given
choices. For the Non-Multiple Choice Questions, write your answer in the box provided.

1. What is the distance covered by a cyclist in 20 seconds 5. A man reaches his destination which is 16 km away,
if he is travelling at a speed of 36 kmph? 9 min late, if he travels at 8 kmph. What should his
(A) 720 metres (B) 18 metres speed be if he wishes to reach 15 minutes ahead of
(C) 300 metres (D) 200 metres the right time?
(A) 10 kmph
2. Mr. Ashok takes 16 hours to go by train to a certain (B) 3 m/sec
town and return by car. He would save 4 hours if he (C) 20/9 m/sec
travelled both ways by car. Find the time he would (D) 12 kmph
lose, if he travelled both ways by train (in hours).
6. A person covered 50% of a certain distance at
15 kmph, 60% of the remaining distance at 18 kmph
and the rest at 45 kmph. The total travel time of the
3. Ram covers a distance of 100 m in a certain time. If person is 9.8 hours. Find the total distance covered.
he increases his speed to 3 times his original speed, (in km)
the time taken reduces by 40 seconds. Find the original
time taken (to cover the same distance of 100 m).
(A) 30 seconds
(B) 120 seconds 7. The distance between two points P and Q is 84 km.
(C) 180 seconds Two persons start at the same time but one
(D) 60 seconds travelling from P towards Q and the other travelling
from Q towards P. If their respective speeds are
4. If Ashok travelled at 4/5th of his usual speed, he 36 kmph and 27 kmph, where do they meet each other?
would reach his destination 15 minutes late. By how (A) 48 km from Q
many minutes would he be early if he travelled at (B) 24 km from P
6/5th of his usual speed? (C) 36 km from P
(A) 12 (B) 10 (C) 15 (D) 20
(D) 48 km from P
Triumphant Institute of Management Education Pvt. Ltd. (T.I.M.E.) HO: 95B, 2nd Floor, Siddamsetty Complex, Secunderabad – 500 003.
Tel : 040–27898195 Fax : 040–27847334 email : info@time4education.com website : www.time4education.com SM1001906/64
8. Towns P and Q are 80 km apart. Cars A and B are 15. M and N are two points that are 8 m apart. Anand
stationed at towns P and Q respectively. If they start and Ajay started simultaneously from M and N
simultaneously towards each other, they would meet respectively. Anand moved towards N at 3 m/sec.
in an hour. If both start simultaneously in the same Ajay moved towards M at 5 m/sec. After reaching
direction, the faster car would overtake the slower car their destinations, both turned back at their original
in 4 hours. Find the speed of the faster car (in kmph). speeds towards their starting points. Find the total
distance (in m) that Ajay would have travelled before
crossing Anand for the second time.
(A) 12 (B) 13 (C) 15 (D) 14
9. A cat on seeing a dog 100 m away turns around and
16. P and Q are two points, 10 km apart. Anand started
starts running away at 24 kmph. The dog spots him
from P towards Q and at the same time, Ashok
one minute later and starts chasing the cat at a
started from Q towards P. They crossed each other
speed of 33 kmph. After how much time, from the
after 1 hour. After that, Anand reduced his speed by
start of the cat’s run, will the chase end?
2 kmph and Ashok increased his speed by 2 kmph.
(A) 160 s (B) 220 s
They reached their destinations simultaneously.
(C) 260 s (D) 280 s
Find Anand’s initial speed (in kmph).
10. Train A starts at 6 a.m. from city P towards city Q at
a speed of 54 kmph. Another train ‘B’ starts at
9 a.m. from P towards Q at 72 kmph. If the distance
between P and Q is 1440 km, find at what distance 17. Mahesh travelled from Hyderabad to Tirupati at a
from Q would the two trains meet each other? certain speed and returned at a certain speed. His
(A) 648 km (B) 792 km average speed for the entire trip was the average of
(C) 486 km (D) 954 km his onward and return speeds. He travelled a total
distance of 1200 km in 12 hours. Find his onward
11. Towns P and Q are 265 km apart. Car A started speed (in kmph).
from P at 60 km/hr at 8:00 a.m towards Q. Car B (A) 100 (B) 80 (C) 60 (D) 40
started from Q at 40 km/hr at 10:00 a.m, towards P.
At 11:00 a.m, B stopped for half an hour. A stopped 18. A person travels one-third of a certain distance
for 28 minutes at town S, which is at a distance of AB at x kmph, one-fourth of the remaining distance
210 km from P. Find the time at which B crossed A. at 2x kmph and the remaining distance at 3x kmph.
(A) 11:35 a.m. (B) 11:54:30 a.m. If his average speed for the entire journey is
(C) 11: 52:30 a.m. (D) 11:48 a.m. (x + 2) kmph, then find the total distance he covers.
(A) 60 km (B) 50 km
12. P and Q are two stations. Train A started from P (C) 40 km (D) Cannot be determined
towards Q at 6:00 a.m at 90 kmph. At the same
time, train B started from R, an intermediate station 19. Rahul ran around a square plot ABCD of side 0⋅48 km
60 km from P, and travelled towards Q at 60 kmph. once. He ran the distance AB, BC, CD and DA at
Train C started from Q towards P at 7:00 a.m at speeds of 4 kmph, 6 kmph, 4 kmph, and 6 kmph
120 kmph. All the trains crossed each other respectively.
simultaneously. Find PQ (in km). (i) Find his average speed from A to C (in kmph).
(A) 2⋅4 (B) 3⋅6 (C) 4⋅8 (D) 7⋅2

(ii) Find his average speed for the entire distance


(in kmph).
13. Two cars left simultaneously from two places (A) 2⋅4 (B) 3⋅6 (C) 4⋅8 (D) 7⋅2
P and Q, and headed for Q and P respectively. They
crossed each other after x hours. After that, one of (iii) Find the time taken for the entire distance
the cars took y hours to reach its destination while (in hours).
the other took z hours to reach its destination. Which (A) 0⋅05 (B) 0⋅53 (C) 0⋅4 (D) 0⋅27
of the following always holds true?
20. Two cars C and D start from a junction along
y+z 2yz
(A) x = (B) x = two perpendicular roads at 8:00 a.m. and 9:00 a.m.
2 y+z respectively. If at 12 noon, the cars, which travel at
the same speed, are 150 km apart, then, find the
y2 + z2 speed of each car. (in kmph)
(C) x = yz (D) x =
y+z

14. Ram and Gopi run towards each other starting from 21. Rajesh started in his car from town P towards town
K and L respectively with respective speeds of Q at 51 km/hr. After every minute, the speed of his
2 kmph and 3 kmph. After meeting each other, to car increased by 1 km/hr. The distance between
reach L, if Ram takes 7 hours less than the square
of magnitude of the time (in hours) taken by Gopi to 3775
P and Q is km. Find the time he would take
reach K, find the distance between K and L. 60
(A) 25 km (B) 28 km to reach Q (in minutes).
(C) 30 km (D) 32 km (A) 40 (B) 45 (C) 55 (D) 50

Triumphant Institute of Management Education Pvt. Ltd. (T.I.M.E.) HO: 95B, 2nd Floor, Siddamsetty Complex, Secunderabad – 500 003.
Tel : 040–27898195 Fax : 040–27847334 email : info@time4education.com website : www.time4education.com SM1001906/65
22. A car had to travel a total distance of 600 km. After 29. In a 200 m race, Raja gives Rakesh a start of 20 m
travelling a part of the distance, it developed an and beats him by 20 m. If Raja beats Rakesh by
engine problem. It travelled the remaining distance 4 seconds, find Raja’s speed (in m/sec).
at 4/5th of its usual speed. It arrived an hour late at
its destination. Had the engine problem occurred
after it had travelled 150 km more, the car would
have arrived half an hour earlier at its destination. 30. In a kilometre race, Ram beats Shyam by one
Find the distance it travelled without any problem (in km). minute and Shyam beats Tarun by 30 seconds. If
(A) 150 (B) 250 (C) 200 (D) 300 Ram beats Tarun by 250 m in the same race, find
the time taken by Ram to run the race (in seconds).
23. Everyday, Ashwin starts at 3.00 pm from his home to (A) 180 (B) 270 (C) 360 (D) 330
pick up his son from school. They reach their house at
5.00 p.m. One day, school was over at 3.00 p.m. 31. In a 100 m race, Karna beats Kiran by 20 m and
Ashwin, not aware of this, started from home as usual. Kiran beats Kumar by 7.5 sec. What is Kiran’s
He met his son on the way and they reached home speed if Karna runs twice as fast as Kumar?
20 minutes earlier than usual. If the speed of his car is (in m/sec)
55 kmph, find his son‘s speed (in kmph).

32. In a 500 ft race, Habib beats Akram by 60 ft. If Habib


24. Two persons R and S had to meet at a place 36 km takes 5 paces for every 4 paces taken by Akram,
from where they were at that moment. S, who was what is the ratio of the length of Habib’s pace to that
given a head start of 2 hours, reached the of Akram?
destination 12 minutes earlier than R. If R travelled (A) 10 : 11 (B) 11 : 10
18 kmph faster than S, at what speed did R travel? (C) 25 : 22 (D) 22 : 25
(A) 25 kmph (B) 24 kmph
(C) 30 kmph (D) 16 kmph Directions for questions 33 and 34: These questions
are based on the following data given below.
25. In a race, Mohan beats Sohan by 40 m and Sohan
beats Rohan by 80 m. Mohan beats Rohan by 104 m. Two people Allen and Donald start off to complete a race
Find the length of the race (in m). of distance d km, with speeds of a kmph and b kmph
respectively (where b > a). After reaching the halfway
mark, Allen picks up speed and both of them reach the
finishing post together.
26. (i) In a race of length d m, Amar beats Bhavan by
x m and Charan by y m. By what distance does 33. What is Allen’s speed (in kmph) after the halfway
Bhavan beat Charan in the same race? mark?

(A)
(y − x )d m (B)
(y − x )d m (A)
2a − b
(B)
d
d−y d−x ab b−a
d( d − y ) b+a
(C) m (D) (y − x)m (C) (D)
ab
d−x 2 2a − b
(ii) In a race of length d m, Amar beats Bhavan by
34. What is Allen’s average speed over the entire
x m. Bhavan beats Charan by y m. By what
journey?
distance does Amar beat Charan (in m)?
a+b
xy xy (A) kmph (B) a2 kmph
(A) x − y + (B) y − x + 2
d d
(C) a kmph (D) b kmph
xy
(C) x + y (D) x + y −
d 35. A train takes 30 seconds to cross a 200 m long
platform and 40 seconds to cross a 300 m long
27. In a 100 m race, Ajay gives Bala a start of x m platform. Find its length (in m).
(x ≥ 20) and is beaten by him by y m (y ≤ 20). If the
speeds of both are distinct, which of the following
cannot be the ratio of the speeds of Ajay and Bala?
(A) 6 : 5 (B) 4 : 3 36. A train takes a minute to cross a stationary pole. It
(C) 7 : 5 (D) 9 : 10 takes 240 seconds to cross another 3600 m long
train traveling at 54 kmph. Find its speed (in kmph).
28. In a 500 m race A beats B by 25 m or 5 s, what is (A) 108 (B) 144 (C) 90 (D) 72
A’s speed?
37. A train takes 2 minutes to overtake a cyclist traveling
4 5 at 18 kmph and one and a half minutes to overtake
(A) 4 m/s (B) 5 m/s
11 19 a cyclist traveling at 9 kmph. Find its length (in m).
(A) 1200 (B) 800
3 1 (C) 1500 (D) 900
(C) 7 m/s (D) 10 m/s
13 2
Triumphant Institute of Management Education Pvt. Ltd. (T.I.M.E.) HO: 95B, 2nd Floor, Siddamsetty Complex, Secunderabad – 500 003.
Tel : 040–27898195 Fax : 040–27847334 email : info@time4education.com website : www.time4education.com SM1001906/66
38. A train 360 m in length, travelling at a uniform speed 46. P and Q start cycling simultaneously around a circular
overtook a car, travelling parallel to the tracks at track 48000 m long with speeds of 10 m/sec and
72 kmph and passed it in 12 sec. Fortyeight minutes 30 m/sec respectively in opposite directions. After
later, the train starts overtaking a cyclist and passed every crossing, P’s speed increases by 10 m/sec and
him in 9 sec. How much time after the train overtook Q’s speed decreases by 10 m/sec. Find the time
the cyclist would the car over take him? taken by them to cross for the third time (in seconds).
(A) 2 hrs 24 min 36 sec (A) 3600 (B) 7200 (C) 4800 (D) 9600
(B) 3 hrs 49 min 49 sec
(C) 3 hrs 12 min 9 sec 47. Ram and Shyam started simultaneously from the
(D) 2 hrs 24 min 27 sec same point on a circular track 1800 m long in
opposite directions with speeds of 6 m/sec and
39. A boat travels 30 km upstream in 5 hours and 24 km 12 m/sec respectively. After every crossing, they
downstream in 3 hours. Find the speed of the boat exchanged their speeds and continued to travel in
in still water and the speed of the water current the same direction. Find the distance between them
(A) 7 kmph, 2 kmph (measured along the track) when Ram completed
(B) 14 kmph, 1 kmph 1
(C) 7 kmph, 1 kmph 3 rounds. (in m)
4
(D) 8 kmph, 2 kmph

40. A boat takes 9 hours to make a round trip in a river


between two points 24 km apart. It would have taken
48. P and Q are two points on a 1 km long circular track.
4 hours to cover the upstream distance in still water.
The distance PQ, along the track is 200 m. Rohan
Find the speed of the stream (in kmph).
started running from P. Sohan started running
simultaneously in the same direction from Q. Both
reached P for the first time simultaneously. If both
started simultaneously from P in opposite directions,
41. The difference of the squares of the speed of a boat
in still water and the speed of a river is six times the 1
they would take 111 seconds to meet for the
speed of the boat in still water. Find the average 9
speed of the boat in covering a round trip. (Assume first time. Find the time taken by them to meet for
all speeds are in kmph) the first time (in seconds), if they started
(A) 3 (B) 6 (C) 9 (D) 7.5 simultaneously from Q in the same direction.

42. Girish takes 1 minute to complete a round around a


circular track. Harish is twice as fast as Girish,
Suresh is thrice as fast as Harish. All three start at 49. A watch set correctly at 10:00 a.m. on a Sunday
the same point. Find the time taken by them to meet shows 20 min more than the correct time at 6:00 p.m.
at the starting point for the first time (in minutes). on that day. When the clock shows 10:30 p.m. on
the same day, what is the correct time?
(A) 7:00 p.m. (B) 10:00 p.m.
(C) 10:10 p.m. (D) 10:20 p.m.
43. Kavya, Suma and Sowmya are running along a
circular track of length 1120 m in the same direction 50. Dinesh started answering a test at a time between
with respective speeds of 10 m/s, 8 m/s and 7 m/s. 3:00 p.m and 4:00 pm. He noted the position of the
(i) When will they be together again for the first hands of his watch. He ended the test at a time
time? between 4:00 p.m and 5:00 p.m The positions of the
(ii) When will they be together again at the starting hands were interchanged. For how many hours did
point for the first time? the test last?
(A) 280 s, 140 s (B) 756 s, 560 s 13 12 9 10
(A) (B) (C) (D)
(C) 140 s, 1120 s (D) None of these 14 13 10 11

44. Tony and Harry begin to run in opposite directions Directions for questions 51 to 60: Each question is
on a circular path of radius 35 m at 20 m/sec and followed by two statements Ι and ΙΙ. Indicate your
11 m/sec respectively from the same point. What is responses based on the following directives:
the time taken by them to meet for the third time at Mark (A) if the question can be answered using one
the starting point? of the statements alone, but cannot be
(A) 11 min (B) 220 sec answered using the other statement alone.
(C) 8 min 40sec (D) 325 sec Mark (B) if the question can be answered using
either statement alone.
45. Amar, Akbar and Anthony start running in the same Mark (C) if the question can be answered using
direction and from the same point, around a circular Ι and ΙΙ together but not using Ι or ΙΙ alone
track with speeds 7 m/sec, 11 m/sec and 22 m/sec Mark (D) if the question cannot be answered even
respectively. If Akbar can complete 5 revolutions using Ι and ΙΙ together.
around the track in 40 sec, when will they meet for 51. Find the speed of the train, which crosses a signal
the first time after they start? (in sec) post in 10 sec.
Ι. The length of the train is 200 m.
ΙΙ. The train crosses a platform in 40 sec.
Triumphant Institute of Management Education Pvt. Ltd. (T.I.M.E.) HO: 95B, 2nd Floor, Siddamsetty Complex, Secunderabad – 500 003.
Tel : 040–27898195 Fax : 040–27847334 email : info@time4education.com website : www.time4education.com SM1001906/67
52. In a race, Ram gave Shyam a head start of at least 57. If Ram walked from his home to his office at a kmph,
x m and was beaten by him by at most y m. Was he would be a minutes early. Instead had he walked
Ram faster than Shyam? at b kmph, he would be b minutes early, where a
Ι. x ≥ y and b are distinct. Find the usual time he takes to
ΙΙ. x ≤ y reach his office.
Ι. a + b = 10
53. Mohan rowed his boat from a point A in a river ΙΙ. a − b = 2
12 km upstream and returned to A. If the river was
moving at a constant rate, then was his speed in still 58. Trains A and B have their lengths in the ratio 3 : 2.
water more than 3 kmph? A takes x seconds to cross platform P1. B takes
Ι. He took four hours for the upstream journey. x 5
ΙΙ. He took three hours for the downstream journey y seconds to cross platform P2. Is > ?
y 4
54. Two people A and B drove one car on a 500 Km trip. Ι. A and B have equal speeds.
A drove for 5 hours, which was half an hour more ΙΙ. P1 and P2 have equal lengths.
than the time B drove. What was B's average speed?
Ι. B drove 50 km more than A. 59. Train A takes 50 seconds to cross a bridge B1 500 m
long. Train B is twice as long as A and has half its
ΙΙ. A’s average speed was 5 kmph less than that of B.
speed. Find the time taken by it to cross bridge B2.
55. A boat started moving from A to B, what is the Ι. Length of A is 300 m.
speed of boat in still water? ΙΙ. Length of B2 is 1000 m.
Ι. Speed of the river current is 2 m/sec.
60. In a kilometre race, A beats B by x m and B beats C
ΙΙ. The distance between A and the boat is
by y m. Does A beat C by more than 400 m?
increasing at the rate of 1 m/s.
Ι. x = 200 ; y ≥ 200
56. What is the distance covered by a car during a ΙΙ. x = 200 ; y ≤ 200
certain trip?
Ι. The average speed of the car is 50 kmph.
ΙΙ. The car would have covered 100 km more
(in the same time) if the average speed
increases by 25%.
Key
Concept Review Questions

1. (a) A 6. A 13. A 20. 12 27. C 34. C 41. (i) A 47. C


(b) A 7. 30 14. 72 21. D 28. 25 35. B (ii) B 48. C
(c) C 8. B 15. A 22. B 29. B 36. 50 42. 120 49. A
2. 36 9. C 16. B 23. C 30. B 37. B 43. C 50. A
3. A 10. A 17. D 24. 70 31. C 38. 5 44. C
4. A 11. 360 18. D 25. 30 32. D 39. A 45. 105
5. 630 12. B 19. 60 26. B 33. A 40. A 46. 44

Exercise – 5(a)

1. 45 7. 36 13. C 18. C 24. A 30. A


2. B 8. C 14. 70 19. 484 25. 2400 31. 30
3. C 9. D 15. B 20. A 26. A 32. A
4. 6.4 10. D 16. (i) C 21. B 27. C 33. 30
5. C 11. 5.5 (ii) D 22. D 28. B 34. A
6. B 12. D 17. 150 23. D 29. 3 35. A

Exercise – 5(b)
1. D 12. 300 21. D 31. 8 42. 1 53. A
2. 4 13. C 22. D 32. A 43. D 54. B
3. D 14. C 23. 11 33. D 44. A 55. C
4. B 15. C 24. C 34. D 45. 88 56. A
5. A 16. 6 25. 200 35. 100 46. A 57. A
6. 180 17. A 26. (i) B 36. B 47. 450 58. D
7. D 18. D (ii) D 37. D 48. 1000 59. A
8. 50 19. (i) C 27. D 38. D 49. B 60. A
9. C (ii) C 28. B 39. C 50. B
10. B (iii) C 29. 6.25 40. 2 51. A
11. C 20. 30 30. B 41. B 52. A
Triumphant Institute of Management Education Pvt. Ltd. (T.I.M.E.) HO: 95B, 2nd Floor, Siddamsetty Complex, Secunderabad – 500 003.
Tel : 040–27898195 Fax : 040–27847334 email : info@time4education.com website : www.time4education.com SM1001906/68
CHAPTER – 6
TIME AND WORK
Work to be done is usually considered as one unit. It The concept of MANDAYS is very important and useful
may be constructing a wall or a road, filling up or here. The number of men multiplied by the number of
emptying a tank or cistern or eating certain amount of days that they take to complete the work will give the
food. number of mandays required to do the work. The total
number of mandays required to complete a specific task
There are some basic assumptions that are made in the will remain a constant. So, if we change one of the
problems on Time and Work. These are taken for variables - men or days - the other will change
granted and are not specified in every problem. accordingly so that their product will remain constant
(remember from our knowledge of VARIATION, two
(i) If a person (or one member of the workforce) does variables whose product is a constant are said to be
some work in a certain number of days, then we inversely proportional to each other). The two variables -
assume (unless otherwise explicitly stated in the men and days - are inversely proportional to each other,
problem) that he does the work uniformly, i.e., he when work is constant.
does the SAME amount of work everyday.

For example, if a person can do some work in Examples


15 days, he does 1/15th of the work in one day.
6.01. If 15 men take 60 days to complete a job, find
If a person completes the work in 4 days, he does 1/4 th the time taken by 45 men to complete it.
of the work on each day and conversely, if a person can
Sol: Number of Mandays required to complete the
complete 1/4th of the work in one day, he can complete
job = 900. Time taken by 45 men to complete it
the work in 4 days.
900
= i.e. 20 days
If a tap can fill a tank in 20 minutes, then in one minute, it 45
can fill 1/20th part of the tank.
6.02. 18 men take 20 days to complete a job working
(ii) If there is more than one person (or members of 12 hours a day. Find the number of days that
"workforce") carrying out the work, it is assumed that 15 men will take to complete it if they work
each person (or members of the workforce), unless 9 hours a day.
otherwise specified, does the same amount of work
each day. This means they share the work equally. Sol: Total time for which 18 men work = 240 hours.
Number of man hours required to complete the
If two people together can do the work in 8 days it means job = (18) (240) man hours.
that one man can do it in 16 days. This, in turn means, each Number of days taken by 15 men working
person can do 1/16th of the work per day. 9 hours a day to complete it
If a man works three times as fast as a boy does, the man =
(18 )(240 ) = 32
takes one-third of the time the boy takes to complete the (15 )(9)
work. If the boy takes 12 days to complete the work, then Hence, in general we can say that
the man takes 4 days to complete the work.
If M1 men can do W1 work in D1 days working
This method is known as "UNITARY METHOD", i.e., the
H1 hours per day and M2 men can do W2 work
time taken per "Unit Work" or number of persons
in D2 days working H2 hours per day (where all
required to complete "Unit Work" or work completed by
men work at the same rate), then
"Unit Person" in "Unit Time", etc., is what is first
calculated. M 1 D 1 H1 M D H
= 2 2 2
W1 W2
We should recollect the fundamentals on variation (direct
and inverse) here.
6.03. 20 men take 10 days to complete a job working
- Time remaining constant, Work and Men are directly 12 hours a day. Find the number of men
proportional to each other, i.e., if the work increases required to complete a job, twice as large, in
the number of men required to complete the work in 30 days working 8 hours a day.
the same number of days increases proportionately
and vice-versa. Sol: Number of man hours required to complete the
job = (20) (10) (12) = 2400
- Work remaining constant, Men and Days are Number of men required to complete a job twice
2 (2400 )
inversely proportional, i.e., if the number of men
increases, the number of days required to complete as large = = 20
the same work decreases in inverse proportion and (30 )(8)
vice-versa. Alternative method:
- The number of workingmen remaining constant, M1 = 20, D1 = 10, H1 = 12
Work and Days are directly proportional i.e., if the D2 = 30, H2 = 8
2 = 2W 1
work increases, the number of days required to
M1 D1 H1 W2 (20 )(10 )(12 )(2W1 )
complete the work with the same number of working M2 = = = 20 .
men also proportionately increases and vice-versa. W1 D 2 H2 W1 (30 )(8 )

Triumphant Institute of Management Education Pvt. Ltd. (T.I.M.E.) HO: 95B, 2nd Floor, Siddamsetty Complex, Secunderabad – 500 003.
Tel : 040–27898195 Fax : 040–27847334 email : info@time4education.com website : www.time4education.com SM1001906/69
If two persons A and B can individually do some work in Sol: Suppose that A, B and C take a days, b days
p and q days respectively, we can find out how much and c days respectively to complete the job.
work can be done by them together in one day. Since A 1 1 1
can do 1/pth part of the work in one day and B can do Given, + = –––––––– (A)
a b 12
1/qth part of the work in one day, the two of them
together do (1/p + 1/q)th part of the work in one day. 1 1 1
+ = –––––––– (B)
b c 20
From this we can find out the number of days that they 1 1 1
+ = –––––––– (C)
take to complete the work. a c 10
Adding (A), (B) and (C),
 1 1 1  14
If A can do a piece of work in p days and B can do 2 + +  =
it in q days then A and B together can complete the  a b c  60
pq 1 1 1 7
same in days. + + = –––––––– (D)
p+q a b c 60
1 1
∴ By (D) – (A), =
c 30
c = 30
1 1
6.04. A and B can complete a job in 10 days and By (D) – (B), =
12 days respectively. Find the time taken to a 15
complete it, if both A and B work together. a = 15
1 1
Sol: Time taken by them to complete it By (D) – (C), =

=
(10 )(12) = 60 days b 60
b = 60
10 + 12 11

6.05. A and B together can complete a job in 12 days. 6.08. P and Q together can complete a job in
A alone can complete it in 24 days. Find the 12 days. Q and R together can complete it in
time taken by B to complete it. 15 days. P and R together can complete it in
18 days. All the three work together for 3 days
Sol: Part of the job that A and B can complete in a and then R leaves. In how many days can
1 P and Q complete the remaining work?
day =
12
Part of the job that A can complete in a day Sol: Part of the job that can be done by P, Q and R
1 1 1 1 1 
= in a day =  + + 
24 2  12 15 18 
Part of the job that B can complete in a day
Part of the job completed in 3 days
1 1 1
= − = 1  1 1 1  37
12 24 24 = 3  + +  =
∴ B can complete it in 24 days.  2  12 15 18  120
83
6.06. Ajay and Bala working together can complete a Remaining part of the job =
120
job in 16 days. Ajay alone can complete it in
18 days. Both work together for 4 days and then This can be completed by P and Q in
Bala leaves. Find the time taken by Ajay to 83
complete the remaining work. 120 = 8⋅3 days
1
Sol: Part of the job that can be done by both in a day 12
1
=
16 6.09. A can complete a job in 16 days. He started the
Part of the job that can be done by them in work and after 4 days, B joined him. They
1 1 completed the job in 4 more days. Find the
4 days = 4 × =
16 4 number of days in which B alone can complete it.
3
Remaining part of the job = 1
4 Sol: Part of the job done by A in a day =
Time taken by Ajay to complete it 16
A worked for a total of 8 days
=
3
(18 ) = 13.5 days  1  1
4 ∴ A completed (8)   = of the job.
 16  2
6.07. A and B together can complete a job in 1
12 days. B and C together can complete it in Hence, B can complete the remaining of the
2
20 days. A and C together can complete it in
10 days. Find the times taken by each of job in 4 days.
A, B and C to complete it. ∴ B alone can complete the entire job in 8 days.

Triumphant Institute of Management Education Pvt. Ltd. (T.I.M.E.) HO: 95B, 2nd Floor, Siddamsetty Complex, Secunderabad – 500 003.
Tel : 040–27898195 Fax : 040–27847334 email : info@time4education.com website : www.time4education.com SM1001906/70
6.10. P and Q together can complete a job in 9 3
2 From (B) and (C), =
14 days . Q and R together can complete it in q 20
5 q = 60
4 1 3  1 1 
20 days. P and R together can complete it in = − + 
7 r 20  10 60 
16 days. Find the time taken by each of them to
complete the job. r = 30

Sol: Part of the job that P and Q can do in a day 6.12. 10 boys or 20 girls can complete a job in 10 days.
5 Find the time taken by 10 boys and 20 girls to
= complete it.
72
Part of the job that Q and R can do in a day
Sol: Part of the job that can be done by a boy in a
7
= 1
144 day = .
100
Part of the job that P and R can do in a day
Part of the job that can be done by a girl in a
1
= 1
16 day = .
Let the times taken by P, Q and R to complete 200
the job be p days, q days and r days Part of the job that can be done by 10 boys and
respectively.  1   1  1
20 girls = 10  + 20 =
1 1 5  100   200  5
+ = --- (A)
p q 72 ∴ They can complete the job in 5 days.
1 1 7 Alternative method:
+ = --- (B)
q r 144
10 boys are as efficient as 20 girls.
1 1 1 ∴ 10 boys and 20 girls are as effective as
+ = --- (C)
p r 16 20 boys.
Adding (A) and (B) and subtracting (C), 10
∴They will take = 5 days to complete the job.
2 5 7 1 8 2
⇒ = + − =
q 72 144 16 144
6.13. If there are 4 more men in a work force, they will
q = 36 take 8 days less to complete a job. Instead if
substituting q = 36, in (A), there are 8 men more in it, they will take
we get p = 24 12 days less to complete it. Find the ratio of the
substituting q = 36 in (B), number of men and the time they take (in days)
we get r = 48. to complete it.
6.11. To complete a job, P takes half as long as
Q and R together take. Q takes 8 times as long Sol: Let the number of men be x and the number of
as P and R together take. All the three together days they take to complete the job be y.
(x + 4) (y − 8) = xy ⇒ 4y – 8x = 32 ––––– (A)
20
can complete the job in days. Find the time (x + 8) (y – 12) = xy ⇒ 8y – 12x = 96 ––– (B)
3 solving (A) and (B), we get
taken by each of P, Q and R to complete it. x = 8 and y = 24
Sol: Let the times taken by P, Q and R be p days, ∴ x : y = 8 : 24 = 1 : 3
q days and r days respectively.
6.14. 3 men and 4 women can complete a job in
1  1 1
Given = 2 +  10 days. 24 men and 2 women can complete it
p q r in 2 days. Find the time taken by 11 men and
3  1 1 1 3 women to complete it.
⇒ = 2 + +  ––––––––– (A)
p p q r  Sol: Let the parts of the job done by a man and a
woman each day be m units and w units
1 1  1 1
Given =  +  respectively.
q 8  p r  1
Given, 3m + 4w = –––––––– (A)
8 1 1 10
⇒ = +
q p r 1
24m + 2w = ––––– (B)
9 1 1 1 2
⇒ = + + –––––––– (B) Solving for m and w,
q p q r
1 1
1 1 1 3 m= and w =
Given + + = –––––––– (C) 50 100
p q r 20 1
11m + 3w =
3 6 4
∴From (A) and (C), =
p 20 ∴ Time taken by 11 men and 3 women to
p = 10 complete the job is 4 days.
Triumphant Institute of Management Education Pvt. Ltd. (T.I.M.E.) HO: 95B, 2nd Floor, Siddamsetty Complex, Secunderabad – 500 003.
Tel : 040–27898195 Fax : 040–27847334 email : info@time4education.com website : www.time4education.com SM1001906/71
6.15. X is twice as fast as Y and hence takes 30 days 6.20. P and Q together can complete a job in 8 days
less than Y to complete a job. Find the time and 16 days respectively. They work on
taken by X and Y together to complete it. alternate days with Q starting the job. In how
many days will the job be completed?
Sol: Let the time taken by X to complete the job be n
days. Sol: Part of the job completed by P and Q in the first
Time taken by Y to complete the job = 2n days
1 1 3
n = 2n − 30 ⇒ n = 30 2 days = + =
Time taken by X and Y together to complete it 8 16 16
(n) ( 2n) After 5 cycles of 2 days, i.e., after 10 days,
= = 20 days 15
n + 2n th of the job will be completed.
16
6.16. Sita can complete a job in 6 days working 1
8 hours a day. Gita can complete it in 3 days Remaining part = th . Q will work on the
16
working 12 hours a day. In how many days can they th
11 day and he takes exactly one day to
together complete it working 4 hours a day? complete the remaining part.
Sol: Numbers of hours taken by Sita and Gita to ∴ The job will be completed in 11 days.
complete the job are 48 hours and 36 hours
respectively. In general, money earned should be shared by people
1 1 7 doing the work together in the ratio of the SHARE OF
They together can complete + = th
48 36 144 WORK done by each of them.
of the job in an hour.
144 For example, if A does 2/5th of the work, then he should
∴ They can complete the job in hours.
7 get 2/5th of the total earnings for the work. If the
∴ They can complete the job in remaining 3/5th of the work is done by B and C in the
144 ratio of 1 : 2, then the remaining 3/5th of the earnings
7 = 36 days . (after paying A) should be shared by B and C in the ratio
4 7 of 1 : 2. Suppose `500 is paid to A, B and C together for
doing the work, then A will get `200 (which is 2/5 of
6.17. P and Q can complete a job in 20 days and `500), B will get `100 and C, `200 (because the
30 days respectively. P started it and after remaining `300 after paying A is to be divided in the ratio
10 days, Q joined. In how many days will they 1 : 2 between B and C).
complete the remaining work?
Sol: Part of the job completed by P and Q in a day When people work for the same number of days each,
1 1 1 then the ratio of the total work done will be the same as
= + = .
20 30 12 the work done by each of them PER DAY. Hence, if all
Suppose that the remaining work is completed in x the people involved work for the same number of days,
days. then the earnings can directly be divided in the ratio of
 1   1  work done per day by each of them.
Then x   + 10  =1
 12   20 
x=6 6.21. A, B and C can complete a job in 4 days, 5 days
and 6 days respectively. They work together and
6.18. A contractor decided to complete a job in
complete it. If their total wage is `3700, find A’s
30 days for which he employed 20 men in the
wage.
beginning. After 10 days he released that the
job could not be completed on time. Hence he
Sol: Ratio of the wages of A, B and C = Ratio of the
employed 15 more men and thus completed the
work done by A, B and C = Ratio of the daily
job on time. Find the number of extra days it
work done by A, B and C
would have taken to complete the job if the
1 1 1
additional men were not employed. = : : = 15 : 12 : 10
4 5 6
Sol: Number of mandays required to complete the job 15
= (20) (10) + (20 + 15) (20) = 900 man days. ∴A’s wage = (3700 ) = `1500
If the additional men were not employed, 37
900
number of extra days = − 30 = 15 days. 6.22. X, Y and Z take a job on contract for `8000.
20 4
X and Y started the job and completed th of
6.19. A and B can complete a job in 18 days and 5
36 days respectively. They work on alternate the job. Z then took over and completed the
days with A starting the job. In how many days remaining work. Find Z’s share.
will the job be completed?
Sol: Part of the job completed in the first 2 days 1
1 1 1 Sol: Part of the job completed by Z =
= + = 5
18 36 12 8000
∴ To complete the job, 12 cycles of 2 days i.e., ∴Z’s share = = `1600
5
a total of 24 days will be required.
Triumphant Institute of Management Education Pvt. Ltd. (T.I.M.E.) HO: 95B, 2nd Floor, Siddamsetty Complex, Secunderabad – 500 003.
Tel : 040–27898195 Fax : 040–27847334 email : info@time4education.com website : www.time4education.com SM1001906/72
6.23. P, Q and R can together earn `3100 in 10 days. Sol: Part of the tank filled per minute when all the
Q and R together can earn `1320 in 6 days. 1 1 1 1
P and R together can earn `1050 in 5 days. three pipes are opened = + − =
10 20 40 8
Find R’s daily earning.
∴ They would take 8 minutes to fill the tank.
` 3100 6.27. Pipes P and Q take 24 minutes and 36 minutes
Sol: Total daily wage of P, Q and R = = `310
10 respectively to fill an empty tank. If both take
`1320 18 minutes to fill the tank along with an outlet
Total daily wage of Q and R = = `220 pipe R, find the time R would take to empty the
6
full tank.
`1050
Total daily wage of P and R = = `210
5 Sol: Let the time taken by R to empty the tank be
Total daily wage of P, Q and 2R = `430 r minutes.
∴ R’s daily wage = `120 1 1 1 1
+ − =
24 36 r 18
6.24. Two men undertake a job for `960. They can r = 72
complete it in 16 days and 24 days. They work
along with a third man and take 8 days to 6.28. Pipes X and Y can fill a tank in 30 minutes and
complete it. Find the share that the third man 60 minutes respectively. Both pipes are opened
should get. simultaneously. After how much time should X be
closed so that the tank is filled in 30 minutes?
Sol: The amount payable should be proportional to
the fraction of work done. Sol: Let us say pipe X should be closed after
Part of the job done by the third man n minutes.
i.e. pipe X is in operation for n minutes and pipe Y
 8 8  1
= 1 −  +  = . for all the 30 minutes.
 16 24  6 n 30
` 960 So, + = 1 ⇒ n = 15
∴ The third man should get = `160 30 60
6
6.29. Pipes P, Q and R together can empty a full tank
PIPES AND CISTERNS in 6 hours. All the three pipes are opened
simultaneously and after 2 hours, P is closed.
There can be pipes (or taps) filling (or emptying) tanks The tank is emptied in another 6 hours. Find the
with water. The time taken by different taps (to fill or time in which P can empty the tank.
empty the tank) may be different. Problems related to
these can also be dealt with in the same manner as the Sol: Part of the tank that can be emptied by P, Q
foregoing problems on Work have been dealt with. 1
and R per hour = .
6
There is only one difference between the problems on Part of the tank that was emptied by P, Q and R
regular Work (of the type seen earlier on in the chapter) 1
in 2 hours =
and those in Pipes and Cisterns. In Pipes and Cisterns, 3
a filling pipe or tap does positive work and an emptying Part of the tank which was emptied by Q and R
pipe or a leak does negative work. 1
1−
3 1
6.25. Pipes P and Q can fill a tank in 20 minutes and per hour = =
6 9
30 minutes respectively. If they are opened Time in which P can empty the tank
simultaneously, in how much time can they fill it?
1
= i.e. 18 hours.
1 1
Sol: If a pipe takes x hours to fill a tank and another −
takes y hours to fill it, they can fill it in 6 9
xy
hours. 6.30. A tank has a leak at its bottom which empties it
x+y at 6 litres/minute. It also has a filling tap which
In this problem, x = 20 and y = 30 can fill the tank in 6 hours. The tank takes 18 hours
to become full. Find the capacity of the tank.
xy
∴ = 12
x+y Sol: Let the time that would be taken by the leak to
empty the full tank be x hours.
6.26. Pipes X and Y take 10 minutes and 20 minutes 1 1 1
∴ − =
respectively to fill an empty tank. Pipe Z takes 6 x 18
40 minutes to empty a full tank. Find the time x=9
taken to fill the empty tank if all the three pipes ∴Capacity of the tank = (6) (9) (60)
are opened simultaneously. = 3240 litres.

Triumphant Institute of Management Education Pvt. Ltd. (T.I.M.E.) HO: 95B, 2nd Floor, Siddamsetty Complex, Secunderabad – 500 003.
Tel : 040–27898195 Fax : 040–27847334 email : info@time4education.com website : www.time4education.com SM1001906/73
Concept Review Questions
Directions for questions 1 to 35: For the Multiple Choice Questions, select the correct alternative from the given
choices. For the Non-Multiple Choice Questions, write your answer in the box provided.

1. If ten men can do a job in ten days, what fraction of 13. X and Y can complete a job in 30 days. If X can
the job can be done by one man in one day? complete it in 60 days, find the number of days
1 1 1 1 taken by Y to complete it.
(A) (B) (C) (D) (A) 45 (B) 90 (C) 75 (D) 60
10 100 20 2
14. Adam can complete a job in 25 days. Adam and
2. A man can eat an apple in one day. How many
Chris together can complete it in 93/8 days. In how
apples can six men eat in six days?
many days can Chris alone complete the job?
(A) 125/8 (B) 10 (C) 25 (D) 15

15. A works 4 times as fast as B. If B completes a job in


3. If a man cuts the grass of a lawn in T minutes, what
60 days, then in how many days can A and B
part of the lawn can he cut in 30 minutes?
together complete the same job?
(A) 30/T (B) T-30 (C) T/30 (D) 30-T
(A) 24 (B) 12 (C) 18 (D) 15
4. X men can complete a work in 120 days. If there
16. A can complete a piece of work 3 times as fast as B. If
were 10 men more, the work would be completed in
A and B together can complete the work in 6 days, how
20 days less. Find the value of X.
many days would B alone take to complete the work?
(A) 8 (B) 12 (C) 4 (D) 24

17. P and Q working together, can complete a job in


5. Ten men can do a piece of work in 15 days. How
48 days. The ratio of their rates of doing work is
many men are needed to complete a work which is
3 : 5. Find the time taken by the faster person to
five times as large as the first one, in 10 days?
complete the job. (in days)

6. Nine men can complete a job in 15 days. If a man


18. P can complete a job in 60 days. Q is 25% less
works thrice as fast as a woman, find the number of
efficient than P. Find the time in which Q can complete
days taken by 15 women to complete the job.
it (in days).
(A) 20 (B) 24 (C) 27 (D) 36
(A) 75 (B) 80 (C) 90 (D) 45
7. A man can complete a job in four days working ten
19. The ratio of the rates of doing work of P, Q and R is
hours a day. In how many days can he complete it if
3 : 4 : 5. If they completed a job working together,
he works eight hours a day?
what part of it did P complete?
1 20
(A) (B)
4 47
8. Three men can complete a job in 3 days, working
3 hours a day. In how many days can 9 men 1
complete it working 1 hour a day? (C) (D) None of these
3
(A) 6 (B) 1.5 (C) 3 (D) 9
20. Amar, Bharat and Charu can complete a job in
9. If X’s rate of doing work is 40% more than that of Y, 12, 24 and 24 days respectively. If they all work
find the ratio of their rates. together, how long will they take to complete the
(A) 7 : 5 (B) 5 : 7 (C) 49 : 25 (D) 25 : 49 same work?
(A) 18 days (B) 6 days
10. The times taken by X and Y to complete a job are in (C) 20 days (D) 16 days
the ratio of 5 : 6. Find the ratio of the work they can
complete in an hour. 21. Anand, Bhanu and Chandra can complete a work in
(A) 6 : 5 (B) 5 : 6 (C) 25 : 36 (D) 1 : 1 4 days. Anand and Chandra can do the same work
in 8 and 16 days respectively. How many days will
11. The ratio of the times taken by A, B and C to Bhanu alone take to complete the same work?
complete a job is 3 : 4 : 6. Find the ratio of the work (A) 16 (B) 20
they can complete in an hour. (C) 8 (D) 12
(A) 6 : 4 : 3 (B) 4 : 3 : 2
(C) 2 : 3 : 4 (D) 3 : 4 : 6 22. P and Q can complete a job in 12 days. Q and R
can complete it in 20 days. R and P can complete it
12. A can complete a job in 60 days. B can complete it in 15 days. Find the time taken by P, Q and R
in 15 days. Find the number of days taken by them working together, to complete it.
to complete the job, if they work together. (A) 10 days (B) 20 days
(C) 5 days (D) 8 days

Triumphant Institute of Management Education Pvt. Ltd. (T.I.M.E.) HO: 95B, 2nd Floor, Siddamsetty Complex, Secunderabad – 500 003.
Tel : 040–27898195 Fax : 040–27847334 email : info@time4education.com website : www.time4education.com SM1001906/74
23. P and Q can complete a job in 10 days. Q and R 29. Find the number of days for which 120 kg of ration
can complete it in 12 days. P and R can complete it will be sufficient for a family of 6 members if each
in 20 days. Who is the slowest of the three workers? member consumes 2.5 kg of rations per day.
(A) P (B) Q
(C) R (D) Cannot be determined

24. Somu and Ramu can do a job in 60 and 40 days 30. Two pipes X and Y can fill a cistern in 6 and 12 minutes
respectively. They earned `450 for completing work respectively. How long will it take to fill the cistern, if
together. What is the share of Somu? (in `) both the pipes are opened simultaneously?
(A) 9 minutes (B) 4 minutes
(C) 8 minutes (D) 3 minutes

25. Gautham and Karan can complete a job in 4 hours 31. Tap A can fill an empty tank in six hours. Tap B can fill it
and 12 hours respectively. If they work alternatively in nine hours. Find the time taken (in hours) if the
for one hour each, with Gautham starting the job, two taps are opened together to fill the tank.
when will the job be completed? (in hours)

32. Tap X can fill a tank in 10 hours. Tap Y can fill it in


15 hours. If the two taps fill the tank together, what
26. X and Y can do a piece of work in 12 days and 18 days fraction of the tank is filled by X?
respectively. If they work on alternate days starting with (A) 1/10 (B) 1/6 (C) 2/3 (D) 3/5
X how many days will they take to finish the job?
(A) 141/2 (B) 141/3 33. Pipe A can fill an empty tank in 9 hours. Pipe B can
(C) 151/3 (D) 151/2 empty a full thank in 18 hours. If both pipes are
opened simultaneously when the tank is empty, find
27. A can complete a job in 20 days. B can complete it the time taken to fill the tank. (in hours)
in 30 days. If they work on alternate days, the job (A) 24 (B) 27 (C) 18 (D) 36
would be completed in the
(A) least number of days if A starts the job. 34. It takes 3 hours to fill a tank but due to a leakage it
(B) least number of days if B starts the job. takes 4 hours. How long would the leak take to
(C) the same time irrespective of whoever starts the empty a full tank?
(A) 14 hours (B) 16 hours
job.
(C) 10 hours (D) 12 hours
28. X can complete a job in 6 days. Y can complete it in 35. A tank can be filled by two pipes individually in
10 days. If they work on alternate days, the job 30 and 20 minutes respectively. There is an
would be completed in the emptying pipe which can empty the full tank in
(A) least number of days if X starts the job. 60 minutes. If all three pipes are opened
(B) least number of days if Y starts the job. simultaneously then, how much time does it take to
(C) the same time irrespective of whoever starts the fill the empty tank? (in minutes)
job.

Triumphant Institute of Management Education Pvt. Ltd. (T.I.M.E.) HO: 95B, 2nd Floor, Siddamsetty Complex, Secunderabad – 500 003.
Tel : 040–27898195 Fax : 040–27847334 email : info@time4education.com website : www.time4education.com SM1001906/75
Exercise – 6(a)
Directions for questions 1 to 30: For the Multiple Choice Questions, select the correct alternative from the given
choices. For the Non-Multiple Choice Questions, write your answer in the box provided.

1. Anwar would have to incur `600, `900 and `1200 as 9. Raman can do a piece of work in half the time taken
the expenses if he got a job done by A, B and C by Kapil. Sunil can do the same work in one-third of
respectively. The daily wages of A, B and C are the time taken by Raman. All three of them work on
`100, `60 and `40 respectively. Find the cost to it for 30 days after which Kapil leaves. Sunil and
Anwar of getting the job done by all three of them (in `). Raman complete the remaining work in 18 more
(A) 600 (B) 800 (C) 750 (D) 900 days. How many days would it take for Raman alone
to complete the total work?
2. A group of 50 salesmen plan to achieve their target (A) 52 (B) 414 (C) 138 (D) 207
for the next 30 days by working 12 hours a day. Due
to various reasons they put in only 10 hours a day 10. The efficiency of a man is reduced by half every
for the first 15 days. Now, if 10 men leave and the two hours. At maximum efficiency, he could have
rest continue working for only 10 hours a day, how completed the job in 150 hours. How many hours
many days more than the initially estimated time will does it take him to complete the job, if his efficiency
they require to meet their target? becomes maximum after every 8 hours, and then
(A) 11¼ (B) 12¾ (C) 13¼ (D) 13¾ reduces as mentioned above?

3. P and Q can complete a piece of work in 20 days,


Q and R in 15 days and P and R in 12 days. In how
many days can P, Q and R respectively complete
11. A man starts a piece of work. Starting from the
the work if they work individually?
second day onwards, every day a new man joins.
(A) 30, 60, 20 (B) 45, 60, 180
With every new man joining, the work that each man
(C) 30, 45, 90 (D) 45, 60, 20
can do per day doubles. The work is completed in
4. A team of two people A and B, who can complete a 5 days. On which day would they have completed
piece of work together in 40 days, begin the work. the work, if the work that each of them could do
C joins them after 8 days and the work gets per day had remained constant?
completed in 24 days after C joins them. In how
many days can C alone complete the work?

12. Beginning with the second day, the amount of work


Ram can do per day keeps doubling everyday he
5. Akbar can do a piece of work in 40 days and Ajay works. Saleem can always do the work done by
can do the same in 60 days. Akbar works on the Ram in 40% less time. Owing to this increasing
piece of work alone for some days. Then Ajay joins ability, Saleem and Ram complete the job together
him and together they complete the work 10 days in 2 days. How many days would it have taken if
earlier than the time Akbar alone would have taken. their efficiencies had remained constant?
After how many days did Ajay join Akbar? (A) 4 (B) 6 (C) 5 (D) 3
(A) 19 (B) 12 (C) 15 (D) 20
13. Vivek, Rameshwar and Bhuvan divide a work
6. P can complete a job in 27 days. He starts it and amongst themselves in the ratio of 2 : 3 : 5. Their
after three days Q joins him. They work together for rates of work are in the ratio 1 : 2 : 3. It takes Vivek
six days. P then leaves and R takes his place. 12 days to complete his part. What is the amount of
Q and R complete the job in 12 more days. If Q work completed by them in 8 days from the start?
takes at most 54 days to complete the job, then
(A) 29/40 (B) 11/45
which of the following cannot be a possible value of
(C) 4/5 (D) 31/45
the number of days taken by R to complete it?
(A) 36 (B) 34 (C) 40 (D) 38
14. Praveen, Shiva and Sunny, each working alone,
take 20, 10 and 5 days respectively to make
7. P, Q and R are three machines. They produce
electronic gadgets. The ratio of the rates of P, Q and 2000 hats, each. If the defects in their production
R is 3 : 4 : 5. P worked for 6 days, Q worked for are 10%, 20% and 40% respectively, approximately,
8 days and R worked for 10 days. They manage to how many days will it take them working together to
produce 400 gadgets. How many gadgets would make 10,000 non-defective hats?
they have produced in the same time if P’s rate (A) 22 (B) 20 (C) 18 (D) 20.4
doubled and Q’s rate tripled?
15. Pipe A can fill a cistern in 18 minutes. Pipe B can fill
the cistern in 24 minutes. Pipe C can empty the
cistern in 36 minutes. At 10 a.m., pipe A is opened.
8. A man builds 1/8th part of a wall every day. Out of At 10 : 10 a.m, pipe C is opened. At 10 : 15 a.m,
the length of the wall built per day, 20% falls off at the pipe B is opened. The time at which the cistern
end of the day till the wall is completely built. In how would be full is approximately
many days can he complete the construction of the wall? (A) 10 : 40 (B) 10 : 36 a.m.
(A) 8 (B) 10 (C) 91/5 (D) 94/5 (C) 10 : 27 a.m. (D) 10 : 20 a.m.
Triumphant Institute of Management Education Pvt. Ltd. (T.I.M.E.) HO: 95B, 2nd Floor, Siddamsetty Complex, Secunderabad – 500 003.
Tel : 040–27898195 Fax : 040–27847334 email : info@time4education.com website : www.time4education.com SM1001906/76
16. A number of crabs are kept in a jar of height 40 cm. 24. P and Q are filling pipes which can fill a tank in
One crab climbs up by 3 cm in a minute and in the 15 and 20 minutes respectively. R is an emptying
subsequent minute it is pulled down by the other pipe which can empty the full tank in 30 minutes.
crabs by 1 cm. If this cycle of alternate up and down The three pipes are operated continuously one after
movements of the crab repeats until it reaches the the other in the order of P, Q and R, each being kept
top, then in how many minutes will the crab reach opened for 2 minutes until the tank is filled. After
the top of the jar? (Assume once it reaches the top it how much time will the tank be full?
is not pulled down)? (A) 30 minutes (B) 322/3 minutes
(A) 40 mins (B) 39 mins (C) 36 minutes (D) 34 minutes
(C) 382/3 mins (D) 371/3 mins
25. Two taps can normally fill a cylindrical tank in
17. Niranjan, Rajesh and Vinayak can complete a piece 16 hours and 48 hours. But a leak which can empty the
tank in 24 hours is present at (3 4 ) of the tank’s
of work in 10, 15 and 12 days respectively. All the th
three of them started working and after an integral
number of days (y) Niranjan stopped working and height from the base. Find the time taken to fill the
Rajesh stopped working exactly (y) days before the tank if the taps are opened simultaneously (in hours).
work got completed. If the number of days, taken to
complete the work is an integer, in how many days
did the work get completed?
26. A swimming pool is fitted with three pipes of uniform
rate of flow. The first two pipes operating
simultaneously can fill the pool in half the time that
18. Krishna and Rama can complete a piece of work in the third pipe alone takes to fill the tank. The second
20 days and 30 days respectively. With the help of pipe, to fill the pool, takes 12 hours more than the
Gopi, they complete the work in 6 days. If the total first pipe working alone and 8 hours more than the
amount paid for the work is `720, what are the daily third pipe working alone. Find the time taken by the
earnings of Gopi? three pipes individually to fill the pool (in hours).
(A) `40 (B) `480 (C) `360 (D) `60 (A) 12, 16, 18 (B) 16, 28, 20
(C) 12, 24, 16 (D) 8, 20, 12
19. P, Q and R together can complete a job in 5 days.
The wages paid to P, Q and R for completing the job 27. A, B and C are three taps connected to a tank. Time
were `4050, `5400 and `6750 respectively. In how taken by C to fill the tank is 9 2 times the time taken
many days can P working alone, complete the job?
(A) 20 (B) 24 (C) 30 (D) 36 by A and B to fill it. The time taken by A to fill it is
5 6 times the time taken by B and C to fill it.
20. Alok and Sachin agree to complete a piece of work A, B and C take 20 11 hours to fill it. Find the time
in 20 days. They also agree to forfeit double the
taken by B to fill it (in hours).
amount of wages corresponding to the uncompleted
part of work, if they fail. If Alok alone can complete the 1 2
(A) 3 (B) 5 (C) 6 (D) 10
work in 40 days and they lost 1/3rd of the pay due for 3 3
the total work, in how many days can Sachin alone
complete the work? 2
28. X and Y are filling pipes. If the bottom rds of a
(A) 30 (B) 60 (C) 40 (D) 80 3
tank is filled by X and the rest is filled by Y, the tank
21. (i) Rakesh and Ramesh take 30 days and 60 days
respectively to complete a job. They work on 2
will be filled in 14 minutes. If the bottom rds of the
alternate days to complete it with Rakesh 3
starting the job. Find the time in which the job is tank is filled by Y and the rest is filled by X, the tank
completed (in days). will be filled in 16 minutes. Find the time taken by
(A) 60 (B) 80 (C) 40 (D) 90 the two pipes to fill the tank together (in minutes).
(ii) If Rakesh and Ramesh had instead taken
10 days and 12 days respectively to complete
the job, find the time in which the job would
29. If a piece of work can be done by 6 men and
have been completed (in days).
8 women in 10 days or by 8 men and 22 women in
1 5 1 5 days, in how many days will 34 women do a piece
(A) 10 (B) 10 (C) 11 (D) 10
3 6 2 of work thrice as large?
22. A contractor was assigned a task to be completed in
120 days. He initially hired 48 men to complete the
task. But after 60 days, only two–fifths of the task
was completed. How many more men should he hire 30. There are t taps numbered 1, 2 and so on till t, each
after 60 days in order to complete the task on time? of which can fill a cistern. The rate of filling of the nth
tap is such that it is equal to twice that of all the taps
23. A tank is full of water. A drain pipe, which can empty from 1 to (n − 1) put together. If the 18th tap can fill the
the full tank in 60 minutes, is opened. 18 minutes empty cistern in 2 minutes, then find the time in which
later another pipe which can fill the empty tank in the 15th tap alone can fill the empty cistern.
30 minutes is opened. After how much time, in total, (A) 27 minutes (B) 34 minutes
is the tank full again (in minutes)? (C) 54 minutes (D) 72 minutes
(A) 18 (B) 20 (C) 36 (D) 40
Triumphant Institute of Management Education Pvt. Ltd. (T.I.M.E.) HO: 95B, 2nd Floor, Siddamsetty Complex, Secunderabad – 500 003.
Tel : 040–27898195 Fax : 040–27847334 email : info@time4education.com website : www.time4education.com SM1001906/77
Exercise – 6(b)
Directions for questions 1 to 45: For the Multiple Choice Questions, select the correct alternative from the given
choices. For the Non-Multiple Choice Questions, write your answer in the box provided.

1. Rajdeep and Pranav can do a piece of work in 10. Anushka takes m times the time taken by Bhanu
15 days and 24 days respectively. Pranav works and Chawla to complete a job. Bhanu takes m times
alone for 10 days and leaves. In how many days can the time taken by Anushka and Chawla to complete
Rajdeep complete the remaining work? it. Chawla takes m times the time taken by Anushka
(A) 8 (B) 8¾ (C) 8¼ (D) 7¼ and Bhanu to complete it. Find m.
(A) 1 (B) 2 (C) 3 (D) 1/2
2. Raj can build a wall in 18 days and Kiran can do the
same in 30 days. After Raj had built half the wall, 11. A gas station needs 80 people to fill air in
Kiran joins him. What is the total number of days 20,000 tyres in 84 days. How many people does the
taken to build the wall? gas station need to fill air in 30,000 tyres in 63 days?
(A) 24 (B) 145/8 (C) 151/2 (D) 161/2

3. Raman and Rajan together complete half the work


in 18 days. Rajan and Rajiv complete the remaining 12. A frog was at the bottom of a 80 m deep well. It
half in 12 days. If Rajiv alone can complete the work attempted to come out of it by jumping. In each jump
in 36 days, then how many days will it take for it covered 1⋅15 m but slipped down by 0⋅75 m. Find the
Raman alone to complete the work? number of jumps after which it would out of the well.
(A) 198 (B) 201 (C) 200 (D) 199

13. The work done by Ananath in 12 hours is equal to


4. Kaushik is one and a half times more efficient than the work done by Anand in 15 hours, which in turn is
Ravi. Kaushik can do a piece of work in 20 days. equal to the work done by Arjun in 20 hours. If
What portion of the total work can both of them working together they complete the work in
together complete in 10 days? 10 hours, in how many hours can each of them,
(A) 3/10 (B) 4/5 (C) 9/10 (D) 7/10 working alone, complete the work?
(A) 20, 30, 40 (B) 30, 40, 20
5. Had then been one men less, then the number of days (C) 30, 24, 40 (D) 24, 30, 40
required to do a piece of work would have been one
more. If the number of man-days required to complete 14. Pradeep can work 2/3 times as fast as Abishek and
the work is 56, how many workers were there? Antony together. Pradeep and Antony together can
work twice as fast as Abishek. If Antony alone takes
45 days to complete a job, how long (in days) would
Pradeep and Abishek individually take to complete
6. In 8 days, Peter can do as much work as Pan can the job?
do in 12 days. To do a certain job both together take (A) 30, 36 (B) 36, 30
36 days. In how many days can Pan, working alone, (C) 45, 60 (D) 60, 45
complete the job?
(A) 60 days (B) 80 days 15. Two men or 5 women can complete a piece of work
(C) 108 days (D) 90 days in 15 days. In how many days can 4 men and
5 women complete the same work?
7. X can complete a job in 36 days and Y can complete
(A) 8 (B) 7 (C) 5 (D) 4
it in 45 days. Z can complete the job in z days.
Z started the job. After 28 days, X and Y joined. The 16. The work done by a man in 3 days is equal to the
job was completed in 4 more days. Find z. work done by four boys in 2 days. How many men
are needed to do a piece of work in 48 days which
72 boys can do in 16 days?
8. P can complete a job in 24 days and Q can do it in
72 days. P, Q and R start the job. After six days,
P leaves. In 12 more days, Q and R complete the 17. A rectangular tank has dimensions 5 m x 3 m x 2 m.
job. Find the time (in days) in which R can complete There are three inlet pipes – P, Q, R, which have
another job, which is six times as big. filling rates of 2 m3/hr, 3 m3/hr and 5 m3/hr
(A) 108 (B) 144 (C) 216 (D) 72 respectively. At 9:00 a.m, when the tank was empty,
9. Working in pairs, PQ, QR and RP can complete a P was opened. Q was opened at 9: 30 a.m. and R
job in 24 days, 20 days and 30 days respectively. was opened at 10:30 a.m. The time at which the
Find the respective times taken by P, Q and R tank would be full is _____.
individually to complete the same job (in days). (A) 11:54 a.m. (B) 11:48 a.m.
(C) 12:42 p.m. (D) 12:54 p.m.
240 240
(A) 48, 80, (B) 80, 48, 18. A man, a woman and a boy can do a piece of work
7 7
in 2, 4 and 8 days respectively. How many boys
240 240 must work together with 1 man and 1 woman to
(C) 80, , 48 (D) 48, , 80 complete the work in 1 day?
7 7 (A) 5 (B) 4 (C) 2 (D) 1
Triumphant Institute of Management Education Pvt. Ltd. (T.I.M.E.) HO: 95B, 2nd Floor, Siddamsetty Complex, Secunderabad – 500 003.
Tel : 040–27898195 Fax : 040–27847334 email : info@time4education.com website : www.time4education.com SM1001906/78
19. 40 men who can complete a job in 96 days started a 27. Chakri and Bharat can complete a piece of work
job. After 24 days, 20 men joined. After 32 more individually in 24 days and 36 days respectively.
days, N men left. The remaining men completed the They work on alternate days starting with Chakri.
remaining work in 20 more days. Find the value of N. Further, they get a holiday after every four days and
after the holiday the person who worked on the last
day before the holiday now starts the work. After
how many days from the start will the work get
20. Jadeja, Bhangar, Balaji and Dravid can do a piece of completed?
work in 8, 16, 32 and 64 days respectively. Dravid (A) 142/5 days (B) 36 days
starts the work and Balaji joins him after 1/4th of the (C) 352/3days (D) 35 days
work is done, Bhangar joins them after half the work 28. A man can begin a work at his maximum rate; but
is done and Jadeja joins them after 3/4th work is afterwards the rate at which he works follows a
done. How many days does it take to complete the cyclic pattern. Every two hours, it reduces by half
work? but after 8 hours, it comes back to its maximum
(A) 30 (B) 32 (C) 252/35 (D) 2424/35 level. He can complete a job in 151 hours at his
maximum rate. How many hours would he take to
21. A machine of type A which has to produce a set of complete the job if his rate follows the cyclic pattern?
1500 bolts, can do so in 30 days. The machine (A) 301 (B) 601 (C) 641 (D) 321
breaks down after 10 days. A machine of type B
completes the remaining work in 10 days. In 29. Sonia can complete a piece of work in 20 days,
30 days how many bolts can both of them together while Priyanka and Anjali can do it in 30 days each.
produce? Sonia works on the first day. Sonia and Priyanka
work on the second day and Sonia, Priyanka and
Anjali work on the third day. Also Sonia works on the
fourth day, Sonia and Priyanka work on the fifth day
22. Machines P, Q and R can do a piece of work in 20, and Sonia, Priyanka and Anjali work on the sixth day
30 and 60 days respectively. Machines P, Q and R and this cycle is repeated till the work is completed.
start the work together. Machine P goes out of order In how many days is the work completed?
after 5 days. After 3 more days machines Q and R (A) 12 days (B) 131/3 days
also go out of order. Machine P got repaired by then (C) 14 days (D) 161/2 days
and it completes the remaining work. What portion
of the total work did machine P do? 30. (i) A man starts a job, working at the rate of 1 unit
(A) 3/4 (B) 11/20 a day. After each day, another man joins the
(C) 7/20 (D) 3/5 working group. The man who joins on the
xth day of the job works until the job is
23. A man started a job. Starting from the second day, completed, at x units per day. The job is
each day a new man joined with which the capacity completed in 4 days. Find the time it would have
of each man doubled. The job was completed in taken have the man who started the work to
6 days. On which day will the job be completed if the complete the job alone (in days).
joining of a new man on a day results in each man (A) 20 (B) 10 (C) 16 (D) 24
working at thrice the rate as he did on the previous
(ii) Four men start a job. After each day, a man
day?
leaves. The xth man who leaves works at
(A) 6th (B) 5th (C) 4th (D) 3rd
x units/day. The job is completed when the last
man leaves. What part of the total wages will
24. Twenty five persons start a job of digging over an area
the last man get?
of 330 m2. From the second day, a new person joins
the group an each day. Each person digs 1 m2 per day. 7 3 8 2
(A) (B) (C) (D)
Find the time taken to complete the job (in days). 15 5 15 3

31. Prakash, Pranay and Pramod working alone can do


a piece of work in 20, 30 and 60 days respectively.
25. Some workers have been divided into two groups – All three of them start the work together, but after
A and B – depending on their rate of doing work. x days Prakash leaves and then after y more days
Three workers from A and six from B take 20 days to Pranay leaves and Pramod completes the remaining
complete a job. Eight from A and 4 from B take work. If Pranay had not left, Pramod and Pranay
10 days to complete it. Find the time taken by one would have completed the remaining work in
worker from each group to complete it (in days). (y + 6) days after Prakash had left. If both Prakash
(A) 90 (B) 108 (C) 72 (D) 54 and Pranay had stayed, the work would have been
completed in (x + 6) days. What is the total number
26. Anil can complete a piece of work in 6 days and of days taken to complete the work?
Mukesh can complete it in 8 days each working
alone. Consider two cases in which they work on
alternate days. The first case when Anil starts and
32. In a farm, each cow eats twice as much grass as
the second when Mukesh starts. What is the
each sheep. The cost of grass for 10 cows and
difference of the number of days taken in the
40 sheep for 20 days is ` 900. Find the cost of grass
two cases?
for 20 cows and 10 sheep for 18 days (in `).
(A) 1/2 (B) 1/4 (C) 1 (D) 13/4
(A) 600 (B) 675 (C) 750 (D) 800
Triumphant Institute of Management Education Pvt. Ltd. (T.I.M.E.) HO: 95B, 2nd Floor, Siddamsetty Complex, Secunderabad – 500 003.
Tel : 040–27898195 Fax : 040–27847334 email : info@time4education.com website : www.time4education.com SM1001906/79
33. The cost of grass for 20 cows and 30 sheep for 41. N taps numbered from 1 to N are fitted to a tank.
30 days is `720. If the 30 sheep eat double the The rate at which the nth tap, for n = 1, 2, …….
grass eaten by the 20 cows, then what is the cost of N fills the tank, equals the sum of the rates of all the
grass eaten by 20 sheep in 15 days?(in `) taps numbered below it. If the sixth tap can fill it in
80 minutes, find the time in which the ninth tap can
fill it (in minutes).

34. George and Gagan together repair a bridge in


45 days and receive `13,500. If Gagan is three
times as efficient as George, what is the amount of 42. A pipe can fill a tank in 4 hours, while a leak which is
money he earns in 10 days? at one-fourth the height of the tank can empty upto
(A) `2,000 (B) `2,250 that part in 2 hours. If both are operated
(C) `2,500 (D) `2,750 simultaneously and initially the tank is full, then
when will it be one-fourth full?
35. Gokul, Govardhan and Ganesh can do a piece of (A) 2 hours (B) 2 1/3 hours
work in 10, 20 and 30 days respectively. They begin (C) 11/2 hours (D) 6 hours
a new job of similar nature and each of them works
on it for one third of the total period of work. If they 43. Two taps, which can fill a tank in 12 hours and
get `6,600 for the new job, how much should 36 hours, are opened simultaneously. When the
Govardhan get, given that the amounts distributed tank was supposed to be full, it was found that only
are in proportion to the work done by them? (5/6)th of it was full due to a leak at the bottom. Find
(A) `1,800 (B) `2,200 the time in which the remaining part of the tank
(C) `3,300 (D) `2,400 would be filled (in hours).

36. A piece of work when done by a man, a woman and


a child costs respectively `720, `810 and `1,080.
Their respective daily wages are `60, `45 and `30. 44. A pipe is filling a tank of 200 litres capacity at
If a family consisting of a husband, wife and their 5 litres/hr. Because of a leak it takes 80 hours more
child is engaged to complete this work, how much to fill the tank. What is the rate in litres/hr at which
would it cost? (in `) the water is leaking?
(A) 60 (B) 3 (C) 3.33 (D) 80

45. Three taps x, y, z can fill a tank in 18, 24 and


37. A group of men are building a wall. After half the 36 hours respectively. x is opened first and y is
wall has been built, double the number of men join opened after time ‘t’ and 'z' is opened after ‘2t’,
the original group. The wall gets completed measured from the time x was opened. If the time
6 days earlier than scheduled. What is the total taken for the tank to fill up is 9/2 hours less than
number of days the initial group of men would have double the time that it would have taken had x, y, z
taken to complete the wall? been opened simultaneously, what is the time in
(A) 18 (B) 16 (C) 14 (D) 12 hours after which y was opened?
(A) 4 (B) 43/4 (C) 44/5 (D) 41/2
38. Six small pumps and three large pumps are fitted to
rd Directions for questions 46 to 55: Each question is
a tank. Each small pump works at 2 3 the rate of followed by two statements Ι and ΙΙ. Indicate your
each large pump. If all the pumps work together, responses based on the following directives:
what fraction of the time taken by a single large
pump, will they take to fill the tank? Mark (A) if the question can be answered using one
1 1 1 1 of the statements alone, but cannot be
(A) (B) (C) (D)
6 7 8 9 answered using the other statement alone.
Mark (B) if the question can be answered using
39. Two pipes A and B which can fill a tank in 20 and either statement alone.
30 hours respectively were opened simultaneously. Mark (C) if the question can be answered using
But there was a leak and it took 3 hours more to fill Ι and ΙΙ together but not using Ι or ΙΙ alone
the tank. In how many hours can the leak empty the Mark (D) if the question cannot be answered even
tank? using Ι and ΙΙ together.

46. If ten men take six hours to do a piece of work, then


how long will five boys take to do the same work?
40. Two pipes P and Q can fill a cistern in 12 and Ι. A boy works at 3/4 the rate of a man.
18 hours respectively. Both the pipes were opened ΙΙ. Five men and five boys take ten hours to do the
at 10:00 a.m. and the cistern was full at 6:00 p.m. work.
What could be the minimum possible duration for
which one of the pipes must have been closed 47. How long will it take for the tank to get filled when
during that interval? the taps A and B are opened simultaneously?
(A) 4 hours Ι. Tap A is a filling tap, and can fill it in two hours.
(B) 21/2 hours ΙΙ. Tap B is an emptying tap, and can empty it in
(C) 2 hours three hours.
(D) 4/3 hours
Triumphant Institute of Management Education Pvt. Ltd. (T.I.M.E.) HO: 95B, 2nd Floor, Siddamsetty Complex, Secunderabad – 500 003.
Tel : 040–27898195 Fax : 040–27847334 email : info@time4education.com website : www.time4education.com SM1001906/80
48. What is the time taken to fill the tank, if the two pipes 52. Ajay and Bala completed a job working on alternate
A and B are turned on alternately for one minute each? days. Find the time taken by them to complete it.
Ι. The two pipes together fill the tank in six minutes. Ι. Had they worked together they would have
ΙΙ. One pipe can fill the tank in ten minutes. completed the job in 20 days.
ΙΙ. Times taken by Ajay and Bala working
49. A, B and C are paid a total amount of `529. How individually to complete a similar job are 30 days
much should A be paid? and 60 days respectively.
Ι. A and B together completed 19/23rd of the work.
ΙΙ. B and C together completed 8/23rd of the work. 53. Pipes A and B can fill a tank in t hours. A can fill it in
(t + a) hours. B can fill it in (t + b) hours. Find t.
50. From a tank full of water, the water is discharged Ι. 4a = 9b
into a tub through a pipe. What is the quantity of ΙΙ. ab = 144
water discharged from the tank in a minute?
Ι. The tank was emptied in 30 minutes. 54. Rohan and Sohan can complete a job in 12 days.
ΙΙ. The dimensions of the tub are 3 m × 2 m × 2 m. Who is the least efficient among Rohan, Sohan and
Mohan?
51. There are two groups of people P and Q. In P each Ι. Rohan and Mohan can complete the job in 15 days.
person works with the same efficiency. In Q each ΙΙ. Mohan and Sohan can complete the job in 20 days.
person works with the same efficiency. Is the time
taken by one person from P and one person from 55. Pipes A, B and C can be used for filing or emptying
Q to complete a job more than 6 days? with the same capacity. If A and B are used for filling
Ι. Two persons from P and three persons from and C is used for emptying, a tank would be filled in
12 6 hours. Find the time taken by C to fill it.
Q can complete the job in days. Ι. If all the pipes are used for filling, the tank would
5
be filled in 2 hours.
ΙΙ. Three persons from P and two persons from
ΙΙ. If B and C are used for filling and A is used for
68 emptying, the tank would be filled in 6 hours.
Q can complete the job in days.
35

Key

Concept Review Questions

1. B 6. C 11. B 16. D 21. A 26. B 31. 3.6


2. 36 7. 5 12. 12 17. 76.8 22. A 27. C 32. D
3. A 8. C 13. D 18. B 23. C 28. A 33. C
4. 50 9. A 14. D 19. A 24. 180 29. 8 34. D
5. 75 10. A 15. B 20. B 25. 6 30. B 35. 15

Exercise – 6(a)

1. C 9. D 17. 6 24. B
2. A 10. 320 18. D 25. 15
3. A 11. 16 19. A 26. C
4. 120 12. D 20. B 27. C
5. C 13. C 21. (i) C 28. 7.2
6. B 14. D (ii) B 29. 15
7. 728 15. D 22. 24 30. C
8. D 16. C 23. C

Exercise – 6(b)

1. B 9. C 17. D 25. C 32. B 40. D 48. A


2. B 10. B 18. C 26. B 33. 160 41. 10 49. A
3. 72 11. 160 19. 12 27. B 34. B 42. D 50. C
4. D 12. D 20. D 28. D 35. A 43. 1.8 51. A
5. 8 13. D 21. 4500 29. A 36. 810 44. C 52. B
6. D 14. A 22. D 30. (i) A 37. A 45. D 53. A
7. 40 15. C 23. B (ii) C 38. B 46. B 54. C
8. C 16. 9 24. 11 31. 28 39. 60 47. C 55. A

Triumphant Institute of Management Education Pvt. Ltd. (T.I.M.E.) HO: 95B, 2nd Floor, Siddamsetty Complex, Secunderabad – 500 003.
Tel : 040–27898195 Fax : 040–27847334 email : info@time4education.com website : www.time4education.com SM1001906/81
CHAPTER – 7
AVERAGES, MIXTURES AND ALLIGATIONS
AVERAGES If there are n items and they are denoted by Q1, Q2, Q3,
....., Qn, then the average of these n items is given by
"Average is a very simple but effective way of 1 n
representing an entire group by a single value.” Average = P + ∑ (Qi − P)
n i =1
"Average" of a group is defined as:
The extent to which this method will simplify the
Sum of all items in the group calculation will depend on the selection of the arbitrary
Average =
Number of items in the group value P. It should be selected in such a way that the
positive and negative deviations cancel out each other to
"Sum of all the items in the group" means "sum of the
the extent possible. Then the final figure left for division
values of all the items in the group".
will be relatively small making the division easier.
A batsman’s performance can be expressed as the For example, the cricketer that we considered above
average number of runs scored per innings rather than scored the following runs in seven innings: 35, 56, 45,
giving the scores in individual innings. For example, let 43, 67, 70 and 48. Now, to find his average, we take an
us say a cricketer scored the following runs in 9 different arbitrary figure, say 50 and first find the deviations of
innings in a year: 35, 56, 124, 29, 0, 87, 98, 45 and 75. each of the scores from this figure. The deviations of the
Then his average score (per innings) for the year is scores from 50 are –15, +6, –5, –7, +17, +20 and –2.
35 + 56 + 124 + 29 + 0 + 87 + 98 + 45 + 75 The sum of these deviations is +14.
= 61 Hence the average of the cricketer's scores is
9
14
Similarly, if there are 60 students in a class, instead of 50 + = 52
talking of the height of each individual student, we can 7
talk of "average" height of the class. The average height Please note that the number P (= 50 above) can be any
of the class of students is equal to the sum of the heights value. Let us work out the same example taking a
of all the students of the class divided by the number of different value for P. Let us take P equal to 45. The
students in the class. deviations of the scores from P are −10, +11, 0, –2, +22,
+25 and +3. The sum of these deviations is 49. Hence
Average is also called the "mean" or mean value of all the average is 45 + 49/7 = 45 + 7 = 52.
the values.
WEIGHTED AVERAGE
POINTS TO REMEMBER
When two groups of items are combined together, then
(i) If the value of each item is increased by the same we can talk of the average of the entire group. However,
value p, then the average of the group or items will if we know only the average of the two groups
also increase by p. individually, we cannot find out the average of the
(ii) If the value of each item is decreased by the same combined group of items.
value p, then the average of the group or items will
also decrease by p. For example, there are two sections A and B of a class
(iii) If the value of each item is multiplied by the same where the average height of section A is 150 cm and that
value p, then the average of the group or items will of section B is 160 cm. On the basis of this information
also get multiplied by p. alone, we cannot find the average of the entire class
(iv) If the value of each item is divided by the same (of the two sections). As discussed earlier, the average
value p (p ≠ 0), then the average of the group or height of the entire class is
items will also get divided by p. total height of the entire class
(v) The average of a group of items will always lie total number of students in the entire class
between the smallest value in the group and largest Since we do not have any information regarding the
value in the group - i.e., the average will be greater number of students in the two sections, we cannot find
than the smallest value and less than the largest the average of the entire class. Now, suppose that we
value in the group. are given that there are 60 students in the section A and
40 students in section B, then we can calculate the
AN EASY METHOD TO CALCULATE average height of the entire class which, in this case will be
AVERAGES 60 x 150 + 40 x 160
equal to = 154 cm.
As already discussed, the average of a group of items 60 + 40
whose values are given can be found out by the rule This average height 154 cm of the entire class is called
given at the beginning of this section. However, in most "weighted average" of the class.
of the cases, we do not need to perform such elaborate
additions and divisions. The calculation of averages can The above step in calculating the weighted average of
be simplified greatly by taking some arbitrary number (P) the class can be rewritten as below:
as a starting point, take the deviations (differences) of 60 x 150 + 40 x 160
the given items (Qi) from this arbitrary number, find the 60 + 40
average of all these deviations (Qi – P) and algebracially 60 40 3 2
add it to the arbitrary number (P) to give the correct = 150 + 160 = 150 + 160
average of the given items. 100 100 5 5

Triumphant Institute of Management Education Pvt. Ltd. (T.I.M.E.) HO: 95B, 2nd Floor, Siddamsetty Complex, Secunderabad – 500 003.
Tel : 040–27898195 Fax : 040–27847334 email : info@time4education.com website : www.time4education.com SM1001906/82
It is clear from the above step that we would have been If q1 is the quantity (or number of items) of one particular
able to calculate the average height of the entire class item of quality p1, and q2 be the quantity (or number of
even if we had not been given the number of students in items) of the second item of quality p2 are mixed together
the individual sections but only the ratio of the number of to give a new mixture, then the weighted average value
students in the two sections (which in this case is 3 : 2). (p) of the quality of the mixture is given by

Even if there are more than two groups of items to be


p1 q1 + p2 q2
combined, then also the weighted average can be p=
calculated by the same method. For example, if three q1 + q2
sections in a class have their average marks as
75, 76 and 79 respectively and their respective strengths Even if there are more than two groups of items mixed,
are 30, 35 and 35, then the average mark of the entire the weighted average rule can be applied. We will only
class is given by have to take figures (as shown in the formula for the
30 x 75 + 35 x 76 + 35 x 79 two groups) for all the groups in the numerator as well as
= 76.75 the denominator and calculate the weighted average.
30 + 35 + 35
For example, if there are four groups of quantities q1, q2,
The method of deviations we used for calculating q3 and q4 whose respective qualities are p1, p2, p3 and
averages can be applied to calculate weighted average p4, then the weighted average quality of the group can
also. Here, that method will involve finding out deviations be written as
from the arbitrarily chosen number and calculating the
weighted average of these deviations. In the above
example, if we take 70 as the arbitrary figure, then the p1 q1 + p2 q2 + p 3 q3 + p 4 q4
p=
deviations of the three observed values given from q1 + q2 + q3 + q4
70 are +5, +6 and +9. The weighted average of these
deviations is
30 x5 + 35 x 6 + 35 x 9 675 A mixture can also be a solution - that is, a liquid mixed
= = 6.75.
30 + 35 + 35 100 with another liquid which is normally water. The
concentration of the solution is expressed as the
Hence, the weighted average will be proportion (or percentage) of the liquid in the total
70 + 6.75 = 76.75 solution.

The arbitrary figure chosen can be any figure and if it is For example, if 10 litres of pure alcohol is mixed with
selected, as in the previous case, between the smallest 40 litres of water, then in a total solution of 50 litres, there is
and largest observed figures, some of the deviations will 10 litres of alcohol. Hence the concentration of this solution
be positive and some negative making the final division is 0.2 (= 10/50) or 20%.
relatively simpler. For example, in the above case, if we
take with 76 as the arbitrary figure, the deviations are Similarly, if 30 litres of pure milk is mixed with 10 litres of
–1, 0 and +3. Then the weighted average will be water, the concentration of this solution can be
expressed as 75% (= 30/40) milk or 25% water.
30 x (-1) + 35 x 0 + 35 x ( +3) 75
= = 0.75
30 + 35 + 35 100 We can also have two solutions mixed together to give a
Hence, the weighted average will be new solution. Such problems can also be handled in the
76 + 0.75 = 76.75 same manner as other mixtures. In the weighted
"Weighted Average" can be defined or calculated for any average rule, the quality of the constituents (p1, p2, etc.)
MIXTURE. will then be the concentrations of various solutions mixed
together.
MIXTURES
Examples
Mixing of two or more qualities of things produces a
mixture. When two items of different qualities are thus
mixed, the quality of the resultant mixture lies in between 7.01. Find the average of the scores of the tests
the qualities of the original constituent items, i.e., it will taken by Ram given that his scores were 78, 82,
be higher than the lowest quality and lower than the 84, 86, 87, 89, 93 and 95.
highest quality of the items being mixed.
Total score in all the tests
In the above example that we took, the "quality" that we Sol: Average score =
Number of tests taken
looked at was the height of the students. We could also
78 + 82 + 84 + 86 + 87 + 89 + 93 + 95
have taken their weights or the marks scored by them or = = 86⋅75
any other "quality" or "parameter" and calculated the 8
"weighted average" value of that particular "quality" for
the entire group. 7.02. The monthly incomes of Raja, his wife and their
son are `6000, `4000 and `1880 respectively.
Similarly, if two types of a product of different prices per
Find their average monthly income.
unit are mixed, the unit price of the resultant mixture will
lie between the prices of the two types that form the
mixture. Sol: Average income
6000 + 4000 + 1880
Here, the average quality is essentially the weighted = = ` 3960
3
average of the two constituent items.
Triumphant Institute of Management Education Pvt. Ltd. (T.I.M.E.) HO: 95B, 2nd Floor, Siddamsetty Complex, Secunderabad – 500 003.
Tel : 040–27898195 Fax : 040–27847334 email : info@time4education.com website : www.time4education.com SM1001906/83
7.03. Rajiv purchased three dozen mangoes at 7.08. The sales recorded by a salesman in 6 days of
`10 per dozen, two dozen mangoes at `15 per a week are as follows: `14,000 per day for the
dozen and five dozen mangoes at `16 per first 3 days, `13000 for the fourth day, `24000 for
dozen. Find the average cost per dozen of the the fifth day and `26000 for the sixth day. Find his
mangoes that he purchased. daily average sales over this period.

Sol: The cost of first three dozen mangoes Sol: Total sales for the first 3 days = (3) (14000)
= (3) (10) = `30 = `42000
The cost of next two dozen mangoes Sales on the fourth day = `13000
= (2) (15) = `30 Sales on the fifth day = `24,000
The cost of next five dozen mangoes Sales on the sixth day = `26000
= (5) (16) = `80 Total sales on the 6 days = `105000
Total cost of the mangoes purchased = `140. 105000
Average cost per dozen Average sales per day = = `17500
6
Total cos t of Mangoes 140
= = = `14.
Number of dozens 10 7.09. Tarun earned an average of `1500 per month
from January to April in a year. He earned an
7.04. The average age of 5 men is 20 years. Their average of `1600 per month from May to
average age increased by 1 year when a new man October in that year. His earning in the month of
joined them. Find the age of the new man. December of that year was `300 more than his
earning in the month of November of that year.
Sol: Total age of 5 men = (5) (20) = 100 years His average monthly earnings for that year was
Total age of 6 men = (6) (21) = 126 years `1675. Find his earnings in the month of
The age of the new man = 126 – 100 November.
i.e. 26 years
Sol: Total earnings of Tarun from January to April
7.05. Six kilograms of wheat costing `18 per kg is
= (1500) (4) = `6000
mixed with nine kilograms of wheat costing
Total earnings of Tarun from May to October
`12 per kg. Find the price per kg of the mixture.
= (1600) (6) = `9600.
Sol: Total cost of 6 kg = (6) (18) = `108. Let his earnings in November be `x
Total cost of 9 kg = (9) (12) = `108. His earnings in December = ` (x + 300)
Average cost of the mixture 6000 + 9600 + x + x + 300
= 1675
Total cos t 108 (2) 12
= = = ` 14.40 per kg. x = 2100.
Total quantity 15
7.10. The average age of a group of friends is
7.06. A batsman had an average of 20 runs in the
37 years. If 6 new friends whose average age is
first 6 tests. In the second to the seventh tests
35 years join them, the average age of the
his average score was 21. If he scored a total of
entire group becomes 36 years. How many
30 runs in the first and the seventh tests, find
people were there in the group initially?
his score in the seventh test.
Sol: Total score in the first 6 tests = (6) (20) = 120. Sol: Let the initial number of people in the group be n.
Total score in the second to the seventh test The total age of the initial group of friends = 37n
= (6) (21) = 126. years
Let his scores in the first and seventh tests be a The total age of the six friends who joined the
and b respectively. Total score in the second to group = 35 × 6 i.e. 210 years.
the seventh test − Total score in the first 6 tests Given that, 37n + 210 = 36 (n+6)
=6 ⇒ 37n – 36n = 216 – 210
∴b − a = 6 ––––––––– (1) ∴ n = 6.
Given, b + a = 30 ---------(2)
Solving (1) and (2), b = 18. 7.11. The average age of the boys in a class is
18 years. The average age of the girls in that
7.07. The average marks of three sections in the class is 12 years. If the ratio of the numbers of
tenth class were 90, 120 and 150. If the number boys and girls in that class is 3 : 2, find the
of students in these sections are 30, 40 and 50 average age of the class.
respectively, find the average mark of the tenth
class. Sol: Let the number of boys in the class be 3x.
⇒ Number of girls in that class = 2x
Sol: Total mark of the first section = (90) (30) = 2700 Total age of the boys = (18) (3x) = 54x years.
Total mark of the second section = (120) (40) Total age of the girls = (12) (2x) = 24x years.
= 4800 Average age of the class
Total mark of the third section = (150) (50) = 7500 54 x + 24 x
= = 15 ⋅ 6 years
Average mark of class X =
Total mark 3 x + 2x
Number of students
2700 + 4800 + 7500 15000 7.12. If 10 litres of 80% alcohol is mixed with 15 litres
= = = 125. of 60% alcohol, find the concentration of the
30 + 40 + 50 120 resulting solution.
Triumphant Institute of Management Education Pvt. Ltd. (T.I.M.E.) HO: 95B, 2nd Floor, Siddamsetty Complex, Secunderabad – 500 003.
Tel : 040–27898195 Fax : 040–27847334 email : info@time4education.com website : www.time4education.com SM1001906/84
Sol: Quantity of alcohol in 10 litres = (0.8) (10) 7.13. How many kilograms of rice costing `6 per kg
= 8 litres. Quantity of alcohol in 15 litres = (0.6) must be mixed with 6 kg of rice costing `8 per kg
(15) = 9 litres. so that the resulting mixture costs `7⋅20 per kg?
Concentration of the resulting solution
8+9 Sol: Let x kg of rice costing `6 per kg be used for
= × 100 = 68% .
10 + 15 mixing.
Using alligation rule,
ALLIGATIONS x 8 − 7 ⋅ 20 2
= = ⇒x=4
6 7 ⋅ 20 − 6 3
We will take the Weighted Average rule discussed in the
previous section and rewrite the formula such that the 7.14. A bag contains a total of 120 coins in the
quantity terms come on one side and the price terms denominations of 50 p and `1. Find the number
come on the other side. If we do this we get the rule of 50 p coins in the bag if the total value of the
q1 p − p2 coins is `100.
=
q2 p1 − p
This is called the RULE OF ALLIGATION. This rule Sol: Let the number of 50 p coins be x.
 50 x 
+ 1 (120 − x )
connects quantities and prices in mixtures. This can also
Total value of the coins = ` 
q p −p  100 
be written as 1 = 2
q2 p − p1 50 x
In a descriptive manner, the Rule of Alligation can be + 120 − x = 100
100
Quantity of Cheaper ⇒ x = 40
written as
Quantity of Dearer Alternative method:
Rate of Dearer - Average Rate (100 ) (100 )
= Average value per coin =
Average Rate - Rate of Cheaper 120
500
= paise.
This rule is a very powerful rule and is useful in problems 6
on weighted averages and mixtures. This rule is also 500
useful in a number of problems which can be treated as 100 −
Using allegation rule,
x
= 6 = 1
mixtures and applied to parameters other than price
120 − x 500 2
also. We will take examples where alligation rule can be − 50
applied. 6
⇒ 2x = 120 – x ⇒ x = 40.
In actual practice, to apply alligation rule, we do not need
to remember the above formula at all. It can be made 7.15. A vessel has 60 litres of solution of milk and
very simple by representing the rule pictorially. The water having 80% milk. How much water should
above formula can be represented as follows: be added to it to make it a solution in which milk
is 60%?
Dearer Price Cheaper Price
Sol: Let the quantity of water to be added be x litres.
Average Price  80 
Quantity of milk in the vessel =   (60)
 100 
= 48 litres ⇒ 48 = 0⋅6 (60 + x)
(Average Price (Dearer Price ∴ x = 20
– Cheaper Price) – Average Price)
7.16. Two vessels contain mixtures of milk and water.
{We write the dearer and cheaper prices in one line with The ratios of milk and water are 3 : 10 and 4 : 5
some gap in between them. Then, we write the average in the first and second vessels respectively. In
price in between these two but in the line below the line
what ratio should they be mixed so that the
in which dearer and cheaper prices are written. Then,
resulting mixture contains milk and water in the
take the differences of quantities as shown along the
arrows and write along the same direction of the arrows ratio 79 : 155?
continued, i.e., in a diagonally opposite corner. The Sol: Milk content in the first milk solution
difference between the top left hand quantity (Dearer 3 3
Price) and Average Price has to be written at the bottom = =
3 + 10 13
right hand corner. Similarly, the difference between the
top right hand corner (Cheaper Price) and the Average Milk content in the second milk solution
Price has to be written at the bottom left hand corner. 4 4
= =
Now the ratio of the two quantities in the bottommost line 4+5 9
will give us the ratio of the quantities of Dearer and Given that the milk content of mixture of the
Cheaper varieties. Please note that since we took
79
Dearer Price on the top left corner, the ratio of the above two solutions =
bottom left figure to that of the bottom right figure will 79 + 155
give the ratio of Dearer Quantity to Cheaper Quantity} 79
=
234
Triumphant Institute of Management Education Pvt. Ltd. (T.I.M.E.) HO: 95B, 2nd Floor, Siddamsetty Complex, Secunderabad – 500 003.
Tel : 040–27898195 Fax : 040–27847334 email : info@time4education.com website : www.time4education.com SM1001906/85
Lower Higher If there is P volume of pure liquid initially and in
concentration concentration each operation, Q volume is taken out and
of milk of milk replaced by Q volume of water, then at the end of
3 4 n such operations, the concentration (k) of the
13 9 P − Q 
n
liquid in the solution is given by   =k
 P 
Concentration
79 mixture This gives the concentration (k) of the liquid as a
234 of milk PROPORTION of the total volume of the solution.

If the concentration has to be expressed as a


percentage, then it will be equal to 100k.
4 79 25 79 3 25
− = : − = If the volume of the liquid is to be found out at the
9 234 234 234 13 234
Ratio of mixing = 1 : 1 end of n operations, it is given by kP, i.e., the
concentration k multiplied by the total volume P of
7.17. In what ratio must Anand mix two varieties of the solution.
barley costing `24 per kg and `38 per kg so that
by selling the mixture at `40 per kg he would 7.18. A vessel has 400 litres of pure milk. 40 litres of
milk is removed from the vessel and replaced
1
make 11 % profit? by water. 40 litres of the mixture thus formed is
9 replaced by water. This procedure is repeated
once again. Find the percentage of milk in the
40 (100 ) resultant solution.
Sol: Cost price of the mixture =
1
100 + 11 Sol: Let v litre be volume of milk with a concentration
9
= `36 per kg of c1%.
If x litres of the solution is removed and
Cost of replaced with water, the new concentration is
Cost of dearer
cheaper variety  v−x 
variety  × c1  %
24 38  v 
Given that initial concentration is 100%. (pure milk),
Cost of the v = 400, x = 40 and the replacement is done
36 thrice.
mixture
∴ Concentration of milk in the resultant solution
3
2 12  400 − 40 
=   × 100 = 72.9%
 400 
∴ The ratio of the costs of the cheaper to the
dearer variety = 2 : 12 = 1 : 6.

Triumphant Institute of Management Education Pvt. Ltd. (T.I.M.E.) HO: 95B, 2nd Floor, Siddamsetty Complex, Secunderabad – 500 003.
Tel : 040–27898195 Fax : 040–27847334 email : info@time4education.com website : www.time4education.com SM1001906/86
Concept Review Questions
Directions for questions 1 to 35: For the Multiple Choice Questions, select the correct alternative from the given
choices. For the Non-Multiple Choice Questions, write your answer in the box provided.

1. The average of the numbers 41, 45, 49, 53, 57, 61, 14. Sachin wrote a total of six tests. His average mark in
65, 69 and 73 is those was 65. His average mark in the first five tests
was 60. Find his mark in the sixth test.

2. Find x, if the average of 17, 26 and x is 22.


(A) 18 (B) 22 (C) 1 (D) 23
15. The average age of a class of twenty students is
15 years. The average increases by three years, if
3. The average of N numbers is x. If each number is
the age of a teacher is included. What is the age of
doubled, find the new average.
the teacher?
(A) x (B) 2x (C) x/2 (D) x + 2
(A) 68 years (B) 86 years
(C) 78 years (D) 87 years
4. The average of a set of numbers is b. If each
number is decreased by 50%, find the new average.
16. The average age of a group of students is 16 years.
(A) b (B) 2b
If a 66 year old teacher joins the group, the average
(C) b/2 (D) None of these
increases by one year. What is the number of
students in the group?
5. Find the average of the numbers from 21 to 30.

6. Find the average of all the two digit numbers 17. A total of 55 chocolates were distributed among
divisible by 10. 13 children such that each girl gets 3 chocolates and
(A) 40 (B) 50 (C) 45 (D) 60 each boy gets 5 chocolates. Which of the following
is the number of girls?
7. Find the average of all odd numbers less than 50. (A) 8 (B) 3 (C) 4 (D) 5
(A) 26.5 (B) 25.5 (C) 26 (D) 25
18. The average salary per month of a man for the first
8. Find the average of all the multiples of 12 less than four months, next four months and the last
100. four months of a year are `6000, `8000 and `13000
(A) 48 (B) 54 (C) 60 (D) 66 respectively. Find his average salary per month in
that year (in `)
9. The average age of a class of 15 students is
15 years. A student whose age is 15 years joins the
class. Find the new average age of the class
(in years).
19. In an office there are 20 employees. The average
heights of the male employees is 180 cm. The
average height of the female employees is 170 cm.
10. The average age of a class of 20 students is Find the average height of all the employees (in cm).
(A) 172 (B) 174
20 years. A student whose age is 20 years leaves
the class. Find the new average age of the class (C) 176 (D) Cannot be determined
(in years).
20. In a class, there are 90 students. 30 of them are
(A) 20 (B) 21 (C) 22 (D) 19
boys. Their average mark in a test is 75. The
average mark of the girls in the test is 90. Find the
11. There are five members in a family. Their present
average age is 25 years. Find their average age average mark of the class in the test.
after five years.
(A) 26 years (B) 27 years
(C) 30 years (D) 35 years
21. In a class, there are 50 students. The average
weight of all the girls is 30 kg. The average weight of
12. The present average age of a group of friends is
65 years. What is their average age thirty years ago, 30 of the boys is 40 kg. Which of the following can
given that all the friends are at least 35-years old now? be a possible value of the average weight of the
entire class (in kg)?
(A) 35 years (B) 30 years
(C) 40 years (D) 25 years (A) 31 (B) 33 (C) 35 (D) 37

13. The monthly income of Amar, Bhavan and Chetan in 22. The average age of the boys in a class is ten years.
The average age of the girls in the class is
January 2005 were `4000, `4500 and `6500
eight years. There are 50% more boys than girls in
respectively. Find their average monthly income in
the class. Find the average age of the class (in years).
that month.
(A) `5000 (B) `6000
(C) `5500 (D) `6500
Triumphant Institute of Management Education Pvt. Ltd. (T.I.M.E.) HO: 95B, 2nd Floor, Siddamsetty Complex, Secunderabad – 500 003.
Tel : 040–27898195 Fax : 040–27847334 email : info@time4education.com website : www.time4education.com SM1001906/87
23. In a class there are 50 boys and 30 girls. 29. In what ratio should water and 84% milk solution be
The heights of boys and girls are in the ratio 4 : 3. mixed to obtain 48% milk solution?
Find the average height of the boys and girls in the (A) 4 : 3 (B) 9 : 13 (C) 2 : 3 (D) 3 : 4
class. (taken in cm)
(A) 155 30. Vessel A has 20 litres of a mixture of milk and water
(B) 160 having 75% milk. Vessel B has x litres of a mixture
(C) 165 of milk and water having 60% milk. The contents of
(D) Cannot be determined the vessels are mixed to form a mixture having
66% milk. Find x.
24. The average weight of all the students in a class
equals the average of the average weight of the
boys and that of the girls, which of the following
holds true? 31. A vessel has 30 litres of a mixture of milk and water
(A) The numbers of boys and girls in the class are equal. containing 80% milk. How many litres of water must
(B) The average weights of the boys and the girls be added to the vessel to reverse the ratio of
are equal. milk and water?
(C) At least one of (A) and (B) (A) 60 (B) 75 (C) 90 (D) 120
(D) Neither (A) nor (B)
32. Two solutions of sulphuric acid are mixed in the ratio
25. A vessel has 20 litres of a mixture of milk and water 2 : 3. The concentrations of sulphuric acid in the
having 60% milk. Five litres of pure milk is added to first and second solutions are 10% and 20%
the vessel. Find the percentage of milk in the respectively. Find the concentration of sulphuric acid
new solution. % in the final mixture.
(A) 16% (B) 14% (C) 12% (D) 18%
%
33. A milkman has 15 litres of pure milk. How many
26. In what ratio must two kinds of coffee which cost litres of water have to be added to it so that he gets
`80 per kg and `108 per kg be mixed such that the a 60% profit by selling at cost price?
resultant mixture costs `96 per kg? (A) 9 (B) 10 (C) 8 (D) 12
(A) 1 : 2 (B) 2 : 3 (C) 3 : 4 (D) 2 : 1
34. From 90 litres of pure milk, 9 litres is withdrawn and
27. A trader mixes two varieties of rice one costing at replaced by water. 9 litres of the mixture is then
`15 per kg and the other at `25 per kg so that by withdrawn and replaced by water. Find the ratio of
selling the mixture at `30 per kg he makes a profit. milk and water in the present mixture.
In what ratio he mixed them? (A) 19 : 81 (B) 19 : 100
(A) 1 : 2 (B) 1 : 1 (C) 81 : 19 (D) 81 : 100
(C) 2 : 1 (D) Cannot be determined
35. In the above question, find the quantity of milk in the
28. Five kilograms of sugar costing `15 per kg is mixed final mixture (in litres).
with ten kilograms of sugar costing `12 per kg. The
mixture is sold at 20% profit. Find the selling price of
the mixture (in `/kg).

Triumphant Institute of Management Education Pvt. Ltd. (T.I.M.E.) HO: 95B, 2nd Floor, Siddamsetty Complex, Secunderabad – 500 003.
Tel : 040–27898195 Fax : 040–27847334 email : info@time4education.com website : www.time4education.com SM1001906/88
Exercise – 7(a)
Directions for questions 1 to 30: For the Multiple Choice Questions, select the correct alternative from the given
choices. For the Non-Multiple Choice Questions, write your answer in the box provided.

1. Find the average of the first 20 multiples of 50. 9. The students of three classes, A, B and C take a
(A) 500 (B) 512.5 test. The average per student marks of the classes
(C) 525 (D) 537.5 A and B put together is 71. The average marks per
student of the classes B and C put together is 76.
2. The average of eleven distinct positive integers is The average per student marks of the class A and C
21. If the average of the first 6 is 23 and the average put together is 79. Find the range of the average
of the last six is 22, find the sixth integer. marks (p) of all the three classes put together.
(A) 72 < p < 76 (B) 73 < p < 75
(C) 71 < p < 77 (D) 73.5 < p < 77.5

3. The temperatures on Sunday, Monday and Tuesday 10. Just before the last match in a season, the total
were 28° C, 30° C and 32° C respectively. If the number of runs scored by Sachin Tendulkar added
temperature of each day from Tuesday to Friday up to 2100. In his last match he scored 101 runs. As
(both days inclusive) was 1° C less than that of th e a result his average score for the season went up by
following day, what was the difference in the one run. Find the total number of matches he played
average temperatures of the first four days of the in that season if he got out in every match.
week and that of the last three days of the week? (A) 31 (B) 5
(C) 71 (D) Either 31 or 71
(A) 4.5° C (B) 4.25° C
(C) 0.5° C (D) 4.75° C
11.
Number of Average
4. The average age of a board of 10 advisors of a Class
students marks
company is the same as it was 3 years ago, on A 32 83
account of the replacement of one of the older B 58 76
advisors by a younger man. How much younger is C 82 85
the new man than the director whom he replaced? D 48 90
(A) 24 years (B) 40 years
(C) 52 years (D) None of these Four classes A, B, C and D take an algebra test and
the average score of each of the classes is as
5. The average weight of A, B, C, D is 60 kg. The indicated above. Find the average score of all the
average weight of A, B, C, D, E is 57 kg. The weight four classes combined.
of F is 5 kg more than that of E. The average weight (A) 82.3 (B) 84.5
of B, C, D, E, F is 61 kg. Find the weight of A (in kg). (C) 85.6 (D) None of these

12. The average of n numbers is 16. If 5/8th of the


numbers are doubled and 3/8th of the numbers
6. The average of n numbers is a. If one of the become 10/3 times their original values, by what
numbers, y, is replaced by x, the average becomes percentage does the current average exceed the
b. Find the relation between n, a, x, y and b. original average?
(A) 20%
1 x−y 1 a −b (B) 30%
(A) = (B) = (C) 25%
n a−b n y−x
(D) Cannot be determined
1 x+y 1 a+b
(C) = (D) = 13. The average weight of all the students of classes
n a+b n x+y Ι and ΙΙ equals the average of the average weight of
the students of the two classes. There are twice as
7. The average weight of N boys in a group is 36 kg. If many students in class ΙΙ as in class Ι. The sum of
20 other boys whose average weight is 30 kg join twice the average weight of the students of class Ι
the group, the average weight of the group would be and the average weight of the students of class ΙΙ is
the same as what it would be if 5 boys whose 60 kg. Find the average weight of class Ι (in kg).
average weight is 40 kg leave the group. Find N.
(A) 10 (B) 15 (C) 20 (D) 25

8. The students of three classes X, Y and Z wrote a 14. Ajay attempted to add ten two-digit numbers. One of
test. The average marks of the students of X, Y and them, a, was the reverse of one of the others. If a
Z are 70, 81 and 90 respectively. The average was replaced by another two-digit number, b and the
marks of the students of X and Y is 76. The average reverse of a was replaced by the reverse of b and
marks of the students of Y and Z is 86. Find the the average was found, it would be 2.2 more. The
average marks of the students of all the three sum of the digits in b exceeds the sum of the digits
classes to the nearest integer. in a by
(A) 1 (B) 2 (C) 3 (D) 4

Triumphant Institute of Management Education Pvt. Ltd. (T.I.M.E.) HO: 95B, 2nd Floor, Siddamsetty Complex, Secunderabad – 500 003.
Tel : 040–27898195 Fax : 040–27847334 email : info@time4education.com website : www.time4education.com SM1001906/89
15. There are N students in a class. Their class teacher 23. There are two containers having mixtures of
gave them a task of finding the average of the hydrochloric acid and water. In container 1, the ratio
first N natural numbers and each of them left out a of hydrochloric acid and water is 1 : 2 and in
different number and found the average of the container 2 the ratio of hydrochloric acid and water
remaining numbers. The average of the averages is 4 : 1. Find the amount of the mixture that should
obtained by all the students was 21. Find N. be taken from container 1 in order to make 28 litres
(A) 39 (B) 40 (C) 41 (D) 38 of a mixture containing equal amount of water and
hydrochloric acid. (in litres)
16. Two alloys A and B contain copper and zinc in the
ratio 4 : 9 and 5 : 6 respectively. If equal weights of
the two are melted together to form a third alloy, find
the ratio of the weights of copper and zinc in the
24. The ratio of alcohol and water in three mixtures of
third alloy named C.
alcohol and water is 3 : 2, 4 : 1 and 7 : 3. If equal
(A) 109 : 167 (B) 113 : 164
quantities of the mixtures are drawn and mixed, the
(C) 109 : 177 (D) 107 : 158
concentration of alcohol in the resulting mixture will
be_____.
17. Two varieties of wheat are mixed in the proportion of
(A) 65% (B) 70% (C) 75% (D) 80%
3 : 4 and the mixture is sold at `28 per kg at a profit
of 40%. If the second variety of wheat costs `3 more
25. Fresh dates contain 90% water while dry dates
than the first variety of wheat, find the cost price of
contain 28% water. How many kilograms of dry
the first variety of wheat.
dates can be obtained from 36 kg of fresh dates?
(A) `128/7 per kg (B) `120/7 per kg

149
(C) `141/7 per kg (D) ` per kg
7
26. From 700 litres of milk, 70 litres are drawn and
18. A man buys milk at `4 per litre, mixes it with water and replaced with water. 70 litres of this mixture is then
sells the mixture at the same price. If his profit is 25%, again drawn and replaced with water and the
find the amount of water mixed with each litre of milk. process is continued two more times. Find the
(A) 0.25 litres (B) 0.5 litres quantity of milk present in the solution (in litres) now.
(C) 0.75 litres (D) 0.6 litres (A) 432.6 (B) 459.2
(C) 481.2 (D) 495.6
19. A vessel has 10 ml of a solution of milk and water
containing 20% milk. x ml of milk was added to the 27. From a vessel containing only alcohol, six litres are
vessel to reverse this ratio. y ml of water was then drawn and replaced with water. six litres of the
added to the vessel to reverse the ratio once again. mixture is now taken out and replaced with water.
Find x + y. The ratio of alcohol to water now is 9 : 16. How
many litres of alcohol was there initially?

20. The average salary of the employees of two groups


A and B in an organization is `4000. The average 28. In what proportion should milk and water be mixed
salary of the group A employees is `4800. The to reduce the cost of litre of milk from `18 per litre to
average salary of the group B employees is `3000. `16?
If there are 10 more employees in A than in B, find (A) 8 : 1 (B) 6 : 1 (C) 10 : 1 (D) 7 : 1
the number of group A employees.
(A) 35 (B) 50 (C) 45 (D) 40 29. A vessel contains a mixture of 100 litres of milk and
water. The concentration of milk is 90%. 10% of the
21. Raju bought 40 kg of a certain variety of coffee contents of the vessel are withdrawn and replaced
powder costing `240/kg and 60 kg of another with an equal amount of water. The minimum
variety of coffee powder costing `360/kg. He number of times that this procedure must be carried
mixed the two varieties of coffee powder and sold out such that the concentration of milk in the vessel
4 th 2
of the mixture at `360/kg and the rest at is less than 66 % is
5 3
`270/kg. The overall profit percentage made by .
Raju is
8 4
(A) 9 % (B) 11 % 30. Two vessels P and Q were partially filled with milk.
13 7 80% of the contents of P were transferred to Q. 60%
5 2 of the contents of Q were then transferred back to P.
(C) 5 % (D) 17 %
11 19 At this stage, the ratio of the quantities of milk in
P and Q is 23 : 12. Find the ratio of the initial
22. In what proportion can three varieties of sugar volumes of milk is P and Q.
priced at `10 per kg, `12 per kg and `18 per kg, be (A) 5 : 3 (B) 10 : 6
mixed so that the price of the mixture is `14 per kg? (C) 15 : 11 (D) 5 : 2
(A) 2 : 2 : 5 (B) 2 : 3 : 4
(C) 1 : 3 : 4 (D) 3 : 4 : 5

Triumphant Institute of Management Education Pvt. Ltd. (T.I.M.E.) HO: 95B, 2nd Floor, Siddamsetty Complex, Secunderabad – 500 003.
Tel : 040–27898195 Fax : 040–27847334 email : info@time4education.com website : www.time4education.com SM1001906/90
Exercise – 7(b)
Directions for questions 1 to 40: For the Multiple Choice Questions, select the correct alternative from the given
choices. For the Non-Multiple Choice Questions, write your answer in the box provided.

1. A’s weight equals the average weight of B, C and D. 9. There are eleven people in a group. The average
B’s weight equals the average weight of A, C and D. age of the oldest and the youngest is 11 years.
The average weight of C and D is 30 kg. Find the If any one person leaves the group, the maximum
average weight of A and B. and the minimum average of the remaining are
(A) 15 kg (B) 30 kg 12 years and 11 years respectively. Find the
(C) 60 kg (D) 45 kg average age of the entire group, in years.
7 5
(A) 9 (B) 10 (C) 11 (D) 11
2. The average weight of 18 students of a class is 11 11
15 kg. If a student leaves the class, the average
weight of the class drops by 0.8 kg. Find the weight 10. A boy finds the average of ten positive integers.
of the student (in kg) who left the class. Each integer is a two-digit number. By mistake, the
boy interchanges the digits of one of the numbers.
Due to this, the average becomes 1.8 less than the
previous one. Find the difference between the
3. In a group of 11 people, x is 32 years old and y is two digits of the number.
4 years younger than x. If x and y are replaced by (A) 1 (B) 2 (C) 3 (D) 4
two other people, the average age of the group drops
by 1 year. Find the average age of the two people 11. There are 60 persons. The age of each person
(in years) is a two digit number. The average age of
replacing x and y.
the persons is A years. One of the persons whose
(A) 26 years
age is ‘ab’ is replaced by a new person whose age
(B) 28 years
is ‘ba’. If the average age of the new group is
(C) 29 years 0.8A years. Find the maximum value of A.
(D) 24.5 years (A) 12 (B) 10.8
(C) 11.4 (D) Data inconsistent
4. A basket ball player played nine matches. The
average number of points he scored per match was 12. Just before the last match of a season, the total runs
16. His points in the ith match were two less than that of Rahul for the season added up to 1200. In his
in the (i – 1)th match. Find the average number of last match, he scored 20 runs and his average
points scored in the second and the eighth matches. (Number of runs / Total matches) remained unchanged.
(A) 8 (B) 12 (C) 16 (D) 18 Find the number of matches that he played in all.

5. There are nine two-digit numbers with distinct tens


digits. The units digit of each number is one less
than its tens digit. Find the average of the units 13. The members of a club meet for lunch every
digits. Monday. Last week, just before the bill was
presented, six of the members were called for an
official meeting and hence they left. The remaining
members were presented with a bill of `2160. It was
6. The average score in Mathematics of a class of customary to divide the bill equally. To cover the
36 students is 60. If the top two scores are share of those who left, each member had to pay
excluded, the average goes down by 2. If the `81 more. How many members met for lunch?
second highest score of the class is 90, find the (A) 16 (B) 12 (C) 18 (D) 24
highest score of the class.
(A) 98 (B) 96 (C) 97 (D) 95 14. Arjun ate a number of chocolates on each of the
5 week days of a certain week. On Tuesday, he ate
2 more than on Monday and 8 less than on
7. The average score of a class of 32 students is 70.
Wednesday. On Friday, he ate 4 more than on
If the top score and the least score, which differ by
Thursday and 6 less than on Wednesday. The
70, are excluded, the average score of the
average number of chocolates he ate on the
class drops by 1. Find the top score. first three days and the last two days are in the ratio 4 : 3.
Find the number of chocolates he ate on Thursday.
(A) 3 (B) 4 (C) 5 (D) 6

8. Of five numbers, the first number is thrice the third, 15. There are nine three-digits numbers with distinct units
the fourth number is two less than the first, the digits. Each number is reversed and the reversed
fifth number is one-seventh of the second and the number is subtracted from the original number. The
results were found to have an average of 0. If for each
second number is three less than thrice the first.
number, the hundreds digit is not less than its units
Find the fifth number, if the average of the numbers
digit, find the average of the hundreds digits of the
is 16.2.
greatest and the least numbers.
(A) 3 (B) 4 (C) 5 (D) 6 (A) 4 (B) 5 (C) 6 (D) 7

Triumphant Institute of Management Education Pvt. Ltd. (T.I.M.E.) HO: 95B, 2nd Floor, Siddamsetty Complex, Secunderabad – 500 003.
Tel : 040–27898195 Fax : 040–27847334 email : info@time4education.com website : www.time4education.com SM1001906/91
16. The average of the ages of a man and his only son 24. A sum of `7.75 is made up of 100 coins, which are
exceeds the average of the ages of his wife and his in the denominations of 5 paise and 10 paise. Find
son by 25%. The age of his son is 30 years less the number of 5 paise coins.
than the age of his wife whose age is 10 years less (A) 50 (B) 55 (C) 75 (D) 45
than his age. Find the age of his son (in years).
25. `8000 is lent in two parts. A part is lent at 10% p.a.
and the other part is lent at 20% p.a. Both parts are
lent at simple interest for a year. The total interest
17. There are two classes A and B. The average weight realized from them is `960. Find the difference of
of the students in class A is 40 kg. The average the parts (in `).
weight of the students in class B is 60 kg. A student, (A) 4800 (B) 2400 (C) 1600 (D) 6400
whose weight is x kg left A and joined B. As a result,
the average weight of A, as well as that of 26. A businessman lends `1800 in two parts, one at
B decreased. Which of the following must be true? 10% and the other at 12% interest. At the end of the
(A) 35 < x ≤ 40 (B) 40 < x < 60 year, the average interest he obtained worked out to
(C) 60 ≤ x < 65 (D) 30 < x ≤ 35 be 10.5%. Find the interest earned by the
businessman from the part which was lent at 10%.
Directions for questions 18 and 19: These questions (A) `135 (B) `150
are based on the following data. (C) `200 (D) `250

Each of four persons A, B, C and D, has a certain sum of 27. Raju covered a part of a total distance of 250 km at
money. The average amount with them is `200. If the 30 kmph and the remaining part at 80 kmph. He
money with A increases by 100% and that with covered the entire distance in 5 hours. Find the
B increases by 200%, then the average money with the distance he covered at 80 kmph (in km).
four increases by `62.50. If the percentage increases of
the amounts with A and B gets swapped, average
money with A and B will be `275.
28. A person travels 250 km in 5 hours in two parts. He
18. Find the money A originally has. travels the first part of the journey by scooter at an
(A) `150 (B) `200 (C) `175 (D) `180 average speed of 45 kmph, and the second part by
car at an average speed of 70 kmph. Find the
19. If C gives `50 to D, he will have twice the money distance he travelled by scooter.
that D would then have. Find the amount that (A) 160 km (B) 180 km
C originally has. (C) 150 km (D) 200 km
(A) `500 (B) `400
(C) `475 (D) None of these 29. A litre of pure milk is added to 6 litres of 30% milk.
Find the percentage water in the solution.
20. In an office, the average salary of the employees is (A) 50% (B) 55% (C) 65% (D) 60%
`8000. If the average salary of all the 50 clerks is
`3000 and the average salary of all the managers is 30. A 350 ml solution contains 10% of milk. How much
`10000, find the difference in the number of clerks water should be added to it in order to bring down
and the number of managers. the percentage of milk to 7% in the solution?
(A) 15 ml (B) 80 ml
(C) 125 ml (D) 150 ml

21. The average cost per fruit of 5 apples and 31. A vessel contains pure milk costing `20 per litre.
4 mangoes is `48 and the average cost per fruit of A certain quantity of water, which is free of cost, is
7 apples and 8 mangoes is `60. Find the total cost added to it. As a result the cost of the mixture
of 24 apples and 24 mangoes. becomes `18 per litre. Find the ratio of milk and
(A) `1332 (B) `2424 (C) `2664 (D) `3996 water in the mixture.
(A) 8 : 1 (B) 1 : 9
22. The students of three classes P, Q and R take a (C) 1 : 8 (D) 9 : 1
test. The average mark per student of P and Q put
together is 60. The average mark per student of 32. A milkman buys milk at `5 per litre. He mixes it with
Q and R put together is 64. The average mark per water and sells the mixture at the cost price and
student of P and R put together is 72. If the average thereby makes a profit of 25%. Find the quantity of
mark per student of all the classes put together is x, water he mixed with each litre of milk (in litres).
which of the following is a possible value of x?
(A) 61.5 (B) 61.9 (C) 62.5 (D) 68.3

23. Four classes P, Q, R and S take up a test. The ratio 33. Vessels A and B contain mixtures of milk and water.
of the number of students in P, Q, R and S is 2 : 3 : 6 : 4. The ratios of milk and water in A and B is 4 : 5 and
The ratio of the average marks of P, Q, R and S is 3 : 1 respectively . In what ratio should the contents
5 : 4 : 3 : 2.If the average marks of all combinations of of A and B be drawn and mixed, to obtain a mixture
three sections are considered, the greatest is 52. having milk and water in the ratio 3 : 2?
Find the least. (A) 27 : 28 (B) 23 : 24
(C) 11 : 12 (D) 35 : 36

Triumphant Institute of Management Education Pvt. Ltd. (T.I.M.E.) HO: 95B, 2nd Floor, Siddamsetty Complex, Secunderabad – 500 003.
Tel : 040–27898195 Fax : 040–27847334 email : info@time4education.com website : www.time4education.com SM1001906/92
34. Two vessels contain alcohol and water in the ratio (i) Find the capacity of the vessel. (in litres)
3 : 4 and 65 : 79. In what ratio should the solution in
the first vessel be mixed with the solution in the
second, so as to get a solution with alcohol and
water in the ratio 4 : 5? (ii) What is the least number of such additional
(A) 7 : 16 (B) 16 : 7 replacements required, so that the vessel
(C) 36 : 85 (D) 85 : 36 contains less than 75% milk?

35. A grocer had two varieties of rice, A and B. A and B


were priced at `20 per kg and `30 per kg
respectively. He mixed them in the same ratio as Directions for questions 41 to 45: Each question is
their prices and sold the mixture at `39 per kg. followed by two statements Ι and ΙΙ. Indicate your
(i) Find the cost of the mixture which he would responses based on the following directives:
have obtained if he had interchanged the
quantities of A and B (in ` per kg). Mark (A) if the question can be answered using one
(A) 20 (B) 22 (C) 24 (D) 26 of the statements alone, but cannot be
(ii) Find the percentage of profit that he currently answered using the other statement alone.
makes. Mark (B) if the question can be answered using
(A) 40% (B) 45% either statement alone.
(C) 50% (D) 62.5% Mark (C) if the question can be answered using
Ι and ΙΙ together but not using Ι or ΙΙ alone
36. Three varieties of rice, A, B and C costing `6/kg, Mark (D) if the question cannot be answered even
`9/kg and `12/kg are mixed together in a certain using Ι and ΙΙ together.
2
ratio. The mixture is sold at 66 % profit for `15/kg. Of 41. Is the average age in a class of 20 students more
3
the total of 100 kg of the mixture, 50 kg is variety B. than 15 years?
Find the quantity of variety A (in kg). Ι. Eighteen students in the class are each
15 years old.
ΙΙ. The average age of four of the students in the
class is less than 15 years.
37. Two varieties of tea costing `280 per kg and
`180 per kg are mixed in certain ratio to form 42. A certain alloy contains lead, copper and tin. How
varieties P and Q. P and Q are mixed in the ratio many kilograms of tin is contained in 60 kilograms of
1 : 2 to form variety R which is sold for `288 per kg the alloy?
at 20% profit. If P costs `200 per kg, in what ratio Ι. By weight, the alloy has 2/5th lead and
were the two varieties mixed to form Q? 3/16th copper.
(A) 2/1 (B) 5/2 (C) 4/1 (D) 8/3 ΙΙ. By volume, the alloy has 1/3rd lead and
1/3rd copper.
38. A local grocer mixed three qualities of coffee T1, T2
and T3 at `56 per kg, `64 per kg and `80 per kg in 43. If the arithmetic mean of six numbers is 30, then
the ratio 1 : 2 : 4. To 4 kg of this mixture, he added how many numbers are greater than 30?
p kg of T1 and 4p kg of T3. The final mixture so Ι. Three of the six numbers are equal to 40.
formed is sold for `87.60 per kg at 20% profit. Find p. ΙΙ. Three of the six numbers are equal to 20.
(A) 1/3 (B) 5/9 (C) 2/3 (D) 4/11
44. Between two mixtures x and y – each containing
39. A vessel is full of a mixture of milk and water, with
9% milk. Nine litres are withdrawn and then replaced milk and water, the concentration of which mixture is
with pure water. If the milk is now 6%, how much more?
does the vessel hold? Ι. x has three parts of water to seven parts of milk.
(A) 27 litres (B) 18 litres ΙΙ. y has seven parts of water to thirteen parts of
(C) 36 litres (D) 40 litres milk.

40. A vessel is filled to its capacity with pure milk. 45. In a certain class, x boys and y girls took a test. Was
Nine litres are withdrawn from the vessel and the average score of x boys greater than 83?
replaced with an equal amount of water. Nine litres Ι. The sum of all the scores was 85(x + y).
of the mixture is again withdrawn and then replaced ΙΙ. The average score of y girls was 86.
with an equal amount of water. After these changes,
the vessel contains 17.1 litres of milk less than it did
initially.

Triumphant Institute of Management Education Pvt. Ltd. (T.I.M.E.) HO: 95B, 2nd Floor, Siddamsetty Complex, Secunderabad – 500 003.
Tel : 040–27898195 Fax : 040–27847334 email : info@time4education.com website : www.time4education.com SM1001906/93
Key
Concept Review Questions

1. 57 8. B 15. C 22. 9.2 29. D


2. D 9. 15 16. 49 23. D 30. 30
3. B 10. A 17. D 24. C 31. C
4. C 11. C 18. 9000 25. 68 32. A
5. 25.5 12. A 19. D 26. C 33. A
6. B 13. A 20. 85 27. D 34. C
7. D 14. 90 21. D 28. 15.60 35. 72.9

Exercise – 7(a)

1. C 6. B 11. D 16. C 21. A 26. B


2. 39 7. A 12. D 17. A 22. D 27. 15
3. B 8. 82 13. 20 18. A 23. 18 28. A
4. D 9. D 14. B 19. 150 24. B 29. 3
5. 30 10. D 15. C 20. B 25. 5 30. D

Exercise – 7(b)

1. B 9. D 17. B 25. A 33. A 40. (i) 90


2. 28.6 10. B 18. A 26. A 34. A (ii) 1
3. D 11. D 19. D 27. 160 35. (i) C 41. C
4. C 12. 61 20. 75 28. B (ii) C 42. A
5. 4 13. A 21. C 29. D 36. 25 43. C
6. A 14. B 22. C 30. D 37. C 44. C
7. 120 15. B 23. 41.8 31. D 38. D 45. D
8. D 16. 5 24. D 32. 0.25 39. A

Triumphant Institute of Management Education Pvt. Ltd. (T.I.M.E.) HO: 95B, 2nd Floor, Siddamsetty Complex, Secunderabad – 500 003.
Tel : 040–27898195 Fax : 040–27847334 email : info@time4education.com website : www.time4education.com SM1001906/94
Solutions for SM1001906
Chapter – 1 9 (p + 3) = 63 ⇒ p + e = 7 --- (3)
(Simple Equations) subtracting (1) from (2),
p − e = −1 -- (4)
Concept Review Questions Adding (3) and (4),
2p = 6 ⇒ p = 3. Choice (B)
Solutions for questions 1 to 30:

1. Let the number be x 11. Let the costs of each pen and each eraser be `p and `e
x respectively
3 x − = 24 3p + 3e = 17
3
3p + 4e = 18
x=9 Ans: (9)
Adding both equations,
2. Let the number be x. 7 (p + e) = 35
1 1 1 1 p+e=5
4 + × × × x= x The cost of 1 pen and 1 eraser is `5 Ans: (5)
2 3 6 12
x x
⇒4+ = 12. 10m + 9a = 104 ---------------- (1)
36 12 27m + 25a = 285 ---------------- (2)
⇒ x = 72 Choice (C) Solving (1) × 3 and (2)
30m + 27a = 312
3. Let the two numbers be x and y. 27m + 25a = 285
x + y = 18 ------------- (1) ----------------------------
x−y=4 ------------- (2)
⇒ 3m + 2a = `27 Choice (C)
On Solving (1) and (2)
x = 11 and y = 7 Choice (A)
13. Let the costs of each chocolate, each milk shake and each
4. 3x + 6y = 12 ⇒ x + 2y = 4 cake be `ch `m and `c respectively.
4x + 8y = 16 ⇒ x + 2y = 4 2ch + 3m + 4c = 190
As both equations are identical, they have infinite solutions. 4ch + 8c = 320
Choice (D) The coefficients of ch and c are proportional, we can
determine m, but not ch or c.
5. 3x + 9y = 21 ⇒ 6x + 18y = 42 2ch + 4c = 160
6x + 18y = 45 ∴ 3m = 190 − (2ch + 4c) = 30
No value of x and y can satisfy both equations. ∴ m = 10. Choice (A)
Choice (A)
14. Two digit number = TU
6. Two equations a1x + b1y = c1 and a2x + b2y = c2 will have a
unique solution New number = TU4 = 100T + 10U + 4 Choice (B)
a b
If 1 ≠ 1 . As the given equations satisfy this condition, 15. Let the two digit number be xy
a2 b2 Given 2x + 11y = 10x + y
they have a unique solution. Choice (B) 5
⇒ 8x = 10y ⇒ x = y⇒x:y=5:4
4
7. 8x + y − 10 = 0 ------------- (1)
As 54 is the only number where the above condition is
4x + 2y − 13 = 0 ------------- (2)
satisfied, with the given ratio, the number must be 54
Solving (1) × 2 and (2) uniquely. Ans: (54)
16x + 2y − 20 = 0
4x + 2y − 13 = 0 16. The difference between any two digit number and the
− − + number formed by reversing its digits is equal to 9 times the
----------------------- difference of its digits
12x = 7 ∴ the required difference = 9 (3) = 27 Choice (B)
7 16
x= and y = Choice (D)
12 3 17. Let the number be abc with a > b
Difference of abc and cba = abc − cba
8. 3x + 9y + 12z = 18 = 100a + 10b + c − (100c + 10b + a) = 99 (a − c)
⇒ x + 3y + 4z = 6 ---------------- (1) ∴ 99 (a − c) = 198
and 2x + 3y + 4z = 8 ---------------- (2) a−c=2 Ans: (2)
Solving (1) & (2)
2x − x = 8 − 6 ⇒ x = 2 Choice (A) 18. The difference between a three digit number and the
number formed by reversing its digits is always divisible by
9. Let the costs of a puff and a samosa be `p and `s is 99 but is not always divisible by any greater number.
respectively Therefore, 99 is the greatest such number. Ans: (99)
5p + 2s = 48 . . . . . (1)
p = 2s . . . . . (2) x
19. Let the fraction be
(1), (2) ⇒ 5 (2s) + 2s = 48 ⇒ s = 4 y
p=8
x=y−5 ---------------- (1)
The cost of a puff is `8. Ans: (8)
and 4x = y + 1 ---------------- (2)
10. Let the costs of each pen and each eraser be `p and `e Solving (1) and (2)
respectively x = 2, y = 7
4p + 5e = 32 --- (1) 2
5p + 4e = 31 --- (2) ∴ the fraction is Choice (C)
7
Adding (1) and (2),
Triumphant Institute of Management Education Pvt. Ltd. (T.I.M.E.) HO: 95B, 2nd Floor, Siddamsetty Complex, Secunderabad – 500 003.
Tel : 040–27898195 Fax : 040–27847334 email : info@time4education.com website : www.time4education.com SM1001961/1
x Their total present age is 3x + 27 = 90
20. Let the fraction be ∴x = 21
y
Wife’s age is 42 years and man's age is 48 years.
y=x+2 Choice (A)
⇒x−y+2=0 ---------------- (1)
x 3 Exercise – 1(a)
also, =
y +1 4
⇒ 4x − 3y − 3 = 0 ---------------- (2) Solutions for questions 1 to 30:
Solving equations (1) & (2)
x = 9 and y = 11 1. 7(x + 2) + 3(y − 2) = 8 ------- (1)
∴ the fraction is 9/11. Choice (D) 3(x − 2) + 7(y + 2) = 8 ------- (2)
From (1)
21. Let the three consecutive numbers be 2x, 2(x + 1), 2(x + 2) 7x + 3y = 8 − (14 − 6)
1 1 7x + 3y = 0 ------- (3)
then × 3 (x + 1) = × 2(x + 2) From (2)
3 4 3x + 7y = 0 ------- (4)
⇒x=2 From (3), (4)
∴ the three numbers are 4, 6, 8 Choice (A) x = 0, y = 0 Choice (C)

22. Let the number of 50 np coins with Nalini be x. 2. 2x + 3y − 6z = 18 ------- (1)


Number of `1 coins with her will be (30 − x) 4
Amount = x × 50 + (30 − x) 100 = 2000 2y − x + 4z = 12 ------- (2)
3
⇒ x + 60 − 2x = 40 ⇒ x = 20
∴ number of `1 coins with her is (30 − 20) = 10 2
(1) ×   + ( 2) gives
Ans: (10) 3
2
23. Let Sekhar have x number of `500 notes with him, then the 4 y = (18 )   + 12
number of `100 notes with him is (22 − x). 3
⇒ x × 500 + (22 − x)100 = 7000 ⇒ 4y = 12 + 12
⇒ x = 12 Ans: (12) ⇒ 4y = 24 ⇒ y = 6 Ans: (6)

24. Let the amount with Amar be `a 3. x + 2y + 3z = 14 ------- (1)


Total amount with the others = ` (150 − a) 2x + 3y + z = 11 ------- (2)
3x + y + 2z = 11 ------- (3)
a = (150 − a )
1
(1) + (2) + (3) gives
4 6(x + y + z) = 36 ⇒ x + y + z = 6 Choice (A)
⇒ a = 30 Choice (C)
4. 3x + y − 3z = 11
25. Let the amounts P, Q and R be `p, `q and `r respectively. ⇒ 3(x − z) + y = 11 ------- (1)
p + q + r = 1250 2(x − z) + 5y = 29 ------- (2)
p = (q + r )= (1250 − p)
3 3 Let x − z = a
7 7 ⇒ 3a + y = 11 ------- (3)
⇒ 7p = 3750 − 3p ⇒ p = 375 Choice (D) 2a + 5y = 29 ------- (4)
(3) × 5 − (4)
26. Let Sreedhar’s age be s years ⇒13a = 26
s + 20 = 6(s − 20) ⇒a=2
⇒ 5s = 140 ⇒ s = 28 Choice (C) y=5
a=x−z
27. Let the present ages of Ramesh and Suresh be r years and ⇒y+x−z=5+2=7 Choice (A)
S years respectively
r = 3s 5. In order that the equations may be consistent the
r + 2 = 2 (s + 2) co-efficients of the corresponding terms shall be proportional.
⇒ r = 2s + 2 = 3s 32 5 4
s = 2r = 6 ⇒ = =
96 15 2p
∴ Ramesh’s present age is 6 years. Choice (D)
∴p = 6 Choice (B)
28. Let the present ages of Ashok and Bala be a years and b
years respectively 6. As the system has a solution, the coefficients are not
a − 4 = 4 (b − 4) proportional.
⇒ a = 4b − 12 . . . . . (1) 3 k +3

a + 3 = 3 (b + 3) k 6
⇒ a = 3b + 6 . . . . . (2) ⇒ k(k + 3) ≠ 18
(1), (2) ⇒ 4b − 12 = 3b + 6 If k (k + 3) = 3(6) or (− 6) (− 3)
⇒ b = 18 and a = 60 i.e., if k = 3 or − 6, the system does not have a unique
Ashok’s present age is 60 years. Choice (C) solution.
∴ Any value other than k = 3 and k = −6 will result in a
29. m = 20 years unique solution. Choice (D)
After x years, Mrudhula's age is 20 + x.
The twins will be of ages x each.
x
Total = 20 + x + x + x = 50 7. Let the fraction be .
⇒ x = 10 Ans: (10) y
⇒ 6y = 8x – 2
30. Let the man’s age 27 years ago be x. ⇒ 3y = 4x – 1 ------- (1)
His wife’s present age is 2x. 10x = 10y – 20
His present age is x + 27 ⇒x–y=–2 ------- (2)

nd
Triumphant Institute of Management Education Pvt. Ltd. (T.I.M.E.) HO: 95B, 2 Floor, Siddamsetty Complex, Secunderabad – 500 003.
Tel : 040–27898194/95 Fax : 040–27847334 email : info@time4education.com website : www.time4education.com SM1001961/2
(1) + 3 × (2) gives, 14. Let the initial amount = A
4x – 3x – 1 – 6 = 0 Cost of 1 apple = (A/10)
⇒x=7 Cost of 1 orange = (A/15)
Substituting in (1), Cost of 1 watermelon = (A/2)
y = 27/3 = 9 As all three types of fruits are bought, the minimum shall be
y 9 one.
Answer = = Choice (D) As oranges are to be maximum, others are one each.
x 7 Amount spent = 6A (as per data).
8. Let the speeds of John and Peter be x and y respectively Hence, the number of oranges
= [6A – {(A/10) + (A/2)}] / (A/15)
(in kilometres per hour)
= [6A – (6A/10)] / (A/15) = (54A/10) / (A/15) = 81
⇒ x = y + 10 - (1)
Choice (B)
x + 15 = 2y - (2)
y = 25 15. Let the present ages of the man and the son be m years
x = 35 and s years respectively.
∴ John’s speed is 35 km/hr Ans: (35) m − 10 = 2 (s − 10) + 35
m = 2s + 25
9. Let the number of girls in class A and class B be x and y
(i) let us say after x years the father will be twice his
respectively
son’s age.
x=y+7
m + x = 2 (s + x)
⇒x–y=7 - (1) 2s + 25 + x = 2s + 2x
x + 4 = 2y 25 = x Choice (C)
x – 2y = – 4 - (2)
Solving (1) and (2), we get (ii)Let us say after x years the father will be thrice his
y = 11, x = 18 son’s age
∴Number of boys in A = 35 – 18 = 17 2s + 25 + x = 3 (s + x)
Number of boys in B = 35 – 11 = 24 Choice (D) 25 = s + 2x
We have only 1 equation but 2 unknowns. So x cannot be
10. Let the number of marbles with Nitya and Satya be n and s
found. Choice (D)
respectively.
n+1=s−1 16. Data is tabulated below
⇒n−s+2=0 ---------------- (1) Bhangar Karthin
s + 1 = 2(n − 1) Runs x + 10 y
⇒ 2n − s − 3 = 0 ---------------- (2) Balls y + 5 x
Solving (1) & (2) x + y + 10 = 50
i.e. 2n − 2s + 4 = 0 x + y = 40→ (1)
2n − s − 3 = 0 y + 5 = x − 15
------------------------- x − y = 20 → (2)
⇒ s = 7, n = 5 Choice (B)  40 + 20 
⇒ x =   = 30
11. Let the initial amount with Gopi, Murthy and Hari in rupees  2 
be x, y and z respectively. Runs scored by Bhangar = 30 + 10 = 40 Ans: (40)
x
Gopi gives to Murthy y 10
2 17. Karthik’s run rate = = = 0.33 Choice (D)
x x x 30
They now have , + y and z
2 2
18. A + B + C + D = 90000 ---------------- (1)
Murthy gives half of his amount to Hari A + C + D = 4B ---------------- (2)
They now have also, B + D = 4/5 (A + C) ---------------- (3)
x x y x y From (1) & (2)
; + ; + + z
2 4 2 4 2 4B + B = 9000
Hari gives half his amount to Gopi. ⇒ B = `1800
x x y x z From (2) and (3)
Gopi now has  + + + +  and this is equal to x. B + D = 4/5 (4B − D)
2 8 4 4 2 5B + 5D = 16B − 4D
x x y z ⇒ 11B = 9D
⇒ x= + + +
2 8 4 2 11
⇒D= × 1800 = `2200 Choice (C)
3x y z 9
⇒ = +
8 4 2
19. Let the times for which P, Q, R worked on the project be
⇒ 3x = 2y + 4z = 2 x 45 = 90, as (y + 2z) is given equal to 45. p, q, r respectively (all in hours)
⇒ x = 30 Ans: (30)
2 2 22  4
12. Let the number of 100 gm weights be x and number of p= q , q = r ∴ p =  r = r
3 5 3  5  15
500 gm weights be y.
100x + 500y = 8000, as per data. p : q : r = 4 : 6 : 15
x + 5y = 80 → (1) 3
R worked for of the total number of hours that all the
the 2nd relation is x + y = 20 → (2) 5
Subtracting, 4y = 60, y = 15, x = 5 three worked. Choice (B)
Answer = 500 gm Choice (D)
20. Let the costs of an apple, a banana and an orange be `a,
13. If cost of a pencil is `5 then the only feasible values for cost
`b and `c respectively.
of a pen and cost of a ruler are 6 and 4 respectively.
4a + 6b + 8c = p
∴ Cost of 10 rulers = 40 5a + 8b + 11c = q
∴ Number of pencils he purchased with `40 is 8a + 16b + 20c = r
40 Going back from the options, only Choice (D) satisfy the
= =8 Ans: (8)
5 given equation. Choice (D)
Triumphant Institute of Management Education Pvt. Ltd. (T.I.M.E.) HO: 95B, 2nd Floor, Siddamsetty Complex, Secunderabad – 500 003.
Tel : 040–27898194/95 Fax : 040–27847334 email : info@time4education.com website : www.time4education.com SM1001961/3
21. Let the correct number be 10x + y (2) − (1) gives
∴10y + x − (10x + y) = 10x + y − 4 0.25 = 9(y − x) + 9.25
8y – 19x = −4 9(x − y) = 9
⇒ 19x – 8y = 4 ------- (1) x − y = 1 → (3)
3(x + y) = 10x + y − 10
7x − 2y = 10 ------- (2) 27. w = 10x + y + 0.25
Solving (1) and (2), we get So, the actual weight w = 10x + y + 0.25
x = 4, y = 9 So, in the actual weight after the decimal part 0.25 must
Number = 49, Product of the digits = 36 Choice (C) appear, and in the integral part, the ten’s digit should be
more than between the unit’s digit. Both these conditions
Solutions for questions 22 and 23: are satisfied for (A), not for choices (B) and (C).
Choice (D)
Let the number of toys sold by the shopkeeper be ‘ab’. Let the
selling price of each toy be ` ‘cd’. Let his initial stock be s. 28. In the indicated weight, ten’s digit is 1 more than the units
Actual stock left = s − ‘ab’. digit and this is satisfied by Choice (A) only. Choice (A)
Stock as shown on record = s − ‘ba’
s − ba = s − ab + 81 29. Present time = P(say)
Given that, P + t min = 12 p.m. ……….(1)
10a + b − (10b + a) = 81 P – 44 min = 9 a.m. + 7t min .……….(2)
a−b=9 P = 12 p.m. – t min
= 9 a.m + (7t + 44) min
∴ a = 9 and b = 0
⇒ 3 hr – 44min = 8t min
22. ab = 90 Choice (C) ⇒ 180 min – 44 min = 8t min
⇒ 17 = t ⇒ P = 11 : 43 a.m. Ans: (11, 43)
882 882
23. Mistaken sale price = = = 98 = `98 30. If the marks for each correct, each wrong and each
' ba' 09 unanswered question were 4, –1, 0 respectively he would
have got 25 marks. But he actually got 5, 0, 1 marks
Actual sale price = `89 Choice (A)
respectively. i.e., for each question, he actually got 1 mark
more. There were 35 questions. His actual score
24. Let the children be C10, C9, ……, C1 in decreasing order of
= 25 + 35 = 60. Ans: (60)
their ages.
Let each child have `x at the beginning. The following table Exercise – 1(b)
gives the amounts with each of them at the end of each
transaction. Solutions for questions 1 to 35:
1 → (x – 9), (x + 1) (x + 1) ………… (x + 1)
x x x
2 → (x – 9), (x – 7) (x + 2), (x + 2) ……. (x + 2) 1. (a) x+ + = +5
3 → (x – 9), (x – 7), (x – 5), (x + 3), (x + 3),………(x+3) 2 3 4
4 → (x – 9), (x – 8), (x – 5), (x – 3), (x + 4), (x + 4) …….. x x x
⇒ x + + − =5
(x + 4) 2 3 4
x(12 + 6 + 4 − 3 )
At the end, the total sum with children who have given ⇒ =5
some money 12
= (x − 9) + (x − 7) + (x − 5) + (x − 3) = 4x − 24 12 x 5 60
The total sum with those children who have not given any ⇒ x= = Choice (A)
19 19
money = 6 – (x + 4) = 6x + 24
∴6x + 24 = 2(4x − 24) (b) 99x + 101y = 400 ------- (1)
99y + 101x = 400 ------- (2)
⇒ 2x = 72 x = 36 Choice (A)
(1) + (2) gives
200(x + y) = 800
25. Let the hundreds, tens and units digits be x, y, z
⇒x+y=4 ------- (3)
respectively.
(100x + 10y + z) is the 3-digit number = 11(10x + z) (1) − (2) gives
= (100x + z) + (10x + 10z) 2(y − x) = 0
∴y = x + z ⇒y=x ------- (4)
Given y − x = 1∴z = 1 From (3) and (4)
Note: If (x – y) is taken as 1, z = – 1, which is inadmissible. 2x = 4, x = 2 and y = 2 Choice (D)
Ans: (1) 6 7
(c) + =7 ------- (1)
( 2x − y ) (3 y − x )
26. In addition to the data in Q-25, y − z = 1
4 14
∴y=1+z=1+1=2 + =6 ------- (2)
x=y−z=1 ( 2 x − y ) (3 y − x )
∴Number is 121. Ans: (121) From (2)
2 7
Note: If (z – y) is taken as 1, then x = −1, which is + =3 ------- (3)
( 2 x − y ) (3 y − x )
inadmissible.
(1) − (2) gives
Solutions for questions 27 and 28: 4
=4
(2x − y)
Let the actual weight of food grains be w.
⇒ 2x− y = 1 ------- (4)
Case Ι:
Similarly
If it is kept in the pan of weight 0.5 kg
(3) x 3 − (1) gives
Total weight (including that of food grains) = w + 0.5
14
On the other side of the balance let an indicated weight "xy" be =2
kept, w + 0.5 = 10x + y + 0.75 → (1) 3y − x
Case ΙΙ: ⇒ 3y − x = 7 ------- (5)
If food grains are kept in the other pan of the balance (4) + 2 × (5) gives
w + 0.75 = 10y + x + 9.5 + 0.5 → (2) 5y = 15 ⇒ y = 3 and x = 2 Choice (A)
Triumphant Institute of Management Education Pvt. Ltd. (T.I.M.E.) HO: 95B, 2nd Floor, Siddamsetty Complex, Secunderabad – 500 003.
Tel : 040–27898194/95 Fax : 040–27847334 email : info@time4education.com website : www.time4education.com SM1001961/4
2. Let the two parts be x grams and y grams ⇒ 49x + 14y + 7z = 343
x + y = 1000 2x + 5y + 7z = 37
5 -----------------------------
x + y = (x − y) 47x + 9y = 306
4
------------------------------
x – y = 800
Choice (D)
⇒ x = 900
y = 100 Choice (C) 13. Let the cost of each pen, each eraser and each sharpener
be `p, `e and `s respectively.
3. Given that P, Q and R are successive even natural
2p + e + 3s = 23 …….(1)
numbers in ascending order. As we need Q, we express
6p + 3e + s = 45…….(2)
P and R in terms of Q.
14p + 7e + 21s = 161…(3)
P = Q − 2 and R = Q + 2 But (1), (3) are equivalent.
∴ 5 (Q + 2) = 7(Q − 2) + 8 ⇒ Q = 8 Ans: (8) ∴ We effectively have only 2 equations (1) and (2) or (2)
and (3).
A +B In order to find the cost of each pen, the ratio of the
4. =A −B
3 coefficients of erasers and sharpeners must be the same.
3A − 3B = A + B As this is not the case, Choice (D) follows.
2A − 4B = 0 Choice (D)
A − 2B = 0 ------- (1)
B + 40 = 2A + 10 14. Let the cost of each brown pen be b paise, cost of each
white pen = (b + 70) paise. Suppose Anand bought
2A − B = 30 ------- (2)
B brown pens. He must have bought (30 − B) white pens.
⇒ 4A − 2B = 60
3A = 60 Total cost = bB + (30 − B) (b + 70)
⇒ A = 20 and B = 10 Choice (C) 30b − 70B + 2100 = 3200
3b − 110
5. Let the number of baskets made by Meena and Reena in B= . Among the given choices, B has a feasible
7
one hour be m, r respectively.
3m = 2r + 1 ------------- (1) value only when b = 60. Choice (D)
5m = 4r − 1 ------------- (2)
Solving (1) and (2) Solutions for questions 15 to 17:
m = 3 and r = 4 Choice (C)
Let the number of pens, erasers and sharpeners that Rakesh
6. Let the number be x. purchased be p, e and s respectively.
p + e + s = 38
9 2
x − x = 4235 ⇒ x = 990 Ans: (990) p<e<s
2 9 ∴ (p, e, s) can be only (11, 12, 15) or (11, 13, 14).

7. Let the number of coins with Ashok and Balu be a and b 15. In either of the above combinations p = 11
respectively. Choice (A)
Given that, a − 20 = 3(b + 20) ⇒ a = 3b + 80
and a − 30 = 2(b + 30) ⇒ a = 2b + 90 16. (p, e, s) = (11, 13, 14) results in the minimum expenditure.
∴2b + 90 = 3b + 80 ⇒ b = 10 Choice (B) Minimum expenditure = ` 117. Choice (C)

8. Let the number of `20 notes with Alok be x. Number of 17. Expenditure = 2p + 3e + 4s.
`5 notes with him = 30 − x. Given that, This will be minimum, if e and s have their minimum values,
20x + 5(30 − x) = 300 in which case p has its maximum value
⇒ x = 10 Choice (B) ∴ minimum expenditure occurs when e = s = 11 and
p = 16
9. Number of sequence copies made = 7500 − 1500 = 6000. Minimum expenditure = ` 109. Choice (B)
Cost of making these copies (in `) = 7080 − 1500 = 5580.
18. Let the shares of A, B and C be A, B and C respectively.
5580
y= = 0.93 Ans: (0.93) then, A + B + C = 5600 ------------- (1)
6000 B + C = 3A ------------- (2)
A + C = 9/5 B ------------- (3)
10. Let the cost of each pen, each eraser and each sharpener
Solving (1) & (2)
be `p, `e and `s respectively.
4A = 5600 ⇒ A = `1400
4p + 5c + 6s = 47 ….. (1)
6p + 7c + 9s = 69 …… (2) Solving (1) & (3)
By 3 × (1) − 2 × (2), we get e = 3 × 47 − 2 × 69 = 3
14 B
Choice (A) = 5600
5
11. Let the cost of each pen, each eraser and each sharpener ⇒ B = `2000
be `p, `e, `s respectively. Share of C
5p + 8e + 118 = 54 ------------ (1) = 5600 − 1400 − 2000 = `2200 Choice (C)
3p + 5e + 7s = 34 ------------ (2)
By multiplying (1) by 2 and subtracting thrice (2), we get
19. In each city, Aswin spends `2 more than half of what he
p+e+s=6 Ans: (6)
has when he entered the city i.e., he is left with `2 less than
half of at the beginning. Therefore, when he entered each
12. Cost of 1 pen = x
city he has double of 2 more than what he had when he left
Cost of 1 pencil = y
the city.
Cost of 1 eraser = z
When he entered the third city he has 2(150 + 2) or 304
2x + 5y + 7z = 37 - (1)
7x + 2y + z = 49 - (2) When he entered the second city he has 2 (304 + 2) or 612
Multiplying equation (2) by 7 and subtracting equation (1) When he entered the first city he has 2(612 + 2) or 1228
gives the required value. Ans: (1228)

Triumphant Institute of Management Education Pvt. Ltd. (T.I.M.E.) HO: 95B, 2nd Floor, Siddamsetty Complex, Secunderabad – 500 003.
Tel : 040–27898194/95 Fax : 040–27847334 email : info@time4education.com website : www.time4education.com SM1001961/5
N By substituting c = 2a in (1), it becomes
20. Let the fraction be . 2d − 2a = b → (4)
D
Subtracting (4) from 2 × (3), we get
Given that
b
N+1 2 12a = 3b ⇒ a =
= ⇒ 2D – 3N = 1 → (1) and 4
D +1 3
b
N−2 3 As c = 2a, c =
= ⇒ 5N – 3D = 4 → (2) 2
D−2 5
Applying, 3 × (1) + 2 (2), we get b 3b 3b
By substituting a = in (4), we get 2d = ⇒d=
(−9N) + (10N) = N = (3 × 1) + (2 × 4) = 11 4 2 4
Put N = 11 ∴a:b:c:d=1:4:2:3
Substituting in (1), 2D = 1 + 33 = 34 ⇒ D = 17 ∴ ‘abcd’ can be 1423 or 2846. Ans: (2)
N + 4 11 + 4 15 5
Required fraction = = = = 27. Let the number of questions correctly answered by Ajay be x.
D + 4 17 + 4 21 7
Number of questions wrongly answered by him = 200 − x.
Choice (C)
1 5
21. As the system has infinite solutions, the coefficients are His mark = 2x − (200 − x) = x − 100 = 360
2 2
proportional
⇒ x = 184
k
+2 Upon interchanging of the marks, his marks would be
3 = 184(1/2) − 16(2) = 60
i.e., = 3 Choice (B)
k 2k
k  28. Only the first two children have been mentioned
k − 4  = 0 individually. Let the number of chocolates with the
3  first child, the second child and all the others be a, b, c
k = 0 or 12. If k = 0, the second equation becomes respectively. If the first child attempted to double the
inconsistent number of chocolates with each of the others, he would fall
∴ k = 12 Ans: (12) short by two chocolates.
∴a+2=b+c
22. Let the number be ‘ab’. Also, a − 3 = b + 18
4(a + b) = 10a + b − 18 ⇒ b + 6 = 2a ∴c = 23 Ans: (23)
Also, 10a + b − 18 = 10b + a − 9 ⇒ a − b = 1
∴ b + 6 = 2(b + 1) 29. Suppose Alok started with `A. He gave `x to his friend in
b = 4 and a = 5 each round. The amounts with him at the end of the
∴ Product of digits = 20 Choice (B) first, second and third rounds are (in `)
2A − x, 4A − 3x and 8A − 7x respectively.
23. Let the number be ab. It’s reverse is ba.
∴ab + ba = 99 8
Given that, 8A − 7x = 0 ⇒ x = A
i.e., 10a + b + 10b + a = 99 7
⇒a+b=9 8
(a, b) can be (1, 8), (2, 7), (3, 6) ,……..(9, 0) As x = A + 20, A = A + 20 ⇒ A = 140 Choice (D)
7
∴ ab has 9 possibilities. Ans: (9)
24. Let the hundreds, tens and units digits be x, y, z 30. Let the present ages of Ram and Shyam be r years and
respectively. s years respectively.
The number is 100x + 10y + z r − 6 = 4(s − 6) ⇒ r = 4s − 18
x+y+z=8 - (1) And r + 6 = 3(s + 6) ⇒ r = 3s + 12
y = 3(x + z) - (2) ∴ 4s − 18 = 3s + 12 ⇒ s = 30, r = 102
From the third condition, we get x = z ∴ their present combined age is 132 years. It has to increase
∴y = 6x and 2x + y = 8 by 18 years to become 150 years. This will happen if the
∴x = 1 age of each increases by 9 years. Choice (B)
y=6
Number = 161 Choice (A) 31. The difference of Ashok’s age and his son’s age is always
24 years. As x years ago, Ashok was 48, his son was 24.
25. Let the number N be ‘abc’. Also 5x years ago Ashok was 36, his son was 12
Let the sum of the remaining numbers be R. ∴ The difference of these two times is 4x = 12 or x = 3.
R + cba = R + abc + 11(a + b + c) ∴ Ashok’s present age is 48 + 3
⇒ 99(c − a) = 11(a + b + c) His son’s present age is 24 + 3
⇒ 8c − 10a = b ……(1) The sum is 78. Choice (C)
5
As b ≥ 0 and c ≥ a , c > a
4 1 1
32. Let the two numbers be and
∴ 8 (Difference of a and c) x y
= 8(c − a) = 6 + 2a 1 1
8c − 10a = 6 …..(2) + =4 → (1)
x y
From (1) and (2), b = 6 Choice (B)
1 1
and x − y = 2xy ⇒ − =2 → (2)
26. Let the number be ‘abcd’ y x
Given that,
b + c = 2d → (1) 2 1
(1) + (2) gives =6 ⇒ y=
b + 6a = 2(c + d) → (2) y 3
d + 5a = 2b → (3) (1) − (2) gives
Let us that the equations (1), (2) and (3) as the linear 2
equations in a, b and c and express the values of a, b and c =2 ⇒ x = 1
x
in terms of b.
∴Product of the reciprocals = (x) (y) = (1) (1/3) = (1/3)
c
By (2) − (1), we get 3c = 6a ⇒ a = Choice (C)
2
Triumphant Institute of Management Education Pvt. Ltd. (T.I.M.E.) HO: 95B, 2nd Floor, Siddamsetty Complex, Secunderabad – 500 003.
Tel : 040–27898194/95 Fax : 040–27847334 email : info@time4education.com website : www.time4education.com SM1001961/6
33. Let the larger number be a. Using both the statements we have
Smaller number = 250 − a man’s age = 42 + 2 = 44 years
a2 − (250 − a)2 = 12500 ∴ Clearly man’s age < wife’s age + son’s age. Choice (C)
(a + 250 − a) (a − (250 − a)) = 12500
⇒ 2a − 250 = 50 40. As statements Ι and ΙΙ individually do not relate to Uno and
Zen, they are not sufficient.
∴ a = 150 Choice (C)
Using both, we have Palio = 4/5 (Alto);
34. Let the number of marbles with Dinesh, Eswar, Ganesh Alto = Zen − 2
and Harish be d, e, g and h respectively. 4
Also Alto gives 45 × = 60 km
Given that d + e + g + h = 20 → (1) 3
1 1 ∴ Zen = 62 km
d= (e + g + h) = (120 − d)
2 2 ∴ Zen gives more mileage. Choice (C)
d = 40
Chapter – 2
similarly, e =
1
(120 − e) ⇒ e = 30 and (Ratio – Proportion – Variation)
3

g=
1
(120 − g) ⇒ g = 24 Concept Review Questions
4 Solutions for questions 1 to 30:
h = 120 − (d + e + g) = 120 − (30 + 40 + 24) = 26
Ans: (26) 9
1. Given a = b
8
35. Before doubling, the amounts with Bhavan, Chetan and
3b 3b 2
80 = = Choice (A)
Dinesh, each of them must have had = `40. 4a 9  3
2 4 b 
∴ Amar must have then had `80 + `120 i.e. `200. Similarly 8 
we can work out the amounts with each of them before the
other doubled the amounts. The results are summarized below. 2. Given 2.4p = 0.08q
2.4
q= p = 30p
Amar Bhavan Chetan Dinesh 0.08
Finally 80 80 80 80 q + p 31p 31
= = Choice (C)
Before Amar q − p 29p 29
200 40 40 40
doubles
Before Bhavan 3. Let a = 2x b = 3x
100 180 20 20 3a + 4b = 3 (2x) + 4 (3x) = 18x
doubles
Before Chetan 4a + 5b = 4 (2x) + 5 (3x) = 23x
50 90 170 10
doubles 3a + 4b 18
= Choice (B)
Before Dinesh
25 45 85 165 4a + 5b 23
doubles
4. a:b=2:3=4:6
Choice (D) a:c=2:3=6:9
∴a:b:c=4:6:9 Choice (A)
Solutions for questions 36 to 40:
5
5. p : q = 5 : 6⇒ p = q
x 3y 6
36. Statement Ι: + =1
2 4 4 4
q:r= ⇒q= r
Statement Ι is sufficient. 7 7
Statement ΙΙ: 9y + 6x = 12, we can see that this is 5  4  10
equivalent to 3y + 2x = 4. ∴ p =  r  = r
6  7  21
⇒ The value of 2x + 3y; value can be found, so statement
ΙΙ is sufficient. Choice (B) ∴ p : q: r =
10r 4
: r :r = 10 : 12 : 21 Choice (A)
21 7
37. From statement ΙΙ, if the cost of 12 shirts is doubled, it will
be the same as 72 pants ⇒ cost of 12 shirts is the same as 6.
a b c d 2 6 14 1
x x x = x x x =
4
Choice (D)
the cost of 36 pants. So, only statement ΙΙ is sufficient and b c d e 3 7 25 2 25
the difference is zero. Choice (A)
7. Let P = 5x q = 4x
38. There are two possibilities 5x = a + b --- (1)
Ι. 21 attempts __ all correct. 4x = a − b--- (2)
ΙΙ. 25 attempts __ 22 correct. From (1) and (2),
Hence, the question cannot be answered even after using 2a = 9x and 2b = x
both the statements. Choice (D) ∴ a : b = 2a : 2b = 9 : 1 Choice (B)

39. From statement Ι, we have man’s age = wife’s age + 2 years 8. Number of sweets received by Sita =
4
(35) = 20
But to know whether man’s age = wife’s age + son’s age, 7
we should know the ages of at least two of them. Ans: (20)
∴ Statement Ι alone is not sufficient.
From statement ΙΙ, we have 9. Let the numbers be 2x and 5x.
son’s present age = 12 years 2x + 4 4
=
wife’s present age = 42 years 5x + 4 9
But to know whether man’s age = wife’s age + son’s age or 18x + 36 = 20x + 16
not, we should know the relationship between the ages of x = 10
the man and wife or man and the son. 2x = 20 and 5x = 50
∴ Statement ΙΙ alone is not sufficient. ∴ The numbers are 20 and 50 Choice (D)
Triumphant Institute of Management Education Pvt. Ltd. (T.I.M.E.) HO: 95B, 2nd Floor, Siddamsetty Complex, Secunderabad – 500 003.
Tel : 040–27898194/95 Fax : 040–27847334 email : info@time4education.com website : www.time4education.com SM1001961/7
10. A : B : C = 7 : 5 : 4 s1 3
let A’s age = 7k B’s age = 5k ∴ < .
s2 4
c’s age = 4k = 32 ⇒ k = 8
= 7k + 5k + 4k = 16 x 8 = 128 years Ans: (128) s1 13
Only Choice (C), i.e. = satisfies this condition.
s2 20
11. Let the ages of the four members be 9k, 8k, 3k and
Choice (C)
2k years respectively.
9k + 8k + 3k + 2k 20. Let the present ages of the husband and the wife be
Average age = = 22
4 5x years and 4x years respectively.
⇒k=4
∴ The age of the eldest family member (i) Ratio of their ages 20 years ago
= 9 x 4 = 36 Choice (A) 5 x − 20 5(x − 5 ) + 5 5
= = > (some positive quantity)
12. Let the number of marbles with A and B be 10x and 11x 4 x − 20 4(x − 5 ) 4
respectively 5
Total number of marbles = 21x ∴ the ratio must exceed . Only choice (D) satisfies
4
∴ the total number of marbles must be divisible by 21. Only
Choice (D) violates this condition. Choice (D) this condition Choice (D)

13. Let the ratio of the number of boys and girls in the class be (ii) Ratio of their ages 20 years hence
a : b. Number of boys and girls will be ak and bk. 5 x + 20 5(x + 5 ) − 5 5
= = <
Given ak + bk = 70 4 x + 20 4(x + 5 ) 4
70
K= . 5
a+b ∴ the ratio must be less than
4
Since k is an integer a + b must be a factor of 70; from the Only Choice (B) satisfies this condition. Choice (B)
options
The sum of the ratio in options (A), (B), (C) are factors of 21. Let the four numbers be x , 2x, 3x, 4x
70. But the sum of ratio 9 : 2 is not a factor of 70. The sum of squares = 480
∴ 9 : 2 is not a possible ratio. Choice (D) ⇒ x² + (2x)² + (3x)² + (4x)² = 480 ⇒ 30x² = 480
14. Let the number of gents in the party be g. ⇒ x² = 16
3 ⇒x=4
Number of ladies in the party = g ∴ The numbers are 4, 8, 12, 16 Choice (C)
4
Number of ladies in the party if 8 ladies join the party
q+r p+r p+ q
3 5 22. Given that = = =k
= g + 8 = g ⇒ g = 16 Ans: (16) p q r
4 4
q + r = pk, p + r = qk and p + q = rk
15. Let the present ages of the mother and son be m and s years q + r + p + r + p + q = k(p + q + r)
respectively (p + q + r) (2 − k) = 0
m:s=4:1 ∵ p + q + r ≠ 0,
⇒ m = 4k, s = k ∴2−k=0
after twelve years
⇒k=2 Choice (D)
4k + 12 2
=
k + 12 1 23. x ∝ y² ⇒ x = k x y²
⇒k=6 4 = k x 122
∴ the present ages of the mother and son are 24 and k = 4/122
6 years respectively. Choice (C) When y = 18,
x = ky² ⇒ x = 4/12² x 18 x 18 = 9 Choice (B)
16. Let the number of chocolates A and B be 3k, 4k
3k − 4 5 24. Area x (Diagonal)2
=
4k + 4 9 A x D2
∴ A had 3k = 24 chocolates initially. Ans: (24) ⇒ A = KD2
A A 32 72
17. Let the numbers of students in A, B and C be 3x, 7x and 8x ⇒ 21 = 22 ⇒ 2 = 2
D1 D 2 8 D 2
respectively.
If 10 students leave C and join B, C and B would have ⇒ D2 = 12 units. Ans: (12)
8x − 10 students and 7x + 10 students respectively.
S1 D
7x + 10 = 80 25. = 1
x = 10 S2 D2
Total number of students = 18x = 180 Ans: (180) S1 = 10400, D1 = 26 and D2 = 24
SD
18. Let the incomes of A and B be `4x and `3x S 2 = 1 2 = 9600 Ans: (9600)
Their savings be `5y, `6y D1
Their expenditures are (4x – 5y) and `3x – 6y
⇒ 4x – 5y = ¾ (4x) ⇒ x = 5y 26. As the distance to be covered is constant, S1 T1 = S2 T2
∴ the expenditure of A = 4x – 5y = 4(5y) – 5y = 15y S1 = 50, T1 = 6 and S2 = 100
∴ the expenditure of B = 3x – 6y = 3(5y) – 6y = 9y ST
T2 = 1 1 = 3 Choice (B)
∴ the ratio of the expenditure of A and B is S2
= 15y : 9y = 5 : 3 Choice (B)
27. P = k (Q + R) where k is a proportionality constant.
s1 e s s +e e
19. If < 1 then 1 < 1 1 < 1 P1 = k (Q1 + R1)
s2 e2 s2 s 2 + e2 e 2 P2 = K (Q1 − 1 + R1 − 1) = k (Q1 + R1 − 2) = P1 − 2k
e1 + s1 3 e 4 As k is unknown, the change in P cannot be found.
In the given problem, = and 1 = Choice (D)
e2 + s2 4 e2 5

Triumphant Institute of Management Education Pvt. Ltd. (T.I.M.E.) HO: 95B, 2nd Floor, Siddamsetty Complex, Secunderabad – 500 003.
Tel : 040–27898194/95 Fax : 040–27847334 email : info@time4education.com website : www.time4education.com SM1001961/8
28. From the given information, A varies directly with the 5. 3x − 4y + 2z = 0 ---(Ι)
product of B and C. 4x − 2y − z = 0 -----(ΙΙ)
A BC From (II)
∴ 1 = 1 1
A 2 B 2C2 z = 4x − 2y
Substituting z = 4x – 2y in I, we get
A1 = 6000, B1 = 20, C2 = 30, B2 = 40 and C2 = 60
3x − 4y + 8x − 4y = 0
AB C
A 2 = 1 2 2 = 24000 Ans: (24000) 11x = 8y
B1C1 Again, z = 4x − 2y
multiply by 4 on both sides we get
29. A α B 4z = 16x − 8y; substituting 8y = 11x
A α 1/C 4z = 16x − 11x = 5x
⇒ A α B/C 5 11
x:z:y=x : x: x = 8 : 10 : 11 Choice (A)
AC A C 4 8
⇒ 1 1= 2 2
B1 B2
p−x 1 3p − q
6. = ;x= Choice (B)
20 × 30 A 2 × 42 q+ x 3 4
⇒ =
65 84
7. Two numbers are in the ratio 4 : 7
⇒ A2 = 24 Choice (A)
Let the numbers be 4k and 7k
Let the number added to each be L.
30. From the given information, X varies inversely with the
Given that, (4k + L) + (7k + L) = 75
product of Y and Z.
⇒ 11k + 2L = 75 --------- (1)
X Y Z
∴ 1 = 2 2 4k + L 8
X2 Y1 Z1 It is also given that, =
7k + L 17
X1 = 30, Y1 = 8, Z1 = 7, ⇒ 12k + 9L = 0 ----------- (2)
Y2 = 16 and Z2 = 21 −9 L −3
X YZ ⇒k= = L
X2 = 1 1 1 = 5 Ans: (5) 12 4
Y2 Z 2 Substituting the value of k in (1) we get L = –12

Exercise – 2(a) Alternate method:


Ratio of the resulting numbers = 8 : 17
Solutions for questions 1 to 30: Sum of the resulting numbers = 75
Hence, the numbers obtained after addition are
1. a/b = 3/7 (8/25) x 75 = 24 and (17/25) x 75 = 51
i.e., 4k + L = 24 and 7k + L = 51
a
4  + 5 12 + 5 On subtraction,
4a + 5b 7k − 4k = 51 − 24 ⇒ 3k = 27, k = 9 and L = −12
=  
b 47
= 7 =
2a + 2b a 6 20 Ans: (–12)
2  + 2 +2
b 7
8. Let the amounts initially with Mohan and Sohan be 8x and
5x respectively. Let us say Mohan spends y each day.
Alternate method:
Substituting the values of a and b as 3k and 7k respectively 8 x − y(9) 10
= ⇒ x = 3y
in
4a + 5b
, 5 x + (y / 6 )(9 ) 11
2a + 2b Let us say Mohan and Sohan have amounts which are in
12k + 35k 47k 47 the ratio 18 : 35 after t more days.
we get = = Choice (A)
6k + 14k 20k 20 8 x − t.y 18 8(3 y ) − t.y 18
= i.e. =
5 x + t.y / 6 35 5(3 y ) + t.y / 6 35
2. a : b = 2 : 3 = (2 × 4) : (3 × 4) = 8 : 12
b : c = 4 : 3 = (4 × 3) : (3 × 3) = 12 : 9 t = 15
The ratio of the amounts with them would be 18 : 35 after
 9  9  27  6 more days. Ans: (6)
c : d = 2 : 3 =  2 ×  :  3 ×  = 9 :  
 2  2  2 
9. Let the present age of Kishore be x years,
27 Vipin's present age is 2(x – 1) years
a : b : c : d = 8 : 12 : 9 : = 16 : 24 : 18 : 27 Given the ratio of the sum of their present ages to the
2
difference of their present ages, is 19 : 5
Choice (B)
3 x − 2 19
i.e. = ∴x=7
3. Let a, b and c be the weights of the Bimal, Basu and Bali x−2 5
respectively. sum of present ages = (k + v) = 19 years Ans: (19)
a/b = 2/3 = 8/12; c/b = 3/4 = 9/12; a : b : c = 8 : 12 : 9
Given the sum of the weights (8 + 12 + 9 = 29 parts) is 203 kg 10. Ratio of prices = 5 : 8 : 13
i.e. one part is 7 kg. So Basu’s weight i.e. 12 parts is Ratio of number of balls = 5 : 4 : 3
12 x 7 = 84 kg Ans: (84) Ratio of amounts spent = (5 × 5) : (8 × 4) : (13 × 3)
= 25 : 32 : 39
4. Number of boys = 7x Total number of parts of the ratio=25 + 32 + 39 = 96
Number of girls = 3x Total amount spent, as per data = `768
7x 2 Value of each part of the amounts' ratio=768/96 = 8
= Amount spent on costliest variety
3 x + 15 3
= Value of 39 parts = 39 × 8
21x = 6x + 30
Number of costliest variety balls Paul purchased is
15x = 30 , x = 2
= (39 × 8)/13 = 24 Choice (D)
∴ Number of girls = 6 Choice (D)

Triumphant Institute of Management Education Pvt. Ltd. (T.I.M.E.) HO: 95B, 2nd Floor, Siddamsetty Complex, Secunderabad – 500 003.
Tel : 040–27898194/95 Fax : 040–27847334 email : info@time4education.com website : www.time4education.com SM1001961/9
11. ( a + c ) + (c + e ) + ( a + e ) 2(a + c + e ) a + c + e
Brother 1 2 3 4 5 6 7 8 9 k= = =
(b + d) + ( d + f ) + (b + f ) 2(b + d + f ) b+d+ f
Amount
a+c a+c +e (a + c + e ) − ( a + c ) e
with the x 2x + 2 3x + 3 4x + 4 ... ... ... ... 9x + 9 Now = =k = = =k
brother b+ d b+d+ f (b + d + f ) − (b + d) f
c +e a+c +e
9x + 9 Similarly = =k
= 10, x = 9; d+ f b+ d+ f
x a + c + e ) − (c + e )
th a
5 brother gets (5x + 5) = 45 + 5 = 50 Choice (D) k= = and lastly,
(b + d + f ) − ( d + f ) b
12. Let p. q, r and s represent the apples received by Karan, ( a + c + e ) − (a + e ) c
Kiran, Kumar and Khanna. k = = , hence all the choices (A), (B)
(b + d + f ) − (b + f ) d
Given q : (p + s) = 1 : 2,
and (C) are true. Choice (D)
⇒ q : (p + s) : (q + p + s) = 1 : 2 : (1 + 2) = 1 : 2 : 3 ------ (1)
r : (p + q + s) = 2 : 5-------- (2)
s = 2 + q ------- (3) 15 12
17. In the two cases, the greatest parts are N and N
and p : r = 1 : 2 or r : p = 2 : 1 ------- (4) 42 42
From (2) × 3 and (1) × 5, we have 3N
r : (p + q + s) : (p + s) : q = 6 : 15 : 10 : 5 Their difference = =6 N = 84 Ans: (84)
42
Combining this with (4) × 3, we have
r : p : (p + q + s) : (p + s) : q = 6 : 3 : 15 : 10 : 5 18. Let the two numbers be “a” and “b”.
⇒ r : p : q : (p + q + s) : (p + s) = 6 : 3 : 5 : 15 : 10
ab = 9
⇒ r : p : q : (p + q + s − p + q) : (p + q + s) : (p + s)
Squaring on both sides
= 6 : 3 : 5 : (15 − 3 + 5 ) : 15 : 10 ab = 81 ––––– (1)
b2 = 243a --- (2)
⇒ r : p : q : s : (p + q + s) : (p + s) = 6 : 3 : 5 : 7 : 15 : 10 multiplying both the sides with b, we get
From (3), s − q = 2 and this is satisfied by the number of b3 = 243 × 81
parts of s and q in the above ratio. b = (35 × 34)1/3 = 33 = 27 ⇒ a = 3
⇒ The number of parts in the above ratio are the actual The larger of the two numbers = 27 Choice (A)
values.
⇒ p = 3, q = 5, r = 6, s = 7; i.e., p + q + r + s = 21 19. Let P = ab and Q = cd Ps = a + b and Qs = c + d
Choice (A)
10a + b 10c + d
=
a+b c +d
9 th
13. David received of the number of toffees distributed by (10a+b) (c+d) = (10c+d) (a + b)
17 10ac + bc + 10ad + bd = 10ac + ad + 10bc + bd
1 th ad = bc ……(1)
Alok. Amitha received of the number of toffees Let a = 1, b = 0, c = 2 d = 0
16
distributed by David. This would correspond to the minimum value of Ps+Qs,
which is 3. Choice (D)
1  9 
 T  = 18
16  17  d1 d2 48 d2
⇒ T = 544 Ans: (544) 20. = ; = ⇒ d2 = 147 Choice (D)
t12 t 22 16 49
14. Quantity of diesel purchased = x lts.
∴ Cost per liter of diesel = 510/x 21. Let the sum of money be `x
510 × 3
The sum is sufficient to pay A’s wages for 55 days and B’s
Cost per liter of kerosene = = 306/x wages for 66 days.
x×5
x x
Since equal volumes are purchased, amount spent on Daily wage of A = and daily wage of B =
55 66
306
kerosene = × x = ` 306 Choice (B) Number of days that the sum is sufficient to pay the wages
x x
of both workers = = 30 Ans: (30)
x x
15. Amount spent on kerosene = x +
Amount spent on petrol = 5x 55 66
∴ Cost involved = 6x ---------- (1)
Ratio of prices per liter of Petrol, diesel and kerosene is 22. V α r2h; ⇒ V = k.r2h;⇒ V1/( r12h1 )
15 : 5 : 3
When equal volumes of all are purchased, the amounts are = V2 / ( r2 2h 2 ) = k
also in the ratio 15 : 5 : 3. Hence, 66/(9 × 7) = 308/(6r²); ⇒ r = 7 Choice (C)
Amount spent on diesel, in this case = 510
Total amount spent = (23 × 510)5 = 23 × 102----- (2) 23. Let K, M and S be the kinetic energy, mass and speed of a
From (1) and (2), 6x = 2346 body respectively.
x = 391 Given:
∴ Amount spent on petrol = 391 × 5 = `1955 K ∝ S² (when M is kept constant)
Choice (C) and K ∝ M (when S is kept constant)
⇒ K ∝ MS² ⇒ K = CMS²
a+c c +e a+e where C is the constant of proportionality.
16. = = =k Given that when M = 2 kg, S = 10 m/s, K = 100 joules
b + d d+ f b+ f
⇒ 100 = C x 2 x 10² ⇒ C = 1/2 ⇒ K = 1/2 MS²
Using the relationship each ratio When M = 20 kg and S = 1 m/s
K = 1/2 x 20 x 1² = 10
sum of numerators ∴ A body of mass 20 kg moving with a speed of 1 m/s has
=k=
sum of deno min ators a kinetic energy of 10 joules. Choice (C)

Triumphant Institute of Management Education Pvt. Ltd. (T.I.M.E.) HO: 95B, 2nd Floor, Siddamsetty Complex, Secunderabad – 500 003.
Tel : 040–27898194/95 Fax : 040–27847334 email : info@time4education.com website : www.time4education.com SM1001961/10
24. Let the fixed income and the royalty that he gets be
27. v = 40 – k 3 n , where v is the speed when n wagons are
denoted by F and R respectively.
Total income (T) = F + kB (∴R α B). attached.
46000 = F + 2000k If n = 27, v = 34
66000 = F + 3000k 3
34 = 40 – k 27
Solving these, k = 20. F = 6000
Total income on setting 6000 books = F + 6000k 34 = 40 – k(3) ⇒ k = 2 ∴ v = 40 – 2 3
n
F + 6000k 6000 + 6000( 20) Minimum speed required = 30
Income per book = = = 21.
6000 6000
∴ 30 = 40 – 2 3
n ;
Ans: (21)
3
⇒5= n
25. A α (B + C) ⇒ A = k1 (B + C) n = 125
B α x ⇒ B = k2 x So, a maximum of 125 wagons can be attached.
1 k Choice (B)
Cα ⇒C= 3
x x
28. Let the initial weight of the stone (before breaking) be 6 w.
 k  Weights of the broken stones are w, 2w and 3w
∴ A = k1 k 2 x + 3 
 x  respectively.
Initial value of the stone (in `) = 10, 872
As A = 3, when x = 1,
10,872 α 36 w2
3 = k1 [k 2 + k 3 ] ---- (Ι)
10,872 = 36 w2 k
As per data, A = 3, when x = 2 Total value of the pieces of the stone
= (12 + 22 + 32) w2 k = 14 w2 k
 k  Loss in the value = 22 w2 k
∴ 3 = k1 2k 2 + 3  ----- (ΙΙ)
 2 
10872
= 22 × = 22 × 302 = 6644
(Ι) – (ΙΙ) gives 36
k k  k3  Choice (C)
− k1 k2 + 1 3 = 0 ⇒ k1  − k 2  = 0
2  2 
29. distance = d weight = w
⇒ k1 = 0 Given d α (1/w)
k dw = k (constant)
or k2 = 3 are the possible cases Let weights of 3 pieces be x, 3x, 2x
2
k k k 11 k
k1 cannot be 0 (as A can’t be 0). ∴ Sum of distances = + + = = 22
x 3 x 2x 6 x
k
Hence k2 = 3 k = 12x
2
Weight of unbroken stone is
From (Ι)
w = 6x = k/2 as d. w = k
3k1 k 3 d x k/2 = k ⇒d=2m Ans: (2)
3=
2
k1 k3 = 2. ⇒ k1 k2 = 3 −2 = 1. 30. Fixed charge = x, unit rate = y, x and y are in Rupees.
5/4(x + 100y) = x + 200y
k1k 3 2 5x + 500y = 4x + 800y -------- (1)
Hence A = k1k2 x + =x +
x x x + 50y = 700 ----------------(2) (data)
When x = 4, y = 2, x = 600
Choice (A)
2 9
A=4+ = = 4.5 Ans: (4.5)
4 2 Exercise – 2(b)

26. The quantity of the balance of food after the transfer is such Solutions for questions 1 to 40:
that
(900 − 300) = 600 soldiers, consumed at the rate of 1. Let a + b − c = 5x
3 kg/day/soldier, for 25 days ------- (1) b + c − a = 6x
If the soldiers were not transferred, 900 soldiers would a + c − b = 7x
have consumed it at the rate of 2.5kg/day/soldier, the same (a + b − c) + (b + c − a) = 11x
food. ---------- (2) ∴ 2b = 11x
The data can be tabulated as: (a + b − c) + (a + c − b) = 12x
∴ 2a = 12x
Soldiers Consumption rate Number of days (b + c − a) + (a + c − b) = 13x
600 3.0 25
∴2c = 13x
900 2.5 How many?
∴ a : b : c = 2a : 2b : 2c = 12 : 11 : 13
Choice (B)
Number of soldiers and the number of days for which
food lasts are inversely proportional. The number of
soldiers increased; hence, number of days decreases. 20p 2 − 40pq
2. Given, = 20 ⇒ 20p2 − 60pq − 80q2 = 0
Hence multiplication factor is (600/900). pq + 4q2
Consumption rate and number of days are also inversely
proportional. Hence, multiplication factor is 3.0/2.5 ⇒ p2 − 3pq − 4q2 = 0
Applying the above rates of variation, the number of days ⇒ (p − 4q) (p + q) = 0
= 25 x (600/900) x (3.0/2.5) = 20 days
The initial stock was to last for 30 days. p
∴ = 4 or − 1
⇒ Soldiers were transferred after 30 − 20 = 10 days q
Ans: (10) ∴Choice (C) is possible. Choice (C)

Triumphant Institute of Management Education Pvt. Ltd. (T.I.M.E.) HO: 95B, 2nd Floor, Siddamsetty Complex, Secunderabad – 500 003.
Tel : 040–27898194/95 Fax : 040–27847334 email : info@time4education.com website : www.time4education.com SM1001961/11
3. We will find it convenient to change the notation slightly. Number of girls who are day scholars and travel to school
Let A : B = 3 : 4 and c : d = 2 : 3 1 2  2
Let A = 3x ∴ B = 4x by bus =  (2y ) = x
23  3
Let c = 2y ∴ d = 3y
1 2
A 3 c 2 + B 3 d2 x+ x
To find E = Required part = 12 3 =1 Choice (C)
AB 2 d2 + A 2Bcd 3x 4
As each term has 3 upper case letters and 2 lower case,
12. The data is tabulated below.
E is homogeneous and we can evaluate it.

3 3 ( 2 2 ) + 4 3 (32) x3 y2 White Black Green Total


E=
3( 4 )(3 ) + 3 ( 4)(2)(3) x y
2 2 2 3 2 3 4
3 5
3 + 42 19 9 12 20 41
= = Choice (D)
3( 4 ) + 6 18
The total number of white and green balls is a multiple of
4. Let the scores of Ajay be a and v respectively. 29. Among the choices, only 58 is a multiple of 29.
a + 2v = 310 ……(1) Choice (C)
v + 2a = 290 ……(2)
Solving (1) and (2) 13. Let the number of coins with Amar, Bhavan and Chetan be
a = 90 and v = 110 A, B and C respectively.
∴ a : v = 9 : 11 Choice (A)
The data is tabulated below
5. Let the first and second parts be a and b. A B C AB AC BC 2(ABC)
3 5 4 12
3a 25 a 25
Given, = ⇒ = ∴ A = 6 − 4 = 2, B = 6 − 5 = 1 and C = 6 − 3 = 3
2b 4 b 6
∴B:C=1:3 Choice (D)
25
a= (93) = 75 Ans: (75)
31 14. Let the price of tea last year be `5x per kg. Let the price of
coffee last year be `7y per kg.
6. Let the fraction be x/y
x−3 1
6
(5x )+ 8 (7y )= 48
= ⇒ 2x – y = 11 → (1) 5 7
y+5 2
3 
x+2 5x =
20
(7y ) ⇒ y = 3 x ⇒ 6x =  x  = 48
= 1; x – y = –2 → (2) 21 4 4 
y
⇒ x = 4 ⇒ 5x = 20 Choice (B)
Subtracting (2) from (1), we get x = 13; y = 15
∴ Fraction = 13/15 Choice (C)
2
15. Rohan was supposed to get th of the total amount.
7. Let the number of students in three classes A, B and C be 9
3x, 7x and 8x respectively. 1 1 1
But the actual ratio of division was : : = 6: 4: 3 .
7 x + 10 8 2 3 4
Given, = ⇒ x = 10
8 x − 10 7 6
∴ he got th of the total amount.
Total = 18x = 180 Ans: (180) 13
8. Let the numbers of sweets received by Ram, Shyam and 2 6
Tarun be r, s and t. As ≈ 0.2 and > 0.4, Rohan gained. He gained by
9 13
r 5 s 2 4
= and = = 6 2
s 4 t 3 6  −  117 = ` 28 Choice (C)
3 9
6
∴r:s:t=5:4:6⇒t= (60) = 24 Ans: (24)
15 16. Let the quantities of milk be 3xml, 4x ml, 5x ml and 6x ml
and 7x ml.
9. Let the 3 parts be x, y, z. Let the capacity of each vessel be 100 ml.
∴ x + y + z = 66 --------- (Ι) Total capacity = 500 ml
z=x+y Substituting z in (Ι) 2x + 2y = 66
⇒ x + y = 33 --------- (ΙΙ) 3x + 4x + 5x + 6x + 7x =
60
(500) ⇒ x = 12
100
Given y = 2x – 3 Substituting y in (ΙΙ),
∴ 3x = 36, 4x = 48, 5x = 60, 6x = 72, 7x = 84
we get 3x – 3 = 33 x = 36/3 = 12
The last 3 vessels contain at least 50% milk.
∴ y = 21 and z = 33 Ans: (3)
Ratio of x, y and z is = 12 : 21 : 33 = 4 : 7 : 11
Choice (D)
17. Let the monthly income of Ashok be `3x. Let the expenditure
of Ashok be `4y.
10. Number of weighing stones of 500 gms = 5000/500 = 10
Monthly income of Bala = `4x
∴ Number of 100 gm weights = (3/5) × 10 = 6
Expenditure of Bala = `5y
Choice (A)
Savings of Ashok and Bala are `(3x − 4y) and `(4x − 5y)
respectively.
11. Let the number of boys in the class be x.
Number of girls in the class = 2x. 1
y
3x − 4y 3 4 3
Number of day scholars who are boys and girls travel to Ratio = = − which is less than . Only
11  1 4x − 5y 4 4x − 5y 4
school by bus =  x  = x. Choice (B) violates this condition. Choice (B)
4  3  12

Triumphant Institute of Management Education Pvt. Ltd. (T.I.M.E.) HO: 95B, 2nd Floor, Siddamsetty Complex, Secunderabad – 500 003.
Tel : 040–27898194/95 Fax : 040–27847334 email : info@time4education.com website : www.time4education.com SM1001961/12
18. Data can be tabulated as follows: 23. Total number of questions is 22.
Number of English questions = (7 × 22)/11 = 14
Manoj Shiva Time taken for English questions
1. Income Ratio 3 : 4 = (60 × 7)/15 = 28 minutes
2. Savings Ratio 2 : 3 Number of questions that can be answered in 18 minutes
3. Spending = 18/(28/14) = 9 Choice (D)
(Expenditure) - -
m p
Manoj's expenditure is 2/3rds
of his income. Let Manoj's 24. When two ratios like and are equal, each of them is
monthly income be `36. (an arbitrary number is chosen in n q
such a way that no fractions are encountered in the m−p
calculation) equal to , provided n ≠ q.
n−q
( )
Hence, Manoj's expenditure is 36 x (2/3) = 24 ---- (1)
Hence, Manoj's savings is 36 − 24 = 12 (a 2 + c 2 ) − b 2 + c 2
∴k= =a+b Choice (C)
Ratio of savings of Manoj and Shiva = 2 : 3 (a + c ) − (b + c )
Hence, Shiva's savings = 3 x (12/2) = 18 ------- (2)
Income Ratio = 3 : 4
p + q q+r p+r
As Manoj's income = `36, Shiva's shall be 25. As = =
4 x (36/3) = `48------------- (3) r p q
Shiva's expenditure = Income – Savings p + q+ q+r +p +r
= 48 − 18 = 30 --------- (4) Each of them equals = =2
r +p+ q
From (1) and (4); Ratio of expenditures of Manoj and Shiva
is 24 : 30 = 4 : 5 Choice (B) If p + q + r ≠ 0, then k = 2
p+q
If p + q = − r, then =−1
19. Let the numbers satisfying the condition be denoted by xyz. r
z = x + y, z − y = y − x z = 2y − x = x + y ∴ k = −1
y = 2x Sum of all the possible values of k is 1. Choice (A)
z = 3x
∴x:y:z=1:2:3
2x 2 − 4 x + 3 2x 2 − 3 x + 5
∴ (x, y, z) can be (1, 2, 3), (2, 4, 6) or (3, 6, 9) 26. =
Choice (C) 4x − 3 3x − 5
By componendo and dividendo
20. Let the present ages of the mother, her husband and her
daughter be x years, y years and z years respectively. ( 2 x 2 − 4 x + 3 ) + ( 4 x − 3 ) ( 2 x 2 − 3 x + 5 ) + (3 x − 5 )
=
x + z = 60 4x − 3 3x − 5
The mother would attain her husband’s age after y − x
years. 2x 2 2x 2
y+y−x ⇒ = ⇒ x = 0 or 4x − 3 = 3x − 5
∴ =2 4x − 3 3x − 5
z+y− x
⇒ x = − 2 or 0 Choice (D)
x = 2z
From (1), z = 20 Ans: (20) 27. (10a + b) (10c + d) = (10b + a) (10d + c)
100ac + 10bc + 10ad + bd
21. = 100bd + 10ad + 10bc + ac
ac = bd
Larger plot Smaller plot Combined a d a b
= or = Choice (A)
b c d c
Area M 8x 29y – 8x 29y
Area B 9x 33y – 9x 33y 28. Let the lengths of the larger and the smaller parts be x units
and y units respectively.
29 y − 8 x 13 x+y y x y
=6 + 1− 6 = 0
= ⇒ x = 2y. y x y x
33y − 9x 15
2
x x x  x 
Ratio of the area under maize cultivation in the larger plot   + −6=0  + 3  − 2  = 0
y y y  y 
and that under barley cultivation in the smaller plot     
8x 16 y 16
= = = Choice (B) x x
33 y − 9 x 33 y − 18 y 15 As + 3 > 0, = 2 Choice (A)
y y
22.
29. Let the numbers be a and b
Number of Time per a : 9 = 9 : b ⇒ ab …….(1)
Total time a : b = b : 6561 ⇒ b2 = 6561a …….(2)
questions question
Solving (1) and (2)
Mathematics a 2x 8y a = 1 and b = 81 Ans: (81)
English b x 7y
30. Let Ahmed’s age = x, Ahmed’s brother’s age = a
Let Mohammed’s age = y, Mohammed’s brother’s age = b
Given 15y = 60 x + y a+b
y = 4∴ 8y = 32 min 7y = 28 min x/y = a/b; = (using compoendo and dividendo)
x − y a−b
(a ) ( 2x ) 8
= [a/b = 16/28 = 4/7] x+y x−y 1 a+b
(b ) ( x ) 7 = = as per data∴ =2
a+b a −b 2 x+ y
∴ Ratio of the number of English questions to the number Choice (A)
of Maths = b/a = 7/4 Choice (B)

Triumphant Institute of Management Education Pvt. Ltd. (T.I.M.E.) HO: 95B, 2nd Floor, Siddamsetty Complex, Secunderabad – 500 003.
Tel : 040–27898194/95 Fax : 040–27847334 email : info@time4education.com website : www.time4education.com SM1001961/13
31. x = k (y2 + 4) P1V1 P2 V2
39 = k(13) ⇒ k = 3 36. = ;
T1 T2
⇒ 60 = 3(y2 + 4)
y2 = 20 – 4 = 16, y = ± 4 (but, from the condition y > 0 xy (3 x ) ( 2 y )
= , T2 = 600 K
⇒ y = 4) Ans: (4) 100 T2

32. Let the three-digit numbers satisfying the given conditions ∴ T2 – T1 = 500 K Ans: (500)
be denoted by abc
Q
a+c 37. P α , other factors being constant where Q and V are
b= ……(1) V
2 mass and volume of the gas.
Let the volumes of three chambers be V1, V2, V3
ab bc
= respectively.
a +b b+c V1 V2 V3
10a + b 10b + c Let pressures in 3 chambers be P1, P2 and P3
=
a+b b+c P1V1 = P2 V2 = P3 V3 ----(1)
[Since PV = KQ and as Q is same for all chambers, KQ is
(10a + b) (b + c) = (10b + c) (a + b) constant]
10ab + b2 + 10ac + bc = 10ab + ac + 10b2 + bc V1 : V2 : V3 = 1 : 2 : 3
ac = b2 So, P1 = 2P2 = 3P3
2
 a+c  6 6 6
From (1), ac =   ∴, P1 P2 : P3 = : :
 2  1 2 3
(a − c)2 = 0 P1 : P2 : P3 = 6 : 3 : 2 Choice (C)
∴a=c
From (1), b = c 38. H = K T − 20 where K is a constant.
∴a=b=c H 16
∴ Nine numbers satisfy these conditions. Choice (C) K= = =4
T − 20 36 − 20
33. Let the required distance be x m.
D = kt2 where k is a constant. 20 = 4 T − 20 T = 45 Choice (B)

D 500 39. Let the number of members be x.


k= = =5
t2 10 2 T=F+V
(110)(50) = F + 50K ….(1)
x = Distance the body falls in 9 seconds, Distance the body (80) (80) = F + 80K ….(2)
falls in 8 seconds = 5(92) − 5(82) = 85 m Ans: (85) In (1) and (2) K is a constant.
Solving (1) and (2)
34. Let Q = A + B y + C(3 y ) F = 4000 and K = 30
130x = 4000 + 30x
Where A, B and C are constants. x = 40 Ans: (40)
60 = A + B + C ⇒ B + C = 60 − A
40. Constant part Variable rate Units Total
230 − A
230 = A + 8B + 4C ⇒ 2B + C = x (Patna) y 100 x+100y = 500
4 5x + 900y
5/3x (Bangalore) y 300 = 1100
729 − A 3
729 = A + 27B + 9C ⇒ 3B + C = Solving the above two equation, we get y = 2; x = 300
9
5
2B + C − (B + C) = 3B + C − (2B + C)  3  x = 5/3 × 300 = `500
 
230 − A 660 − A 230 − A
∴ − (60 − A )= − Required difference = 500 − 300 = 200 Choice (A)
4 9 4
Solutions for questions 41 to 45:
Among the choices given, only Choice (C) satisfies the
equation above. Ans: (30) 41. Let 1 Rupee coins, be x; 50 paise coins be y and 25 paise
coins be z.
35. Let cost of supply = s From statement Ι, x + y + z = 20.
Price per unit = p From statement ΙΙ, x = 6 and y : z = 6 : 1.
s = k 1 + k 2 p + k 3 p2 Combining statements Ι and ΙΙ, we get x = 6, y = 12, z = 2.
Given, s = 9, when p = 1 Choice (C)
⇒ 9 = k1 + k2 + k3 ----(I)
s = 24, when p = 2 42. Let the number of technical staff be 3x and non-technical
⇒ 24 = k1 + 2k2 + 4k3 ---(II) staff be 10x.
s = 47 when p = 3 From statement Ι,
⇒ 47 = k1 + 3k2 + 9k3 ----(III) 80 40
(II) – (I) gives 15 = k2 + 3k3 ----(IV) Total number of graduates = (3x) + (10x)
(III) – (II) gives 100 100
23 = k2 + 5k3 ---(V) = 2.4x + 4x = 6.4x
(IV) – (V) gives 6. 4 x
Percentage of graduates = × 100 = 64%
8 = 2k3 10 x
k3 = 4 So statement Ι alone is sufficient.
⇒ k2 = 15 − 12 = 3 From statement ΙΙ,
⇒ k1 = 9 − 4 − 3 = 2.
32 x
Hence, when p = 4 The required ratio = × 100%
s = k1 + 4k2 + 16k3 = 2 + 12 + 64 = 78 Choice (D) 65 x
So statement ΙΙ alone is also sufficient. Choice (B)
Triumphant Institute of Management Education Pvt. Ltd. (T.I.M.E.) HO: 95B, 2nd Floor, Siddamsetty Complex, Secunderabad – 500 003.
Tel : 040–27898194/95 Fax : 040–27847334 email : info@time4education.com website : www.time4education.com SM1001961/14
43. Let x be the number of boys and y be the number of girls A A
x 5 8. A% of + % of
From statement Ι, = ⇒ 4x = 5y B B
y 4 A
A A B
From statement ΙΙ,
x−4
=1⇒x=y+4 A=   + (A )
y 100  B  100
Combining both the statements, we get the value of y which A2 A2 A2
is the number of girls. Choice (C) = + =
100 B 100 B 50 B
44. From statement Ι, we don’t know what percentage of boys A C A AC
C% of =  =
and what percentage of girls are intelligent, so we can’t B 100  B  100B
answer the question.
A2 AC
From statement ΙΙ, =
50B 100B
20% of boys and 20% of girls are adults so (100 – 20)%
= 80% of students are not intelligent, whatever be the ratio A2
of boys and girls. Choice (A) 50B
C= =2A Ans: (2)
A
1 100B
45. From statement Ι, we have x = kg (assuming total as
8
a ab
1 kg) 9. a% of b = (b ) =
100 100
3 3 1 
Also B = kg i.e.,  × kg  b ab
16 2 8  b% of a = (a) =
100 100
As we do not know about others, we cannot find who
331 / 3
received least from this statement alone.
From statement ΙΙ, we have
331/3% of (a + b) = (a + b)
100
3 1 1
A= kg; Y = kg = (a + b)
8 8 3
Again statement ΙΙ alone is not sufficient, as we do not
ab ab 1
know about others. + = (a + b)
Using both, we know about four persons out of five. We can 100 100 3
find how much Z received and hence, also the one who 2ab 1
received the largest part. Choice (C) = (a + b)
100 3
Chapter – 3 6 a+b
=
(Percentages – Profit & Loss – Partnerships) 100 ab
6 1 1
Concept Review Questions = +
100 a b
Solutions for questions 1 to 50:  1 1
6% =  +  %. Choice (A)
a b
40 x 2000
1. × 50 = × 80 ⇒ x = = 25
100 100 80 10. Let the value of the property be `x
Choice (B) The value of the part that Ganesh owns (in `)
1
83
200 = 3 ×x= 5 x
2. × 100 = 250% Ans 250 100 6
80
35 
Three fourths of this part is worth `  x
3. Let the number be x. 4  6 
32% of x = 256
5
32 i.e., ` x
⇒ x = 256 8
100
⇒ x = 800 Choice (B) 5
x = 5, x = 8 Ans: (8)
8
4. 60% of x = 60 + 60% of 60
60 11. Let the total marks be x
= 60 + (60) = 60 + 36 = 96. Choice (C) 80% of x = 720
100
720 ×100
x= = 900 marks
508 80
5. 1013/5% = % = 508/500 = 127/125 Choice (D) 90 × 900
5 90% of x = = 810 marks
100
6. Let the number be x. Alternate method:
 60   70  If 80% of x = 720
    (x ) = 1680 90% of x = 90/80 x 720 = 810 Choice (C)
 100   100 
42 12. Let the maximum mark in the test be M.
⇒ x = 1680 ⇒ x = 4000. Ans (4000) 35
100 Pass mark = M
100
7. 40% of y = 75 + 20% of 1500 The student failed by 15 marks.
40 20 ∴ Pass mark R = 230 + 15 = 245
y = 75 + (1500)
100 100 35
M = 245 ⇒ M = 700 Choice (A)
0.4 y = 375 ⇒ y = 937.5. Choice (C) 100
Triumphant Institute of Management Education Pvt. Ltd. (T.I.M.E.) HO: 95B, 2nd Floor, Siddamsetty Complex, Secunderabad – 500 003.
Tel : 040–27898194/95 Fax : 040–27847334 email : info@time4education.com website : www.time4education.com SM1001961/15
1 Alternate method:
25 − 17
13. Reduction in the tax (in `) = 2 (9200) Any quantity which decreases by x% must be increased by
100 100 x
= 690 Ans: (690) % to become its original value
100 − x
14. Let the number of votes polled be x. 100 x
As x = 20, = 25 Choice (A)
70 100 − x
Number of votes secured by Ashok = x = 0.7x
100
He won by a majority of 0.4x votes. 25. Let the original price be `x
0.4x = 168 ⇒ x = 420 Choice (B) Increase in the price = `0⋅2x
Price after the increase = `1·2x
15. Let the original price be ` x 0 ⋅ 2x
Increase = ` 0⋅3x % decrease = (100 ) = 16 2 %
1 ⋅ 2x 3
x + 0⋅3x = 260 x = 200 Ans: (200)
Alternate method:
16. Profit = `300.
Any quantity which increases by x% must be decreased by
Profit percentage =
300
(100 )% = 50% Choice (A) 100 x
% to become its original value
600
100 + x
17. Ratio of profits of P and Q = Ratio of the investments of 100 x 2
P and Q = 1: 2 As x = 20, = 16 % Choice (C)
100 − x 3
2
Q’s profit = (9600 ) = ` 6400 Ans: (6400)
3 5 2
26. Ratio = :
6 3
18. Let the original price be `x.
Let the first number be 5x,
30
x− x = 63 ⇒ x = 90 Choice (C) Second number is 4x
100
5x − 4x
% less = × 100
19. Let the marks of Mohan and Sohan be m and s 5x
respectively. = 20% Choice (A)
 25  5
m = s 1+  = s. 27. Let the price of fan B initially be `x
 100  4
Price of fan A initially = `2x
m = minimum mark required to pass i.e. pass mark = 35.
5
s = 35 Increase in the price of A =
10
(2x ) =` 0 ⋅ 2x
4 100
s = 28. Ans: (28)
Decrease in the price of B =
20
(x ) =` 0 ⋅ 2x
20. Let Ravi’s salary after the increment be 4x 100
∴ Before the increment it was 3x and the total family income As the increase equals the decrease, the sum of the prices
was 15x. After Ravi’s increment, the family income is 16x. does not change.
Required percentage = ∴ required percentage change = 0% Ans: (0)
Ravi' s new salary 4x
(100%) = (100%) = 25%.
Total family income 16 x 28. Suppose there is a group of quantities. If all are increased /
Choice (D) decreased by the same percentage, their sum will
increase/decrease by the same percentage. Otherwise we
21. As the salaries of the two persons are equal and since the can say that the change in the sum is greater than the least
percentage increase in the salary for one person equals the percentage and less than the greatest percentage.
percentage decrease in the salary for the other, the The salaries of some of the employees are increased by a
increase in the salary and the decrease in the salary must different percentage compared to that of the others.
be equal. ∴ We cannot determine on the percentage change in the
∴ the total salary of the 2 persons does not change. total salary. We can say it has to be between 10% and 20%.
Choice (C) Choice (D)

22. If any quantity becomes n times. 29. Let the initial price of the article be `100
We say it increased by (n − 1) 100%. Decrease in the price = `20
As the population quadrupled, it increases by Price after the decrease = `80
(4 − 1) 100% = 300%. Choice (C) Increase in the price = `16
Price after the increase = `96
23. Let Bala’s marks be 100. ∴ the price effectively decreases by 4% Choice (C)

Arun’s mark = 100 +


20
(100 ) = 120 30. Let the initial price of the article be `100
Increase in the price = `10
100
Price after the increase = `110
Required % =
20
(100 ) = 16 2 %. Choice (B) Decrease in the price = `11
120 3 Price after the decrease = `9
∴ the price effectively decreases by 1%
24. Let the original price be `x
Decrease in the price = `0⋅2x Note: It can be seen that in problems 1 and 2 the
Price after the decrease = `0⋅8x final results are the same although the order of operations vary
∴ in problems on successive percentages the operations
0 ⋅ 2x
% increase = (100 ) = 25% can be performed in any order as the result does not
0 ⋅ 8x depend on the order of the operations. Choice (C)

Triumphant Institute of Management Education Pvt. Ltd. (T.I.M.E.) HO: 95B, 2nd Floor, Siddamsetty Complex, Secunderabad – 500 003.
Tel : 040–27898194/95 Fax : 040–27847334 email : info@time4education.com website : www.time4education.com SM1001961/16
31. Let the number be x. 42. Total cost price (80) (12·50) = `1000
20 Total profit = (80) (4⋅50) = `360
Decrease in it = x = 0.2x Total selling price = `1360
100
Total selling price of 20 articles = `360
Its new value = 0.8x
Selling price of each of the remaining articles
20
Increase in the new value = (0.8x) = 0.16x 1360 − 360 2
100 = = ` 16 Choice (B)
Final value of the number = 0.96x which is < x 60 3
∴ The number decreased.
x − 0.96x 43. Let the M.P. be `100
It decreased by (100)% i.e., 4% 30
x First discount (in `) = (100) = 30
Choice (C) 100
Price after this discount = `70
32. Let the first, second and the third numbers be f, s and 100 10
respectively. Second discount (in `) = (70) = 7.
100
 40  Price after this discount = `63
f = 100 1 −  = 60 Total discount = `37
 100  Equivalent single discount percentage = 37%
 50  Ans: (37)
s = 100 1−  = 50
 100  44. Let his C.P. = `100
Required percentage = 60 (100)% = 120%. Choice (D)
 25 
His M.P (in `) = 100 1+  = 125
33. The salary Raja’s as a percentage of his family’s  100 
total income increased by 10 percentage points.
Choice (C) 30
His discount (in `) = (125) = 37.50
100
34. Profit = `36 = 6% of CP His S.P. = `87.50
6 His loss = `12.50
⇒ × CP = 36 ⇒ CP = `600 Ans: (600)
100 12 .5
His loss percentage = (100) %
100
35. Suppose 2 articles have the same cost price and one is sold at = 12.5% Choice (D)
x% profit and the other is sold at y% loss. The overall profit/loss
1 45. Let his C.P. = `100
percentage is given by (difference of x and y)%
2
 40 
As x = 20 and y = 10, overall M.P (in `) = 100 1+  = 140
Profit/loss percentage = 5% profit Choice (D)  100 

36. Let the selling price be `x 12


profit in (`) = (100) = 12
Cost price = `0⋅9x 100
Profit = `0⋅1x S.P. = `112
0 ⋅ 1x discount = `28.
Profit percentage = (100 ) = 11 1 % Choice (C) discount ( Q Discount = M.P – S.P)
0 ⋅ 9x 9
28
= (100) % = 20% Ans: (20)
 1  140
 100 + 33 
 100 + 20   3 
37. Selling price = 50 
 100   100  46. Let the marked price be `x.
  Selling price = `400
 
 6  4  1
100 − 11
= 50     = ` 80 Ans: (80) 9 x = 400 ⇒ x = 450
 5  3  Ans: (450)
100
38. Let the cost price of the item be `x
47. Ratio of profits of Ajay and Vijay = 9 : 12 = 3 : 4
Profit = `0⋅2x
x + 0⋅2x = 60 ⇒ x = 50
To gain 30% profit must be `0⋅3x
Ajay’s share =
3
(3500 ) =` 1500 Choice (A)
7
∴ selling price must be `1⋅3x = `65 Choice (B)
48. Ratio of profits of Ramesh and Suresh
39. Let the cost price of each in be `x = (9000) (10) : (6000) (12) = 5 : 4
Profit made on selling 5 m = cost price of 2 m = `2x
Suresh’s share = (4500 ) = ` 2000
4
Cost price of 5 m = `5x Choice (D)
9
2
Profit percentage = (100 )% = 40 % Choice (C)
5
49. Annual income of Ashok =
6
(12000 ) = `720
100
60 − 50
40. Required percentage = (100 )% = 20% Ans: (720)
50
Choice (C) 50. Market value = `105
12600
41. Profits of the company, dealer and shopkeeper are `10, Number of shares purchased = = 120
`15 and `25 respectively. 105
∴ The shopkeeper got the highest profit Choice (C) Annual income of Ajay = (5) (120) = `600 Choice (A)
nd
Triumphant Institute of Management Education Pvt. Ltd. (T.I.M.E.) HO: 95B, 2 Floor, Siddamsetty Complex, Secunderabad – 500 003.
Tel : 040–27898194/95 Fax : 040–27847334 email : info@time4education.com website : www.time4education.com SM1001961/17
Exercise – 3(a) 100
7. Total food grain production in 2001= × 1000 = 4000
25
Solutions for questions 1 to 35:
 100 − 4 
1. Let the price in 2003 be 100.
Production of rice in 2002 =   × 1000 = 960t
 100 
Price in 2004 = 110
Rice production as a percentage of food grain production,
With respect to the price in 2004, its price is less by
in 2002 = 25 + 5 = 30%.
110 − 100 1 ∴Total food grain production in 2004
× 100 = 9 %. Choice (B)
110 11 100
= × 960 = 3200. Ans: (3200)
30
2. Let the total marks be x and the pass marks be p
0⋅38x = p + 18 –––––– (1)
8. From the given information, the percentage of employees
0⋅27x = p − 37 –––––– (2)
who took retirement through the scheme
∴(1) and (2) give 0⋅11x = 55
9120
55 = x 100 = 20%.
⇒x= = 500 and p = 0.38x – 18 = 172 45600
0.11
Now by taking 100 as the base, we can calculate the
172 answer easily.
Required percentage = × 100 = 34⋅4%.
500 Out of 100 employees, 40 applied for VRS.
Ans (34.4) But PSU allowed only 85% of 40 i.e. 34 employees to take VRS.
∴34% – 20% = 14%
3. Let the initial cost of house hold items be x i.e., 14% of the total employees did not take retirement,
100 + 20 although their applications are not rejected.
Present cost = x = 1.2x. Choice (B)
100
Let the initial salary of family be y. 9. Number of students in the school = 2,000
100 + 10 Number of students in higher secondary = 40% of 2,000 = 800.
Present salary of family = y = 1⋅1y
100 Number of girls in higher secondary = 28% of 800 = 224
Initially 0⋅2y = x Number of girls who passed = 112.
⇒ 1⋅2x = 0⋅24y Overall percentage of students who passed in higher
Percentage of present salary being spent on household secondary = 40% of 800 = 320.
Number of boys who passed = 320 − 112 = 208.
0 ⋅ 24 y 240 9
items = ×100 = = 21 %. Choice (A)
1 ⋅ 1y 11 11
Choice (D) 10. Let the number of votes Q gets be x
Number of votes P gets = 2.2x
4. Let the manufacturing cost in 2001 be `100 Number of votes R gets = 2.2x − 3,50,000
Transportation cost in 2001 = `30 5
x − 2.2x − 3,50,000 = ( x + 2.2x + 2.2x − 3,50,000)
Total cost in 2001 = `130 100
20 x = 2,50,000
Manufacturing cost in 2002 = 100 + (100) = ` 120 Number of votes polled = 5.4x − 3,50,000 = 10,00,000
100 Choice (B)
1
33 11. Let the incomes of A, B, C in the month considered be a, b, c
Transportation cost in 2002 = 3 (120 ) = ` 40 respectively.
100
Total cost in 2002 = `160 Amount spent by A, B, C are 75 a , 80 b , 62.5 c
100 100 100
160 −130 respectively
Required percent = (100) = 18.75%
160
75 : 80 : 62.5 = 42 : 64 : 55
Ans: (18.75) a b c
100 100 100
5. Let the male population and the female population in 2001
80 62.5
be x and y respectively. Let 75 a = 42k , b = 64k , c = 55k
100 100 100
25 3x
Male population in 2002 = x − x= a = 56k, b = 80k, c = 88k
100 4
Total Income = a + b + c = 644000
20 6y 224k = 644000
Female population in 2002 = y + y= 1 1
100 5 Income of A = a = 56k = (224k) = (644000)
4 4
6y 4 = 161000 Ans: (161000)
44
5 = 9 = 4 ⇒ x = 2y
3x 4 5 12. Total number of matches currently played = 60
100 − 44 75
4 9 Number of matches lost = × 60 = 45.
2 100
Males form 66 % of the population in 2001.
3 The Indian cricket team loses the next 12 matches
Choice (B) ⇒ Total number of matches lost = 45 + 12 = 57.

6. Let the length and the breadth be ℓ and b respectively. ℓ ≥ b. For a success rate of 50% the minimum matches to be
100
If ℓ = b, percentage increase in the perimeter is 15%. played = 57 × = 114.
100 − 50
Also if ℓ > b, percentage increase is more than 15%.
(Since the 57 matches should be equal to 50% of total
∴ Percentage increase can be 16%. Choice (D) matches played. Hence 100% - total matches).
Choice (D)
Triumphant Institute of Management Education Pvt. Ltd. (T.I.M.E.) HO: 95B, 2nd Floor, Siddamsetty Complex, Secunderabad – 500 003.
Tel : 040–27898194/95 Fax : 040–27847334 email : info@time4education.com website : www.time4education.com SM1001961/18
13. Let the salary in 2003 be x.  x  y 
The tax paid by him in 2003 is 0⋅2x  1+  1− 
 100   100 
 93 ⋅ 75 
Salary in 2004 = x1 +  = 1⋅9375x
100 
2
 y 2 
If x = y, this equals (4 )(12)  1−

which is less than (4)
This salary > 10000  100 
 
Given tax difference = 500
(12). But he had more than (4) (12).
(1⋅9375x − 10000) 0⋅2 + 1000 − 0⋅2x = 500
∴ x > y.
(1⋅9375) (0⋅2)x − 1000 − 0⋅2x = 500.
0⋅1875x = 1500 
2
x   y 
2
, (4 )(12 )1+
100 y
x = 8000. If x =
100 − y  1− 
Salary in 2004 = 1⋅9375x = `15500. Choice (A)  100   100 
2
 100   100 − y 
= (4 )(12)   = (4 )(2)
14. Let the number of sharpeners that Anil bought be x
 
Total expenditure = 2.5s (25) + s(x) 
In the conditional case, total expenditure = 2.5s(x) + s(25)  100 − y   100 

1 100 y
This is 9 % less than the actual total expenditure ∴x> must hold true. Choice (A)
11 100 − y

 1  18. 0⋅82CP + 990 = 1⋅15CP


 9 
2.5x + s(25) = [2.5s(25) + xs] 1 − 11  990 = 0⋅33CP
 100  CP = `3000
 
  To make a profit of 10% he should sell at
s = 20 Ans: (20) 3000 × 1⋅1 = `3300. Ans: (3300)

15. Ratio of market shares of Margo, Palmolive, Dove and the 880
new soap in 2001 = 0.9(40) : 0.9(30) : 0.9(30) : 10 19. Cost price of the first shirt = = `800
1⋅ 1
= 36 : 27 : 27 : 10
36 880
Market share of Margo in 2002 = (90) = 32.4% Cost price of the second shirt = = `1100
100 0⋅8
Choice (B) ∴Overall cost price `1900
Overall selling price = 880 × 2 = `1760
16. Let the original price of that jewel be ` P.
140
 x  x  Loss = × 100 = 7⋅36%. Choice (D)
P1+   1−  = P −100 1900
 100   100 
20. Let the initial cost be `100.
 x  x  P −100 It went up by 40% i.e., new cost price = `140
⇒ 1+  1−  = P
 100   100  Initial Selling Price = `120.
(as the profit is 20%)
 x  x  Present Selling Price = 140(1⋅2) = `168
⇒ (P − 100 )1+   1−  = 2304 Percentage increase in selling price
 100   100 
168 − 120
 P −100  = × 100
⇒ (P −100)  = 2304 120
 P 
⇒ P2 − 2504P + 10000 = 0 48
= = 40% Ans: (40)
(P − 2500) (P − 4) = 0 1⋅ 2
P must be greater than 2304 since each up down cycle
reduces the price effectively. 21. Let the Marked price = `x
∴ P = 2500
100 − 40
Discounted price = × x = 0⋅6x.
 x  x 
Note:− (P −100 ) 1+  1−  100
 100   100 
The dealer reduces the price further by 20%

(P −100)1+x 
 1−
x  
 −100 1+
x 
 1−
x 
 ⇒ Selling price =
100 − 20
× 0⋅6x
 100   100   100   100  100
= P − 100 − (some number which is less than 100) = Some = 0⋅48x
number which is more than P − 200 Given 0⋅48x = 9600 ⇒ x = 20000.
∴ 2304 > P − 200 Choice (C)
2504 > P
Only Choice (D) satisfies this condition. Choice (D) 22. Let the total cost price of the shopkeeper be c
Total profit made by the shopkeeper
17. Number of goats with Sunil at the end of 2002

 x  y  18  2  27.5  3  30  8  5  1 
= (4 )(12) 1+  c +  c +  c  = 25 c
=  
 1− 100  100  9  100  5  100  45  6  100 
 100  

Number of goats with Sunil at the end of 2003 Total profit 5


Overall profit % = .100% = 25 %
Total cos t price 6
 x  y 
= (4 )(12) 1+  1− 
 100   100  Choice (B)

Triumphant Institute of Management Education Pvt. Ltd. (T.I.M.E.) HO: 95B, 2nd Floor, Siddamsetty Complex, Secunderabad – 500 003.
Tel : 040–27898194/95 Fax : 040–27847334 email : info@time4education.com website : www.time4education.com SM1001961/19
23. Let the initial selling price of each mango be `x. 28. Total cost of printing (in rupees) = 8[24 – 2(2)] [14 – 2(2)]
∴New selling price = `1.25x. (2000) = (1600) (2000) = 3.2 million.
Overall selling price now = 1.25x (80) = `100x. Charge for 2000 pages to a profit of 8% must be `3.2 million
∴ As there was 25% overall profit on selling, (1.08). Then charge per page must be

Overall cost price =


(100x ) (100)
= `80x. 3.2 million (1.08 )
125 = `1728 Choice (A)
2000
If the remaining 80 apples are sold at `x, then the selling
price = `80x. 29. Venus Serena
∴Overall C.P = Overall S.P, hence no profit or no loss.
Months 3 3
Alternate method: Investment 50,000 50,000
Months 9 9
Let the cost price of 1 mango = `1 Investment 40,000 50,000
Cost Price of 100 mangoes = `100 …. (1)
Profit percentage expected = 25% Venus’s profit percentage
∴Proposed sale price per mango = `1.25 …… (2)
20 mangoes are rotten; 80 mangoes are available for sale. (3)(5) + (9)( 4 )
= × 100
If these mangoes are sold at the original sale price of `1.25 [(3)(5 ) + (3 )(5) + (9) (4) + (9) (5)]
per mango, the money received from the sale of 80 mangoes
= 80 (1.25) = `100 …… (3) 15 + 36 51 5100
= × 100 = × 100 = %
As (1) and (3) are equal, there is neither profit nor loss. 30 + 36 + 45 111 111
Choice (A)
1700 35
= % = 45 % Choice (D)
Note: Information that sale price is increased by 25% is 37 37
redundant because, no mangoes are sold at this price.
30. Investment Ratio of Mehta, Mehra, and Mihir = 1 : 2 : 3
24. Let the initial cost price be `x. Ratio of time periods of Mehta, Mehra and Mihir
Initial SP = x + 25 = 12 : 8 : 4 = 3 : 2 : 1
Now, the cost price is reduced by `25 Profit would be distributed in the ratio of
New CP = (x – 25) (Amount Invested × Time)
Selling price is reduced by 25%. Here the ratio = 3 : 4 : 3
3 3 × 600000
∴New SP = (x + 25) × . Mehta gets = 180000. Ans: (180000)
4 10
Profit = 25%
5 3 31. Let Goyal’s salary be C and the total profit be P.
∴(x − 25) × = (x + 25) ×
4 4 p
C = 0⋅2 (p − C) ∴C = .
5x − 125 = 3x + 75 6
2x = 200 ⇒ x = 100. Ans: (100) 5p
Hence is shared in the ratio of their capitals
25. Let the correct weight be 100 kg 6
Let its cost be `100. i.e. 20,000 : 30,000 or 2 : 3 and Goyal’s share will be
S.P = `120 2 5p p
× =
Weight usually weighed = 80 kg 5 6 3
Present weight is 20% more = 96 kg
p p
Selling Price = `120 + = 4800 ⇒ p = `9600. Choice (C)
Cost of 96kg = `96 6 3

120 − 96 24
Profit percentage = × 100 = × 100 = 25%. 32. Let the profit be p. Hence half the profit i.e.,
p
will be
96 96 2
Choice (B)
p p p
shared equally i.e., , and remaining will be shared
26. Let the selling price of one television be x. 4 4 2
1 in the ratio 24 : 36 or 2 : 3
Total money made = (50x) = 25x 3 p 3p
2 ∴Gayathri will get × =
(since one TV is given free with each one bought by 5 2 10
customer.) 3p p 11p
Given total cost = 10,000 × 50 = 500000 Total earnings of Gayathri = + = –––– (1)
10 4 20
Loss = 15(x) = 15x
Had the entire profit been shared in the ratio of their
Selling price + loss = cost price ⇒ 25x + 15x = 500000
capitals,
40x = 500000
x = 12500 3 12
Gayathri would have got p= p –––––– (2)
Selling price of each television = `12500. Choice (C) 5 20
12p 11p
27. Let the cost price for Balu be C ∴Given – = 2500 [from (1) and (2)]
20 20
Price at which Balu sold the article
∴p = `50000. Choice (D)
= C  5  5  = 25 C
 4  6  24 1395
33. Number of shares bought = = 15
C 93
Profit made by Balu = .
24 He sells `1000 worth stock i.e., 10 units of stock at 95
25 i.e., gain of 2 × 10 = `20.
Profit percentage = 100 % = 4 1 % = % Remaining 5 shares are sold at 80, i.e., loss of 5 × 13
24 6 6 = `65. Net Loss of `45, in the sale and purchase transaction.
Ans: (25) Choice (D)

Triumphant Institute of Management Education Pvt. Ltd. (T.I.M.E.) HO: 95B, 2nd Floor, Siddamsetty Complex, Secunderabad – 500 003.
Tel : 040–27898194/95 Fax : 040–27847334 email : info@time4education.com website : www.time4education.com SM1001961/20
34. Dividend = `3600 6. It can be seen that it depreciates by (18 − 1.5(n − 1)) in the
3600 nth year.
Total number of shares = = 600 ∴ In the seventh year it depreciates by 9%.
6
(since it is a 6% stock) ∴ required amount = 9% (4,00,000) = 36,000
Ans: (36000)
Premium of 5% ⇒ cost of stock = 100 + 5 = 105
Total amount invested by Ismail = 105 × 600
7. Present salary of Mrs. Rai = 20000
= `63000. Ans: (63000)
This is a gain of 25% from last month’s earnings.
⇒ Last month’s salary of Mrs. Rai.
35. Let the total investment be 110 × 90 × x.
1st case 100
= × 20.000 = 16000
125
110 × 90 110 x
Number of 4% stock at 90 = x= = 55x Mr. Rai’s salary has come down to `20000. This represents
2 × 90 2 a 33⋅33% drop.
110 × 90 100
Number of 8% stock at 110 = x=
90
x = 45x Mr. Rai’s salary last month = × 20000
2 × 110 2 66 ⋅ 66
= 30000
Total returns = (55x × 4) + (45x × 8) = 580x Total income of Mr. Rai and Mrs. Rai last month
2nd case = 30000 + 16000 = 46000. Choice (C)
110 × 90
Number of 8% stock at 110 = × x = 90x 8. Let the monthly income of Ram be `100 initially.
110 Initial expenditure of Ram = `70.
Total returns = 90x × 8 = 720x Initial Saving of Ram = `30
Given 720x – 580x = 3500 ⇒ x = 25 Final monthly income of Ram = `126
His investment = 110 x 90 x 25 = 247500. Final expenditure of Ram = `84
Choice (B) Final savings of Ram = `42
Percentage increase his savings = 40% Choice (A)
Exercise – 3(b)
9. Let its original area be x sq cm.
Solutions for questions 1 to 45:
40
Area after the first cut = x − x = 0.6x
1. Let B’s salary be x. 100
100 − 20 50
A’s salary = × x = 0⋅8x Area after the second cut = 0.6x − x (0.6x ) = 0.3x
100 100
C’s salary = 10000 and it is also 25% more than B’s salary Given, 0.6x − 0.3x = 30 ⇒ x = 100 Ans: (100)
100 + 25
⇒ × x = 10000. 10. Let the breadth be 100 units initially. Let the length be
100 100 units initially.
x = `8000. New breadth
A’s salary = 0⋅8x = `6400 Choice (D)
 20  30  20 
= 100 − (100)  +  100 − (100 ) 
2. Let the numbers of students in school B = x  100  100  100 
Number of students in school A = 1⋅3x
Given 1. 3 x – x = 0⋅3x = 120 130
= (80) = 104 units
⇒ x = 400 100
Total number of students in both the schools
= (1 + 1⋅3)x = (2⋅3) x400 = 920. Ans: (920) New length =
Area
New Breadth
3. Let the initial price be `100
10000 2
 2  4  2  = = 96 units
Final price = 100 + (100)  +  100 + (100)  104 13
 100  100  100 
= `106.08 100 − 96
2
(100)100% 11
∴ Percentage increase = 6.08% Choice (B) ∴ Required percent = 13 =3 %
100 13
25 Choice (D)
4. Let the pass mark be x. Rahul’s mark = x + x =150
100 11. Let the cost price of the laptop be 100x.
x = 120 Selling Price (15% profit) = 115x
165 − 120 If the selling price was 10% lower i.e., 103⋅5x,
Required percent = (100) = 37.5% Profit was `1050 ⇒ 3.5x = 1050 ⇒ x = 300
120
Cost price of the laptop = 100x = 30,000.
Ans: (37.5)
Choice (C)
30 12. Let the minimum number of matches be x. Let us say, of
5. Number of girls = (1000) = 300
100 these x matches y matches were won. Number of matches
80
Number of boys = 1000 − 300 = 700 won after the coach took over = (60) = 48 .
Number of UGs = 1000 − 600 = 400 100
Let the number of female UGs be f. Number of male 48 + y
Given, (100 ) = 60 ⇒ 48 + y = 0.6(60 + x)
UGs = 400 − f ⇒ 400 − f − f = 120 60 + x
⇒ f = 140
140 ⇒ x would be minimum when y = 0.
Required percent = (100) = 20% Choice (A) In this case, x = 20 Ans: (20)
700

Triumphant Institute of Management Education Pvt. Ltd. (T.I.M.E.) HO: 95B, 2nd Floor, Siddamsetty Complex, Secunderabad – 500 003.
Tel : 040–27898194/95 Fax : 040–27847334 email : info@time4education.com website : www.time4education.com SM1001961/21
13. Let Mr. Singh’s initial income per hour be `100 17. Let the initial total income in 2001 be 100.
Present income per hour = `120 The total income of the family in 2001
Let the number of hours per month be 100, initially = 30 + 42 + 9 + 24 = 105
Present number of hours = 80 After 20% increase, total income of the family
Earlier monthly income = 100 × 100 = 10000 105 × 120
Present monthly income = 120 × 80 = 9600 = = 126
100
Percentage decrease in his income
∴Increase = 21 = Increase in salary of A.
400 21
= × 100 = 4%. Required percentage increase = × 100 = 70%.
10000 30
Choice (C)
Alternate method:
18. Let the number of questions attempted by Ram be x. Let
Mr. Singh’s income (I) is the product of his hourly rate (R) the pass mark be p.
and the number of hours he works (n).  70x  30
i.e., I = rn Ram’s mark = 2   − 100 x = 1.1x
 100 
6R
 60 
 (100 − x) − 100 (100 − x )
R increases by 20% to and 40
5 Shyam’s mark = 2 
 100 
4n ⇒ 1.1x = p + 40 ……(1)
n decreases by 20% to .
5 ⇒ 0.8(100 − x) = p + 25 ….(2)
Solving (1) and (2), x = 50 and p = 15 Choice (A)
6R 4n
∴The income changes to = x
5 5 19. Percentage secured by Ram
(1.1) (50 ) 1.1(50)
24 = (100 )% = (100)% = 34.375%
= I , i.e., it decreases by 4%. Choice (C) 80( 2) 80( 2)
25
Choice (D)
14. Let the income in Dec 2014 be I and in Feb 2015 be J. 20. The price of a computer four years ago was `50000. Now,
Amount spent on utility bills it is `32805
2  75  I i.e., it has decreased by a factor of
– in Dec 2014 =   I= 32805
15  100  10 = 0 ⋅ 65610
50000
If each year the price decreases by a factor of r, in
– in Feb 2015 = 7  72  J =
21 J 4 years it decreases by a factor of r4.
48  100  200 i.e., r4 = 0⋅65610
∴r = 0⋅9
2 i.e., Each year the price decreases from p to 0⋅9p or
The second amount is 16 % more than the first
3 decreases by 10%. Choice (C)

I  7  = 21J I = 9 J 21. Let the whole sale price index on 1st April be points. As per data,
 
10  6  200 10  x  x 
p − p1 +  1 −  = 160
 100  100 
I is 10% less than J. Ans: (10)
 x 2   x2 
15. Let the total distance be 100 km. Let the total fuel be p − p1 − = 160 p  = 160 …… (1)
 10000   10000 
10 litres. As 10% of the fuel was used to cover 20% of his    
total journey driving conditions, a litre is required for driving
every 20 km (under given driving conditions). The second equation:
∴ Total fuel required for conditions which were different  x 2   x 2  x 2 
= Fuel required for the 60% of the journey p1 − − p1 − 1− = 134⋅4
 10,000   10,000  10,000 
   
60
(100 )
 x 2  x 2
= 100 = 3 litres . p 1 − ⋅ = 134⋅4 ……… (2)
20  10000  10000
 
∴ 7 litres is required for driving the balance 40 km.  x 2 
from (1), 160  1 − = 134⋅4
 10000 
40  
20 −
Required percent = 7 × 100% = 71 3 % x2
20 7 1− = 0⋅84 x = 40%
10000
Choice (A)
∴from (1), p = 1000. Choice (C)
16. Let the total salary of the family be `100.
22. Let the total cost be `100. Costs of P, Q and R are `10, `20
In the year 2001 the family saved 100 − 30 = 70
and `20 respectively
In the year 2002, the total salary of the family
= 30 + 42 + 9 + 24 = 105 Let costs of others is 100 − (10 + 20 + 20) = `50
Increase in each of the costs of P, Q and R is 2
To save 70, they have to spend 105 − 70 = 35
Increase in the total cost is `15
35 Increase in the cost of the others = 15 − 3 (2) = `9
Required percentage of spending = × 100
105 9
Percentage increase in the cost of the others = × 100
= 33⋅33%. 50
Choice (C) = 18%. Ans: (18)

Triumphant Institute of Management Education Pvt. Ltd. (T.I.M.E.) HO: 95B, 2nd Floor, Siddamsetty Complex, Secunderabad – 500 003.
Tel : 040–27898194/95 Fax : 040–27847334 email : info@time4education.com website : www.time4education.com SM1001961/22
23. Let the initial values of the length and the breadth be 31. Let the marked price be ` x.
l and b respectively. Let the cost price be `100
Intial perimeter = 2(l + b) Profit = `20
New perimeter = 2(1.1l + 1.2b) Selling Price = `120
2(1.1l + 1.2b) − 2(l + b) Discount = `0.2x
% increase in perimeter = × 100% =
2(l + b ) Given, x − 0.2x = 120 ⇒ x = 150
∴ The marked price is 50% above the cost price.
10l + 20b 10b
= 10 + Ans: (50)
l+b l+b
l≥b 32. Let the cost of a car be 100x.
∴l + b ≥ 2b After a discount of 100000, the profit = 15%
1 b ⇒ Selling price of the car = 115x, and
≥ Marked price of the car = 115x + 100000
2 l+b
Given that the M.P of the car is 25% more than the C.P of
10b the car ⇒ 125x = 115x + 100000
10 + ≤ 15
l+b ⇒ 10x = 100000, x = 10000
% increase in perimeter ≤ 15%. Choice (B) Marked price of the car = 115x + 100000 = 1250000.
Choice (D)
24. Let the cost price of P’s article P be `x
 20  33. For Samuel
Selling price of P’s article P = `  X + X  = `1.2X
 100  Let’s assume the marked price of each VCD is `100
For Simon
This is also the cost price of Q’s article
Cost price of each VCD = 0⋅75 × 100 = 75
 25  Cost price of 100 VCDs = 7500
Selling price of Q’s article Q = `2x 1 +  = `1.5X
 100  Now, undiscounted price of 25VCDs = price marked by
1.5x -1.2x = 90 ⇒ x = 300 Choice (C) Samuel for 25 VCDs = 2500
So, profit on 100 VCDs = 2500
25. Let the cost of 100 radios be `100. 2500
Cost of 65 radios = `65 Percentage of profit = × 100 = 33⋅33%.
7500
Selling price of 65 radios = `100
Choice (D)
100 − 65
Profit percentage = × 100
65 34. Let the cost of 10000 hard disks be `10000
35 11 After a discount of 10%, cost = `9000
= ×100 = 53 %. Choice (A) 20% of the hard disks are damaged.
65 13
⇒ Number of hard disks to be sold = 8000
26. Let his cost price be `x per watch Selling price = 20% more than the price at which he bought
10 Total sales = `(8000 × 1⋅2 × 0⋅9) = `8640
Given, x = 60 ⇒ x = 600
100 9000 − 8640 360
Loss percentage = ×100 = × 100 = 4%.
10 9000 9000
⇒ Profit = [( 600) ( 600)] = `36000 Ans: (36000)
100
Alternate method:
27. Suppose Rakesh purchased 2x chocolates.
He bought x chocolates at 15 paise each and x chocolates Ratio of CP of undamaged disks and damaged disks
at 20 paise each. = 80% : 20% = 4 : 1
Total cost price = 15x + 20x = 35x paise Undamaged disks are sold at a price which is 20% more
Selling price of each chocolate = 20 paise than the CP;
Total selling price = (20)(2x) = 40 x paise ⇒ profit percentage on these = (+)20%
Profit = 5x paise Damaged disks are to be taken as unsaleable;
5x = 1000 ⇒ x = 200 ⇒ 2x = 400 Choice (D) ⇒ loss percentage on these = (–)100%
Overall percentage of profit/loss
28. Number of articles sold = 20% of 50 = 10 = [4 × (+20) + 1(-100)]/(4 + 1) = (–20/5) = –4
Cost of 5 articles = `1200 ∴ 4% loss Choice (D)
Cost of 1 article = `240
Cost of 10 articles = `2400 Note:
Selling price of 10 articles = 2400 + 1200 = `3600 1. Information about number of disks is redundant.
Selling price of 1article = `360 2. The statement about the discount (10%) received at
Number of articles remaining = 40. the time of purchase, is also redundant.
Total value of the remaining articles at selling price
= 40 × 360 = 14400. Ans: (14400) 35. Let his profit / loss percentage be 3x%.
His discount percentage = 2x%
29. Let the cost prices for Ajay and Dinesh be `a and `d
respectively.  3x   2x 
His selling price = 200 1+  = 300 1− 
Cost price of Balu = `1.560.  100   100 
∴ Ajay’s article’s value increases by `0.56a.
Cost price of David = `0.8d.  3x   2x 
If he made a profit, 200 1 +  = 3001− 
Selling price of David = `0.56d  100   100 
0.56a + 0.56d = 28000
a + d = 50000 Choice (A) 25
x=
3
30. Let the cost price of the article = 100x
Marked price = 135x Profit percentage = 25
For profit to be 20%, selling Price = 120x  3x   2x 
135x − 120x If he made a loss 200 1−  = 300 1− 
Percentage of discount = × 100  100   100 
135x
200 − 6x = 300 − 6x which is not possible.
100 1 ∴ He made 25% profit.
= = 11 %. Choice (A) Choice (B)
9 9
Triumphant Institute of Management Education Pvt. Ltd. (T.I.M.E.) HO: 95B, 2nd Floor, Siddamsetty Complex, Secunderabad – 500 003.
Tel : 040–27898194/95 Fax : 040–27847334 email : info@time4education.com website : www.time4education.com SM1001961/23
36. Let the cost price to Feroze be `100 42. Let the investments of A, B, C and D be `3x, `4x, `5x and
Feroze marks up the price to `130 `6x respectively. Let the salary of B be `S.
He sells it at 20% discount; at `104 4x + S 9
Sohail marks up the price to 20% more than `100, equal to Given =
5 x + S 10
`120
The least sale price = 104 ⇒ S = 5x
To sell it at `104, percentage discount to be offered Total amount of profit = 18x + 2S
= 28x = 84000 ⇒ x = 3000
16 40 1 ∴B’s salary is 5x i.e., 15000 Ans: (15000)
= × 100 = = 13 %. Choice (D)
120 3 3
43. Let the number of registered voters be T
37. Let the cost price of Girish be `100. 90
Marked price of Girish = `160 Number of voters who voted = T
100
Let the maximum number of discounts be n.
160(0.9)n ≥ 100 ……(1) 90  90  81
Number of votes that were valid =  T = T
100  100  100
Maximum value of n satisfying (1) is 4. Ans: (4)

38. If there is a lower limit on the discount, there is a


corresponding upper limit on the profit. If there is an Number of votes that P got = 65  81 T 
upper limit on the discount, there is a corresponding lower 100  100 
limit on the profit. Neither statement (A) nor (B) is of this
type. Consider Choice (C). Number of votes that P won by = 65− 35 81 T i.e.
If cost price (c) = 300, marked price (M) = 400. 100  100 
Profit percentage = discount percentage, Profit (p) :
 81 
0.3 
81 
Discount (D) = 3: 4. T  = 0.3  T  = 9720 ⇒ T = 40000
As P+ D = 100, P = (3/7) (100) and D = (4/7) (100),  100   100 
i.e., profit percentage (p) Ans: (40000)
(3 / 7)(100) 2 44. The data is tabulated below. All the amounts are in
= (100%) = 14
(300) 7 thousands of rupees.
= discount percentage (d).
6 Month Satish Sanjay Sunil
If P ≥ D, (in particular if p = d), P = 42 ,
7 1st 20 20 20
1
while D = 57 , i.e., D can be greater than P. 2nd 21 22 20
7
∴ Choice (C) is false. Consider Choice (D). If p ≤ d 3rd 22 22 23
(in particular if p = d),
4th 23 24 23
2
p = d = 14 % , th
7 5 24 24 23

6 1 6th 25 26 26
P = 42 and D = 57 ,
7 7 7th 26 26 26
i.e., D ≥ P. If p < d, D is definitely greater than P. This is
8th 27 28 26
true. Choice (D)
9th 28 28 29
39. Let the labelled weight of dal be 100 gm
Let the labelled price of dal of 100 gm be `100 10th 29 30 29
Amount of dal in the packet = 80 gm
Price paid by the customer = `90 (because of 10% 11th 30 30 29
discount)
12th 31 32 32
Quantity to be received for `90 = 90 gm.
Dal to be added = 90 − 80 = 10 gm
∴ the ratio of their profit shares is
10
Percentage of dal to be added = × 100 = 12⋅5%.  20 + 31   22 + 30 
80
 2 12: 2  (5) + 20 + 32: 20 + 20 + 32 + 3(3)(26)
Ans: (12.5)    2 

40. Ratio of the profit shares of Gopal, Hari and Karthik = 51(6) : 52(6) : 72 + 9(26) = 153 : 156 : 153
= (8000) (I12) : (12000) (x) : (16000) (x) We see that Sanjay gets the greatest share.
= 96 : 12x : 16x Choice (B)
16x > 96 > 12x
∴ x > 6 and x < 8 45. Number of stocks at 5% premium for `19950
As x is an integer, x = 7 19950
= 190
∴ Required ratio = 24 : 21 : 28 Choice (B) 105
Dividend per year = 190 × 4 = `760 (4% stock)
41. Let the investments of P and Q be `p and `q respectively. Value of stock purchased at 5% discount for `19950
Salary of Q = `10000 = 19950/95 = 210
p
(50000 ) =
p
( 40000 ) + 8000 Dividend income from it = 6 × 210 = `1260
p+q p+q Total dividend = 1260 + 760 = 2020.
p:q=4:1 2020
4 Effective yield percentage = × 100 = 5⋅06%.
Profit share of p = (40000) = `32000 Ans: (32000) 19950 × 2
5 Choice (C)

Triumphant Institute of Management Education Pvt. Ltd. (T.I.M.E.) HO: 95B, 2nd Floor, Siddamsetty Complex, Secunderabad – 500 003.
Tel : 040–27898194/95 Fax : 040–27847334 email : info@time4education.com website : www.time4education.com SM1001961/24
Solutions for questions 46 to 55: 6000 3
Ratio of the revenues is =
8000 4
46. Each statement is not sufficient.
Hence either statement alone is sufficient.
Combining statements Ι and ΙΙ,
Choice (B)
25
S.P = 20 + × 20 = 25.
100 55. Reduced price = 120 – d
M.P ≤ 30. ∴ Discount ≤ 5. 25
From statement Ι, d = 120 × = 30
5 2 100
Hence, the discount percentage ≤ × 100 = 16 %
30 3 30
Required percentage = × 100 %
∴ Discount percentage may or may not be less than 16% 120 − 30
Choice (D) So statement Ι alone is sufficient.
From statement ΙΙ,
47. From statement Ι, profit = S.P – C.P so d = (1/3) (120 – d) ⇒ 4d = 120 ⇒ d = 30
C.P = 10 [S.P – C.P] ⇒ C.P = 10(165 – C.P)
30
so we can find the cost price of the cycle Required percentage = × 100 %
10 120 − 30
From statement ΙΙ, profit = × 160 = `16 So statement ΙΙ alone is also sufficient. Choice (B)
100
So the cost price of the cycle = 160 – 16 = `144
Chapter – 4
∴ Statement ΙΙ alone is sufficient. Choice (B)
(Simple Interest and Compound Interest)
48. Cost price of the article is not known so we can’t find the Concept Review Questions
profit percentage, so statement Ι alone is not sufficient
From statement ΙΙ, Solutions for questions 1 to 25:
18 Selling Price = 21 Cost Price
21 − 18 1. Interest earned on a sum of `P invested for T years at
Profit percentage = × 100 .
18 PTR
R% p.a. under simple interest = ` .
Statement ΙΙ alone is sufficient. Choice (A) 100

49. Combining statements Ι and ΙΙ, the percentage of income Interest earned =
(5000 ) ( 2) (10 )
25 65 100
saved is × × 100. Choice (C)
100 100 = `1000 Choice (A)

50. Combining statements Ι and ΙΙ, 2. The interest on a sum remains the same each year under
simple interest.
(7 × 12) = 12 × (x) ⇒ x = 7
(as they share equal profits) ∴ Total interest = 3(1000) = `3000 Ans: (3000)
∴ B joined after 12 – 7 = 5 months. Choice (C)
3. Let the sum be `x then,
51. Combining statements Ι and ΙΙ, we can say that they did not x×6×5
invest equal capitals because inspite of B getting = 306 ⇒ x = `1,020 Choice (D)
100
25% remuneration, they got equal profits. Choice (C)
4. Let the sum be `x
52. From statement Ι,
Let the population be x, and the female literacy rate be a%. x × 11× 6
Males = 0.6x so females = 0.4x S.I. = 25410 =
2 × 100
0.6 x × 25 0. 4 x × a 25
+ = x, solving, we can get the ⇒ x = `77,000 Choice (C)
100 100 100
value of a. 5. Let the time period in years be x then,
This can also be obtained by Alligation the rule of from
statement ΙΙ, 15000 × 5 × x
4000 =
x 100
Female literacy = × 100 % = 25%
4x 16
⇒x= years
So statement ΙΙ alone is also sufficient. Choice (B) 3
16
53. Either of the statements alone is not sufficient as earning Number of months = × 12 = 64 Ans: (64)
3
and the rate of commission is given in different statements.
Combining statements Ι and ΙΙ,
6. Let the sum be `x.
let the sales values be `x
7. 5 2.5 x × 37 × 6 x × 33 × 6
1500 = × (x) + [x – 9000] × . then, − = 54
100 100 100 100
Solving we can get the value of x. Choice (C)
x×4×6
⇒ = 54 ⇒ x = `225 Choice (A)
54. Let the selling price of item Y be `a 100
then the selling price of item X be `0.9a
From statement Ι, 7. Let the rate of interest be R% p.a.
Let the number of units of item Y sold be k Interest for 2 years = ` 600
Then the number of units of item X sold be 1.2k
(3000 ) (2) (R )
Revenue from item Y = ak 600 = R = 10.
Revenue from X = 0.9a × 1.2k = 1.08ak 100
Required ratio = ak : 1.08 = 25 : 27 The rate of interest is 10%.
From statement ΙΙ, For two years it is 20%. Choice (A)

Triumphant Institute of Management Education Pvt. Ltd. (T.I.M.E.) HO: 95B, 2nd Floor, Siddamsetty Complex, Secunderabad – 500 003.
Tel : 040–27898194/95 Fax : 040–27847334 email : info@time4education.com website : www.time4education.com SM1001961/25
24000 ×18 × 5 17. Let the sum be `P.
8. Simple Interest = = `21,600 Let the rate of interest be R% p.a.
100
Amount = Principal + Simple Interest  R 
3
 R 
2 
= 24000 + 21600 = `45,600 Ans: (45600) P  1 +  − 1 +   = 2420
 100   100  
 
9. Let the sum be `100.
 R 
4
 R 
3 
Sixteen times of sum = `1,600 P 1 +  − 1 +   = 2662
Interest = `1,500  100   100  
 
100 × 30 × x
1500 = 2
100  R  R
P 1 +  = 2420 --(1)
⇒ x = 50 i.e. 50 years. Choice (D)  100  100
3
10. If a sum takes T years to become N times at simple interest  R  R
P 1 +  = 2662 --(2)
at R% p.a., RT = 100 (N – 1)  100  100
100 (1.84 − 1) Dividing (2) by (1),
Rate of interest = = 12% p.a.
7 R
Choice (C) 1+ = 1.1 ⇒ R = 10
100
11. Let the sum be `x. Alternate method:
Then Amount = `10x.
Interest = `9x. (100) The interest on a sum for the kth year under
Let the time period be r years and rate of interest be r%. compound interest, interest being compounded annually is
x×r ×r 1+ R
⇒ 9x = ⇒ r = 30% Choice (C) (Interest for the (k – 1) th year)
100 100
 R 
2420 1 +  = 2662
12. Let the total amount invested be ` 2x  100 
Amount invested in each scheme = ` x
∴ R = 10
(x )(1)(15) + (x )(1)(20) = 0.35 x
Ans: (10)
Total S.I (in `) =
100 100 18. Let the sum be `100
0.35x = 1400 Amount after eight years = `400
x = 4000 Amount after sixteen years = `1,600
2x = 8000 Ans: (8000) i.e., it takes 8 + 8 = 16 years for the sum to become
sixteen times of itself.
13. Let its present value be `P.
100 400 1600
 6 (10 ) 
P 1 +  = 2400
 100 
P = 1500 Choice (D) 8 8

14. Let the rate of interest be R% p.a. 8 + 8 = 16 years Choice (D)


Amount at the end of 2 years = `630
19. Let the sum be `x.
 R  
2
Then, the amount after three years = `2.197 × x
3630 = 3000  1 + 
 100    100 + r 
3
  ⇒ 2.197 × x = x  

2  100 
 R 
(1.1)2 = 1 + 
100 
3
  100 + r 
⇒ (1.3)3 =  

R  100 
1.1 = 1 + 10 = R Choice (A)
100 100 + r
⇒ 1.3 =
100
15. Interest earned on a sum of ` P invested for T years at
⇒ 130 = 100 + r
R% p.a. under compound interest
⇒ r = 30% Ans: (30)
 R 
T 
= `P 1 +  − 1 20. If a sum takes T years to become N times under compound
 100  
  interest, interest being compounded annually at R% p.a.
T
 10 
2   R 
Interest earned = 5000 1 +  − 1 = `1050 ⇒ 1+  = N
 100    100 
 
⇒ R = (N1/T − 1)100
Ans: (1050)
 1 
 33.1  3 
16. Let the sum be `P and the rate of interest be R% p.a. ∴ Rate of interest =  1 +  − 1 100 = 10% p.a.
2   100  
 R   
P 1 +  = 6000 -- (1)
 100  Choice (C)
3
 R  20
P 1 +  = 7200 -- (2)
 100  21. Rate of interest =
2
% = 10% per half year.
Dividing (2) by (1),
 10 
2 
1 + R = 1.2 Interest = 200 1 +  − 1 = `42 Ans: (42)
100  100  
 
R = 20 Choice (B)

Triumphant Institute of Management Education Pvt. Ltd. (T.I.M.E.) HO: 95B, 2nd Floor, Siddamsetty Complex, Secunderabad – 500 003.
Tel : 040–27898194/95 Fax : 040–27847334 email : info@time4education.com website : www.time4education.com SM1001961/26
22. In all the cases, the rate of interest per year is the same. 3. Under Compound interest policy, amount
If this is the case, then the interest will increase with the n
 r 
number of times compounding is done per year. = P1 + 
∴ Choice (A) will result in the maximum interest.  100 
Choice (A) 5
 r 
Given, P1 +  = 199065.6 –––––– (1)
23. The effective rate of interest per annum is that rate of  100 
interest, which will fetch the same rate of interest if the
3
interest was calculated annually. Hence it will be the same  r 
P1 +  = 138240 –––––– (2)
for each year. Choice (A)
 100 
24. Rate of interest = 10% per quarter. A sum of `P under  r 
2
compound interest when invested at R% p.a. would ⇒ (1) ÷ (2) gives 1.44 = 1 + 
N  100 
 R  3
become `P 1 +  P = 138240/(1.2) = `80000 Choice (B)
 100 
A sum of `100 would become 4. Amount that `10000 becomes under compound interest
 10  10 20 30
100 1+  = 146.41 =1000(1+ )(1 + )(1 + ) = 17160
100 100 100
 100 
amount that `10000 becomes under simple interest
∴ This is the effective rate of interest is 46.41% p.a.
  20  
Choice (D) = (10000) 1+ 3   = 16000
  100  
25. Let the sum be `P. interest realized is `1160 less Choice (D)
Let the rate of interest be R% p.a.
 x +1   x  5. The compound interest for the (n + 1)th year is the same as
R  R 
Ι1 = P  1 +  − 1 –  1 +  − 1 the amount for one year on a principal equal to the nth year
 100    
 
100  interest.
 
x
∴1996.5 = 1885[(1 + (r/100)];
 R   R − 1 ⇒ r = 10% ⇒ r = 10% p.a. Choice (C)
= P 1 +  1 + 100 
 100   
6. When compounded annually, the interest accrued for the
x
PR  R  nth year is r% more than the interest accrued for the
= 1 + 
100  100  (n – 1)th year, where r is the annual rate of interest.
 r 
y ∴I5 = 1 +  I4
Similarly Ι2 =
PR 
1 +
R 
  100 
100  100 
 r  r
As Ι1 > Ι2, x > y ⇒ 1464.1= 1 +  (1331) ⇒ 1+ = 1.1⇒ r= 10%
100 
Choice (A)
 100
Exercise – 4(a) Interest accrued for the first two years when `12000 is
invested in the same scheme
Solutions for questions 1 to 25:  10  
= 1200 1 +  − 1 = 12000[(1.1)2 –1]
 100  
Pnr
1. Given P + = 18600
100 = 12000 × 0.21 = 2520 Choice (C)
 nr 
P 1 +  = 18600 n = 5, r = 11  3 × 20 
 100  7. In 3 years `10000 amounts to 100001 +  under
 100 
18600
⇒P= = 12000 simple interest and is `16000
5 × 11
1+ In the next 2 years, under compound interest `16000
100 2
 20 
amounts to 16000 1 +  = `23040 Choice (D)
Alternate method:
 100 
A = P (1 + nr%), ⇒ 18600 = P(1 + 55%) = 1.55 × P
⇒ 18600 × 100 = 155P, ⇒ P = 12000 Ans: (12000) 8. Let the amount that `8000 becomes after 4 years be x. Then
(x + 14440) (1.1)2 = 30,250
 nr  Or x + 14440 = 30250/1.21
2. For Simple Interest P 1 +  = 10080 x + 14440 = 25000; 10560 = x
 100  Now, let the rate of simple interest for the first 4 years be
n = 2, r = 10 r% Then 8000(1 + 4r/100) = 10,560; 320r = 2560, r = 8
P = 10080/1.2 = 8400 Ans: (8)
n = 2, r = 10, P = 8400
2 9. Let P1 be the sum invested in scheme A; and P2 be the
 10 
A = 84001 +  = 10164 sum invested in scheme B.
 100  P1 + P2 = 34000 –––––– (1)
∴ CI = 10164 − 8400 = `1764 P1 (1 + 20/100)2 = 9 x P2(1 + 60/100)2
P1 x (1.2)2 = 9 x P2 x (1.6)2
Alternate method: 2
 3 ×1.6 
P1 = P2 ×   ; P1 = P2 × 16
A = P(1 + nr%) ⇒ 10080 ⇒ P(1 + 20%);  1.2 
10080 P1 16
Under CI, A = P(1 + R%)n = (1.1)2 = 10,164 = –––––– (2)
1.2 P2 1
Hence, CI = 10,164 – P = 10,164 – 8400 = 1764. From (1) and (2), P2 = `2000 Ans: (2000)
Choice (B)
Triumphant Institute of Management Education Pvt. Ltd. (T.I.M.E.) HO: 95B, 2nd Floor, Siddamsetty Complex, Secunderabad – 500 003.
Tel : 040–27898194/95 Fax : 040–27847334 email : info@time4education.com website : www.time4education.com SM1001961/27
10. Let the first, second and third sums be `f, `s and `t The value that 4 crores would amount to in 24 years at the
respectively. 24
24  1
 5   5  
2 3
5   R   
f  1 +  = s1 +  = t 1 +  = A (say) same rate = 4c  1 +  = 4c  2 6  = 64c
 100   100   10   100   
 
A A A 20 400 8000
f :s:t = : : = : : Ans: (64)
21 441 9261 21 441 9261
20 400 8000 2
 r 
= 8820 : 8400 : 8000 = 441 : 420 : 400 Choice (D) 16. CI – SI = 30 = P   . Also, Pr/100 = 600
 100 
11. Suhaas borrows at 28% compound interest and pays back ∴ 600 × r/100 = 30
in 1 year (let him borrow P1) r = 5%
1 ⇒ P × 5/100 = 600
 28 
P11 +  = 38400 P = `12000 Choice (B)
 100 
P1 (1.28) = 38400 P1 = 30000 17. The difference between the compound and simple interest
Let Bhanu borrow P2 for 3 years is given by
Given P1 + P2 = 54000; P2 = 24000  r  3  r  
2

4×r P   + 3 



24000  1 +  = 38400  100   100  
 100  
4r/100 = 1.6 – 1= 0.6 ⇒ r = 15% Choice (C)  20  3  20  
2
∴P   + 3   = 3200
 100   100  
12. Puneet borrows `32000 at 10% simple interest 
32000 ×10 × 3 P × 16/125 = 3200
Simple interest for 3 years = = `9600
100 P = `25000
Compound interest for 3 years Alternate method:
= 32000 (1.2)3 – 32000 = `23296
Puneet would have gained (23296 – 9600) = `13696 Under CI, amount for 3 years = (1.2)3 P = 1.728P
32000 ×10 × 2 Under SI, amount for 3 years = (1 + 3 × .2)P = 1.6P
Simple interest for 2 years = = `6400 Difference in amounts = Difference in interests.
100
Hence, 1.728P – 1.6P = 3200; ⇒ 0.128P = 3200.
Compound interest for 2 years ⇒ P = (3200/0.128) = 25000. Choice (D)
= 32000(1.2)2 – 32000 = `14080
Puneet would have gained (14080 – 6400) = `7680 18. He borrows `2500000
Puneet would have gained an extra amount of At the end of the first year it becomes
13696 – 7680 = `6016 2500000 × (1 + 12/100)1 = `2800000
Alternate method: He repays `500000 –––––– (1)
`2300000 is the principal for the 2nd year.
Profit in the case of 3 years = P(1.2)3 – P(1 + 3 × 0.1) This becomes 230000 (1 + 12/100)1= `2576000
= 1.728P – 1.3P = 0.428P Which is repaid by the person, at the end of 2nd year.
Profit in the case of 2 years = P(1.2)² – P(1 + 2 × 0.1) In the first year, (on his earnings side), he earns 20% on
= 1.44P – 1.2P = 0.24P `2500000 = `500000.
Difference in the two profits = 0.428P – 0.24P Out of this, (1) is repaid. Investment remains as `250000.
= 0.188P = 0.188 × 32000 = 6016. Choice (C) This earns 20% during the second year. So, it becomes
`3000000. Out of this `2576000 is repaid. So, he makes,
13. If compounding is done annually r = 20%, n = 2, 3000000 – 2576000
Let P = 100, A = 100(1.2)2 = 144 = `424000 Choice (A)
If compounding is done half-yearly
r = 10%, n = 4, A = 146.41 Difference = 2.41 19. Loan amount = `24000
If P = 100 then, difference = 2.41 Simple Interest = 12% per annum
1084 .5 24000 × 12
If difference is 1084.5, P = ×100 = `45000 After 1 year, interest = = 2880
2.41 100
Ans: (45000) 19200 × 12
Interest for the second year = = 2304
100
14. Sum = P, C.I = r%, n = 10 years
(Since 20% of the principal has been repaid)
 nr 
  14400 × 12
Given, Pe  100  = a .P Interest for the third year = = 1728
100
10 x
r 9600 × 12
e 100 = a Interest for the fourth year = = 1152
100
Given e = a2 hence er/10 = e2 ⇒ r = 20 Total interest = 2880 + 2304 + 1728 + 1152 = 8064
Choice (B)
Alternate method:
15. Let the rate be R % p.a. Repayment at the end of every year = 20% of the principal
18
2.4c 1 + R 
= 20% of 24000 = `4800
= 19.2c
Hence, interest accrued every year decreases by 12% of
 100 
`4800.
18 decrease in interest per year = 12% of 4800 = `576
⇒  R 
1 +  =8 Interest for the first year = 12% of 24000 = `2880
 100  Hence, interest accrued in 4 years
1
= 2880 + (2880 – 576) + (2880 – 2 × 576) + (2880 – 3 × 576)
⇒ R = 4 × 2880 – 576 (1 + 2 + 3)
1+ = 26
100 = 11,520 – 3456 = 8064 Ans: (8064)

Triumphant Institute of Management Education Pvt. Ltd. (T.I.M.E.) HO: 95B, 2nd Floor, Siddamsetty Complex, Secunderabad – 500 003.
Tel : 040–27898194/95 Fax : 040–27847334 email : info@time4education.com website : www.time4education.com SM1001961/28
20. Let the annual instalment be `x. 4000 ×12 4000 ×12
Let (FV)i denote the value of the ith instalment at the end of = =
(1 − 0.71) 0.29
the fifth year.
Sum of future values of all the instalments = `1450 4000 ×12 4000 × 12
( x ) (8) ( 4 ) ≅ ≃ = 160000. As the actual
(FV)1 = x + 0.29 0.3
100 denominator is slightly less than 0.3 the actual answer
( x ) (8 ) (3 ) should be slightly more than `160000.
(FV)2 = x +
100 Actual answer is `165000 as per the options.
Choice (B)
( x ) (8 ) ( 2 )
(FV)3 = x +
100 25. The value of the first instalment (at the time the sum was
( x ) (8 ) (1) 1200
(FV)4 = x + borrowed) = = 1000.
100 1. 2
(FV)5 = x
The value of the second instalment (at the time the sum
x
∴, 5x + (8) (4 + 3 + 2 + 1) = 1450 1152
100 was borrowed) = = 800.
⇒ 5x + 0.8x = 1450
(1.2)2
1450 The value of the third instalment (at the time the sum was
⇒ x= = 250
5. 8 2592
borrowed) = = 1500.
(1.2)3
Alternate method:
1 Sum borrowed = Total value (at the time the sum was
Each instalment must be less than `( th of 1450) borrowed) = `3300 Ans: (3300)
5
= `(290). From among the choices given, only Choice (C) Exercise – 4(b)
satisfies the condition. Choice (C)
Solutions for questions 1 to 40:
21. Present worth (in `) of `20000 due after 3 years at CI of
10% p.a. 1. Let the principal be P.
20, 000 Let ‘I’ be the interest for one year
= ≃20,000 × (0.9)3 = 14580 Choice (C)
3 P + 3I = 2832 –––––– (1)
 10 
 1+  P + 5I = 3120 –––––– (2)
 100  Subtracting (1) from (2), I = 144 and P = `2400
Choice (A)
22. Principal = `100000
At the end of first year it amounts to 2. Let the sum be `P
100000(1 + 8/100)1 = 108000
Out of this he repays `10000  10 
Extra interest = P   (2) = 4000
So, amount due at the end of 1st year  100 
= 108000 – 10000 = `98000.
P = 20000 Ans: (20000)
At the end of the second year this amounts to
98000 [1 + 8/100]1 = 105840.
3. Let the rate of interest be R% p.a.
Out of this he repays `10000
∴At the beginning of the third year amount due from him  R 
= 105840 – 10000 = `95840 Choice (D) Extra interest = (10000)   (2) = 4000
 100 
23. Let the sum be `P. R = 20 Choice (C)
2
CI (2nd year) = P1 + 5  − P1 + 5  ( Q A2 – A1) 4. Let the sum be `P.
 100   100  Let the rate of interest be R% p.a.
3 2  R 
CI (3rd year) = P1 + 5  − P1 + 5  (P)   (10) = 2P – P = P
 100   100   100 
R = 10
Difference = P(1.05)3 – 2(1.05)2 – (P(1.05) = 42 Let us say it will take T years for it to become 5 times itself.
3 2
 21   21   21 
P  − 2 20  − P 20  = 42 ⇒ P = 16000  R 
(P) 
 20       (T) = 4P T = 40 Choice (B)
 100 
Ans: (16000)

24. Cost of the car = `525000 5. Let the sum be `P. let the rate of interest be R% p.a
Cash payment = `125000  R 
∴Principal of loan to be discharged in instalments (P)   (4) = P
= `400000  100 

Pr  R 
Each instalment X = (P)   (5) = 12500
  100 n   100 
100 1 −   
  100 + r   12500
  P= = 10000
25
×5
400000 × 12 100
=
  100  3   R 
Required interest = P   (3) = `7500 Choice (B)
100 1 −   
  112    100 
 
Triumphant Institute of Management Education Pvt. Ltd. (T.I.M.E.) HO: 95B, 2nd Floor, Siddamsetty Complex, Secunderabad – 500 003.
Tel : 040–27898194/95 Fax : 040–27847334 email : info@time4education.com website : www.time4education.com SM1001961/29
Solutions for questions 6 and 7: R 3 R 2
P[(1+ ) – (1+ ) ] = 17280 → (2)
100 100
Let the sum that Rohan lent to P be `x
Sum he lent to Q = `(100000 – x). R 2 R 
P(1 + )
 10
2  x+
20 
(100000 − x ) = 32000 100  100 
Dividing (2) by (1), = 1.2 ⇒ R = 20
 100 100  R  R 
P(1 + )
x = 40000 100  100 
Let the effective rate at which Rohan lent the entire sum be R% p.a. Choice (C)
 R 
(100000)   (2) = 32000
 100  14. Let the rate of interest be r%
then, r% of interest of nth year
R = 16
= Difference of interest for nth and (n + 1)th year.
⇒ r% of 1256 = (1413 − 1256)
6. `40000 Ans: (40000)
r
7. 16% Choice (C) ⇒ × 1256 = 157 ⇒ r = 12.5% Choice (D)
100
8. Let us say he lends `P
15. Let the present value be `x.
Let us say the least number of years it takes the sum to
fetch an interest at least equal to itself be N. The rate of 87880
interest for the ith year = 2i % p.a. x= = 40000 Ans: (40000)
30 3
(1 + )
 2   4   6   2N  100
P   +   +   + ....  ≥ P
 100   100   100   100 
n
 8 
N(N + 1) ≥ 100 16. Let the time period be n years 1875 1 +  = 2187
If N = 9, N (N + 1) < 100  100 
If N = 10, N (N + 1) > 100
n n 2
N = 10 Choice (B)  108  2187  27  729  27 
 100  = 1875  25  = 625 =  25  ⇒ n = 2
9. Let P1 = `x      
P2 = `4x. Choice (B)
P3 = `5x
P4 = `2x 17. Interest for the third year
 10  2 2
Simple interests on P1, P2, P3 and P4 are (x)   (1)  r   r   21   5 
= P 1 +    = 12000     = `661.5
 100 
 100   100   20   100 
 20  2 6
(4x)  (5),(5 x )( )( 4 ) and (2x) ( )(10)i.e
 100  100 100 Alternate method:
`01x, `4x, `0.4x and `1.2x respectively Interest for the first year = 5/100 x 12000 = `600
4x – 0.1x = 7800 x = 2000 Interest for the second year = 600 + 5/100 x 600 = 630
The total simple interest = 5.7x = `11400 Ans: (11400) Interest for the third year = 630 + 5/100 x 630
= `661.5 Choice (B)
10. Let the sum that Ajay borrowed from Balu be `X. The sum
he borrowed from Chetan = `2x 18. Let the sum be `P and the total interest for the second and
Let the sum he added b `y third years be `X.
18  10   20 
He earned (3x + y) − x  − 2x   20 
100  100   100  Interest for the first year = P   = 0.2 P
0.04x + 0.18y = 4400  100 
3x = 60000 ⇒ x = 20000 ⇒ y = 20000 Choice (D) Interest for the second year = (Amount at the end of
two years) - (Amount at the end of one year)
11. Let the present value be `x = P [(1.2)2 – 1.2] = 1.2 P (0.2)
17280 Interest for the third year = P[(1.2)3 – (1.2)2] = 1.2 P (0.24)
x= = 10800 Ans: (10800)
 20  0.2 P = 4000 ⇒ P = 20000
1 + 3 
 X = 0.528 P ⇒ `10560 Ans: (10560)
 100 
19. Let the sum be P and the rate be r%
12. Let the sum be `P.
Let the rate of interest be R% p.a.  r 
2
P 1 +  = 1200 –––––– (1)
P (1 +
R 3
) = 266200 → (1)  100 
100
 2r  2
P1 +  = 1323 –––––– (2)
100 
R 4
P (1 + ) = 292820 → (2) 
100
R 2
 2r 
Dividing (2) by (1), (1 + ) = 1.1  1 + 
100  100  1323
⇒ R = 10 Choice (B) Dividing (2) by (1), =
2 1200
 r 
 1 + 
13. Let the sum be ` P.  100 
Let the rate of interest be R% p.a.
2
R 2 R  100 + 2r  441 5
P [(1+ ) – (1 + )] = 14400 → (1)   = ⇒ r=5 % Choice (C)
100 100  100 + r  400 19

Triumphant Institute of Management Education Pvt. Ltd. (T.I.M.E.) HO: 95B, 2nd Floor, Siddamsetty Complex, Secunderabad – 500 003.
Tel : 040–27898194/95 Fax : 040–27847334 email : info@time4education.com website : www.time4education.com SM1001961/30
20. Sum = P; C.I = r%; In 3 years, amount = 27P 28. Let the values of each instalment that he should have paid
3 3 and that he actually paid be `x and `y respectively.
 r   r 
P 1 +  = 27P ⇒ 1 +  = 27
 100   100   20 
320320  
r  100  = 209664
⇒ 1+ = 3 r = 200. x=
2
100  100 
1 −  

Compounding half yearly, the amount will be  120 
6
 100 
P1 +  = P x 26 = 64P  20 
 100  320320  
y=  100  = 152064
Additional Amount received = 64P – 27P = 37P 3
Choice (B)  100 
1 −  
 120 
21. Excess interest = 10000 x – y = 57600 Choice (D)

 20 
3   10   20   30   29. Let the sum Ashok borrowed from the bank lending at
1 +  − 1 − 1 +  1 +  1 +  − 1
 100    100   100   100  
10% p.a. be `x
  Sum he borrowed from the other bank = `(84000 – x)
= `120. Ans: (120) 10 20
x+ (84000 –x) = 13200
100 100
22. Let the sum be `P x = 36000
20 2 20 difference = 84000 – 2x = 12000 Ans: (12000)
P(1 + ) – P (1 + ) = 400
2(100) 100
P = 40000 Choice (B) 30. If a sum is doubled in the same time at simple interest as
well as compound interest, the rate of interest under simple
23. As the rate of interest is increasing by 20 percentage points interest is more than rate under compound interest.
annually, the rate of interest per half year increases by Choice (A)
10 percentage points.
31. Let the sum that Anil gave Bala be P.

Half Interest for the


Interest of 75 3
Principal the end of Amount Bala ℓent P i.e. P at 10% p.a CI for 3 years
year half year 100 4
half year
3
1 40000 2000 2000 42000 3  10 
P1 +  = 3993 ⇒ P = 4000 Ans: (4000)
2 42000 6300 8300 48300 4  100 
3 48300 12075 20375 60375
P x 15 2
32. PR2/10000 = 112.5; = 112.5
∴ The required interest will be `20375. Choice (C) 10000
P = `5000 Choice (B)
24. Let the sum be `P.
 20  33. Difference between the simple interest and the compound
Pe(5)   = 60000 interest, interest being compounded annually, on a sum of
 100  `P at R% p.a for 2 years
P = 22075 ≈ 22000 (to the nearest thousand) Ans: (22000)
2
n  R 
 r  = P   . As P = 8000,
25. Given P e (nr / 100 ) − P1 +  = 952000
 100   100 
Given r = 100 e = 2.71 n = 3 2
 R 
952000 and P   = 320, it follows that R = 20 Ans: (20)
⇒P= = 80,000 Choice (C)  100 
(2.71)3 − 23
34. Let the sum be `P.
26. `20000 deposited at the beginning of the first year Let the rate of interest be R% p.a
becomes, in 2 years, an amount equal to
20000 x (1.05)2 = `22050  R 
(i.e., at the beginning of the third year) 0.4P = (P)   (2)
 100 
`20000 deposited at the beginning of the second year
becomes in 1 year (20000) × (1.05) = `21000 20 = R
∴Amount at the beginning of the third year x 20 2
= 22050 + 21000 = `43050 (1 + )P = P(1 + ) ⇒ x = 44 Choice (D)
100 100
Now `10000 is withdrawn
∴There is `33050 in the account on which 5% interest for
35. In case of simple interest SI, P = I, given data.
one more year is earned.
Final amount = 33050 x 1.05 = `34702.5 Choice (D) 5 Pr
⇒P= or r = 20%
100
27. Value of `72000 at the end of the first year = `86400. Since
`x was repaid at the end of the first year, in order to clear Now, if the sum is lent at CI at 20% p.a, let it double in n years
the loan at the end of the second year, the amount to be
n n
repaid must be  20  6
2P = P 1+  2 =  
20  100  5
(86400 – x) + (86400 – x)
100 n lies between 3 and 4. After four complete years, the sum
1.2 (86400 – x) = 57600 becomes for the first time, more than twice itself.
x = 38400 Ans: (38400) ⇒ n = 4; Ans: (4)

Triumphant Institute of Management Education Pvt. Ltd. (T.I.M.E.) HO: 95B, 2nd Floor, Siddamsetty Complex, Secunderabad – 500 003.
Tel : 040–27898194/95 Fax : 040–27847334 email : info@time4education.com website : www.time4education.com SM1001961/31
36. Let the sum be `100. 43. Statement Ι is not sufficient
Hence the interest is `100 From statement ΙΙ,
100 × 25 × R Let the sums be a, b and c, then
∴ 100 =
3 ×100 a(100 + 2 × 5 ) b(100 + 3 × 5 ) c (100 + 4 × 5 )
= = =k
⇒ R = 12% p.a 100 100 100
∴ New rate of interest is 24% p.a. 11a 23b 6c
= = =k
Let after n years, the sum becomes twice of itself. 10 20 5
n a : b : c = 276 : 264 : 253 Choice (A)
 24 
∴ 200 = 100  1 +  ⇒ 2 = (1.24)
n
 100  44. From statement Ι,
Instead of 1.24, if 1.25 is taken up for n = 2,
R.H.S = 1.5625 3
 R 
For n = 3, R.H.S = (1.5625) (1.25) i.e., 1.95 125 = 100 1 +  so we can find the rate of interest (R).
Even for 1.25, it doubles in fourth year, hence for 1.24 also,  100 
it doubles in fourth year.
∴ In 4th year, it doubles. Choice (A) From statement ΙΙ,

3 3
37. Let the sum be `2P.  R   R 
1.5x = x 1 +  ⇒ 1.5 = 1 + 
`P lent at simple interest fetches `0.2 P as interest each
 100   100 
year. `P lent at compound interest fetches `0.24 P as
interest in the second year and `0.3456 P as interest in the So we can find the rate of interest.
fourth year
0.24 P - 0.2P = 400 So each statement alone is sufficient. Choice (B)
P = 10000
Required difference = 0.3456P – 0.2P = `1456 45. Either of the statements alone is not sufficient as the rate of
Choice (C) interest and the simple interest earned in 5 years is given in
different statements.
38. The smaller and the larger parts are Ra.1000 and `3000 Combining statements Ι and ΙΙ,
`1000, in 2 years, under simple interest amounts to (1000) p(5 )(6)
10 (2) = 600 so we can get the value of p.
[1 + ] = `1200 100
100 Compound Interest = {p [1 + 6 /100]5 – p} Choice (C)
`3000 in 2 years, under simple interest amounts to (3000)
30 (2) Chapter – 5
[1 + ] = `4800
100 (Time and Distance)
`4000, in 2 years under simple interest amounts to (4000)
Concept Review Questions
20
(1 + (2) = `5600
100 Solutions for questions 1 to 50:
The interest realized is `400 less. Choice (C)
1. (a) 36 km/hr × 5/18 = 10 m/s Choice (A)
39. The amount that `1000 becomes under compound interest
2 (b) 12.6 × 5/18 = 3.5 m/s Choice (A)
 10 
= (1000) 1 +  = `1210 (c) 216/35 × 5/18 = 2 m/s Choice (C)
 100 
The amount that `3000 becomes under compound interest 2. s = 2.5 m/s × 18/5 = 9 km/hr
2 t = 4 hours
 30 
= (3000) (1 + 1 +  = `5070 d = 9 × 4 = 36 km. Ans: (36)
 100 
The amount that `4000 becomes under compound interest 3. 6 m/s × 18/5 = 108/5 km/hr
2 d = s × t ⇒ 108/ 5 × 15/4 = 81 km Choice (A)
 20 
= (4000) (1 + 1 +  = `5760
 100  1
4. Time ∝
The interest realized is `520 less Choice (D) speed
1 1 1
40. Let the value of the machine two years ago `x ∴ Ratio of the times = : : =4:3:2
2 3 4 6
 90 
then, x   = 24300 Choice (A)
 100 
⇒ x = `30,000 Ans: (30000) 5. Total distance = (Average speed) x (time)
2 × 35 × 45
Solutions for questions 41 to 45: = × 16 = 630 km. Ans: (630)
80

41. Statements Ι and ΙΙ, give the same information. 6. As speed and time are inversely proportional, if speed
∴ The question cannot be answered as the principal is not decreases to 5/6th, time taken will be increasing to 6/5th of
given. Choice (D) original time.
6t
p(8)R ∴ – t = 10 ⇒ t = 50 minutes. Choice (A)
42. From statement Ι, = 36,000 5
100
So, statement ΙΙ alone is not sufficient. 7. As the speed is increased to 1.25 times the original, time
p(8)R taken will be decreased by 4/5 times of actual.
From statement ΙΙ, = p.
100 4t
∴t– = 6 ⇒ t = 30 minutes. Ans: (30)
∴ R = 12.5%. 5
Statement ΙΙ alone is sufficient. Choice (A)
Triumphant Institute of Management Education Pvt. Ltd. (T.I.M.E.) HO: 95B, 2nd Floor, Siddamsetty Complex, Secunderabad – 500 003.
Tel : 040–27898194/95 Fax : 040–27847334 email : info@time4education.com website : www.time4education.com SM1001961/32
270 19. Let the speeds of Ram and Shyam be 5x kmph and 4x kmph
8. Time taken by them to meet = respectively. Suppose Ram overtakes Shyam after t hours.
Re lative speed
12 12
270 t= =
= = 3 hours 5x − 4x x
50 + 40
∴ Ram will overtake Shyam after traveling 5x t = 60 km.
∴ The meeting time is 12 : 00 p.m Choice (B) Ans: (60)
9. Distance that X would have traveled by 10 : 00 a.m.= 30 k.m. 20. Let PQ = d km.
Distance between the cars at 10 : 00 a.m. = 150 k.m. d
Speed of A = km/min
 150  20
Meeting time =   hours after 10 : 00 a.m. i.e.

 30 + 20  d
Speed of B = km/min
1 : 00 p.m. Choice (C) 30
d
10. Let the speeds of P and Q is 3x kmph and 4x kmph Time they would take to meet each other =
d d
respectively. Distance that P would have traveled by +
10 : 00 a.m. = 9x km. They would meet after another 20 30
9x = 12min
= 9 hours i.e. at 7 : 00 p.m. Choice (A) Ans: (12)
4x − 3x
21. The train has to travel a distance of 800 m to cross the
Total dis tan ce electric pole. Hence, the time it takes to cross the pole
11. Average speed =
Total time 800
= = 72 seconds. Choice (D)
XY = (4) (90) or 360 km. Ans: (360) 5
40 ×
8
( 2.5 ) ( 40 ) + ( 2.5)(60 ) 40 + 60
12. Average speed = = length of the train
5 2 500
22. Speed of the train = =
= 50 kmph. Choice (B) time taken to cross the pole 25
= 20 m/sec = 72 kmph Choice (B)
Note: Whenever a person covers a journey in 2 parts such
that he covers each part for the same time, his average 23. The distance to be travelled by the train to cross a platform
speed for the journey will be the average of the speeds at of length 750 m is 750 + 650 i.e., 1400 m. At 72 km/hr. or
which he travelled each part. 1400
20 m/s. the train takes i.e. 70 seconds to cross the
20
(3) ( 40 ) + (5) ( 60 )
13. Average speed = = 52.5 kmph. platform. Choice (C)
8
Choice (A) 400 + 300
24. Time taken = = 70 seconds Ans: (70)
14. Let x km be the distance between Hyderabad and Tirupathi. 5
(36 )  
x+x 8
Average speed =
x x
+ 25. Lt = 300 m; Lb = 120 m
60 90
t = 3m = 180 seconds.
2 ( 60 ) (90) s = 1500/180 = 25/3 m–1
= = 72 kmph Ans: (72)
60 + 90 s = 25/3 × 18/5 m–1 = 30 km/hr Ans: (30)
4
15. His average speed = th of the usual average speed. 26. Given speed of the train is 72 kmph
5
5
Choice (A) F = 72 × = 20 m/s.
18
5 The time taken to cross the person is 12 sec.
16. Average speed of Alok = (usual average speed of Alok) ∴ The length of train = speed time
4
L = (12) (20) = 240 m. Choice (B)
1
∴ His time taken ∝
Average speed 27. l1 = 250m
4 s1 = 90km/hr = 25ms–1
∴ His time taken would be th of the usual time. l2 = 200m, S2 = 36km/hr = 10ms–1
5 But Sp = 25 – 10 = 15 ms–1
Choice (B) D = l1 + l2 = 450m
T = 450/15 = 30 seconds Choice (C)
17. d1 = x km.
S1 = 60km/hr 28. Relative to the boy the speed of the train is 99 + 9 i.e., 108 km/hr.
T1= d/s = x/60 750
D2 = 3x, S2 = 90km/hr Hence, the time it takes to cross him = = 25.
5
T2 = x/30 = x/60 108 ×
Total time = x/30 + x/60 = x/20 8
Total distance = 3 x + 1x = 4x Ans: (25)
∴ Average speed = 4x/ x/20 = 80 km/hr Choice (D)
29. Required distance = Sum of the lengths of the trains
18. Let the time for which the car has to travel to reach Y at the = 1000 m. Choice (B)
scheduled time be t hours.
XY = 60 (t + 2) = 80 (t – 2) ⇒ t = 14 1000 4
30. Ratio of the speeds of A and B = =
60 (14 + 2) 480 1000 − 250 3
Required average speed = = kmph.
14 7 Choice (B)
Choice (D)

Triumphant Institute of Management Education Pvt. Ltd. (T.I.M.E.) HO: 95B, 2nd Floor, Siddamsetty Complex, Secunderabad – 500 003.
Tel : 040–27898194/95 Fax : 040–27847334 email : info@time4education.com website : www.time4education.com SM1001961/33
31. When Eswar finishes the race, Girish would have run 600 600
200 – (10 + 10) = 180 m. (ii) Time = = = 30 seconds
Re lative speed 20
200 10
∴ Ratio of Eswar’s and Girish’s speeds = = Choice (B)
180 9
Choice (C) 600 600
42. Time = LCM ( , ) = LCM (40, 60) = 120 seconds.
15 10
1000 5
32. Ratio of the speeds of X and Y = = Ans: (120)
1000 − 200 4
4 43. Number of rounds completed by Ram in one hour
∴ Y’s speed = (10) = 8 m/sec. Choice (D) (2) (3600 ) = 120
5 =
60
33. Ganesh beats Harish by 10m or 2 seconds. Number of rounds completed by Shyam in one hour
∴ Harish must have taken 2 seconds to run the final 10 m.
=
(4) (3600 ) = 240
10 60
∴ Harish’s speed = = 5 m/sec Choice (A)
2 ∴ Shyam would have completed 120 more rounds than
Ram. Choice (C)
Ganesh' s speed 100
34. = 44. The relative speed does not change when the two
Harish' s speed 90
exchange their speeds. ∴ Both will reach the starting point
10 5 simultaneously. Choice (C)
Ganesh’s speed = (5) = 5 m / s Choice (C)
9 9
45. Angle covered by the hour hand from 12 : 00 p.m. to 2 : 30 p.m.
35. Akbar gives Birbal a start of 2 s and Birbal covers 10 m in = (Angle covered by from 12 : 00 p.m. to 2 : 00 pm) +
these 2 s. (Angle covered by it from 2 : 00 p.m. to 2 : 30 p.m.)
∴ Birbal’s speed is5 m/sec. = 2 (30°) + 30 (1/2) ° = 75°.
∴ He takes 18 seconds, more to cover the 90 m. Angle covered by the minute hand from 12 : 00 p.m. to 2 : 30 p.m.
∴ Akbar covered 100 m in 18 seconds. = (Angle covered by it from 12 : 00 p.m. to 2 : 00 p.m.) +
100 5 (Angle covered by it from 2 : 00 p.m. to 2 : 30 p.m.) = 2 (0°)
His speed = m/sec= 5 m/sec Choice (B) + 30 (6°) = 180°
18 9 ∴ The angle between the hands would be 105° at 2 : 3 0 p.m.
Ans: (105)
36. Let the time taken by R to run the race is t seconds.
Time taken by Q to run the race = (t + 30) seconds. 46. The hands of a clock will be at an angle of θ where θ is any
Time taken by P to run the race = (t + 50) seconds. angle satisfying 0° < θ° < 180° for a total of 44 times in a
∴ P beats R by 50 seconds. Ans: (50) day. Ans: (44)
37. Let the speeds of A, B and C be a m/sec, b m/sec and 47. The hands of a clock will be at an angle of 0° for 22 times in
c m/sec respectively. a day and at an angle of 180° for 22 times in a day .
a 100 10 Choice (C)
= =
b 100 − 10 9
48. Every 12 hours the hands of a clock coincide 11 times.
b 100 5
= = 12
c 100 − 20 4 ∴ The hands of a clock coincide once every hours or
11
a  a   b  25
=  = 5
c  b   c  18 65
11
minutes. Choice (C)

25 − 18 12
∴ A beats C by (100 ) = 28 m. Choice (B) 49. The hands of a clock coincide every hours i.e.
25 11
5
38. Speed of Anand’s boat downstream = 8 kmph. 65 min. We know that hands of a clock coincide at
11
40
Time taken = = 5 hours Ans: (5) 5
8 12 : 00. So, they also coincide at 12 : 00 + 1 : 05
11
39. Let AB = d km. 5
= 1 : 05 . Choice (A)
Let the speed of his boat in still water be x kmph. Let the 11
speed of the river be y kmph.
d 50. Since the hands of a clock take more time to coincide than
x+y= --(1)
6 5
the normal 65 minutes the clock is losing time.
d 11
x–y= --(2)
8 Choice (A)
7d d
Solving (1) and (2), x = and y = Exercise – 5(a)
24 24
∴x:y=7:1 Choice (A) Solutions for questions 1 to 35:

600 600 600 1. Let the usual time taken by the man to reach his office be t.
40. Time = LCM ( , , ) = LCM (60, 40, 30) The speed is 3/4ths the normal speed.
10 15 20
Hence, time is 4/3rd. (4/3t) – t = 1/3 t = 20
= 120 seconds Choice (A)
t = 60 minutes
Had the person walked at 4/3 of his usual speed, time
600 600 taken by him = 3t/4 = 3/4 (60) = 45 minutes
41. (i) Time = = = 60 seconds.
Re lative speed 10 Ans: (45)
Choice (A) 2. Let the usual time taken by the boy be t.
Triumphant Institute of Management Education Pvt. Ltd. (T.I.M.E.) HO: 95B, 2nd Floor, Siddamsetty Complex, Secunderabad – 500 003.
Tel : 040–27898194/95 Fax : 040–27847334 email : info@time4education.com website : www.time4education.com SM1001961/34
Distance between his home and school 400
= 5(t – 4/60) = = 36 sec
= 4(t + 2/60) → (1) (60 − 20 )  5 

Solving (1) we have t = 28/60 hr.  18 
Time taken by the boy to go to school if he walks at Alternate method:
6 km/hr = 5(28/60 – 4/60)/6 = 1/3 hr = 20 min.
Choice (B) The distance covered in both the cases is equal to the
length of the train.
3. Hence, ratio of relative speeds = inverse ratio of the times
S T taken.
⇒ (60 + 12) /(60 – 20) = (x)/12, where x is the required
time.
7m From the above, x = 36 sec. Ans: (36)
8. Time taken by Train U to exit the tunnel
When M meets ‘N’ for the 2nd time, sum of the total Length of the tunnel + length of the train U
=
distances travelled by M and N = 7 + (7 + 7) = 21 m. Speed of B
(M covered from S to T, 7 m;
200 + 150
N covered from T to S, 7 m; =
and they together covered ST 7 m.) 25
The time of travel from the start to the time of second Train U exits the tunnel after 350/25 = 14 sec.
meeting is the same for both. Hence, distance = time x In 14 sec, front end of train T’s engine travels
(sum of the speeds). = 14 x 15 = 210 mts
21 That means the front end of engine is 10 mts out of the
Time taken = = 3 sec. tunnel.
(3 + 4) ∴ The length of the train T still in the tunnel
Total distance travelled by ‘M’ alone = 3 × 3 = 9 m. = 100 – 10 = 90 m Choice (C)
Choice (C)
9. Let the point at which the ends cross each other be x mts
4. Let the person cover x m in the 1st second. In the from the point of entry of the slower train.
2nd second, he covers x/4 m. Hence in 2 seconds, he ∴ The time elapsed before the ends meet for the two trains
covers x + x/4 = 5x/4 m 100 + x 200 − x + 150
In the 3rd second, he covers 1/4(5x/4) m will be equal to =
Hence in 3 seconds, he covers 5x/4 + 5x/16 15 25
= 25x/16 = (5/4)2x m ⇒ 500 + 5x = 1050 – 3x
Thus after 5 seconds, the person would have covered 8x = 550
(5/4)5 – 1x = 625x/256 m x = 68.75 mts
625x/256 = 15.625
x = 15.625/625 x 256 = 6.4 m Ans: (6.4) Alternate method:

5. Distance travelled by the stone in the last two seconds Distance between the rear ends of the two trains, at the
before it reaches the ground be x. instant of entry = 100 + 200 + 250 = 450 metres.
5t2 = 180 Time taken for the two ends to meet
t2 = 36 ⇒ t = +6 = (distance)/(relative speed)
x = 5(6)2 – 5(6 – 2)2 = 5(6)2 – 5(4)2 = 450/(15 + 25) = 11.25 seconds.
= 5(62 – 42) = 5(20) = 100 m Choice (C) Distance covered by the slower train
= 11.25 x 15 = 168.75 metres.
6. Let the length of the train be ‘l’ m, and the speed of the train The required distance = 168.75 – (length of slower train)
be ‘v’ m/sec = 168.75 – 100 = 68.75 metres. Choice (D)
430 + l 550 + l
∴ = = v ⇒ ℓ= 170 10. Distance covered by P, t hours after starting from X = at.
30 36 When Q overtakes P, he would have covered
430 + 170 (a + b) (t – p) = at – ap + bt – bp; and this equals at
= 20 mps
p(a + b)
and v =
30 ⇒t= ------- (1)
18 b
v = 20 × = 72 kmph Let R start q hours after Q started.
5
Distance covered by R when he overtakes P would be
Alternate method: (a + 2b) (t – p – q) = at → (2)
Substituting the values of t from (1) and simplifying we get
Difference of the lengths of bridges = 550 − 430 q=
pa
Choice (D)
= 120 m a + 2b
Difference of the time taken to cross the bridge
= 36 − 30 = 6 seconds 11.
Speed of the train = 120/6 = 20 mps
→SM ←SN
= (20 x 18)/5 = 72 kmph Choice (B)
K
7. Let the length of the train travelling at 60 kmph be L. 50 km

⇒ L In the above diagram, we have SM and SN as the speed of


= 20 ⇒ L = 400 m M and N and K is the meeting pointing of M and N.
(60 + 12 )  5  As distance = (Relative speed) (time),
 18  (SM + SN)5 = 50.
SP + SQ = 10 -------- (1)
Time taken by the train to cross another cyclist travelling in After M and N meet, they move towards their destinations
the same direction as the train, at 20 kmph which they reach at the same time; i.e., the times of travel
are equal.

Triumphant Institute of Management Education Pvt. Ltd. (T.I.M.E.) HO: 95B, 2nd Floor, Siddamsetty Complex, Secunderabad – 500 003.
Tel : 040–27898194/95 Fax : 040–27847334 email : info@time4education.com website : www.time4education.com SM1001961/35
5SM 5SN Time taken by P and Q to reach their meeting point are
Hence = 4 hours and 2 hours respectively. Let M be the meeting
SN + 1 SM − 1
point.
SM2 – SM = SN2 + SN --------- (2) XM = 4P and YM = 2q
Substituting (1) in (2) and solving we have Let the time taken by Q to cover MX be t hours
SM = 5.5 kmph
1
∴P takes t + hours to cover MY.
Alternate method: 3
1
If 't' is the time taken to meet, and t1, t2 are the durations t+
4 3
taken after the meeting to reach the respective destinations We have =
t 2
then, t = t1 t 2 . But it is given t1 = t2.
 1
⇒ 8 = t  t +  ⇒ (3t - 8) (t + 3) = 0
Hence t = t 12 or t = t1 = t2 -------- (1)  3

t = (Q t > 0 ). ∴Q reached his destination


But it is given t = 5 8
∴ t = t1 = t2 = 5 hours. 3
M travelled for 5 hours at x kmph and for another 5 hours at
 8 
(x − 1) kmph. at  2p.m + hours i.e. at  4 : 40 p.m Choice (B)
∴ 5x + 5(x − 1) = 50 ⇒ x = 5.5 kmph Ans: (5.5)  3 

12. Distance ran by the thief in 10 minutes = 1km Solutions for question 16:
Distance moved by the jeep in 10 minutes = 1⋅5km
Distance between the jeep and the thief when the jeep Let us say Anil turned back after t hr. He would have covered
found a gap in the median = 2.5 km 50t km then. Chetan would have covered 10t km. ∴ The two
Additional time in which the jeep would overtake the thief would be 40t km apart. They would meet in
2⋅5 5 40 t 2t
= = hours = 50 minutes more hr .i.e., hr. In this time, Chetan would have
9−6 6 10 + 50 3
∴ Total time = 60 minutes. Choice (D) 20 t 100 t
covered km while Anil would have covered km. Also
3 3
13. Let the speeds of Anand and Bala be x kmph and y kmph
2t
respectively Bala would have covered km.
Distance traveled by them when they met are 6x km and 3
6y km respectively. When Anil picks up Chetan he would be 40t km behind Bala.
6 y 6y Also the ratio of the speeds of Anil and Bala is 5 : 1. Anil
− =5 (with Chetan) would catch up with Bala, when he covers 5x and
x y Bala covers x
y x 5 3 2 ∴ 5x – x = 40t.
− = = − The bike and Bala would have covered distances of 50t km and
x y 6 2 3
10t km respectively
y 3
Comparing both sides, = 20 200 t
x 2 XY = 10t + t + 50 t = km = 40km
3 3
6y 6x 3 2
Total time = + = 6  +  = 13 hours . 3
x y 2 3 ⇒t=
5
Choice (C)

14. Let XY be d km 3
16. (i) Time after which Anil turned back = hours
Let the points of their first and second meetings be denoted 5
by A and B respectively. = 36 minutes Choice (C)
XA : AY = 30 : d − 30
Ratio of the distances covered by man starting from X and
40
man starting from X when they meet for the second time (ii) Bala’s average speed = = 25km / hr
2
= XY + YB : YX + XB t+ t+t
= d + 20 : d + d − 20 3
d + 20 30 Choice (D)
=
2d − 20 d − 30 Note: The distance covered by Chetan until he was picked
d = 70. Ans: (70) up by Anil is the same as that covered by Bala from the
point he was dropped.
15.
M
X Y 17. Since Peter and Paul start at A and have the ratio of their
P speeds as 1 : 2, first time when they meet say at S, the
10 • distances covered by them respectively were in the ratio
P 1 : 2 i.e., Peter would have covered 200 m and Paul would
have covered 400 m
12 • •
P,M A
14 • • Peter Paul
Q
14+t •
1 Q P
14+t+
3 •
S
Let the speeds of P and Q be p kmph and q knph
respectively.
Triumphant Institute of Management Education Pvt. Ltd. (T.I.M.E.) HO: 95B, 2nd Floor, Siddamsetty Complex, Secunderabad – 500 003.
Tel : 040–27898194/95 Fax : 040–27847334 email : info@time4education.com website : www.time4education.com SM1001961/36
At the point S, the ratio of speeds will now become 2 : 1 21.
So, Peter will cover 400 m and reach A and Paul will cover 12
11
200 m and reach A. So, the 2nd meeting point will be at A, 1
the 3rd meeting point will be at S and the 4th meeting point 10 2
will be at A and so on.
So, by the time they meet for the 8th time i.e., at A, Peter S
9 C• 3
would have covered 4 rounds.
Now, both are at A.
When Peter covers 1/4th round, (i.e., 150 m), 8 4
Paul will cover 1/2 round, (i.e., 300 m)
Distance between them is either or 150 + 300 7 5
= 450 m or 600 – (150 + 300) = 150 m. Ans: (150) 6 E
The angle of rotation of minute hand
18. Let the speeds of the faster and the slower runners be
= 6 degrees / minute.
x kmph and y kmph respectively. Time to meet for the first
The angle of rotation of hour hand
time is:
= (1/2) degree/minute.
3 If the duration of the test is 't' minutes, then the sum of the
hrs, and this is given as 1 hour. angles of rotation of the two hands is 6t + t/2
x−y
= 13t/2 degrees ----------- (1)
Hence [3/(x – y)] = 1 ----------- (1) The test started between 2 and 3 o’ clock, when the
Time taken, by the faster and the slower runners hour hand was pointing at S (in the diagram) and the
3 3 minute hand was pointing at E (in the diagram).
respectively, for one lap are and hrs. As the
x y The test ended between 5 and 6 o’ clock, and the position
difference of the above times is given as 2 minutes, of the two hands got interchanged.
This implies that the duration of the test is less than
3 3 1 3 hours; (because 3 hours would have been completed if
= − ---------- (2)
the minute hand came to the point E.)
x y 30
During the test period, the minute hand completes
From (1), we get y = x – 3 which, when substituted in (2) 2 rounds of the dial, and falls short of the angle SCE for the
and solved for y, we get y = 15 kmph. 3rd round; but the hour hand rotates through the
angle SCE.
Alternate method: Hence, sum of the angles of rotation of both the hands
= angle of rotation for 3 rounds of the dial
The faster runner overtakes the slower runner once in = 360 x 3 = 1080 degrees. ---------- (2)
every hour. This implies that the number of rounds (or laps) From (1) and (2),
of the track made by the faster in 1 hour is more than the 13t 2 x 1080
rounds made by of the slower by 1. Let the slower = 1080; t = minutes
complete n rounds in 1 how; then the faster completes 2 13
(n + 1) rounds in 1 hour. 2 x 1080 10
= hours = 2 hours.
Time taken by the faster to complete 1 round is less than 13 x 60 13
that of the slower by 2 minutes, i.e., (1/30)th hour.
1 1 1 Alternate method:
⇒ − = ; Let the time at start be p minutes past 2 and the time at the
n n + 1 30 end be q minutes past 5.
1 1 ∴ At the start, angle made by minutes hand = 6p;
⇒ = hours hand = 60 + p/2.
n(n + 1) 30; At the end of the test, the angles are:
⇒ n2 + n – 30 = 0; (n + 6) (n – 5) = 0 minute hand = 6q
Ignoring the negative value, n = 5. hour hand = 150 + q/2
Track length is 3 km; hence for 5 rounds, the distance Since the positions interchange
covered is 15 km; and the speed is 15 kmph. 6p = 150 + q/2 → (1)
Choice (C) 60 + p/2 = 6q → (2)
p− q
19. Since Pramod runs at half the speed of Prakash, they meet 6(p − q) = 90 −  
 2 
once for every round Pramod makes or for every 2 round
Prakash makes.
∴ If they meet 11 times, Pramod makes 11 round and
13
(p − q) = 90
2
Prakash makes 22 round i.e., Prakash covers 11 times the
circumference more than Pramod, i.e., 11(22/7) (7) (2)m (p − q) = 180 → (1)
= 484 m Ans: (484) 13
We know that the duration from 2 : p to 5 : q is
20. Let the speeds of P and Q be p km / hr and q km/hr 3 hours + (q − p) minutes = 180 + (q – p) minutes
respectively. Substituting from (1)
Same direction: Initial position of P was behind that of Q. = 180 – 180/13 = 180 x 12/13 minutes
When P catches up with Q for the second time, the = (180 x 12)/(13 x 60) hours
difference of the distance covered by them would be (XY + 10
track length) i.e. 8 km. = 36/13 hours = 2 hours Choice (B)
13
8 40
= . . . . . (1) (when the two meet, p would have
p − q 60 22. Starting from 12 noon, angle moved by the hour hand
= (4 x 30) + (30 x ½) = 135
covered a greater distance ∴ p > q)
Angle covered by minute hand
6 − 2 12 = (4 x 360) + (30 x 6)
Opposite direction = . . . . . (2) = [(4 x 0) + 180] = effectively 180.
p + q 60
Angle between the hands of the clock at 4:30 p.m.
Solving (1) and (2) p = 16 and q = 4 Choice (A) = 180 – 135 = 45° Choice (D)

Triumphant Institute of Management Education Pvt. Ltd. (T.I.M.E.) HO: 95B, 2nd Floor, Siddamsetty Complex, Secunderabad – 500 003.
Tel : 040–27898194/95 Fax : 040–27847334 email : info@time4education.com website : www.time4education.com SM1001961/37
23. The first clock will show the correct time when it gains Since L beats N by 125 m, and as they run for the same
24 hours. It gains 4 minutes every hour. time; ratio of speeds of L and N will be
1
To gain 24 hours, 24 x = 360 hours, are required. 500
(4 / 60 ) =
500 − 125
-------- (1)
The second clock will show the correct time when it loses
24 hours. It loses 6 minutes per 1 hour L and N, individually cover the distances of 500 m in
To lose 24 hours, it requires, tL seconds and (tL + 80) seconds respectively.
1
24 x = 240 hours. t L + 80
(6 / 60 ) Hence, ratio of their speeds = ---------- (2)
tL
If both of them have to be correct, then LCM of (360 and
240 i.e. 720) hours or 30 days exactly are required. From (1) and (2)
So, at 8:00 a.m. on 31st January both the clocks will show
t L + 80 500
the correct time. Choice (D) =
tL 375
5 Solving, we have tL = 240 sec.
24. The hands of the correct clock coincide every 65 minutes. Time taken by M to run the race = 240 + 40
11 = 280 sec Choice (C)
∴The first clock loses 80 – (655/11)
= 160/11 min per 80 min 28. During the time Rahim covers 600 metres, Saleem covers
5 300 m, as Rahim’s speed is double that of Saleem. During
The second clock gains 65 – 65
11 the time Rahim covers the balance 400 metres,
= (5/11) min per 65 min Saleem covers 400 m, as their speeds are equal. Saleem had
In 24hours, the first clock loses a head start of 200 m and he covers (300 + 400) = 700 m.
[24 x 60 x 160/11] x 1/80 Hence, when Rahim is at finishing point, Saleem is at 200 + 700
The second clock gains in 24 hours = 900 m metres. Saleem takes 20 seconds to cover the
[24 x 60 x 5/11] x 1/65 balance 100 m. Saleem’s speed = 5 m/s. Choice (B)
24 x 60 x 2 24 x 60
Time difference = + 29. First condition gives 4(b – s) = b + s → (Ι)
11 11 x 13 where, b = speed of the boat in still water (kmph) and s
24 x 60  1  24 x 60 x 27 = speed of stream (in kmph)
= 2 +  = = 271.89 minutes Second condition gives 10/b = 2 ⇒ b = 5 → (ΙΙ)
11  13  11 x 13
From (Ι), (ΙΙ)
≅ 272 minutes Choice (A)
4(5 – s) = 5 + s ⇒ s = 3
∴ Speed of the stream = 3 kmph Ans: (3)
25. City P must be to the west of city Q. Let the local time
difference between the cities be t hrs.
30. Let the speed of the stream be y, then the speed of the
boat in still water will be y + 8
Method 1
The speed of A each way is the same Upstream speed = y + 8 − y = 8 kmph.
Hence, upstream time = (96/2)/8 = 6 hours
∴ The travel time each way must be same Time for downstream = 9 − 6 = 3 hours. Choice (A)
Travel time from P to Q = (4 – t) hrs
Travel time from Q to P = (2 + t) hrs 31. Let the number of steps on either escalator be S
4–t=2+t Let the speed of the person be x steps / sec and that of
t = 1 and 4 – t = 2 + t = 3 either escalator be y steps / sec
PQ = 800(3) = 2400 3
S = 90 (x – y) = 18 (x + y) ⇒x= y
2
Method 2
Total distance traveled=2PQ = 800(4 – t) +800 (2 + t) ∴S = 45y
= 800(6) PQ = 2400 Ans: (2400) Time taken to go up / down (in seconds) using either
S 45 y( 2)
26. escalator, if it is switched off = = = 30
x 3y
H× × ×S Ans: (30)
M
32. In 11 minutes, 4½ tonnes of water is admitted by the leak.
Let H be the home, S be the school and M be the point In 1 minute, the leak admits = 9/2 x 1/11
where Anwar and his wife meet. = 9/22 tonnes of water.
As Anwar travels always at the same speed of 30 kmph, In 1 minute, the pumps can throw out 1/5 tonnes
ratio of HS to HM shall be same as the ratio of times of The net inflow of water per minute = (9/22) – (1/5)
travel. = 23/110 tonnes
⇒ (HS/HM) = [(6PM – 4PM)/2]/[(5.45 PM – 4.0 PM)/2] Time taken to accumulate 184 tonnes of water
⇒ (HS/HM) = (2 hrs/2)/(1 hr 45 min/2) = 1/(7/8) = 8/7 184/(23/110) = 880 minutes
⇒ (HM + MS)/HM = [1 + MS/HM] = 1 + (1/7) Average rate of sailing so that the boat may just reach the
⇒ (MS/HM) = 1/7 shore as she begins to sink = 154/880
As HM and MS are covered by Anwar and the wife during = 10.5 kmph Choice (A)
the same time interval, (MS/HM) shall be equal to the ratio
of their speeds. 33. If the car was stationary, the gunfire would have been
⇒ (1/7) = (speed of the wife)/speed of Anwar heard at an interval of 24 seconds. But because it is
moving towards the source of the sound, sound of the
= (speed of wife)/30
second gunfire was heard after an interval of 22 seconds
⇒ speed of the wife = (30/7) kmph. Choice (A)
only. The distance sound would have travelled in 2 seconds
is the distance travelled by the car in 22 seconds.
27. Let the time taken by L to run 500 m be tL.
∴ 330 x 2 = V x 22 where V is the speed of the car
Time taken by M to run 500 m = tL + 40
If M and N run a 500 m race, time taken by N = tL + 40 + 40 ∴ V = 30 m/sec Ans: (30)
= tL + 80

Triumphant Institute of Management Education Pvt. Ltd. (T.I.M.E.) HO: 95B, 2nd Floor, Siddamsetty Complex, Secunderabad – 500 003.
Tel : 040–27898194/95 Fax : 040–27847334 email : info@time4education.com website : www.time4education.com SM1001961/38
34. 4. Let his usual time be x minutes. Let the distance be d km
d km
300 km Usual speed =
x min ute

A C B 4
If Ashok traveled at th of his usual speed, his usual
5
Let the car develop an engine problem at point C. So, it d 5
covers CB at 3/4th of its original speed. New time taken speed, his time = = x min utes
4d 4
= 4/3rd of the original time taken to cover CB.
5 x
4  1
Given  −1 t = ( t ) = 80, 5
3  3 x − x = 15
4
where, t is the original time taken to cover CB.
x = 60
t = 240 minutes.----------- (1)
When the problem develops 50 km further on, (i.e., at point D) 6
If Ashok traveled at th of his usual speed, his time
5
50 km d 5
= = x
6 d 6
A C D B 5 x
1
Let t, be the time taken to travel the distance DB, if the car He would be early by x = 10 min utes.
had moved with normal speed. 6
1 The ratio of speeds of Ashok in three instances i.e., usual
t1 = (80 − 20) = 60 speed, 4/5th speed and 6/5th speeds, is s : 4s/5 : 6s/5 i.e.,
3 5 : 4 : 6.
⇒ t1 = 180 minutes. (As the car reached 20 minutes 1 1 1 1
sooner) --------- (2) As speed α , time taken ratio is : : i.e.,
From (1) and (2) time 5 4 6
Time taken to cover 50 km = (240 −180) 12 : 15 : 10.
= 60 min = 1 hr Given, 3 parts is representing 15 minutes, hence 2 parts
Speed of the train = 50 kmph. will represent 10 minutes. Choice (B)

 240 
 (50) = 300 − 200 = 100 km
5. By formula
AC = 300 − 
 60  vu
(p + q) = distance
Choice (A) v −u

35. Let the length of a round be R km. Let the speed at which  v x 8  24
⇒   = 16
Hari walked be s km/hr  v − 8  60
R 8v 2
3s + 5   = 9R ⇒ x = 16
S v −8 5
⇒ v = 5v – 40 ⇒ 40 = 4v ⇒ v = 10 kmph
R
4s+10   = 14R
S Alternate method:
d d
5 5 − = difference of times of travel
Solving these, s =  and R =  Choice (A) s1 s 2
3  6
16 16 15 + 9 24 2
− = = =
Exercise – 5(b) 8 s2 60 60 5

Solutions for questions 1 to 50: Solving, s = 10 kmph Choice (A)

1. Distance = speed x time = (36 x 5/18) x 20 6. Let the total distance covered be 100d km
= 200 m Choice (D) 50 d 30 d 20d
+ + = 9.8 ⇒ d = 1.8
2. Let the times taken for traveling to the town by train and by 15 18 45
car be x hours and y hours respectively and 100d = 180 Ans: (180)
Given, x + y = 16 and 2y = 16 − 4 = 12
∴y=6 7. Time after which they meet =
84 4
= hours
∴ x = 10 36 + 27 3
⇒ 2x = 20
∴The first person will cover 36 x 4/3 = 48 km in this time.
∴ He would lose 4 hours. Ans: (4)
So, they will meet 48 km from P. Choice (D)
3. Let the original time taken be ‘t’. Since, his speed increases
8. Let the speeds of the faster car and the slower car be
to 3 times his original speed, new speed = (3)
x kmph and y kmph respectively.
(original speed) ⇒ New time
80
original time Given, = 1 ⇒ x + y =80 ––––– (1) and
= x+y
3
80
Given, t −
t
= 40; ⇒
2t
= 40 ⇒ t = 60 sec. = 4 ⇒ x − y = 20 ––––– (2)
3 3 x−y
Choice (D) Solving (1) and (2), x = 50 Ans: (50)

Triumphant Institute of Management Education Pvt. Ltd. (T.I.M.E.) HO: 95B, 2nd Floor, Siddamsetty Complex, Secunderabad – 500 003.
Tel : 040–27898194/95 Fax : 040–27847334 email : info@time4education.com website : www.time4education.com SM1001961/39
9. 13. Let the speeds of the cars leaving P and Q be p kmph and
q kmph respectively.
px = qy ––––– (1)
D 100 A C pz = qx ––––– (2)
Dividing (1) by (2),
x y
The cat runs away from an initial point A for 1 minute at a =
z x
speed of 24 kmph.
∴ AC = 24 x 60 x 5/18 = 400 m x= yz . Choice (C)
Now, the dog is separated from the cat by a distance of
100 + 400 (DA + AC) = 500 m and this lead has to be 14. Let the time taken by Gopi to reach K after meeting Ram,
covered at a speed of (33 – 24)
= 9 kmph i.e., relative speed. 2 tG
be tG. Then, we have = 2
500
3 tG − 7
∴ Time = = 200 sec
5 Squaring both sides and solving for tG we get tG = 4 hours.
9x Distance between K and L = Distance covered from the
18
meeting point to L by Ram + the Distance covered from the
But, if the time is required from the start of the cat’s run, meeting point to K by Gopi = 2(42 – 7) + 3(4) = 30 km
1 minute has to be added. i.e. 200 + 60 = 260 seconds. Choice (C)
Choice (C)
15. Anand and Ajay would meet for the first time during their
10. onward journeys. They would meet for the second time
during their return journeys. Time taken by Anand to travel
P Q 8
from M to N = sec onds.
3
A 8 40
Distance travelled by Ajay in this time = (5) = m.
3 3
B
40 16
∴ He would be −8 = m from N when Anand
When ‘B’ starts from city P towards city Q, the distance 3 3
which ‘A’ would already have covered = 54×3 = 162 km. reached N.
At 9 a.m., train B is separated from train ‘A’ by a distance of 16 8
∴ Distance between them would then be 8 − = m.
162 km. 3 3
Train B overtakes train A after a time of Anand would cross Ajay in another
162 162
= hrs = 9 hrs. 8
(72 − 54) 18 3 1
= sec onds.
In 9 hrs, the distance travelled by B = 72 × 9 Re lativespee d 3
= 648 km. ∴ Ajay would have travelled a distance of
Distance from city Q, when they meet
40 1
= (1440 − 648) = 792 km Choice (B) + (5) = 15 m when he crossed Anand for the second
3 3
11. By 10:00 a.m, A would have traveled 120 km. At 10:00 a.m, time. Choice (C)
distance between A and B = 145 km. In another hour A and
B would have traveled 60 km and 40 km repeating both 16. Let the speeds of Anand and Ashok before the meeting be
cars would have together traveled 100 km x kmph and y kmph respectively
∴ At 11:00 a.m, distance between A and B = 45 km. In the 10
⇒ x + y = 10
1= ––––– (1)
next half an hour, A would travel 30 km while B would be x+y
stationary. By 11:30a.m, A would have traveled 210 km. Distances travelled by Anand and Ashok before meeting
Now it would stop for 20 minutes. At 11:30a.m, B would are x km and y km respectively. As they reached their
resume its journey and would be 15 km from S. It would destinations simultaneously,
reach S in another 22⋅5 minutes.
y x
∴ Time of meeting = 11:52. 30 a.m. Choice (C) = ⇒ y2 + 2y = x2 – 2x
x−2 y+2
12. 2=x−y ––––– (2)
Solving (1) and (2), x = 6. Ans: (6)

60 km 17. Let his forward and return speeds be x kmph and y kmph
P R x km S Q respectively.
2 xy
Average speed =
x+y
Suppose the trains met at S. Let RS = x km
2xy x+y
PS RS SQ Given, =
= = +1 x+y 2
90 60 120
⇒ (x + y)2 − 4xy = 0
60 + x x ⇒ (x − y)2 = 0
=
90 60 ⇒x=y
120 = x 2xy
∴ =x
x+y
RS SQ
∴ =2= +1 1200
60 120 ∴ Forward speed = Average speed = = 100 kmph
SQ = 120 km 12
PQ = PR + RS + SQ = 300 km. Ans: (300)
Choice (A)
nd
Triumphant Institute of Management Education Pvt. Ltd. (T.I.M.E.) HO: 95B, 2 Floor, Siddamsetty Complex, Secunderabad – 500 003.
Tel : 040–27898194/95 Fax : 040–27847334 email : info@time4education.com website : www.time4education.com SM1001961/40
18. From the given data, which is about the speeds of travel d 600 − d 600
and the average speed, in terms of a variable x, the Given, + = + 1 ––––– (1)
s 4 s
average speed can be determined. As no information about s
5
the total time is available, distance cannot be calculated.
The same can be seen from the calculations shown below. d + 150 450 − d 600 1
⇒ + = + ––––– (2)
Let the total distance AB be d. S 4 s 2
s
5
 d  1  2d  d −  d + 1  2d  
 3  4  3   3 4  3   Subtracting (1) from (2),
 
Total time =   +  
+

150 (150)s −1
x 2x 3x ⇒ − =
s 4s 2
d 1 1 1 d 7  ⇒ S = 75
=  + + =  
x  3 12 6  x  12  Substituting S in (1) or (2),
d = 300. Choice (D)
d
Average speed for the entire journey =
d 7  23. Let the speed of his son be x kmph.
 
x  12  As Ashwin returned home 20 minutes early, he saved a
= 12x/7 = x + 2, 5x/7 = 2, x = 14/5 kmph travel of 10 minutes to school from the meeting point and
Total distance the person covers cannot be determined as the return journey of 10 minutes. Distance he saved
total travel time is unknown. Choice (D) travelling each way = Distance travelled by his son by walk.
His son would have walked for 50 minutes.
19. AB = BC  50   10 
( x )   = (55 )  
(i) Average speed =
(2)(4)(6) = 4⋅8 kmph  60   60 
4+6 x = 11 kmph Ans: (11)
Choice (C)
24. Since R has given S a start of 2 hours and arrives at the
(ii) Using similar method as shown in (i), average speed destination 12 minutes after S, he takes 1 hr 48 min (9/5 hrs)
of Rahul from C to A = 4⋅8 kmph i.e., same as that less than S to reach the meeting point compared to S. If the
from A to C. 36 36 9
∴ Average speed of Rahul for his journey speed of R is x, we have, − =
x − 18 x 5
= 4⋅8kmph. Choice (C)
Solving this equation, we have x = 30 kmph
Choice (C)
total dis tan ce
(iii) Time taken for his journey =
Average speed 25. Let the length of the race be dm
(0 ⋅ 48)(4) = 0 ⋅ 4 hours . When Mohan finished the race, Sohan would have run
= Choice (C) (d − 40)m and Rohan would have run (d − 104)m.
4⋅8
When Sohan finished the race, Rohan would have run
(d − 80)m.
20.
∴ Ratio of speeds of Sohan and Rohan
←(8:00 a.m.)
A d − 40 d d
x ↓(9:00 a.m.) = × =
d d − 104 d − 80
⇒ (d − 40) (d − 80) = d (d − 104)
y ⇒ d = 200. Ans: (200)
150 km
26. (i) Distances run by Bhavan and Charan when Amar
finishes the race are (d − x)m and (d − y)m respectively.
B ∴ Ratio of speeds of Bhavan and Charan = d − x:d − y
∴ When Bhavan finishes the race, Charan would have
The distances covered by C and D are x and y respectively.
d−y
Since C starts early compared to D, he covers more distance run (d) m
by noon compared to D. Hence x > y. If speed of each car is s. d−x
x = 4s and y = 3s
x2 + y2 = (150)2 (by Pythagoras theorem)  d−y 
∴ Bhavan would beat Charan by  d − ( d)  m
Taking x = 4s, y = 3s and solving for s we have,  d−x 
s = 30 kmph. Ans: (30)
=
(y − x ) d m Choice (B)
21. Let the time he would take to reach Q be x minutes d−x
Speed of his car in the xth minute
= 51 + x − 1 = (50 + x) km/hr (ii) When Amar finishes the race, Bhavan would have run
(d − x)m. When Bhavan finishes the race,
PQ =
1
(51 + 52 + ......(50 + x)) Charan would have run (d − y) m. Ratio of speeds of
60
Amar and Chetan = (Ratio of speeds of Amar and

1
(50 + 1 + 50 + 2 + ............50 = x ) = 3775 Bhavan) (Ratio of speeds of Bhavan and Chetan)
60
⇒ 50x + 1 + 2 +………..x = 3775
60
=
d d
=
(d) (d)
d−x d−y d2 − (x + y )d + xy
 x + 1
⇒ x  50 +  = 3775
 2  d
=
⇒ x (101 + x) = 7550 i.e., (50) (101 + 50)  xy 
d −  x + y − 
Comparing both sides, x = 50. Choice (D)
 d 
22. Let the distance it traveled without any problem be d km. ∴ When Amar finishes the race, Chetan would have
Distance it traveled at the reduced speed = (600 − d)km.  xy 
Let its usual speed by S kmph run d −  x + y − m Choice (D)
 d 

Triumphant Institute of Management Education Pvt. Ltd. (T.I.M.E.) HO: 95B, 2nd Floor, Siddamsetty Complex, Secunderabad – 500 003.
Tel : 040–27898194/95 Fax : 040–27847334 email : info@time4education.com website : www.time4education.com SM1001961/41
27. As Ajay gives Bala a start of atleast 20m, Bala has to run a Alternate method:
maximum distance of 80m. As he is beaten by Bala by
atmost 20m, Ajay would have run a minimum of 80m when Distance = d km; speed of Donald = b kmph.
Bala finishes the race. As the speeds of both are distinct, Here, total time taken by Donald
Bala must have run less than Ajay when Ajay finished the race. = (d/b) hours -----(1)
∴ Ajay is faster than Bala. Choice (D) Time taken by Allen = Time taken by Donald
(given data) = (d/b) hours ------- (2)
28. B covers 25 m in 5 s Time taken by Allen for the first half
∴ Speed of B = 5 m/s = (d/2)/a = (d/2a) -------- (3)
Hence time taken by B = 500/5 = 100 s Hence the time taken by Allen for the second half
Time taken by A = (100 – 5) = 95 s = (d/b) − (d/2a) (from (2) and (3)
5 = d(2a − b)/2ab --------- (4)
Speed of A = 500/95 = 100/19 = 5 m / sec Speed of Allen while covering the second half
19
= (d/2)/[d(2a − b)/2ab]
Choice (B)
d x 2ab
= = (ab) /( 2a − b) kmph. Choice (D)
29. When Raja finishes the race, Rakesh would have run 2d ( 2a − b)
200 − (20 + 20) = 160m
200 5 34. The distance is covered in the same time by both of them
∴ Ratio of the speeds of Raja and Rakesh = = and since Donald travels at a constant speed of b, Allen’s
160 4
average speed will be equal to Donald’s speed = b km/hr.
As Raja beats Rakesh by 20m and also beats him by Choice (D)
4 seconds, Rakesh must have taken 4 seconds to run the
last 20 m. 35. Let the length and the speed of the train be Lm and S m
20 /sec respectively
∴ speed of Rakesh = = 5m / sec
4 L + 200
Given, = 30 ⇒ L + 200 = 30S ––––– (1)
5 S
∴ speed of Raja = (5) = 6 ⋅ 25m / sec . Ans: (6.25)
4 L + 300
Given, = 40 ⇒ L + 300 = 40S ––––– (2)
S
30. Let the time taken by Ram to run the race be t seconds. Subtracting (1) from (2), 100 = 10s ⇒ 10 = S
Time taken by Shyam to run the race = (t + 60) seconds. From (1) or (2), L = 100m. Ans: (100)
Time taken by Tarun to run the race
= (t + 90) seconds. Ratio of speeds of Ram and Tarun 36. Let the length and the speed of the train be Lm and
S m/sec respectively.
1000
L = 60s
1000 t
=  5 
1000 − 250 1000 L + 3600 = 240 (S ± (54)  
t + 90  18 
60S + 3600 = 240 (S ± (15)
4 t + 90
= 60S + 3600 = 240S + 3600
3 t or 60S + 3600 = 240S − 3600
t = 270. Choice (B) S = 0 or 20
As S = 0 is not possible, S = 20
31. Let Karna’s speed = k ∴ Speed of the train = 144 kmph. Choice (B)
Kiran’s speed = b
Kumar’s speed = r 37. Let the length and the speed of the train be Lm and Sm/sec
∴100/k = 80/b ------- (1) respectively
(100/b) + 7.5 = 100/r ---------- (2)
  5  
Given that k = 2r L = 120  S − (18 )    - - - - - (1) and
∴(100/b) + 7.5 = 160/b   18  
⇒ 7.5 = 60/b   5 
b = 8 m/s Ans: (8) L = 90 S − (9 )    ----- (2)
 
  18  
32. Distance travelled by Habib/Distance travelled by Akram Solving (1) and (2), we have
= 500/440 S = 12⋅5
Let the length of the pace of Habib be x and that of Akram ∴ L = 900 Choice (D)
be y.
Ratio of speeds = ratio of distances covered in equal 38.
intervals of time = (5x/4y) T

∴ 5x/4y = 500/440 C
x/y = 10/11 Choice (A) T
→ T
C → →
33. Initial speed of Donald = b kmph C C
Initial speed of Allen= a kmph
d d • • • •
Total time taken by Allen= + where x is the speed P Q R S
2a 2x
nd
for the 2 half of the distance. Let the speed of the train be V m/s
d 360
Total time taken by Donald = . Then = 12 ; V = 50 m/s
b V − 20
∴(d/2a) + (d/2x) = (d/b); Similarly let the speed of the cyclist be V1m/s
1 1 1  ab  360
+ = ,  x =  =9
2a 2x b  2a − b  50 − V1
V1 = 10 m/s
Triumphant Institute of Management Education Pvt. Ltd. (T.I.M.E.) HO: 95B, 2nd Floor, Siddamsetty Complex, Secunderabad – 500 003.
Tel : 040–27898194/95 Fax : 040–27847334 email : info@time4education.com website : www.time4education.com SM1001961/42
Let P be the point where the train begins to overtake the 43. l = 1120 m
car Q be the point where the train completely overtake the a = 10 m/s, b = 8 m/s, c = 7 m/s
car R be the point where the car is, when the train begins to
(i) Required answer
overtake the cyclist
S be the point where the train begins to overtake the  l l   1120 1120 
= LCM  ,  = LCM  10 − 8 , 8 − 7 
cyclist. a−b b−c  
In 48 min, when it is just about to overtake the cyclist, it will = LCM (560, 1120) = 1120 s
cover (48) (60 (50) m, i.e. QS = 48(60) (50)m i.e.
QR = 48(60) (20)m (ii) Required answer
And the car will cover (48) (60) (20) m l l l
= LCM  , , 
At this instant the distance between the car and the cyclist a b c
RS is 360 + (48) (60) (50 – 20)
= 360 + (48) (60) (30)  1120 1120 1120 
= LCM  , , 
This distance has to be covered with a relative speed of  10 8 7 
(20 – 10) = 10 m/s = LCM (112, 140, 160) = 1120 s Choice (D)
360 + ( 48) ( 60) (30) 44. Radius = 35 m.
∴Time required =
10 ⇒ Circumference = 2 x 22/7 x 35 = 220 m
= 36 s + 144 min = 2 hr 24 min 36 s Time taken for them to meet for the first time at the starting
As the time required is the time after the train overtook the point is LCM [220/20, 220/11]
cyclist, it is 9 seconds less; i.e., 2 hr.24 min 27 s. = LCM [11, 20] = 220sec
Choice (D) To meet for the third time at the staring point from the start,
they need 220 x 3 = 660 sec = 11 minutes
30 Choice (A)
39. Speed of the boat upstream = = 6 kmph
5 45. Akbar completes 1 revolution in 40/5 = 8 sec
24 Circumference/Akbar’s speed = Circumference/11
Speed of the boat downstream = = 8 kmph
3 ∴Circumference = 88 m
Time taken to meet for the first time is
 8+6   88 88   88 88 
Speed of the boat in still water =   kmph = 7 km/hr LCM of  ,  i.e., LCM of  , 
 2   22 − 11 11 − 7   11 4 
i.e., LCM of {8, 22} = 88
 8 −6  So, they meet for the first time 88 seconds after the start.
Speed of the water current =   = 1 kmph
 2  Ans: (88)
Choice (C)
46. As P’s speed increases by the same amount as that by
which Q’s speed decreases, their relative speed remains
40. Let the seed of the boat in still water and the speed of the
unchanged.
river be x kmph and y kmph respectively. Let the time taken
for the downstream journey be t hours. Time taken for the ∴ Time taken by them to meet for the third time = 3 (Time
taken by them to meet for the first time)
upstream journey = (9 − t) hours.
(x + y) t = 24  48000 
(x − y) (9 − t) = 24
= 3  = 3600 sec . Choice (A)
 10 + 30 
In still water,
y=0 47. Time taken by them to meet for the first time
24 1800
∴ x= =6 = = 100 seconds
9−5 6 + 12
(6 + y) t = 24 ––––– (1) Distances covered by Ram and Shyam in this time are
(6 − y) (9 − t) = 24 ––––– (2) 600 m and 1200 m respectively. After the first meeting,
Solving (1) and (2), y = 2. Ans: (2) their speeds would be exchanged. This would not affect
time interval between consecutive meetings.
41. Let the speed of the boat in still water and the speed of the ∴ In the next 100 seconds, Ram would have travelled 1200 m
river be x kmph and y kmph respectively. and would have reached his starting point and would meet
Shyam there. In this manner
x2 − y2
Given =6 Ram completed a round. In this manner he would have
x completed 3 rounds. When he would have completed
Let the round trip journey be covered between two points 1
another rounds, Shyam would have
d km apart. Total time for the journey 4
12  1 1
d d 2dx completed   = a round.
= + = hours 6 4 2
x + y x − y x2 − y2
 1 1
∴ Distance between them = 18001 − −  = 450m .
2d x2 − y2  2 4
Average speed = = .=6
2dx x Ans: (450)
x2 − y2 48. Ratio of the speeds of Rohan and Sohan = 1000 :
Choice (B) 1000 − 200 = 5 : 4
Let the speeds of Rohan and Sohan be
42. If two or more runners have their speeds as a multiple of 5x m/ minute and 4x m/ minute respectively.
the speed of a runner N the time taken by these runners to 1000 1
meet for the first time at the starting point for the first time is = 111
the time taken by the N to complete one round. 5x + 4x 9
As the speeds of Harish and Suresh and Multiples of the x=1
speed of Girish, required time = Time taken by Girish to 1000
Required time = = 1000 sec onds. Ans: (1000)
complete 1 round = 1 minute. Ans: (1) 5x − 4x
Triumphant Institute of Management Education Pvt. Ltd. (T.I.M.E.) HO: 95B, 2nd Floor, Siddamsetty Complex, Secunderabad – 500 003.
Tel : 040–27898194/95 Fax : 040–27847334 email : info@time4education.com website : www.time4education.com SM1001961/43
49. Correct time Clock time Solutions for questions 51 to 60:
10:00 a.m. 6:00 p.m. 6 : 20
(8 hrs) (8 1/3 hrs) 51. From statement Ι,
? 10 : 30 speed of the train is
(x) (12½ hrs) 200
= 20 m/s
10
Applying the rule of proportions to the durations,
From statement ΙΙ, we do not know the length of the
1
8 platform, so we can’t find the speed of the train.
8 3 = 25 / 3
= So statement ΙΙ alone is not sufficient. Choice (A)
x 1 25 / 2
12
2 52. Let the length of the race be L m.
⇒ x = 12
∴ The correct time is 10:00 a.m. + 12 hours = 10:00 p.m.
Choice (B)
A B C D
50. Let the time at which he began the test be 3 : p p.m. Let the
time at which he ended the test be 4 : q p.m. In the above figure, A and B denote the starting points of
Angle made by the minute hand at 3 : p p.m = 6p°. A ngle Ram and Shyam respectively. D denotes the finishing points of
made by the hour hand with the 12:00p.m position of hands both. C denotes the position of Ram when Shyam finished.
o When Shyam covered BD, Ram would have covered AC.
 p
is  90 +  BD = BC + CD = BC + y
 2 AC = AB + BC = x + BC
Angle made by the minute hand at 4 : q p.m = 6q°. A ngle If Ram had not been faster than Shyam, AC ≤ BD must
made by the hour hand at 4 : q p.m with the 12:00 p.m have been true.
 q
o ∴ x ≤ y must be true.
position of hands is 120 +  . From statement Ι, if x = y, Ram was not faster than Shyam.
 2 If x > y, Ram was faster than Shyam.
As the hands interchanged their positions, Ι. alone is not sufficient.
 q
o
ΙΙ. alone is sufficient. Choice (A)
6p° =  120 +  –––– (1)
 2
53. Let the speed of Mohan’s boat in still water and the speed
 p of the stream be x kmph and y kmph respectively.
 90 + 2  = 6q ––––– (2) From statement Ι, 12 = 4(x − y)
 
⇒3=x−y
subtracting (1), from (2),
∴x>3
p−q Ι is sufficient.
6 (q − p) = −30 +
2 From statement ΙΙ, 12 = 3 (x + y)
−60 4=x+y
q−p = For x ≥ y
13
∴x≥2
Duration for which he took the test = 4 : q p.m − 3 : p p.m
ΙΙ is not sufficient Choice (A)
q−p 1 12
= 1 hr + = 1− = hours .
60 13 13 54. A drove for 5 hours and B drove for 4½ hours.
From statement Ι, dA + 50 = dB and dA + dB = 500
Alternate method: ⇒ dA = 225 and dB = 275.
∴ We can find the average speed of B.
Starting position between Ending position between
3-00 and 4-00 4-00 and 5-00 dA d
From statement ΙΙ, +5 = B
5 4.5
(Ι) (ΙΙ) 2dB d A
⇒ − =5
9 5

10 dB − 9d A
⇒ = 5 ⇒ 10dB − 9d A = 225
x° x° 45
We know dA + dB = 500
So we can find dA and dB and then the average speed.
Let the angle between the hands of the clock be x° in the ∴ It can be answered using either statement alone.
position (Ι) taken from the minutes hand to the hour hand in Choice (B)
the anti-clockwise direction.
By the time the hour hand covers angle of x° to com e to the 55. From statement Ι, speed of the river current = 2 m/s
position ΙΙ, the minutes hand will cover (360° – x). From statement ΙΙ, speed of the boat relative to the speed
⇒ The time taken by the hour hand to cover x° is equ al to of the river is 1 m/s.
the time taken by the minute hand to cover (360° – x°). Using both the statements,
The speeds of hour hand and minute hand are 0.5° pe r minute V – 2 = 1 where V is the speed of the boat in still water.
and 6° per minute respectively. ⇒ V = 3. Choice (C)

x 360 − x 360° 56. Statement Ι alone is not sufficient as time taken for the trip
∴ = ⇒x=
0.5 6 13 is not known.
360 From statement ΙΙ alone, distance travelled in the trip = st
∴ The duration of the test is minutes where s is the average speed and t is the time taken for the trip.
13 × 0.5
Given (1.25s)t – st = 100 ⇒ 0.25 st = 100
=
12
hours. Choice (B) ∴ st can be found
13 Hence ΙΙ alone is sufficient. Choice (A)

Triumphant Institute of Management Education Pvt. Ltd. (T.I.M.E.) HO: 95B, 2nd Floor, Siddamsetty Complex, Secunderabad – 500 003.
Tel : 040–27898194/95 Fax : 040–27847334 email : info@time4education.com website : www.time4education.com SM1001961/44
57. Let the usual time be t hours. 60. Let the speeds of A, B and C be a m/sec, b m/sec and
 a  c m/sec respectively.
Distance between his home and office = a  t − 
 60  a 1000
=
 b  b 1000 − x
= b  t − 
 60  b 1000
=
a2 − b2 c 1000 − y
(a − b) t =
60
a  a  b  1000 2
a+b =     =
t= c  b   c  1000 −1000 ( x + y ) + xy
2
60
Ι is sufficient. a 1000
=
From ΙΙ, as we cannot find a + b, it is not sufficient. c  xy 
1000 −  x + y − 
Choice (A)  1000 
58. Let the length of A be 3z m. Length of B = 2z m. Let the
 xy 
speeds of A and B be a m/sec and b m/sec respectively. A beats C by  x + y −  m
Let the lengths of P1 and P2 be p m and q m respectively.  1000 

3z + p In each statement, x = 200


=x
a 200 y
When 200 + y − > 400
2z + q 1000
=y
b y > 250
Using statement Ι, we cannot say that y > 250. A is not
From statement Ι, a = b sufficient. As y ≤ 200, ΙΙ is sufficient. Choice (A)
x 3z + p
∴ = Chapter – 6
y 2z + q
(Time and Work)
Nothing is known about p and q.
∴ The question cannot be answered. Ι is not sufficient. Concept Review Questions
From statement ΙΙ, p = q Solutions for questions 1 to 35:
x 3z + q  b 
∴ =   1
y 2z + q  a  1. 10 men can do th of the job in a day.
10
Nothing is known about a and b. 1
∴The question cannot be answered. ∴ 1 man can do th of the job in a day.
100
ΙΙ is not sufficient. Choice (B)
Using both statements, we get a = b and p = q
2. 6 men can eat 6 apples in 1 day.
x 3z + p 3 2q x 3 ∴ They can eat 36 apples in 6 days.
∴ = = − .∴ 1 < < Ans: (36)
y 2z + q 2 2 z + q y 2
3. Per min grass cut = 1/T
We cannot answer the question. In 30 minutes = 30/T part will be cut. Choice (A)
Both statements even when taken together are not
sufficient. Choice (D)
4. X men → 120 days
X + 10 men → 100 days
59. Let the lengths of A and B be p m and q m respectively. Let
X (120) = (X + 10) 100
the speeds of A and B be a m / sec and b m / sec
respectively. 6X = 5X + 50 ⇒ X = 50 Ans: (50)

a 5.
b=
2 Men Work Days
10 1 15
p + 500
= 50 –––––– (1) X 5 10
a
1 men → 1/150 (one day’s work)
Length of B = 2 p m
10 days work = 10/150 = 1/15
2p + q 2p + q i.e. 15 men for one work
Required time = = men needed = 15 × 5 = 75 Ans: (75)
b a
2
6. Job = (15) (9) = 135 man days = (135) (C) = women days.
From statement Ι, q and a are unknown. 405
∴ required time cannot be found. Time taken by 15 women to complete the job = or
15
Ι is not sufficient. 27 days Choice (C)
From statement ΙΙ, q = 1000
M1 D1 H1 M2 D 2 H2
2p + q  p + 500  7. = --(1)
= 4   W1 W2
a  a 
2 M1 = M2 = 1
W1 = W 2
From (1), this can be found. ΙΙ is sufficient. D1 = 4, H1 = 10 and H2 = 8
Choice (A) Substituting these values in (1), D2 = 5. Ans: (5)
nd
Triumphant Institute of Management Education Pvt. Ltd. (T.I.M.E.) HO: 95B, 2 Floor, Siddamsetty Complex, Secunderabad – 500 003.
Tel : 040–27898194/95 Fax : 040–27847334 email : info@time4education.com website : www.time4education.com SM1001961/45
M1 D1 H1 M2 D 2 H2 1
8. = --(1) 17. Part of the job completed by P and Q in a day = th.
W1 W2 48
Part of the job completed by the faster person in a day
M1 = D1 = H1 = 3
W1 = W 2 5  1  5
M2 = 9 and H2 = 1 =   =
8  48  384
Substituting these values in (1), D2 = 3 Choice (C)
384
140 7 ∴ The faster person will complete the job in
9. X’s rate = (Y’s rate) = (Y’s rate) 5
100 5 = 76.8 days. Ans: (76.8)
∴ Required ratio = 7 : 5 Choice (A)
1
18. The part of the job which P can complete in a day = .
10. Let the job be 1 unit. 60
Let the times taken by X and Y to complete it be 5x hours The part of it which Q can complete in a day
and 6x hours respectively.  25   1  1
Parts of the job completed by X and Y in a hour are = 1 −    = .
 100   60  80
1 1
units and units respectively. ∴ Q can complete it in 80 days. Choice (B)
5x 6x

1 1 1
Required ratio = : =6:5 Choice (A) 19. P’s rate = (Combined rate of P, Q and R)
5x 6x 4
1
11. Let the job be 1 unit. ∴ P must have done th of the job. Choice (A)
4
Let the times taken by A, B and C the complete it be
3x hours, 4x hours and 6x hours respectively. 20. Work done by Amar one day = 1/12
Parts of the job completed by A, B and (in a hour are Work done by Bharat in 1 day = 1/24
Work done by Charu in 1 day = 1/24
1 1 1
units, units and units respectively. 1 1 1 1
3x 4x 6x Work done by 3 of them in one day = + + =
12 24 24 6
1 1 1 ∴ they take 6 days to finish the work. Choice (B)
Required ratio = : : =4:3:2 Choice (B)
3x 4x 6x
21. Work done by Anand, Bhanu and Chandra in 1 day = 1/4
12. Part of the job completed by A and B in a day Work done by Anand in 1 day = 1/8
Work done by Chandra in 1 day = 1/16
1 1 1
= + = 1 1 1  1
60 15 12 ∴Work done by Bhanu in 1 day = −  + =
∴ They will complete the job in 12 days. Ans: (12) 4  8 16  16
∴Bhanu takes 16 days. Choice (A)
13. Part of the job completed by Y in a day
22. Parts of the job, which can be completed, by P and Q,
1 1 1
= − = 1 1 1
30 60 60 Q and R and R and P in a day are th, th and th
12 20 15
∴ Y will complete the job in 60 days. Choice (D) respectively.
1 1 1
14. Per day ∴ 2 P’s, 2 Q’s and 2 R’s can complete + +
12 20 15
Adam = 1/25
Adam + Chris = 8/75 1
= th of the job in a day.
5
8 1 1
∴ Chris = − = 1
75 25 15 ∴ P, Q and R can complete th of the job in a day.
10
∴ Chris takes 15 days. Choice (D) ∴ They can complete the job in 10 days. Choice (A)

15. Time taken A = A 23. P and Q take less time to complete the job compared to
Time taken B = B Q and R. ∴ R is slower than P. We should new compare
A = 4B R and Q (not P and Q).
B = 60 days ⇒ A = 15 days P and Q take less time to complete the job compared to
∴ In one day P and R. ∴ R is slower than Q.
A does 1/15 of work ∴ R is the slowest of the 3 workers. Choice (C)
B does 1/60 of work
1 1 1 24. 60 days → Somu Time taken
Together in 1 day = + = 40 days → Ramu
15 60 12
Total amount = `450
So, they take 12 days. Choice (B) 1 1
Ratio of shares of Somu and Ramu = : = 2:3
60 40
16. One day's work
2
A = 1/x Somu's share = × 450 = `180 Ans: (180)
B = 1/3x 5
1 1 1
+ = ⇒ x = 8 days 25. Work done by Gautham in 1 hour = 1/4
x 3x 6 Work done by Karan in 1 hour = 1/12
∴ 3x = 24 days 1 1 1
Work done by both (in 2 hours) = + =
∴ B takes 24 days. Choice (D) 4 12 3
∴ total work is done in 3 × 2 = 6 hours. Ans: (6)
Triumphant Institute of Management Education Pvt. Ltd. (T.I.M.E.) HO: 95B, 2nd Floor, Siddamsetty Complex, Secunderabad – 500 003.
Tel : 040–27898194/95 Fax : 040–27847334 email : info@time4education.com website : www.time4education.com SM1001961/46
26. One day's work 35. Time taken by pipe A = 30 minutes
X = 1/12 Time taken by pipe B = 20 minutes
Y = 1/18 Time taken by emptying pipe C = 60 minutes
1 1 5 Per minute
X+Y= + = (in 1 time period) A = 1/30, B = 1/20, C = 1/60
12 18 36
1 1 1 4 1
Work in 7 time periods = 7 × 5/36 = 35/36 All pipes open at one time = + − = =
Remaining work = 1/36 30 20 60 60 15
1 36 ∴ It takes 15 minutes to fill the tank. Ans: (15)
Time taken by X to finish it = =1 3
1 12
Exercise – 6(a)
∴ total time = 7 × 2 + 1/3
↓ Solutions for questions 1 to 30:
1
7 time periods = 14 /3 hours Choice (B)
600
27. Part of the job completed in the first 2 days 1. Time in which A can complete the job = = 6 days .
100
1 1 1
= + + Time in which B can complete the job
12 20 30 900
The complete job involves 12 such parts. = = 15 days
60
∴ Total time = 2(12) = 24 days. Choice (C)
1200
Time in which C can complete the job = 30 days
28. The following points, must be noted. Suppose 2 workers 40
take x days to complete a job working together. If they work Part of the job which can be done by A, B and C in a day
on alternate days, part of the job they would complete in 1 1 1 4
the first 2 days would be (1/x). If x is an integer, they will = + + =
take 2x days to complete the job irrespective of who starts 6 15 30 15
the job. If x is not an integer, the job would be completed 15
faster if the faster of the workers starts the job. Here, the ∴ They would take days to complete it.
4
1 1 8 1
part alone in the first two days is + = = and
 15 
6 10 30 30 8 Cost to Anwar =   (100 + 60 + 40) = Rs. 750
X is the faster worker.  4 
∴ Choice (A) follows. Choice (A) Choice (C)

29. Total quantity of rations consumed by 6 members in a day 2. Total work = 30 (12) (50) man hours = 18000 man hours.
120 Amount of work done in 15 days putting 10 hours a day
= (6) (2.5) = 15 kg. Number of days = =8
15 = 15 (10) (50) = 7500 man hours.
Ans: (8) Work remaining = 18000 − 7500 = 10,500 man hours.
Number of men remaining = 40
30. Part of cistern filled by pipe x in 1 minute = 1/6 Number of hours = 10 per day.
Part of cistern filled by pipe y in 1 minute = 1/12
10500
1 1 1 Number of days needed = = 261/4 days.
∴ part of cistern filled by pipes x and y in 1 minute = + = 40 (10 )
6 12 4
∴ They require 111/4 more days to complete the work.
∴ Time taken by both pipes = 4 minutes. Choice (B)
Choice (A)
1 1 5
31. Part of the tank filled in a hour by both taps = + = 3. Let p, q, r denote the work done by P, Q and R respectively
6 9 18 in one day.
18 1
∴ The tank will be filled in = 3.6 hours by them. p+q= ------------ (1)
5 20
Ans: (3.6) 1
q+r= ------------- (2)
32. Part of the tank filled in a hour by both taps 15
1 1 5 1
= + = p+r= -------------- (C)
10 15 6 12
∴ The tank will be filled in 6 hours by them. In this time Adding the three equations,
6 3 1 1 1 12 1
X can fill = th of the tank. Choice (D) 2(p + q + r) = + + = =
10 5 20 15 12 60 5
1
33. Part of the tank filled in a hour by both pipes p+q+r= ----------- (D)
10
1 1 1
= − = (D) − (2) gives
9 18 18
∴ The tank will be filled in 18 hours by them. 1 1 1
p= − =
Choice (C) 10 15 30
34. Time taken by tap to fill the tank = 3 hours ∴ P can do it in 30 days
Time taken by leak = x hours (D) − (C) gives
Per hour 1 1 1
q= − =
Tap = 1/3; Leak = 1/x 10 12 60
1 1 1
Part of the tank filled in 1 hour = = − ∴ Q can do it in 60 days
4 3 X (D) − (1) gives
1 1 1 1 1 1 1
∴ = − = r= − =
x 3 4 12 10 20 20
∴x = 12 hours. Choice (D) ∴ R can do it in 20 days. Choice (A)
nd
Triumphant Institute of Management Education Pvt. Ltd. (T.I.M.E.) HO: 95B, 2 Floor, Siddamsetty Complex, Secunderabad – 500 003.
Tel : 040–27898194/95 Fax : 040–27847334 email : info@time4education.com website : www.time4education.com SM1001961/47
1 th 9. Let the number of days taken by Kapil be x.
4. In one day A and B together complete of the work 1
40 Work done by Kapil in one day =
x
8
and in 8 days of the work. 2
40 Work done by Raman in one day =
x
32 2×3 6
Work remaining = . Work done by Sunil in one day = =
40 x x
After C joins them work gets completed in 24 days. Work done by 3 of them together in 30 days.
Let C alone do the work in x days.
30 2 × 30 6 × 30 270
= + + =
 1 1 x x x x
A, B and C can do  +  work in one day.
 40 x  Work done by Sunil and Raman in 18 days
36 108 144
32 th = + =
They do of the work in 24 days. x x x
40
270 144
24 24 32 Total work = + =1
Hence, + = ; 24x + 40(24) = 32x x x
40 x 40
⇒ x = 414
8x = 24 × 40
x
⇒ x = 120 days ∴ Time taken by Raman = i.e. 207 days.
C alone can do the work in 120 days. 2
Choice (D)
Alternate method:
10. Given that the man can complete the job in 150 hours at
As A and B worked for 32 days on the whole, work done by maximum efficiency. i.e. at maximum efficiency he can do
32 4 1
them = = . th of the work per hour.
40 5 150
∴ Remaining work i.e. ⅕th of the work is completed by (He works each 2 hours with the same efficiency)
C in 24 days. ∴ In 8 hours he can do :
∴ C takes 24 x 5 = 120 days to complete the work.  1 1 1 1 
Ans: (120) 2 + + + 
 150 2(150 ) 4(150 ) 8(150 )
5. Akbar can do the work in 40 days. 2 ×15 1 th
The work is completed 10 days earlier. = = of the work.
8 ×150 40
i.e. 40 − 10 = 30 days
Akbar worked on it throughout; i.e., for 30 days. Hence, total number of hours = 8(40) = 320.
Let the number of days Ajay worked be x. Ans: (320)
Ajay does (1/60)th of the work in one day.
11. Let the first man do the work in x days. Everyday a new man
30 x 3 x joins. The work done on successive days is tabulated.
⇒ + = + =1
40 60 4 60
Day Work Work
⇒ x = 15 days (increasing power) (constant power)
Number of days after which Ajay joins Akbar
= 30 − 15 = 15 days. Choice (C) 1 1
1
6. P works for a total of 9 days. Q works for a total of 18 days. x x
R works for a total of 12 days. Let the times taken by Q and R 2 2
to complete the job be q days and r days respectively. 2   2
x x
9 18 12
+ + =1
27 q r 4 3
3   3
q ≤ 54 x x
∴ r ≥ 36 8 4
Only Choice (B) violates this condition. Choice (B) 4   4
x x
7. Let the efficiencies of P, Q and R be 3x gadgets/day,  16  5
4x gadgets/day, 5x gadgets/day. 5   5
(3x) (6) + (4x) (8) + (5x) (10) = 400 x = 4  x  x
Required number of gadgets ∴ The work done in 5 days
= (6x) (6) + (12x) (8) + (5x) (10) = 728 Ans: (728)
1 129
= (1 + 4 + 12 + 32 + 80) = = 1 i.e. x = 129
1 th x x
8. Portion of wall built per day = = 12.5%
8 If each man's efficiency had remained constant, the work
20% of this (i.e. 2.5% of the wall) falls off. n(n + 1) 1
Therefore 10% of the wall is completed every day. done in n days is .
2 x
To determine whether the whole of the 10th day is needed
for work, consider work done in 9 days. n(n + 1) 1
In 9 days 90% of the wall is built. On the 10th day, one-tenth When work is completed, =1
2 x
1 / 10 4
has to be built. So, he takes = = days
1/ 8 5 ⇒ n(n + 1) = 2x = 2(129) = 258
4 ⇒ n2 ≅ 256 or n ≅ 16
So, he takes 9 days to construct the wall. We find that 15(15 + 1) < 258 < 16(16 + 1)
5 ∴ Work is completed on the 16th day. Ans: (16)
Choice (D)
nd
Triumphant Institute of Management Education Pvt. Ltd. (T.I.M.E.) HO: 95B, 2 Floor, Siddamsetty Complex, Secunderabad – 500 003.
Tel : 040–27898194/95 Fax : 040–27847334 email : info@time4education.com website : www.time4education.com SM1001961/48
12. Let the work done by Ram on first day be x, 17. Let the work totally take x days from the start.
∴Second day he does 2x work. Niranjan stopped working after y days and Rajesh stopped
Also as A takes 40% less time than Ram, the ratio of time working y days before the completion of the work. Vinayak
taken by Saleem and Ram is 0.6 : 1 ⇒ 3 : 5. worked throughout.
∴ The ratio of efficiencies of Saleem and Reema = 5 : 3. y x−y x 6 y + 4( x − y ) + 5 x
5x 10x ∴ + + =1 =1
∴ On first day Saleem did and on second day . 10 15 12 60
3 3
9x + 2y = 60
 5x 10 x 
Total work = (x + 2x) +  +  = 8x. ∴The only possible solution such that x > y, is x = 6, y = 3.
 3 3  x and y are integers. Also x > y. Ans: (6)
Combined efficiency of Saleem and Ram, when there is no
change = x + (5x/3) = (8x/3) 6
Number of days required = Work/efficiency = (8x)/(8x/3) = 3 18. In 20 days Krishna does 1 work. In 6 days he does th
20
∴ Saleem and Ram can together do it in 3 days. part of work.
Choice (D)
6
∴ He should get x 720 = `216.
13. From the ratio (2 : 3 : 5), 20
2 6
Vivek does of work in 12 days. Similarly, Rama should get × 720 = `144
10 30
10 Hence Gopi should get 720 − (216 + 144) = `360
⇒ Vivek does full work in 12 × i.e., 60 days.
2
From the ratio 1 : 2 : 3, it can be decided that Rameshwar 360
Daily earnings of Gopi = ` = `60 Choice (D)
does in 30 days. Bhuvan does in 20 days. 6
8 8 8 8 + 16 + 24 4
In 8 days they complete + + = = 19. Ratio of the wages of P, Q, R and total = 3 : 4 : 5 : 12
60 30 20 60 5
∴This is also the ratio of the work completed by P, Q and R.
Choice (C)
∴ P completed 1/4 th of the job. Also the job was
14. In 20 days, Praveen makes 2000 hats. completed in 5 days. ∴ P, working, alone, can complete the
= There are 10% defects job in 20 days. Choice (A)
1800 good hats in 20 days, ⇒ 90 good hats/day → (1)
20. Alok and Sachin were to complete a piece of work in 20 days.
Shiva makes 2000 hats in 10 days
They lost (1/3)rd of the pay. It implies that they could not
There are 20% defects.
complete one-sixth of the work after working for 20 days.
⇒ 1600 non-defective hats; ⇒ 160 good hats/day → (2)
Alok alone can do it in 40 days
Sunny in 5 days
Sunny makes 2000 hats ⇒ Sachin alone can do it in x days.
There are 40% defects
⇒ 1200 non-defective hats; ⇒ 240 good hats/day → (C) 20 20 5 20 5 1 1
⇒ + = ⇒ = − =
Total number of non-defective hats / day = 90 + 160 + 240 = 490 40 x 6 x 6 2 3
Number of days taken to make 10,000 non-defective hats
x = 60 days. Choice (B)
10000 20
= = 20 = 20.4 days. Choice (D)
490 49 21. (i) Part of the job completed in the first two days
1 1 1
15. By 10 : 15 a.m., A would have worked for 15 min and C = + =
30 60 20
would have worked for 5 min
Part of the tank filled by 10 : 15 a.m.  1 
job = 20 
 1   1  25  20 
= 15   – 5   =
 18   36  36 ∴ time taken to complete the job = 20 (2)
11 = 40 days
Remaining part to be filled = Note: The time taken to complete the job would be the
36
Time taken to fill the remaining part same if Ramesh starts the job. Whenever the part of
the job completed by two workers working on alternate
11
days in the first two days
= 36 = .4 min 1
1 1 1 = where n is an integer, the time taken to complete
+ − n
18 26 36
the job = 2n days. Choice (C)
The tank would be full at 10 : 19 : 24 a.m.
Choice (D) 11
(ii) Part of the job completed in the first 2 days =
60
16. Every minute it goes by 3 cm and in the subsequent minute
it is pulled back by 1 cm. 11
So, in a time period of 2 minutes it covers 2 cm. After S cycles of 2 days, part of the job completed = .
12
There can be a total of 19 such time periods as when it
reaches the top, it will not be pulled back. 1
So, in 19 × 2 = 38 minutes, it will cover 38 cm. This is completed in 10 days. Remaining part =
12
2
Remaining 2 cm will be covered in minutes On the 11th day, Rakesh would work. He would
3
5
[∵ it climbs 3 cm in 1 min] complete the remaining part in th of that day.
6
2 5
Hence 38 minutes is the total time taken. ∴ total time = 10 days . Choice (B)
3 6
Choice (C)

Triumphant Institute of Management Education Pvt. Ltd. (T.I.M.E.) HO: 95B, 2nd Floor, Siddamsetty Complex, Secunderabad – 500 003.
Tel : 040–27898194/95 Fax : 040–27847334 email : info@time4education.com website : www.time4education.com SM1001961/49
22. Men hired (M) days worked (D) work completed (W) 26. Let the number of hours taken by the three pipes
individually to fill the pool be x, y and z.
M D W
( x )( y ) 1
2 then = (z)
48 60 x+y 2
5
But y = x + 12 ⇒ x = y − 12 and y = z + 8
3
48 + x 60 ( y − 12) ( y ) 1
5 ⇒z=y−8 = ( y − 8)
( y − 12) + y 2
M α W ( Q D constant) (y2 − 12y)2 = (2y − 12) (y − 8)
M1 W 48 2 5 2x 2y2 − 24y = 2y2 − 28y + 96; ⇒ 4y = 96
= 1 = = ⇒ x = 24
M 2 W 2 48 + x 5 3 3x y = 24 ⇒ x = 12 and z = 16 Choice (C)
Ans: (24)
27. Let the times taken by A, B and C to fill the tank be a hours,
b hours and c hours respectively.
23. Tank is full. Total time to empty it is 60 minutes.
18 3 1  ab 
In 18 minutes the part emptied = = of the tank. C=4  
60 10 2 a+b
At that point, the pipe, that can fill the tank in 30 minutes, is
opened. 1 1 1 1
Taking reciprocals both sides, =  + 
When both are working the tank is filled in 2c 9  a b 
1 1 1
− = i.e., 60 minutes.
30 60 60 9 1 1
= + ––––– (1)
3 2c a b
∴ The tank is filled in x 60 + 18 = 36 minutes.
10 5 1 1
Similarly = + ––––– (2)
Choice (C) 6a b c

24. In one cycle of 6 minutes the part of the tank that is filled 1 1 1 11
+ + =
2 2 2 10 1 a b c 20
= + − = =
15 20 30 60 6 1
Adding to both sides of (1),
In order to decide whether the 6th cycle is needed in full or c
in part, consider the situation after 5 cycles. 11 1 1 1 11 1 1
1 5 = + + = =
5× = th of the tank is filled, in the first five cycles. 2c a b c 20 c 10
6 6 1
5 × 6 = 30 minutes are over. Adding to both sides of (2),
a
1
Part of tank to be filled = th. 11 1 1 1 11
6 = + + =
6a a b c 20
After next 2 minutes, the part of the tank that is empty
1 2 1 1 3
= − = , as pipe P fills (2/15)th part. =
6 15 30 a 10
1 2 1 11  1 1  3
Q can fill this part in 20 x = minutes, = − +  =
30 3 b 20  a c  20
⇒ 6th cycle is not required in full.
2
2 2 ∴ b=6 . Choice (C)
∴ The tank is filled in 30 + 2 + = 32 minutes. 3
3 3
Choice (B) 28. Let the time taken by X and Y to independently fill the tank
be x minutes and y minutes respectively.
3
25. Required time = Time taken to fill the bottom th + time 2 1 2 x
4 x + y = 14 and y + = 16
3 3 3 3
1
taken to fill the top th Solving these, x = 12 and y = 18
4
Part of the tank filled when X and Y work together in a
3
The leak will not affect the filling of the bottom th 1 1 5
4 minute = + =
12 18 36
3
∴ Time taken to fill the bottom part is (time taken by the 36
4 ∴ Time taken to fill the tank = min = 7.2 min
5
1
taps to fill the tank). Time taken to fill the top part is th Ans: (7.2)
4
(time taken by the taps to fill the tank along with the leak) 29. (6m + 8w)10 = (8m + 22w)5
Time taken to fill the bottom part
∴ (6m + 8w)2 = 8m + 22w
3  (16 )(48 )  12m + 16w = 8m + 22w
=   = 9 hours
4  16 + 48  4m = 6w ⇒ 2m = 3w
Part of the tank which can be filled by the taps and the leak ∴ work = (6m + 8w)10
1 1 1  3 
each hour = − = =  6 × w + 8 w  10 = 170 w days
12 24 24
 2 
1 1
∴ Time taken to fill the top th = ( 24 ) = 6 hours New work = 170 × 3 days
4 4
∴ Required time = 15 hours. Ans: (15) 170× 3
34 women will do it in = 15 days. Ans: (15)
34

Triumphant Institute of Management Education Pvt. Ltd. (T.I.M.E.) HO: 95B, 2nd Floor, Siddamsetty Complex, Secunderabad – 500 003.
Tel : 040–27898194/95 Fax : 040–27847334 email : info@time4education.com website : www.time4education.com SM1001961/50
30. The given relation of capacities is as follows: Amount of work done by them together in one day = (1/36)th.
Amount of work done by Raman in one day
Sum of 1 1 1
Value of Value capacities of
Capacity Sum of = − =
of nth capacities 36 72 72
n –1 of n first (n –1)
tap of first n taps ⇒ Raman takes 72 days to complete the work alone.
taps
Ans: (72)
14 15 x 2x 3x
15 16 3x 6x 9x 4. Rate of work and time taken to do a work are in inverse
16 17 9x 18x 27x proportion.
Hence, Kaushik is one and a half times more efficient than
17 18 27x 54x 81x Ravi implies, Ravi takes one and a half time more time than
Kaushik, to do the same work.
⇒ Capacity of the 18th tap = 54x; and this is given 1/2 Kaushik takes 20 days.
(as the 18th tap alone fills in 2 minutes) Hence Ravi takes 20(1 + 1.5) = 50 days to do the same work.
1 1 1 1 In 10 days, amount of work completed by both together
⇒ 54 x = ; ⇒ 2x = × = ; 2x is the capacity of 10 10 70 7
2 2 27 54 = + = = th of the work. Choice (D)
the 15th tap alone. Hence, the 15th tap, by itself will fill in 20 50 100 10
54 minutes. Choice (C)
5. Initially let there be x men who take y days.
Exercise – 6(b) xy = 56 ----------- (1)
(x − 1) (y + 1) = xy ---------- (2)
Solutions for questions 1 to 45: ∴ x − y = 1 ---------- (C)
From (1) and (C), as 56 = 8 × 7,
1 x = 8 and y = 7 Ans (8)
1. In one day Rajdeep can do th of the work and Pranav
15
6. Let Peter and Pan take 8 and 12 days respectively to do
1
can do th of the work. 1 unit work. Let the job mentioned in the problem be x units
24 of work. So, they take 8x days and 12x days to do x units of
 1  10 work respectively. Both together can complete x units of
Work done by Pranav in 10 days = 10   =
 24  24 (8 x ) × (12x )
work in = 4.8x days
14 20 x
Work remaining = Given that 4.8x = 36
24
⇒ x = 7.5
Number of days taken by Rajdeep to complete the
∴ To do x units of work, Pan working alone, takes (12)
14 15 3
remaining work = × =8 Choice (B) (7.5) = 90 days. Choice (D)
24 1 4
7. The total work completed is
th
1  1  1 1 1
2. In one day Raj can build of the wall. 28  +  + +  4 = 1 ⇒ z = 40.
18  z   36 45 z 
1 th Ans: (40)
Kiran can build of the wall.
30 8. Let the time taken by R to complete it be r days
1  1 1 1  1 1
To complete the work Raj takes 9 days. 6 + +  + 12 +  = 1 ⇒ r = 36
2
 24 72 r   72 r 
1
Remaining work = . Required time = (6) (36) = 216 days. Choice (C)
2
In one day both can together complete 9. Parts of the job done by P and Q, Q and R and P and R in
1 1 48 4 1 1 1
+ = = a day are , and respectively
18 30 18 × 30 45 24 20 30
1 1  45  45 ∴P+Q=
1
To complete the work, it takes them  = days. ––––– (1)
2 2  4  8 24
1
45 5 Q+R = ––––– (2)
Total number of days = + 9 =14 days. Choice (B) 20
8 8
1
P+R = ––––– (3)
1 30
3. Amount of work done by Raman and Rajan in 18 days = . Adding (1) and (2) and subtracting (3) from the sum,
2
7
1 2Q =
Amount of work done by Raman and Rajiv in 12 days = . 120
2
7
Rajiv alone can complete the work in 36 days. Q=
12 240
Amount of work done by Rajiv in 12 days = .
36 1
From (1), P =
Let Rajan alone complete the work in x days. 80
12 1
Amount of work done by Rajan in 12 days = . From (2), R =
x 48
12 12 1 240
+ = 6x = 12 × 36; x = 72 ∴ P, Q and R take 80 days, days and 48 days to
36 x 2 7
Rajan alone can complete the work in 72 days. complete the job respectively. Choice (C)
Raman and Rajan complete half the work in 18 days.

Triumphant Institute of Management Education Pvt. Ltd. (T.I.M.E.) HO: 95B, 2nd Floor, Siddamsetty Complex, Secunderabad – 500 003.
Tel : 040–27898194/95 Fax : 040–27847334 email : info@time4education.com website : www.time4education.com SM1001961/51
10. Let the times taken by Anushka, Bhanu and Chawla to 15. 2m = 5w and work = 5w × 15d = 75 women days
complete the job be a days, b days and c days respectively. 4m + 5w = 2 × (2m) + 5w = 2 × (5w) + 5w = 15w
 bc  So, an equivalent of 15 women complete a work of
a = m  75 women days in 5 days. Choice (C)
b+c 
Taking reciprocals both sides 16. Work 1 man can do in 3 days = Work 4 boys can do in 2 days.
1 1  1 1 3 man days = 8 boy days → (1)
=  +  ––––– (1)
a m  b c  Work to be done = (72)(16) boy days

1 1  1 1 3
Similarly, =  +  ––––– (2) = (72) (16)   man days
b m  a c  8
1 1  1 1 = 72 x 6 mandays.
And =  +  ––––– (3)
c m  a b  As work is to be completed in 48 days,
x = (72 × 6)/48 = 9. Ans: (9)
Adding (1), (2) and (3) and simplifying
 1 1 1  2 17. All volumes are in cubic metres,
 a + b + c  1 − m  = 0 Capacity of the tank = 30
  
1 1 1 2  1
As + + ≠ 0, 1 − =0 By 10:30 a.m, volume filled = (2) 1  + 3(1) = 6
a b c m  2
∴ m = 2. Choice (B)
Remaining volume to be filled = 24. This will be filled in
11. Number of man-days required to fill 20,000 tyres = 80 × 84 24
more hours i.e., 2.4 hours.
80 × 84 2+3+5
Number of man-days required to fill one tyre =
20,000 The tank would be full at 12:54 p.m. Choice (D)
Number of man-days required to fill 30,000 tyres
80 × 84 18. Let x boys be required to work,
= × 30,000.
20,000  1 1 1
To complete filling 30,000 tyres in 63 days, number of then x  + + = 1
8 2 4
80 × 84 ×1.5
persons required = = 160. Ans: (160)
63 x+6
= 1, x = 2 Choice (C)
8
12. Each jump followed by a slip would have enabled it to cover
0⋅4m. In 198 jumps, it would have covered (0⋅4) (198) 19. Let the rate of work of each man be x units / day.
= 79⋅2m. In the next jump, it would have come out of the The number of men who left after 56 days (i.e. 24 + 32) is
well. Choice (D) N. The total work is
x (40) (96) = x (40) (24) + x (40 +20) (32)
13. Let a, b and c denote the work done by Ananath, Anand + x (60 – N) (20) ⇒ N = 12 Ans: (12)
and Arjun respectively in one hour.
12a = 15b = 30c 20. Dravid starts the work and completes 1/4th work in 16 days.
1 1 1 Balaji joins and together they complete 1/4th work more.
∴a:b:c= : : =5:4:3 For them to complete 1/4th it takes
12 15 30
Let a = 5k, b = 4k, c = 3k 1/ 4 16
= days.
∴They complete the work in 10 hours, working together. 1/ 64 + (1/ 32) 3
1 1
5k + 4k + 3k = ⇒k= Then Bhangar joins and Dravid, Balaji and Bhangar
10 120
complete another 1/4th of the work. For them to complete
1 1 1 1
∴a=5× = b=4× = 1/4th it takes
1/ 4
=
16
.
120 24 120 30 1/ 64 + 1/ 32 + 1/ 16 7
1 1
c=3× = Then Jadeja joins and all four of them complete the
120 40 remaining (1/4)th work.
∴ Ananath, Anand and Arjun will take 24, 30 and 40 days
respectively. Choice (D) For them to complete 1/4th, it takes

14. Let the work done by Pradeep, Abishek and Antony in one 1/ 4 16
= days.
day be a, b and c respectively. 1/ 64 + 1/ 32 + 1/ 16 + 1/ 8 15
2
Then, a = (b + c) ---------- (1)
3 Total number of days
a + c = 2b ------------ (2)
 1 1 1  24
1 = 16 1 + + +  = 24 days. Choice (D)
c= ------------- (C)  3 7 15  35
45
2
From (1), 3a − 2b = 2c or 3a − 2b = 21. Number of bolts produced by type A in 10 days
45
10
1 = (1500) = 500 Remaining = 1000
and from (2), 2b − a = 30
45
3 1 1 Type B produces 1000 bolts in 10 days.
Adding, we get ∴ 2a = a= and b =
45 30 36 ⇒ 3000 bolts are produced in 30 days by type B.
So, Pradeep takes 30 days and Abishek takes 36 days. Together they can produce (3000 + 1500)
Choice (A) = 4500 bolts in 30 days. Ans: (4500)

Triumphant Institute of Management Education Pvt. Ltd. (T.I.M.E.) HO: 95B, 2nd Floor, Siddamsetty Complex, Secunderabad – 500 003.
Tel : 040–27898194/95 Fax : 040–27847334 email : info@time4education.com website : www.time4education.com SM1001961/52
th 3
1 Hence a total of 6 days.
22. In one day, Machine P does of the work.
20 4
1
th By a similar calculation, if Mukesh starts, a total of 7 days
Machine Q does of the work 1
30 are required. Difference = days. Choice (B)
th 4
1
Machine R does of the work
60 27. Each cycle consists of 5 days, as they take one day off. For
Together in 5 days they complete every 4 days of work. Hence work done in one cycle of 5 days
5 (1+ 2 + 3 )
5
+
5
+ = (5) = 1 portion of the work. =
1
+
1
+
1
+
1
+0 = 2
 1
 +
1 

20 30 60 60 2
24 36 24 36  24 36 
3 3 3+6 9
In 3 days Q and R complete + = =
30 60 60 60 2×5 5
= = th
30 9 21 72 36
Remaining = 1 − − =
60 60 60 36
Full cycles required = Integer part of = 7 cycles.
5 21 36 3 5
Portion of work done by P = + = =
20 60 60 5 7 cycles = 7 × 5 = 35 days → (1)
Choice (D) Work remaining to be done on the 36th day
 5  1
23. Let the work done by Rohit on the first day be 1 unit = 1 –  7 ×  = th → (2)
Job = 1 + 2 (2) + 3 (4) + 4 (8) + 5 (16) + 6 (32) = 312 units.  36  36
If the men worked at twice the rate, part of the job 36th day is the first day of 8th cycle.
completed in the first 4 days = 1 + 2 (3) + 3 (9) + 4 (27) All even numbered cycles are begun by Bharat.
=142 units. On the 5th day, 5 (81) Hence, Bharat comes to do work on the 36th day.
= 405 units can be done. Bharat’s rate of work is 1/36th → (3)
∴ the job will be completed on the 5th day Choice (B) (2) and (3) imply, Bharat completes the work on the
36th day. Choice (B)
24. Let us say the job is completed in x days.
On the ith day of work, (25 + − 1) = 24 + i persons would 28. Suppose the man can complete 1 unit 1hr if he works at his
work. maximum efficiency. The job (1) (151) = 151 units.
∴ (24 + i) m2 would be the work done on the ith day
In the first 8 hours, he would complete
∑ (24 + i) = 330
x
 1 1 1  15
i =1 21 + + +  = th of the part. In order to complete
24 + 1 + 24 + 2 + ……….24 + x = 330  2 4 8 4
x (x + 1) 150 units, he requires 40 cycles of 8 hours i.e., 320 hours.
24 x + = 330 In order to complete the last unit, he requires another hour.
2 ∴ total time = 321 hours. Choice (D)
x (49 + x ) (11)(49 + 11)
=
2 2 29. Work done on
Comparing both sides, x = 11. Ans: (11) 1
1st day → Sonia →
20
25. Let the works done by each worker in group A and each
worker in a group B in a day be a units and b units 1 1
respectively. 2nd day → Sonia and Priyanka → +
20 30
1
Given 3a + 6b = ––––– (1)
20 1 1 1
3rd day → Sonia, Priyanka and Anjali → + +
1 20 30 30
8a + 4b = ––––– (2)
10 3 3 3×5 1
1 1 So in a period of 3 days + = =
Solving (1) and (2) we get a = and b = 20 30 60 4
90 300
So, 4 such time periods or 12 days are required to
1 complete the work. Choice (A)
a+b=
72
∴ One worker from each group take 72 days to complete it. 30. (i) The man would work at 1 unit / day. Men joining on the
Choice (C) 2nd day, 3rd day and 4th day would work at 2 units /
26. Case 1 Anil starts day, 3 units / day and 4 units / day respectively.
1 Job = (1) + (1 + 2) + (1 + 2 + 3) + (1 + 2 + 3 + 4)
Anil 2 days = 20 units.
6
1 20
Mukesh Required time = = 20 days. Choice (A)
8 1
7
In a period of 2 days work is done (ii) Work done on the first day = 1 + 2 + 3 + 4 = 10 units
24 work done on the second day = 2 + 3 + 4 = 9 units
21 work done on the third day = 3 + 4 = 7 units work done
In 3 such time periods i.e. 6 days on the fourth day = 4 units.
24
Job = 10 + 9 + 7 + 4 = 30 units.
7 1 Last man who leaves completes (D) (D) = 16 units.
i.e. work is done.Remaining work is and it is done by
8 8
8
6 3 ∴ He will receive th of the total wages.
Anil in = . 15
8 4 Choice (C)

Triumphant Institute of Management Education Pvt. Ltd. (T.I.M.E.) HO: 95B, 2nd Floor, Siddamsetty Complex, Secunderabad – 500 003.
Tel : 040–27898194/95 Fax : 040–27847334 email : info@time4education.com website : www.time4education.com SM1001961/53
31. The data provides three different combinations of working, 35. Ratio of work done by Gokul, Govardhan and Ganesh in
to do the work completely. equal durations of time
(a) Combination 1 : – 1 1 1
All three people working for (x + 6) days. = : : respectively i.e., 6 : 3 : 2
10 20 30
 1 1 1 
Hence, (x + 6)  + +  = 1; ⇒ x + 6 = 10, 3
 20 30 60  ⇒ Govardhan gets (6600) = `1,800. Choice (A)
11
⇒ x = 4 → (1)
36. Number of days required:
(b) Combination 2 : –
All three people for x days, and two people for (y + 6) days. 720
by a man = = 12
 x   1 1  60
Hence x   + ( y + 6)  + 

 10   30 60  810 1080
= 1; Substituting x = 4, the equation becomes (y + 6) by a woman = = 18; and by a child = = 36
45 30
 3  4 6 If 1 man, 1 woman and 1 child work together,
  = 1 – = , y = 6 → (2)
 60  10 10 1 1 1 1
+ + work will be done in 1 day = th
(c) Combination 3: – 12 18 36 6
All three for x days, two people for y days and one ∴ 6 days will be required.
person for z days (say). Cost = 6 × (60 + 45 + 30) = `810. Ans: (810)
 1   3  z
Hence, x   + y   + =1 37. Let x be the number of men initially engaged.
 
10  20  60 Let d be the number of days for which they worked. Hence,
Substituting x = 4, y = 12, and simplifying, x men completed half of the work in d days (1)
z = 18 → (C) Because 2x men joined, total number of working men
Total number of days required under combination 3, is = x + 2x = 3x.
x + y + z = 4 + 6 + 18 = 28 days. This new group completes the remaining work 6 days
earlier; i.e., 3x men complete half of the work in (d – 6)
Alternate method:
days --------- (2)
Prakash, Pranay and Pramod together complete
th
As the product of men and number of days is the same in
 1 1 1  1 both cases, being equal to work done,
 + +  of work per day or, th of the work per
 20 30 60  10 (x) (d) = 3x (d – 6);
⇒ d = 3 (d – 6), 2d = 18 -------- (C)
day. Together, the three take (x + 6) days to complete.
But, 2d is the duration required by x men to complete the work.
x + 6 = 10,
Choice (A)
⇒ x = 4 → (1)
The work remaining after the departure of Prakash can be 38. Let the rate at which each large pump fills the tank be
completed by Pranay and Pramod in (y + 6) days. l litres / hr.
 1 1  x
∴ (y + 6)  +  = (1 – 10 )
2
Rate at which each small pump fills the tank = l litres/ hr.
 30 60  3
⇒ (y + 6) 3/60 = [1 – (4/10)], ⇒ y = 6 → (2) Ans: (28) Rate at which six small pumps and three large pumps fill
2 
32. Let the required cost be `x the tank = 6  l  + 3l = 7l litres / hr.
10 cows = 20 sheep 3 
∴ 10 cows + 40 sheep = 60 sheep 1
20 cows + 10 sheep = 50 sheep ∴ They will take th the time taken by a single large
Let the cost of grass for each sheep be `y/day. 7
(60y) (20) = 900 pump to fill it. Choice (B)
3
y= 39. Pipe A can fill the tank in 20 hours. Pipe B in 30 hours.
4 Total time for both A and B together to fill the tank
x = (50y) (18) = 675. Choice (B) 20 × (30 )
= = 12 hours.
30 + 20
33. The cost of grass for 20 cows and 30 sheep for 30 days
= `720. But it took 3 hours more i.e. 15 hours in total.
Given 30 sheep eat double the grass eaten by 20 cows. Let the number of hours in which the leak can empty the
⇒ Cost of grass eaten by 30 sheep in 30 days tank be x.
2 1 1 1 60
= × 720 = `480 ⇒ = − ⇒x= = 60 hours. Ans: (60)
3 x 12 15 5−4
The cost of grass eaten by 1 sheep in one day
480 8 40. The time from 10:00 a.m. to 6:00 p.m. = 8 hours.
= = .
30 × 30 15 So, one of the pipes worked for 8 hours and the other pipe
worked for x hours.
The cost of grass eaten by 20 sheep in 15 days
8 × 20 × 15 8 x
= = `160 Ans: (160) + = 1 --------------- (1)
15 12 18
x 8
13500 or + = 1 ----------- (2)
34. Money received for one day’s work = = 300 12 18
45
3 We get x = 6 from equation (1)
Amount of money received by Gagan = × 300 So, pipe Q was closed for 2 hours.
4 or
= 225 per one day.  8  20
For 10 days, Gagan receives 225 × 10 = `2,250. From Eqn (2), x = 121 − =
Choice (B)  18  3

Triumphant Institute of Management Education Pvt. Ltd. (T.I.M.E.) HO: 95B, 2nd Floor, Siddamsetty Complex, Secunderabad – 500 003.
Tel : 040–27898194/95 Fax : 040–27847334 email : info@time4education.com website : www.time4education.com SM1001961/54
 20  4 45. To fill the tank, x takes 18 hours, y takes 24 hours and z
So, pipe P was closed for 8 − = hrs
3  3
takes 36 hours. If opened simultaneously, together they

1 1 1 1
Minimum possible time = 11/3 hours. can fill the tank in a hours, here = + +
a 18 24 36
Alternate method: 1 4+3+ 2
= a = 8 hours
a 72
Part of the tank filled, in 1 hour, by the two pipes Time taken to fill the tank as per the schedule of filling is
1
+
1
=
5 (2 × 8) – (9/2) = (23/2) hours.
= th part.
12 18 36 Hence, x worked for 23/2 hours; y worked for [(23/2) – t]
hours and z worked for [(23/2) – 2t] hours.
36
Hence, time required to fill = = 7.2 hour. Hence, the equation of work is :
5
 23 1   23   1   23   1 
Actual time taken to fill = 6:00 p.m. – 10:00 a.m.  2 × 18  +  2 − t   24  +  2 − 2t   36  = 1
= 8 hours.          
Extra time taken 8 – 7.2 = 0.8 hours 23  1 1 1   t 2t 
⇒  + + –  +  =1
2  18 24 36   24 36 
Extra time was needed because one pipe was closed for
some time.
Hence, part filled by 2 pipes in 0.8 hours = Part not filled  23 1  7t 23 7t
due to closure of one tap ⇒  ×  − = 1, ⇒ − 1=
 2 8  72 16 72
5 1
i.e. Part not filled = 0.8 × = th part. ⇒ t = 4.5 hours. Choice (D)
36 9
Minimum time of closure is to be found out; Solutions for questions 46 to 55:
⇒ The faster pipe was closed.
Rate of filling of the faster pipe = 1/12th / hour. 46. From statement Ι,
Hence duration of closure = (1/9)/(1/12) = (12/9) 3 3 15
= (4/3) hours. Choice (D) 1 boy = man ⇒ 5 boys = × 5 men = men
4 4 4
41. Let the rate of filling of the first tap be x litres/hr 10 men take 6 hours to do the work. So 5 boys, which is
Let the rate of filling of the nth tap be denoted by tn 15 10 × 6
equal to men, can take days to do the work.
t2 = x 4 15 / 4
t3 = 2x ∴Statement Ι alone is sufficient.
t4 = 4x = 22x
t5 = 8x = 23x From statement ΙΙ,
∴ tn = 2n − 2x 10 men can complete the work in 6 hours. So, 5 men can
t 6 23 x 1 complete the work in 12 hours.
= = The amount of work done by 5 boys in one hour
t 9 26 x 8
1 1 1
= − =
∴ time taken by the ninth tap =
1
(80) = 10 min utes 10 12 60
8
So 5 boys can do the work in 60 hours.
Ans: (10) Statement ΙΙ alone is also sufficient. Choice (B)

3 47. Either of the statements alone is not sufficient as


42. To fill th part, the filling pipe takes 3 hours. information about Ι and ΙΙ is given in different statements.
4
Combining statements Ι and ΙΙ, the time taken to fill the
3
To empty th part, the emptying pipe takes 1
4 tank is = 6 hrs Choice (C)
2 hours. (1/ 2) − (1 / 3)
∴ When both are operated simultaneously, they will empty
3 1 1 1 48. From statement Ι, work done by the pipes together, in the
th part of the full tank in − = first 2 minutes = 1/6.
4 2 3 6
⇒ Work done in one minute = 1/12
i.e., 6 hours. Choice (D) So the time taken to fill the tank when they are opened
alternately = 12 minutes.
43. The taps can fill the tank in
(12)(36) = 9 hours So statement Ι alone is sufficient.
12 + 36 From statement ΙΙ, we do not know the time taken by the
5 1 other pipe to fill the tank. So we can’t answer the question.
But it this time only th are full. The balance th will be Choice (A)
6 6
 1/ 6  49. From statement Ι, it is not known what part of the total work
filled in 9   or 1.8 hours. Ans: (1.8)
5/6 is completed, so we can’t answer the question.

44. If the pipe alone is operated, the tank would take From statement ΙΙ, B and C did 8/23 of the work.
200 ∴ A did 1 – 8/23 = 15/23 of the work.
= 40 hours to get filled. Because of the leak, it took ∴ He gets 15/23 of 529.
5
∴ Statement ΙΙ alone is sufficient. Choice (A)
80 hours longer, i.e. 120 hours.
120 50. From statement Ι, the question cannot be answered as
i.e., = 3 tanks were filled or 2 tanks extra were filled.
40 there is no information about volume of water emptied.
This is because of the leakage.
So, 2 tanks or 400 litres were leaked in 120 hours. From statement ΙΙ, as there is no information about tank,
we cannot answer the question.
400 Combining both the statements, as the tub holds 12m3 of
So, rate of leaking = = 3.33 litres/hour.
120 water, we can say that the tank has a capacity of 12m3 and
Choice (C) was emptied in 30 minutes. Choice (C)

Triumphant Institute of Management Education Pvt. Ltd. (T.I.M.E.) HO: 95B, 2nd Floor, Siddamsetty Complex, Secunderabad – 500 003.
Tel : 040–27898194/95 Fax : 040–27847334 email : info@time4education.com website : www.time4education.com SM1001961/55
51. Let the times taken by each person of P and each person Using statement ΙΙ, Rohan and Sohan take less time to
of Q to complete the job be p days and q days respectively. complete the job when compared to Mohan and Sohan.
∴ Rohan is more efficient than Mohan. More information is
Using statement Ι, required about the relation between the efficiencies of
Rohan / Mohan and Sohan.
 1  1 1 5
2   + 3  =
 q  12 = 12 ΙΙ is not sufficient
p   Using both statements, Mohan is the least efficient. Both
5 statements are required to answer the question.
 1 1 1 5 Choice (C)
2  +  + =
 p q  q 12 55. Let the times taken by A, B and C to fill the tank be a
minutes, b minutes and c minutes respectively.
1 1 5 1
+ = − 1 1 1 1
p q 24 2q + − = ------------ (1)
a b c 6


1 1
+ <
5 Using statement Ι,
p q 24 1 1 1 1
+ + = ------------ (2)
24 a b c 2
∴Time taken > days.
5 1  1 1 1
(2) – (1) ⇒ −−  = − .
Ι is not sufficient. c  c  2 6
Using statement ΙΙ, c can be found.
Required time can then be found.
 1   1  35 Ι is sufficient.
3   + 2   =
 p   q  68
Using statement ΙΙ,
 1 1  1 35 1 1 35 1 1 1 1 1
3 +  − = ⇒ + = + + − = ------------ (3)
 p q  q 68 p q 192 3q b c a 6
1 1 35 From (1) and (C),
+ >
p q 192 1 1 1 1 1 1
+ − = + −
a b c b c a
192
∴ Time taken < days i.e., less than 6 days.
35 1 1
=
∴ ΙΙ is sufficient Choice (A) a c
a is unknown
52. Whenever the time taken by two men to complete a job ∴ c cannot be found
when they work together is an integral number of days, the ∴ Required time cannot be found.
time taken by them to complete it if they work on alternate ΙΙ is not sufficient. Choice (A)
days would be twice of that. In the given problem, from
statement Ι, time taken by both to complete the job is an Chapter – 7
integral number of days. (Averages, Mixtures and Alligations)
∴ Required time = 2 (20)
= 40 days, Ι is sufficient Concept Review Questions
Using ΙΙ, the time taken when they work together Solutions for questions 1 to 35:
30 × 60
= = 20 1. Total of the numbers = 513
30 + 60
513
∴ Required time = 2 (20) = 40 days. Average = = 57 Ans: (57)
∴ ΙΙ is sufficient. Choice (B) 9

1 1 1 2. Average = 22
53. Given, = +
t t +a t +b 17 + 36 + x
⇒ = 22 ⇒ x = 23 Choice (D)
( t + b) + ( t + a) 1 3
⇒ =
( t + a) ( t + b) t
3. As each number is doubled, the average will also be
⇒ t [(t + b) + (t + a)] = (t + a) (t + b) doubled.
2 t2 + (a + b) t = t2 + (a + b) t + ab ∴ The new average = 2x Choice (B)

t= ab 4. A 50% decrease in each number result in each number


being halved. As each number is halved, the average will
Using statement Ι, ab is unknown. also be halved.
∴ t cannot be found
b
Ι is not sufficient. ∴ The new average = Choice (C)
ΙΙ is sufficient. Choice (A) 2

54. Using statement Ι, Rohan and Sohan take less time to 21 + 22 + 23 + .... + 30
5. Average = = 25.5
complete the job when compared to Rohan and Mohan. 10
∴ Sohan is more efficient than Mohan. More information is
required about the relation between the efficiencies of first + last 21 + 30
or average = = = 25.5
Sohan / Mohan and Rohan. 2 2
Ι is not sufficient. Ans: (25.5)

Triumphant Institute of Management Education Pvt. Ltd. (T.I.M.E.) HO: 95B, 2nd Floor, Siddamsetty Complex, Secunderabad – 500 003.
Tel : 040–27898194/95 Fax : 040–27847334 email : info@time4education.com website : www.time4education.com SM1001961/56
6. The two digit numbers divisible by 10 are 10, 20, 30, 40, 18. Average salary per month
50, 60, 70, 80 and 90. 4 ( 6000) + 4 (8000 ) + 4 (13000)
Sum of the two digit numbers divisible by 10 =
12
= 10 + 20 + 30 + 40 + 50 + 60 + 70 + 80 + 90 = 450
450 6000 + 8000 + 13000
∴required average = = 50 Choice (B) = = `9000 Ans: (9000)
9 3

7. There are 25 odd numbers less than 50 19. Let the numbers of male employees and female employees
1 + 49 be m and f respectively.
Average = = 25 Choice (D) 180 m + 170 f
2 Average height of the employees =
m+f
8. The last multiple of 12 less than 100 is (12) m
Average of all the multiples of 12 180 + 170
12 (1 + 2 + 3 + .... + 8 ) = f . As m/f is unknown, the average height
= m
8 +1
f
(8) (9) cannot be found. Choice (D)
12
= 2 = 54 Choice (B)
8 20. There are 60 girls in the class.
Total marks of the boys = (30) (75) = 2250.
9. Total of the ages of 15 students = (15) (15) = 225 years Total mark of the girls = (60) (90) = 5400.
Total of the ages of 16 students = 225 + 15 = 240 years Total marks of the class = 7650.
240 7650
New average age of the class = = 15 years. Average marks of the class in the test = = 85
90
16
Ans: (15) Ans: (85)

10. Total of the ages of 20 students = (20) (20) = 400 years. 21. There are at least 30 boys in the class. ∴ There are more
Total of the ages of 19 students = 400 – 20 = 380 years. boys than girls in the class.
380 If the average weight of the entire class is w, 30 < w < 40
New average age of the class = = 20 years. and there are more boys than girls w is closer to 40 than
19
30 i.e. w > 35.
Choice (A)
Only Choice (D) satisfies this condition. Choice (D)
11. After 5 years, the age of each family member will be
5 years more. 22. Let the number of girls in the class be g.
∴ The average age of the family will also be 5 years more 150 3
i.e. 30 years. Choice (C) Number of boys in the class = g = g.
100 2
12. As the present ages of all the friends is at least  3 
thirty five years, all of them are born before thirty years. Total of the ages of the boys = 10  g . = 15 g years.
∴Present average = 65  2 
Let their be x friends then sum of their present ages = 64x Total of the ages of the girls = 8g years.
Sum of the their ages before thirty years = 65x – 30 × x Total of the ages of the students = 23 g years.
Average age thirty years ago = 35x / x = 35 Average age of the class
Choice (A)
23 g
= = 9.2 years. Ans: (9.2)
13. Average monthly income of Amar, Bhavan and Chetan in 3
that month g+ g
2
4000 + 4500 + 6500
= = `5000 Choice (A)
3 23. As the ratio of heights of boys and girls is 4 : 3, let their
actual heights be 4k and 3k respectively.
14. Total marks in 6 tests = (6) (65) = 390 The average height of the boys and girls =
Total marks in the first 5 tests = (5) (60) = 300
4k × 50 + 3k × 30
Mark in the sixth test = 390 – 300 = 90. Ans: (90) = 3.625k.
50 + 30
15. Let the age of teacher be t years then, 20 × 15 + t = 18 × 21
t = 78 years Choice (C) Hence the exact average cannot be found. Choice (D)

16. Let there be n students in the group then, 24. Let the numbers of boys and girls in the class be b and g
respectively. Let the average weights of the boys and the
n × 16 + 66 = (n + 1) 17 ⇒ n = 49 Ans: (49)
girls be B and G respectively.
17. Let the number of girls be g. Bb + Gg B +G
Number of boys = 8 – g =
b+g 2
Method 1:
2Bb + 2Gg
5 (13 – g) + 3g = 55 5 = g
= Bb + Bg + Gb + Gg
Method 2: B (b – g) + G (g – b) = 0
Average number of chocolates received by each child (B – G) (b – g) = 0
55 B = G or b = g or both. Choice (C)
=
13
25. Quantity of milk in 20 litres = (0.6) (20) = 12 litres.
55 Upon addition of 5 litres of pure milk, the new solution of
5−
g 13 5 25 litres would contain 17 litres as milk.
= =
13 − g 55 8
−3 17
13 ∴ Percentage of milk = (100) = 68% Ans: (68)
∴g=5 Choice (D) 25

Triumphant Institute of Management Education Pvt. Ltd. (T.I.M.E.) HO: 95B, 2nd Floor, Siddamsetty Complex, Secunderabad – 500 003.
Tel : 040–27898194/95 Fax : 040–27847334 email : info@time4education.com website : www.time4education.com SM1001961/57
26. Method 2:
80 108
5
x − x
96 y 8 = 3
=
15 5 5
12 16 x−0
8
y=9 Choice (A)
Ratio = 3 : 4 Choice (C)
2
27. As we don’t know the percentage of profit at which he sold  90 − 9  81
34. Ratio of milk and present mixture =   =
the rice, we cannot find the average cost price. (Whenever  90  100
we use alligation rule, all the prices should be in either
cost terms or sales terms). Hence required ratio is cannot ∴ Ratio of milk and water = 81 : 19 Choice (C)
be determined. Choice (D)
81
35. Quantity of milk in the present mixture = (90)
28. Cost price of 5 kg. = (5) (15) = `75 100
Cost price of 10 kg. = (10) (12) = `120
Cost price of the mixture = 72.9 litres. Ans: (72.9)
75 + 120
= = `13/kg. Exercise – 7(a)
15
20 Solutions for questions 1 to 30:
Profit = (13) = ` 2.60/kg
100
Selling price of the mixture = `15.60/kg. Ans: (15.60) 1. Average of the first 20 multiples of 50
50(1 + 2 + ...20 )  21 
29. Pure water has no milk i.e., has 0% milk = = 50   = 525 Choice (C)
20  2 
0% 84%
2. Let the 11 distinct positive integers be represented by
a, b, c, . . . . . . k.
48% a + b + c + d + e + f + g + h + i + j + k = 21 (11)
= 231 ------- (1)
36% 48% a + b + c + d + e + f = 23 (6) = 138 ------- (2)
f + g + h + i + j + k = 22(6) = 132 -------- (3)
=3:4 Choice (D) Adding equations (2) and (3) and subtracting equation (1),
we get, f = 39. Ans: (39)
75
30. Quantity of milk in A = (20) =15 litres. 3. Temperatures on Sunday, Monday and Tuesday were
100
28, 30, 32. (all values in degrees celsius).
60 Temperature on Tuesday was 1 degree less than that on
Quantity of milk in B = x litres.
100 Wednesday.
60 66 Hence, temperature on Wednesday was 32 + 1 = 33
15 + x= (20 + x) Using the same logic, the temperatures on Thursday,
100 100 Friday and Saturday were : 34, 35, 36 respectively.
15 + 0.6x = 13.2 + 0.66x ⇒ x = 30 Ans: (30) Average of first four days = (28 + 30 + 32 + 33)/4
= 123/4 = 30.75 -------- (1)
31. Let the quantity of water to be added be x litres. Average of the last three days
Quantity of water in the vessel = 20% of 30 = 6 litres. = (34 + 35 + 36)/3 = 35 ---------- (2)
6 + x 80 % 4 Required difference = 35 – 30.75 = 4.25 Choice (B)
= =
30 + x 100 % 5
4. Let the present average age of ten advisors as well as their
30 + 5x = 120 + 4x ⇒ x = 90 Choice (C) average 3 years ago, be A. Also let the age of the younger
man be Y and that of the director replaced by him be D.
32. Let the quantities of the solutions be 2x litres and 3x litres.
Quantity of sulphuric acid in the first solution 10 A + 30 − D + Y
= A , as the present average is same as
10 10
= (2x) = 0.2x litres.
100 the earlier average.
Quantity of sulphuric acid in the second solution Hence D – Y = 30 years Choice (D)
20
= (3x) = 0.6x litres. 5. Four statements are given in the data. They can be
100 represented as the 4 equations given below. BCD
Concentration of sulphuric acid in the final mixture represents the total weight of B, C, D.
0.2 x + 0.6 x (100 ) A + BCD = 240
= = 16% Choice (A)
2x + 3 x A + BCD + E = 285
BCD + E + F = 305
33. Let the cost price of the milk be `x/litres. E = 45, and F = 50.
Let the quantity of water to be added to the milk be y litres. BCD = 210
Selling price of the mixture = ` x/litre. Hence A = 30
Weight of A = Total weight of A, BCD, E – Total weight
x (100 ) 5
Cost price of the mixture = = ` x/litre. BCD, E = 30 kg. Ans: (30)
100 + 60 8
Cost of water = `0/litre 6. an = y + k ……..→ (1) bn = x + k ……..→ (2), where k is
the sum of the (x – 1) numbers which are other than y and x.
Method 1: Subtracting equation (2) from equation (1)
5
∴ 15x + 0 = x (15 + y) 9 = y 1 a−b
8 n(a – b) = y – x ⇒ = Choice (B)
n y−x

Triumphant Institute of Management Education Pvt. Ltd. (T.I.M.E.) HO: 95B, 2nd Floor, Siddamsetty Complex, Secunderabad – 500 003.
Tel : 040–27898194/95 Fax : 040–27847334 email : info@time4education.com website : www.time4education.com SM1001961/58
36N + ( 20 )(30 ) 36N − (5)( 40 ) 13. Let the average of classes Ι and ΙΙ be A kg and B kg.
7. = ⇒ N = 10 Choice (A) Let the number of students in the classes Ι and ΙΙ be a and
N + 20 N−5
b respectively.
8. Let the number of students in X, Y and Z be x, y and z Aa + Bb A + B
=
respectively. a+b 2
x 81 − 76 5 2Aa + 2Bb = Aa + Bb + Ab + Ba
Applying the rule of alligation, = = (a – b) (A – B) = 0
y 76 − 70 6
As a ≠ b, A = B ------------ (1)
y 90 − 86 4
and = = x : y : z = 10 : 12 : 15 and 2 A + B = 60 ------------ (2)
z 86 − 81 5 solving (1) and (2), we get A = B = 20. Ans: (20)
Let x = 10a, y = 12a and z = 15a
Average marks of X, Y and Z 14. Let a = 10 p + q and b = 10 r + s
70(10a ) + 81(12a) + 90(15a ) 3020 25 Let the sum of all the numbers excluding a and b be x. Let
= = = 81 ≈ 82 the average of the numbers be A.
10a + 12a + 15a 37 37
x + 10 p + q + 10 q + p = 10 A ------------ (1)
Ans: (82) x + 10 r + s + 10s + r = 10 (A + 2.2) ------------ (2)
Subtracting (1) from (2),
9. Let the number of students in the three classes be nA, nB 11 [(r + s) – (p + q)] = 22
and nC and the total scores of students in the three classes
⇒ (r + s) – (p + q) = 2 Choice (B)
be TA, TB and TC
TA + TB = 71(nA + nB)
15. As each student missed a different number, the sum of the
TB + TC = 76 (nB + nC)
all the numbers added by the students = (N – 1) (sum of the
TC + TA = 79 (nC + nA)
--------------------------------- (N − 1)N(N + 1)
first N natural numbers) =
2TA + 2TB + 2TC = 150nA + 147 nB + 155nC 2
⇒ TA + TB + TC = 75 nA + 73.5nB + 77.5nC The number of numbers added by the students in total
T + TB + TC = (N - 1) (N)
Hence p = A (N − 1)N(N + 1)
n A + nB + nC ∴ Average = = 21 ⇒ N = 41
73.5(n A + nB + nC ) 1.5n A + 4n C 2(N − 1)(N)
= + Choice (C)
n A + nB + n C n A + nB + n C
⇒ p > 73.5 -------- (1) 16. Alloy A Alloy B
p can also be written as, Copper 4x 5y
T + TB + TC Zinc 9x 6y
p= A ------- ------
n A + nB + nC
Total wt. 13x 11y
77.5(n A + nB + nc ) 2.5n A + 4.nB
= − ------- ------
n A + nB + n c n A + nB + nC As the quantity to be drawn from the Alloys A and B to form
⇒ p < 77.5 -------- (2) another alloy C must be a multiple of 13 and 11, let us
Hence 73.5 < p < 77.5 Choice (D) choose the quantity drawn as LCM of 13 and 11; which is
143. Hence 13x = 11y = 143 ⇒ x = 11 and y = 13.
10. Let the total number of matches that Sachin Tendulkar Amount of copper in Alloy C = 4x + 5y
played be x. = 4(11) + 5(13) = 109 kg.
2100 2201 Amount of zinc in alloy C = 9x + 6y
+1= = 9(11) + 6(13) = 99 + 78 = 177 kg.
x −1 x Hence the ratio of copper and zinc = 109 : 177
Among the choices, choices (A) and (C) satisfy the Choice (C)
equation above. Choice (D)
17. Cost price of a mixture of two varieties of wheat
11. Average of all the four classes = (32 × 83 × 58 × 76 + 82 × 100
85 + 48 × 90) × 1/220 = 83.4 (approximately) = x 28 = Rs.20
100 + 40
Alternate method: Let the cost price of the first and the second varieties of
Let us assume average mark of all the four classes is 83. wheat be `y per kg and `(y + 3) per kg respectively.
Average deviation of all the four classes combined Let the quantities be 3x kg and 4x kg respectively.
y(3 x ) + (y + 3 )(4 x )
× (83 − 83 )+ × (76 − 83 ) + × (85 − 83 ) +
32 58 82 Cost price of the mixture =
is: = 3x + 4x
220 220 220
7 xy + 12 x 12
× (90 − 83 ) = 0.4 (approximately) =y+ = 20
48 =
220 7x 7
Hence, correct average = 83 + .4 = 83.4 Choice (D) Cost price of the first variety of wheat
12 128
= 20 − =` /kg
12. Total of all numbers = 16n 7 7
Once 5/8th of the numbers are doubled and 3/8th of the Alternate method:
numbers are increased by a factor of 10/3, total of all A mixture of two varieties of wheat is sold at `28 per kg, at
numbers =
5n
(2A ) + 3n  10B  + 16n, where A and B are a profit of 40%.
Hence, the cost price of the mixture = (28)/(1.4) = `20
8 8  3 
Let x and (x + 3) be the prices in rupees of the first and the
the averages of values of the numbers which are
second varieties respectively.
respectively doubled and increased by a factor of 10/3.
The quantities are mixed in the ratio 3 : 4 (given)
5n 5n
A+ B + 16n By alligation equation,
New average = 4 4 . q1 p −p 3 ( x + 3) − 20
n = 2 ; ⇒ =
q2 p − p1 4 20 − x
As A and B are not known, the new average cannot be
determined. Hence the percentage increase in the average 3 x − 17
⇒ = ; ⇒ x = 128/7 Choice (A)
cannot be determined. Choice (D) 4 20 − x
Triumphant Institute of Management Education Pvt. Ltd. (T.I.M.E.) HO: 95B, 2nd Floor, Siddamsetty Complex, Secunderabad – 500 003.
Tel : 040–27898194/95 Fax : 040–27847334 email : info@time4education.com website : www.time4education.com SM1001961/59
18. Cost price of each litre of diluted milk If q1 and q2 are the quantities of mixture 1 and mixture
2 that are mixed, then by alligation equation,
=
100
(4) = ` 16 q1 ( 4 / 5) − (1 / 2) (3 / 10) = 9
100 + 25 5 = =
By the principle of allegation, q2 (1 / 2) − (1 / 3 ) (1/ 6) 5

Amount of water used for mixing It is given that, q1 + q2 = 28;


Amount of milk used for mixing 9
Hence q1 = x 28 = 18 litres Ans: (18)
14
16 4
4−
5 5 1
= = = 3 4
16 16 4 24. In the first, second and third mixtures, th, th and
−0 5 5
5 5
Hence 0.25 litres of water will be mixed with each litre of milk. 7
th of the contents respectively is alcohol. Let x ml of each
Choice (A) 10
of these be drawn and mixed. The resulting mixture of 3x ml
19. Initial quantity of milk in the vessel = 0.2 (10) = 2 ml 3 4 7  21
Initial quantity of water in the vessel = 0.8 (10) = 8 ml will have  x + x + x  ml of alcohol i.e. x ml of
After x ml of milk was added, the ratio of milk and water 5 5 10  10
would become 4 : 1 alcohol ∴ It will have 70% alcohol. Choice (B)
x = 4 (8) – 2 = 30
After y ml of water was added, ratio of milk and water would 25. Let us say we have x kgs of fresh dates and y kg of dry
again become 1 : 4. dates formed from them.
y = 4 (4(8)) – 8 = 120 Amount of dry pulp = 10x/100 = 72y/100
∴ x + y = 150 Ans: (150) y =10x/72 = 5x/36
⇒ when x = 36 kg, y = 5 kg Ans: (5)
20. Let the number of Group A employees be m. The number
of Group B employees = m – 10 26. Quantity of milk present in the solution now
Applying the alligation rule, 4
 700 − 70 
=   x700
m − 10 4800 − 4000  700 
= = 0.8 ⇒ m = 50 Choice (B)
m 4000 − 3000
4
 9 
21. Total cost of the coffee powder bought by Raju = (40) (240) =   x 700
+ (160) (360) = 31200  10 
4 1 = 459.2 litres Choice (B)
Total selling price = (100) + (100) (270) = 34200
5 5
27. Let the amount of alcohol initially present be x litres. After
two successive dilutions,
Overall profit = 3000
3000 8 2 2
Overall profit % = (100) = 9 % Choice (A) x−6 9 3
  = = 
31200 13
 x  9 + 16  5 
22. Let the required proportion be x : y : z x = 15 litres Ans: (15)
10 x + 12y + 18z
= 14 28. Applying the alligation equation, Quantity of milk/quantity of
x+y+z 16 − 0
water = = 8 :1 Choice (A)
Going by the options, we have only Choice (D) satisfying 18 − 16
the above equation.
29. The original mixture had 90 litres of milk and 10 litres of
Alternate method: 9
water ∴It had th of milk
10
Prices of 3 varieties of sugar are `10, `12 and `18 per kg.
Let them be mixed in the ratio x : y : z After the first withdrawal of 10 litres, 9 litres of milk and
Hence, price of the mixture is 1 litre of water is lost.∴ The resulting mixture would have
(10x + 12y + 18z)/(x + y + z) 81 litres of milk. On then adding water the vessel would
But this value is given on `14. 81
have th of its contents as milk
Hence, (10x + 12y + 18z)/(x + y + z) = 14 100
⇒ 10x + 12y + 18z = 14x + 14y + 14z 2
⇒ 4x + 2y – 4z = 0; We want the milk concentration to be at most 66 %
3
⇒ 2x + y – 2z = 0
This equation has infinite solution sets. Hence, from among 2
(i.e less than or equal to 66 %), the milk quantity to be at
the given options, the one which satisfies the equation will 3
be the solution. 2
Option (D), i.e., 3 : 4 : 5 satisfies the equation. most 66 litres. After the second replacement the vessel
3
Choice (D)
729
would have th of its contents as milk. After the third
23. The data can be tabulated as : 1000
6561
Alcohol Water Concentration replacement, the vessel would have th of its contents as
10000
Mixture 1 1 part 2 parts 1/3 2
milk i.e. 65.61 litres of milk (which is less than 66 litres)
Mixture 2 4 parts 1 part 4/5 3
Combined The least number of times the procedure has to be carried
Mixture 1 part 1 part 1/2 out is 3. Ans: (3)

Triumphant Institute of Management Education Pvt. Ltd. (T.I.M.E.) HO: 95B, 2nd Floor, Siddamsetty Complex, Secunderabad – 500 003.
Tel : 040–27898194/95 Fax : 040–27847334 email : info@time4education.com website : www.time4education.com SM1001961/60
30. Let the initial volumes of milk in P and Q be p and q 8. Let the fifth number be x. Then, from the given data,
respectively. second number = 7x
After the first transfer, P would have 0.2p and Q would 7x + 3
have 0.8p + q. After the second transfer, P would have 0.2p first number =
3
+ 0.6(0.8p + q) and Q would have 0.4(0.8p + q)
7x + 3
0.2p + 0.6(0.8p + q) 23 p 5 fourth number = −2
= ⇒ =
0.4(0.8p + q)
Choice (D) 3
12 q 2
7x + 3 7x + 3  7x + 3 
+ 7x + +  − 2  + x = 5 x 16 .2 = 81
Exercise – 7(b) 3 9  3 
Solving, we get x = 6 Choice (D)
Solutions for questions 1 to 40:
9. Let the average of the ages of the 11 men be A years. Let
1. Let A’s, B’s, C’s, and D’s weights be a kg, b kg, c kg and d the ages of the oldest and the youngest be x years and
kg respectively. y years respectively.
If any one person leaves the group the maximum and
b+c +d
a= = 3a = b + c + d ------------ (1) minimum possible average age of the remaining occur if
3 the person aged y years and the person aged x years,
respectively, leave
a+c +d
b= = 3b = a + c + d ------------ (2) 11 A – y = (10) (11) = 110 ------------ (1)
3 11 A – x = (10) (12) = 120 ------------ (2)
Adding (1) and (2), Adding (1) and (2),
3 (a + b) = a + b + 2 (c + d) 22A – (x + y) = 230
a+b=c+d x+y 5
= 11 A = 11 Choice (D)
c+d a +b 2 11
= 30 ∴ = 30. Choice (B)
2 2
10. Let the correct average of the 10 positive integers be
A and the number whose digits are interchanged be (ab)10
2. Total weight of 18 students = 18 x 15 = 270 kg.
10A = (ab)10 + ……….
If a student leaves the class, his weight = the total weight of 10(A – 1.8) = (ba)10 + …….
18 students – the total weight of the remaining -----------------------------------------
17 students = 270 – [17 x (15 – 0.8)] = 28.6 kg 18 = (ab)10 – (ba)10 = 10a + b – (10b + a)
Ans: (28.6)
= 9a – 9b = 9(a – b) Difference of the two digits a and
3. Y’s age is 32 – 4 = 28
b = 18/9 = 2 Choice (B)
Let the average age of 11 people be A
11A = 32 + 28 + k …… eqn. (1), where k is the sum of the 11. The age of the person who is replaced is ab. Let the sum of
remaining ages. the ages of the remaining persons be x
11(A – 1) = a + b + k……eqn. (2), ab + x = 60A and ba + x = 60(0.8A)
where a and b are the ages of the two people replacing ∴(ab + x) – (ba + x) = 60 (A – 0.8A) ( Q ab – ba = 9)
x and y. Subtracting equation (2) from equation (1), ⇒ 9(a-b) = 12A
11 = 60 – (a + b) ⇒ a + b = 49 and the average age of the
⇒ A = (a − b )
3
people replacing x and y = 49/2 = 24.5 years
Choice (D) 4
A has its maximum value when a – b has its maximum
4. Let the number of goals in the first match be a. The number value, which is 9 ∴Maximum (A) is 6.75. But all the ages
of goals in successive matches are a, a + 2, are two digit numbers and A cannot be 6.75, i.e the data is
a + 4, a + 6, a + 8, a + 10, a + 12, a + 14, and a + 16 consistent. Choice (D)
9a + 72 12. Let the number of matches totally played be N
The average number of goals = = a + 8 = 16
9 1200 1200 + 20 1200 1220
= =
N−1 N N−1 N
The average number of goals in the second and eighth 1200N = 1220 (N – 1) = 1220 N – 1220 Hence N = 61
2a + 16
matches = = a + 8 = 16 Choice (C) Alternate method:
2
In the last match, the score is 20; this can be treated as the
average score for the group of matches, whose number is 1.
5. As the tens digits of the 9 numbers are different, the tens The weighted average of this group and the group of
digits must be 1, 2, 3, 4, 5, 6, 7, 8 and 9. The units digits of matches played earlier is also 20. i.e., there is no change in
the numbers must be 0, 1, 2, 3, 4, 5, 6, 7 and 8. Their average; i.e., the average of the other group is 20. Hence,
36 number of matches of that group = 1200/20 = 60. Hence,
average = =4 Ans: (4)
9 total number of matches = 60 + 1 = 61. Ans: (61)

6. Total score of 36 students = 36 x 60 = 2160. 13. Let us say N members met for lunch
If the two highest scores are excluded, total scores of the 2160 2160
= + 81
remaining 34 students = 34 x 58 = 1972. N−6 N
Highest score of the class = 2160 – 1972 – 90 Substituting the choices in place of N in the equation
= 188 – 90 = 98 Choice (A) above, we see that only Choice (A) satisfies it.
Choice (A)
7. Total score = 2240
Total score, excluding the top and the least scores = (69) 14. Let the number of chocolates he ate on Monday = a
(30) = 2070 The number of chocolates he ate on subsequent days are
Let the top and the least scores be t and l respectively. (from Tuesday to Friday) Tuesday are a + 2, a + 10, a, a + 4
t + l = 2240 – 2070 = 170 ------------ (1) respectively.
t – l = 70 ------------ (2)  a + a + 2 + a + 10  a+a+4
Solving (1) and (2), T = 120 Ans: (120) 3   = 4  ⇒ 4 = a
 3   2 
Choice (B)
Triumphant Institute of Management Education Pvt. Ltd. (T.I.M.E.) HO: 95B, 2nd Floor, Siddamsetty Complex, Secunderabad – 500 003.
Tel : 040–27898194/95 Fax : 040–27847334 email : info@time4education.com website : www.time4education.com SM1001961/61
15. Let the numbers be a, b, c, a2 b2 c2, a3b3c3, …..a9b9c9 a + b + c = 132 p + 124 q + 136 r
1 a+b+c 66p + 62q + 68r
Average of the results = (a1b1c1 – c1b1a1 + a2b2c2 - x= =
9 2(p + q + r ) p+q+r
c2b2a2 + a3b3c3 – c3b3a3 + …….. a9b9c9 – c9b9a9)
1 62(p + q + r ) 68(p + q + r )
= [99 (a1 – c1 + a2 – c2 + a3 – c3 +…a9 – c9] <x<
9 p+q+r (p + q + r )
for i = 1 to 9 ai ≥ ci 62 < x < 68
As the average of the results is 0, it follows that ai = ci. ∴ Only 62.5 is the only possible value Choice (C)
As the units digits of the numbers are distinct, the units
digits must be from 1 to 9. The greatest and the least 23. Let the number of students in P be 2x
hundreds digits are 9 and 1 respectively. Let the average marks of P be 5y
Their average = 5 Choice (B) Numbers of students in Q, R and S are 3x, 6x and 4x
respectively.
16. Let the present ages of the father, his only son and his wife Average marks of Q, R and S are 4y, 3y and 2y
be f years, s years and m years respectively. respectively.
Total marks of P, Q, R and S are 10xy, 12 xy, 18 xy and
f +s 5 s+m 8 xy respectively. Greatest weighted average
=  
2 4  2 
 10 xy + 12 xy + 18 xy 10 xy + 12xy + 8 xy 
 , ,
⇒ 4 (f - m) = s + m 2x + 3 x + 6 x 2x + 3x + 4 x
⇒ f – m = 10 and m - s = 30 = maximum  
 12xy + 18 xy + 8 xy 10 xy + 18 xy + 8 xy 
⇒ s + m = 40 and m – s = 30 ⇒ s = 5 Ans: (5)  , 
 3x + 6x + 4x 2x + 6 x + 4 x 
17. As the average weight of A decreased after the student left, 7 1 12 7
his weight must be more than the average weight of A. As the Max (3 y,3 y,2 y, 3 y ) = 3 y
11 3 13 11
average weight of B decreased after the student joined, his
weight must be less than the average weight of B. 7 (52)(11)
⇒3 y = 52 ⇒ y =
His weight must be between 40 kg and 60 kg 11 40
Choice (B) 38
least weighted average = y = 41.8 Ans: (41.8)
13
18. Let a, b, c and d represent the amounts with A, B, C and D
respectively.
If a is increased by 100%, the new value is 2a. 24. Average value per coin = 775 paise/100 coins 7.75 paise/coin.
When b is increased by 200%, the new value is 3b. By the application of the alligation equation.
Hence, net increase in total value is (2a – a) + (3b – b) The number of 5 paise coins and the number of 10 paise
= a + 2b 10 − 7.75 2.25 9
coins = = =
This increase leads to an increase in the average, equal to 7.75 − 5 2.75 11
(a + 2b)/4. Hence the number of 5 paise coins
Given this increase = `62.50 = (9/20) x (100) = 45 Choice (D)
Hence, (a + 2b)/4 = 62.50
⇒ a + 2b = 250 ---------- (1) Note: The problem can also be solved using simultaneous
When the percentage increases are interchanged, then the equation.
amounts with A and B will be 3a and 2b respectively.
Average of 3a and 2b is given equal to 275. 25. Let the smaller part lent be ` x. The larger part lent
Hence, (3a + 2b)/2 = 275; = ` (8000 – x)
⇒ 3a + 2b = 550 -------- (2) 960
Subtracting (1) from (2), 2a = 300; a = 150 Choice (A) The effective rate of interest = (100)% = 12%
8000
19. From equation (1) of Q.33, (which forms common data), Applying the alligation rule,
C + D = 800 – (A + B) = 800 – (150 + 50) 8000 − x 20 − 12 4
= =
= 600 → (5) If C gives `50 to D, x 12 − 10 1
C – 50 = 2(D + 50) ⇒ C = 2D + 150 → (6) x = 1600. ∴ The difference of the parts = 8000 – 2x = 4800
Solving equation (5) and (6) Choice (A)
D = `150 and C = `450 Choice (D)
26. Let the part lent at 10% be `x and that lent at 12% be
20. By using the alligation equation, (1800 – x).
Number of clerks 10000 − 8000 2 x 12 − 10 .5 3
= = = =
Number of managers 8000 − 3000 5 1800 − x 10 .5 − 10 1
Hence x = 3/4 (1800) = 1350 and y = 1/4 (1800) = 450
Number of clerks = 50; ⇒ Number of managers = 125 Interest earned by the business man from the part lent at
Difference in the number of clerks and managers
1350 x 1 x 10
= 125 – 50 = 75 Ans: (75) 10% = = `135 Choice (A)
100
21. Let the cost of each apple be `A and that of each mango
be `M 27. Let the time in which the part which was covered at the
5A + 4M = 9 x 48 = 432 lower speed be x hours. The time in which the other part
7A + 8M = 15 x 60 = 900. 250
was covered = (5 – x) hours. Raju’s average speed =
Adding both the equations and multiplying with 2, we get 5
24A + 24M = `2,664 Choice (C) = 50 kmph
Applying the alligation rule,
22. Let the total marks of P and Q, P and R and Q and R be a,
x 80 − 50 3
b and c respectively. Let the number of students in P, Q = = ⇒x=3
and R be p, q and r respectively. 5 − x 50 − 30 2
a = 60 (p + q) The distance he covered at the higher speed
b = 64 (q + r) = 80 (5 – x) = 160 km Ans: (160)
c = 72 (p + r)
nd
Triumphant Institute of Management Education Pvt. Ltd. (T.I.M.E.) HO: 95B, 2 Floor, Siddamsetty Complex, Secunderabad – 500 003.
Tel : 040–27898194/95 Fax : 040–27847334 email : info@time4education.com website : www.time4education.com SM1001961/62
28. When different distances are travelled at different speeds, 39 − 26
then the average speed, s, is (ii) profit percentage = (100)% = 50%
26
s t + s2 t 2
s = 11 where s1, s2 are the speeds and t1, t2 are Choice (C)
s1 + s 2
the times of travel with respective speeds. 36. Let the quantities of A, B and C used for mixing be a kg,
t s −s b kg and c kg respectively.
Hence, 1 = 2 15(100 )
t2 s − s1 Cost price of the mixture = = `9 / kg
2
In the given problem, s1 = 45 kmph (scooter), 100 + 66
s2 = 70 kmph (car) 3
s = (250/5) = 50 kmph (average speed). Total cost price of the mixture = 6a + 9b + 12 c = 9
t 70 − 50 20 4 (a + b + c) ⇒ a = c
Hence, 1 = = = As b = 50 and a + b + c = 100, a = 25 and b = 25.
t 2 50 − 45 5 1
∴ The quantity of variety A is 25 kg. Ans: (25)
Total time of travel = 5 hours.
4 37. Let the cost of P, Q, R be `p, q, r. per kg (p = 200, given)
Hence, t1, time of travel by scooter = of 5 = 4 hours.
5 20
r+ r = 288 . ∴ r = 240
Distance covered by scooter = 4 x 45 = 180 km. 100
Choice (B) Let x kg of P and 2x kg of Q be mixed to form R. Let the
29. Once a litre of milk is added, we have the total quantity of cost of Q be `q/kg
the solution = 7litres. x(200 ) + 2 x(q)
= 240 ⇒ q = 260
3x
70
(6) Let a kg of the first variety and b kg of the second variety
Concentration of water = 100 x100 = 60% be mixed to form Q.
7
280 a + 180 b a 4
Choice (D) ∴ = 260 ⇒ a = 4b i.e., =
a+b b 1
30. Quantity of milk in 350 ml = 10/100 x 350 = 35 ml
Choice (C)
Once water is added, milk forms 7% of the solution. Hence
the total solution = 100/7 x 35 = 500 ml
38. Let x kg be the quantity of T1 in the mixture.
Hence the amount of water added = (500 – 350) ml = 150 ml
Cost of the mixture of T1, T2 and T3
Choice (D)
56( x ) + 64(2x ) + 80( 4 x )
31. Let the quantities of milk and water that are mixed be = = ` 72 per kg
7x
m litres and n litres respectively
Cost of water = `0 / litres Let y kg of T1 be added to this mixture. The cost of the final
Applying allegation, (72)( 4 ) + 56 y + 80( 4 y )
w 20 − 18 1 mixture =
= = Choice (D) 4 + 5y
m 18 − 0 9
87.60(100 )
32. Selling price of the mixture = `5 / litre This also equals = 73
100 + 20
5(100 )
Cost prices of the mixture = = ` 4/ litre
100 + 25 288 + 376 y 4
⇒ = 73 ⇒ =y Choice (D)
Let the quantity of water mixed be x litres 4 + 5y 11
x 5−4 1
Applying alligation, = =
1 4−0 4 39. Let the capacity of the vessel be x ml.
x = 0.25 Ans: (0.25) Amount of milk originally in the vessel = 9x/100 ml
Amount of milk in the vessel after replacement by water
33. Let x litres and y litres be the respective quantities to be = 6x/100 ml.
drawn from A and B to form the mixture Amount of milk in the 9 litres withdrawn
= 9(9)/100 = 81/100 ml
x + y = (x + y ) ⇒
4 3 3 x 27
= Choice (A) Hence 9x/100 – 6x/100 = 81/100;
9 4 5 y 28 ⇒ 3x/100 = 81/100
⇒ x = 27litres
34. Concentration of alcohol in the 1st mixture = 3/(3 + 4) = 3/7
Concentration of alcohol in the 2nd mixture Alternate method:
= (65)/65 + 79) = (65/144)
In the case of replacement, the relation between the initial
Concentration of alcohol in the combined mixture
and the final concentration is :
= 4/(4 + 5) = 4/9
If q1 and q2 are the quantities of mixture 1 and mixture v − x
C1   = C2
2 that are combined, then, by the equation of alligation,  v 
q1
=
(65 / 144) − (4 / 9) = 1/ 144 = 63 = 7 C1, C2 are the initial and the final concentrations
q2 (4 / 9) − (3 / 7) 1 / 63 144 16 respectively; v is the total volume, x is the volume replaced.
Choice (A) v − 9
Hence, 9%   = 6%; ⇒ v = 27 Choice (A)
35. The ratio in which A and B are mixed = 2 : 3  v 
Let the quantities of A and B mixed be 2x kg and 3x kg
respectively. 40. Let the capacity of the vessel be x litres.
Cost of the mixture formed
(i) quantity of milk in the vessel finally
20( 2x ) + 30(3 x )
= = `26 per kg 2
5x x −9
(i) Cost of the mixture formed by interchanging the = x   = x – 17.1
 x 
20(3 x ) + 30( 2x )
quantities of A and B = = `24 per kg
5x x2 – 18 x + 81 = x2 – 17.1x x = 90 Ans: (90)
Choice (C)
Triumphant Institute of Management Education Pvt. Ltd. (T.I.M.E.) HO: 95B, 2nd Floor, Siddamsetty Complex, Secunderabad – 500 003.
Tel : 040–27898194/95 Fax : 040–27847334 email : info@time4education.com website : www.time4education.com SM1001961/63
(ii) let the number of further replacements be n. 43. Either of the statement alone is not sufficient as the
n+2 information about six numbers is not given in one
x −9 75 x
  ≤ statement.
 x  100 Using both the statements,
x = 90 We can say that three numbers are more than 30.
(0.9)n+2 ≤ 0.75 ------------ (1) Choice (C)
least value of n satisfying (1) is 1 Ans: (1)
44. Using both statements Ι and ΙΙ, we can say that x is more
Solutions for questions 41 to 45: 4
concentrated. (concentration of x is = 57.14% and then
7
41. From statement Ι, 18 students are 15 years old. 6
From statement ΙΙ, the average age of 4 students is less concentration of y is < 50%) Choice (C)
than 15 years. 13
Combining statements Ι and ΙΙ, there are two students
whose age is less then 15 years and the remaining 45. Either of the statements alone is not sufficient as the total
18 students are 15 years old. score of boys and girls is given in statement Ι and total
So, we can say that the average age of 20 students is less score of girls is given in statement ΙΙ.
than 15 years. Choice (C) Using both the statements,
Let the average score of the boys be k
42. Statement Ι alone is sufficient as tin by weight is kx + 86y = 85(x + y) ⇒ kx = 85x – y
y = x(85 – k)
  2 3 
1 −  +   × 60 kg As x and y are positive, k must be less than 85.
  5 16   So the average score of the boys is less than 85 but we
Statement ΙΙ alone gives by volume. So, it is not sufficient. can’t say whether it is more than 83 or not. Choice (D)
Choice (A)

Triumphant Institute of Management Education Pvt. Ltd. (T.I.M.E.) HO: 95B, 2nd Floor, Siddamsetty Complex, Secunderabad – 500 003.
Tel : 040–27898194/95 Fax : 040–27847334 email : info@time4education.com website : www.time4education.com SM1001961/64

You might also like